Sie sind auf Seite 1von 353

Rapid Review of

Radiology
Shahid Hussain
MA, MB, BChir, MRCP, FRCR
Consultant Radiologist
Heart of England NHS Foundation Trust, Birmingham, UK

Sherif Aaron Abdel Latif


MB, ChB, MRCS, FRCR
Consultant Radiologist
Dudley Group of Hospitals NHS Foundation Trust, West Midlands, UK

Adrian David Hall


MB, ChB, MRCP, FRCR
Consultant Radiologist
Dudley Group of Hospitals NHS Foundation Trust, West Midlands, UK

MANSON
PUBLISHING

Dedications
S.H. To my mum
S.L. To my mum and dad, my wife Darine and baby Danny
A.H. - For my parents, my wife Kirstie and my sons Oliver, Tristan and Seb

Copyright 2010 Manson Publishing Ltd


ISBN: 978-1-84076-120-7
All rights reserved. No part of this publication may be reproduced, stored in a retrieval system
or transmitted in any form or by any means without the written permission of the copyright
holder or in accordance with the provisions of the Copyright Act 1956 (as amended), or
under the terms of any licence permitting limited copying issued by the Copyright Licensing
Agency, 3334 Alfred Place, London WC1E 7DP, UK.
Any person who does any unauthorized act in relation to this publication may be liable to
criminal prosecution and civil claims for damages.
A CIP catalogue record for this book is available from the British Library.
For full details of all Manson Publishing Ltd titles please write to:
Manson Publishing Ltd, 73 Corringham Road, London NW11 7DL, UK.
Tel: +44(0)20 8905 5150
Fax: +44(0)20 8201 9233
Website: www.mansonpublishing.com
Commissioning editor: Jill Northcott
Project manager: Jane Fricker
Design: Cathy Martin, Presspack Computing Ltd
Colour reproduction: Tenon & Polert Colour Scanning Ltd, Hong Kong
Printed by: Replika Press Pvt Ltd, Kundli, Haryana, India

Contents
Preface

Contributors

Abbreviations

General Introduction

Chapter 1 Chest Imaging

11

Chapter 2 Abdominal Imaging

71

Chapter 3 Central Nervous System, Head and Neck Imaging

147

Chapter 4 Musculoskeletal Imaging

205

Chapter 5 Paediatric Imaging

289

Chapter 6 Breast Imaging by Pro fesso r Iain L yburn

331

Further Reading

343

Index of Differential Diagnoses

344

General Index

346

Preface
As in all specialties, learning in radiology is a lifelong process. Yet the rate of learning is undoubtedly highest in the
trainee years, when there is a vast amount of information
to assimilate. There are many excellent, comprehensive
radiology texts available to facilitate this, yet learning for
most of us is a more haphazard process than reading such
a text from cover to cover. More commonly, we learn
initially unrelated pieces of information from multiple
sources and these gradually join together like the pieces of
a jigsaw, using the comprehensive textbook as a reference
along the way. The practical aspects of image recognition
and formulating a differential diagnosis become ever more
important, especially as postgraduate exams loom! Again,
there are already excellent radiology atlases and text-based
differential diagnosis guides that become standard reading
for the trainee.
Our aim in writing this text was to bring together the
images with the differential diagnosis information, adding
some practical advice along with it. Such a book cannot be
comprehensive without becoming another large general
text. Instead, we have tried to compile a selection of cases
covering many aspects of radiology, particularly the types
used in radiology vivas. These include the so-called Aunt
Minnie cases, where a classic image becomes simple
pattern recognition, and also the cases where findings need
to be pieced together along with the clinical history to
formulate a differential diagnosis. This selection of cases
will thus provide image and factual learning material for a
broad sample of disorders that will intertwine with material
learnt elsewhere. The format is intended to help the trainee
starting out with practical issues such as how to approach
films and the vocabulary to use; the trainee approaching
exams with a means of self-testing and rehearsing cases;
and also the nonradiologist to practise some more
challenging material. The selection of cases is hopefully
broad enough to provide an introduction to some exam
favourites for the beginner, but also more testing cases for
those in later stages of training.
In order for the candidate to test him- or herself, we
have presented each case as an image (or set of images)
together with the pertinent clinical details. Over the page
is a description of the images as would be given to an
examiner in a long case or a viva situation. This is perhaps

the most important part in getting to the correct diagnosis


since the description contains the relevant positive and
negative imaging findings, and will help to identify the
correct diagnosis as well as narrow down the differentials.
The correct diagnosis for the film is given, followed, in
most cases, by a differential diagnosis list, which contains
the best diagnoses to fit with the images and history. A
discussion of the underlying diagnosis is then presented;
this includes teaching points and main imaging findings
regarding the diagnosis and any other important
conditions that emerge in the course of the discussion.
Each case concludes with practical tips and notes on
further management.
We have tried to include as many plain radiograph
images as possible, since these still constitute the primary
investigation performed in radiology departments.
Subsequent investigation is usually based on these initial
images and therefore their correct interpretation cannot
be understated. In the abdominal chapter, we have also
included many barium contrast images, which are often
poorly performed on in examinations. Subsequent multimodality images have been included ultrasound, CT,
MRI, interventional radiology and nuclear medicine. In
this way, we have presented the imaging cases as they
would appear in both exam situations and in the real
world. For example, chest disease would be initially investigated with a chest x-ray, with subsequent investigation
with a CT. CNS disease is usually initially investigated with
CT/MRI. We have therefore tried to follow the clinical
investigation pathway as closely as possible to present
trainees with the images in the order in which they would
expect to see them, in exams and clinical situations.
The lists of differential diagnoses, radiological signs and
practical tips are often presented in bullet point list format
to allow for rapid revision just before the exams. We have
also indexed the differential diagnosis lists allowing for easy
and quick referral throughout training, and when it comes
time to revise. Hopefully, we have provided a book that
can be referred to throughout the radiologists training,
through various levels of ability.
Shahid Hussain, Sherif Latif and Adrian Hall

Contributors
Professor Iain Lyburn, Consultant Radiologist, Gloucestershire Hospital NHS Foundation Trust for the breast
imaging chapter.
Dr Bernd Wittkop, Consultant Radiologist, Worcestershire Acute Hospitals Trust for the nuclear medicine cases.
For the contribution of images: Dr Anne Gregan,
Consultant Radiologist, Dudley Group of Hospitals NHS
Trust; Dr Peter Oliver, Consultant Radiologist, Dudley
Group of Hospitals NHS Trust; Dr Ruth Shave,
Consultant Radiologist, Dudley Group of Hospitals NHS
Trust; Dr Katharine Foster, Consultant Radiologist,
Birmingham Childrens Hospital NHS Foundation Trust;
Dr Umesh Udeshi, Consultant Radiologist, Worcestershire
Acute Hospitals Trust; Dr Dee Dawkins, Consultant
Radiologist, Sandwell & West Birmingham Hospitals NHS
Trust; Dr Hong Gap-Teo, Consultant Radiologist,

Sandwell & West Birmingham Hospitals NHS Trust; Dr


Colin Walker, Consultant Radiologist, University Hospital
Birmingham NHS Foundation Trust; Dr J Reynolds,
Consultant Radiologist, Heart of England NHS
Foundation Trust; Dr M Djearaman, Consultant
Radiologist, Heart of England NHS Foundation Trust;
Dr A Pallan, Consultant Radiologist, Heart of England
NHS Foundation Trust; Dr S Roy-Choudhury, Consultant
Radiologist, Heart of England NHS Foundation Trust;
Dr S Cooper, Consultant Radiologist, Heart of England
NHS Foundation Trust.
We would especially like to thank Dr L Arkell for the
film library she collated over the course of her career and
passed on at retirement, many cases from which have been
used in the course of this text.

Abbreviations
ABC
ABPA
AD
ADC
ADEM
ADPKD
AF
AIDS
ANA
AP
ASD
AVM
AVN
AXR
BP
CAD
CAM
CC
CDH
CF
CFA
CMV
CNS
COP
CP

aneurysmal bone cyst


allergic bronchopulmonary aspergillosis
autosomal dominant
apparent diffusion coefficient
acute disseminated encephalomyelitis
autosomal dominant polycystic kidney
disease
atrial fibrillation
acquired immunodeficiency syndrome
antinuclear antibodies
anteroposterior
atrial septal defect
arteriovenous malformation
avascular necrosis
abdominal x-ray
blood pressure
computer-aided detection
cystic adenomatoid malformation
craniocaudal
congenital diaphragmatic hernia
cystic fibrosis
cryptogenic fibrosing alveolitis
cytomegalovirus
central nervous system
cryptogenic organizing pneumonia
cerebellopontine

CRM
CSF
CT
CVA
CWP
CXR
DCIS
DWI
EAA
ECG
ECMO
ENT
ERCP

circumferential resection margin


cerebrospinal fluid
computed tomography
cerebrovascular accident
coal workers pneumoconiosis
chest x-ray
ductal carcinoma in situ
diffusion weighted imaging
extrinsic allergic alveolitis
electrocardiogram
extracorporeal membrane oxygenation
ear, nose and throat
endoscopic retrograde
cholangiopancreatography
FAP
familial adenomatous polyposis
FDG
18 fluoro-2-deoxyglucose
FFDM
full-field digital mammography
FLAIR
fluid attenuated inversion recovery
FNA
fine needle aspiration
FNH
focal nodular hyperplasia
GBM
glioblastoma multiforme
GCT
giant cell tumour
Gd-BOPTA gadolinium benzyloxypropionic-tetraacetate
Gd-EOB-DTPA gadolinium-ethoxybenzyldiethylenetriamine penta-acetic acid

Abbreviations

GI
gastrointestinal
HIV
human immunodeficiency virus
HMD
hyaline membrane disease
HPOA
hypertrophic pulmonary osteoarthropathy
HRCT
high-resolution computed tomography
HRT
hormone replacement therapy
HSP
HenochSchnlein purpura
HSV
herpes simplex virus
HU
Hounsfield units
IAC
internal auditory canal
IDC
invasive ductal carcinoma
ILC
invasive lobular carcinoma
IPF
idiopathic pulmonary fibrosis
ITU
intensive treatment unit
IV
intravenous
IVC
inferior vena cava
IVU
intravenous urogram
LP
lumbar puncture
MCA
middle cerebral artery
MCUG
micturating cystourethrogram
MEN
multiple endocrine neoplasia
MIBG
meta-iodobenzylguanidine
MIP
maximum intensity projection
MISME syndrome
multiple inherited schwannoma,
meningioma and ependymoma
MLO
mediolateral oblique
MRA
magnetic resonance angiography
MRCP
magnetic resonance
cholangiopancreatography
MRI
magnetic resonance imaging
MS
multiple sclerosis
MSK
musculoskeletal
NAI
non accidental injury
NEC
necrotizing enterocolitis
NF1
neurofibromatosis type 1
NG
nasogastric
NOS
not otherwise specified
NSAIDs
nonsteroidal anti-inflammatory drugs
OA
osteoarthritis
OKC
odontogenic keratocyst

OPG
PA
PCA
PCP
PD
PDA
PET
PICA
PIE
PMF
PNET
PSC
PVL
RA
RTA
SACE
SAH
SBC
SI
SLE
SMV
STIR
SUFE
SVC
TB
TCC
TE
TIA
TIPS
TME
TNM
TOF
UBOs
UC
US
UTI
VAD
VATS
VHL
VSD

orthopantomogram
posteroanterior
posterior cerebral artery
Pneumo cystis carinii pneumonia
proton density
patent ductus arteriosus
positron emission tomography
posterior inferior cerebellar artery
pulmonary interstitial emphysema
progressive massive fibrosis
primitive neuroectodermal tumour
primary sclerosing cholangitis
periventricular leukomalacia
rheumatoid arthritis
renal tubular acidosis
serum angiotensin converting enzyme
subarachnoid haemorrhage
simple bone cyst
superioinferior
systemic lupus erythematosus
superior mesenteric vein
short tau inversion recovery
slipped upper femoral epiphysis
superior vena cava
tuberculosis
transitional cell carcinoma
time to echo
transient ischaemic attack
transjugular intrahepatic portosystemic
shunt
total mesorectal excision
tumournodemetastases staging
tracheoesophageal fistula
unidentified bright objects
ulcerative colitis
ultrasound
urinary tract infection
vacuum assisted device
video assisted thorascopic surgery
von HippelLindau syndrome
ventricular septal defect

GENERAL
INTRODUCTION
This book has been written primarily for senior radiology
trainees preparing for final radiology exams and in particular
for trainees studying for the Fellowship of the Royal
College of Radiology. With this in mind, the cases
presented herewith have been presented in the exact
manner that the cases are presented in the Royal College
of Radiology FRCR 2B exam. In the exam, long cases and
viva cases are presented with a very minimal but highly
relevant history and the required response is expected to
be presented in a particular way. This format of reporting1
involves giving a Description/Interpretation of the images;
Diagnosis; Differential Diagnosis; and advice on Further
Management. We have laid out the answers here in exactly
this manner and have included a Discussion to give indepth further information about each condition which will
enable the student to answer any questions directed to him
or her in the viva situation. By following the RCR exam
format, the candidate should be ideally prepared for this
exam and for the future as a Consultant Radiologist.

FILM TECHNIQUE
Whatever the imaging modality, the radiologist interprets
images using all the information and clues available, to
produce a differential diagnosis and/or advise on further
investigation and management. Above all else, this must be
done in a SAFE manner, and this often requires one to be
systematic in approach. Secondly, this process must be done
in a SENSIBLE manner it is easy to quote endless lists of
differential diagnoses but if these are not refined for each
individual case, the radiologists input is of little value. The
following discussion concentrates in particular on
performing these tasks in the examination viva scenario.
However, much of the advice is applicable to everyday
practice too, in particular the emphasis on a safe and
sensible approach.

TYPES OF FILM
The types of film one may encounter in an exam/viva are as
follows:

The Aunt Minnie


There are certain disorders that have a characteristic
appearance on imaging that allows one to make an instant
spot diagnosis. It will be assumed the candidate has come
across it before, and thus the best preparation here is

exposure to as many of these cases as possible. Radiological


atlases and film libraries provide ready access to many of
these classic cases, which can then be committed to
memory. You can prepare a ready-made description of
these cases for the viva. If you are sure of the diagnosis,
dispatch the film promptly with your preprepared speech
so that you can progress to the next case as soon as
possible. Of course there may be Aunt Minnie cases that
you havent seen and this may present a problem. Such
cases are often not amenable to working out the diagnosis
you either know it or you dont. The only thing to do is
be methodical in your analysis and description of the
findings so that at the very least you can suggest whether
you feel an abnormality is likely to be longstanding and
benign or otherwise, and make appropriate suggestions on
how you would proceed.

The test of observation


Here, there is an abnormality present that once seen, may
well lead to an easy diagnosis. The abnormality is subtle or
hidden however, such that it tests the candidates
perception and approach to a case. Perceptual ability,
however, is variable, not only between people but also in
the same observer on different days (this is particularly true
in examinations where anxiety levels are high). You must
therefore be systematic in analysing each film if there is no
obvious abnormality to see on first inspection. There are
many different systematic approaches and it is beyond the
scope of this discussion to be more prescriptive. However,
make sure you have a system and use it. Moreover, describe
the process you are going through aloud in the viva so that
the examiner knows that you are practising safe radiology.

The jigsaw puzzle


This type of case presents several findings that once
identified and considered together, lead to a specific or
differential diagnosis. This not only tests perceptual skill
and systematic approach, but also the ability to mentally
cross-reference several differential diagnosis lists for the
various abnormalities identified, to find the best fit
diagnosis. Whenever producing a differential diagnosis in
an examination or real life, it is vital to produce a sensible
list, not just a recital of long lists learnt from books. To do
this, you must use all clues available from the clinical
history and film, and combine this information with
knowledge of the incidence of each possibility in a given
patient population.

General Introduction

The discussion

Further management

In the real world, there are many abnormal radiological


studies that have no specific best fit diagnosis, but rather
a differential diagnosis that cannot be narrowed down
without further investigation. It is easy to assume that all
cases used to test an examination candidate will have a
single correct answer but this is a dangerous assumption
these real world cases with no specific diagnosis are clearly
a good test of how a radiologist will operate in daily
practice. As always, a safe and systematic approach to film
analysis is vital, together with a sensible approach to a
differential diagnosis. Such cases, in particular, also assess
the other role of the radiologist advising on further
investigation and management.

Suggest further investigations to confirm the diagnosis


or suggest further clinical management.

ANALYSING THE CASE


As already emphasized, have a system of analysis for all
types of film. More specific advice on possible approaches
will be given in further chapter introductions.
Use all clues available to you. Background clinical
information is most important when interpreting
radiological studies, so listen very carefully to any
information the examiner provides with the case. Make a
note of the age and sex of the patient if possible using
either identification data on the film or anatomical
information many disorders can be eliminated or
suspected from the differential diagnosis list on the basis of
such simple information. Specific features on the film may
also help narrow down the differential diagnosis the
presence of central venous cannulae and airway intubation
immediately indicates a seriously ill patient for example.
Examiners will often try to help you. For example, they
may offer further information, affirm your suggestions or
perhaps suggest you might like to reconsider something
you have said. It is fairly safe to assume they are not trying
to mislead you deliberately, so do not ignore their hints. If
their hints lead you to reconsider previous statements as
erroneous, do so graciously and honestly you cannot fool
them that you knew all along.

PRESENTING THE CASE


When presenting any radiological film in an exam or as a
report in clinical practice, the approach should be the same.
The Royal College of Radiology has given guidance on the
required format that reports in the long cases/viva should
take and this is as we have presented the cases in this book.
After briefly looking at the film, present the:

Description
Relevant positive and negative radiological findings in a
systematic manner, summarizing the findings at the end of
the description.

Diagnosis
Try to identify relevant information in the history/patient
data/radiological findings/other investigations which
narrows the differential list down.

Differential diagnosis
Give a list of differentials for the findings.

Obviously, the description is presented dynamically at the


same time as evaluating the film and this requires some
mental multitasking. You can afford a brief pause when
the case is presented to make an initial assessment, but after
a few seconds, further silence does not create a good
impression. Introductory statements such as This frontal
chest radiograph of an adult male or this AP radiograph
of the humerus in an unfused skeleton are not only
appropriate, but also buy you another brief moment to
think. In the initial stages of assessing an abnormality prior
to reaching a conclusion, be careful not to use specific
terminology that implies a specific diagnosis.
Even when you are completely unsure of what the
abnormality is, it is still important to keep talking thus
providing evidence that you continue to approach a film
systematically and safely, even when you are uncertain.
Summarizing your negative findings can often be as
important as the positive ones and even if you remain
oblivious to any abnormality after considered review of the
case, verbally excluding acute life-threatening possibilities
in your analysis shows, at the very least, a degree of safety
in your practice.
Asking questions of the examiner to help interpret the
film is usually acceptable, but only after you have made a
considered assessment of the film and have either offered
some thoughts or made it clear that you are merely seeking
final confirmation of a particular possibility you have in
mind. Asking questions early on, before you have made a
systematic analysis of the case, however, only points to
desperation.
The final stage of suggesting further investigation or
management is vital in illustrating that you are a safe and
valuable practitioner. If you have identified a lifethreatening emergency on a film, statements such as I
would contact the referring doctor immediately are
essential. Finally, when considering whether further
investigation should be suggested, you may well note that
the examiner already has a film ready to show you.
It is useful to practise ways of ending a case because
there is danger in continuing to talk and talk about a case
when you have already reached the limit of what you can
interpret from it it is all too easy to dig yourself into a
hole in this situation, making comments that appear
indecisive. If you have already suggested a differential
diagnosis, likely diagnosis and suggested further
management options, you have naturally provided a
conclusion just make sure you look at the examiner when
doing so to clearly make the point that you have finished.
If you are uncertain about what the film shows, it can
be more difficult. However, if you have reached an impasse
and cannot proceed any further, options available include
presenting a summary of your observations, suggesting
further investigations, asking for more information, etc.
(always looking at the examiner).

General Introduction

GENERAL VIVA CONSIDERATIONS


Practice makes perfect.
Practice sessions in groups where you get to watch
your colleagues being tested are particularly useful as
you can learn much from their errors and successes. As
well as honing your radiological skills, take note of
irritating personal habits that might be better resolved
before your real examination (e.g. fidgeting). Practise
your descriptive findings for the classic exam-type
cases so that you can dispatch these cases quickly and
with confidence.
See as much material as possible.
Time spent looking through film libraries and
radiological atlases will build up a knowledge base of
classic exam-type cases, whilst time spent looking at
endless everyday studies not only fosters sensible real
world practice, but will also turn up pathology and
build up your mental database of normal
appearances.
Dont give up.
Anxiety levels are high during examinations and can
lead to foolish errors. However, never assume that a
particular case has gone so badly that overall failure is
inevitable. There are no doubt many candidates who
have failed an exam, not because of the single case
they thought they had failed, but because of the cases

they failed thereafter because of this presumption and


altered mental state. Every case is a new opportunity
to demonstrate your ability, irrespective of how badly
the previous case went.
Be thorough.
At the same time, however, it is advantageous to get
through as many cases as you can. Thus, when you
are presented with an Aunt Minnie case of which
you are confident, proceed quickly and confidently
through it using your preprepared speech.
Do not miss life-threatening or serious conditions.
Make sure you check for conditions such as a fracture
and pneumothorax when you have failed to identify
any other abnormality.
Be safe.
Most of all, show that you are safe and sensible in
your practice.

Reference
1. Royal College of Radiology Guidance on Format of
Reporting Session Reports for FRCR Part 2B;
http://www.rcr.ac.uk/content.aspx?PageID=713

SUMMARY
Use the same structured format for reporting radiology cases, which
consists of:

DESCRIPTION
Give your OBSERVATIONS on the films including relevant positive
and negative findings and then give your INTERPRETATION of
these findings.

DIAGNOSIS
Give the best fit diagnosis for the image findings.

DIFFERENTIAL DIAGNOSIS
Give a limited number of possible differential diagnoses for the image
findings.

FURTHER MANAGEMENT
Give suggestions for relevant further investigations and immediate
management which need to be undertaken.

This page intentionally left blank

Chapter 1

CHEST IMAGING

The approach to plain chest radiographs and to chest CT


scans is essentially the same and it requires a systematic
approach to the images presented. A suggested approach
to these images is presented here, though it is worth finding
a systematic approach that works best for you, covering all
of the important areas and that you find easy to remember.
Ensure that whichever approach you take, it is well
practised and well rehearsed, since it becomes rapidly
obvious to examiners whether or not you have looked at
and regularly reported this type of film before. As always,
you should be able to modify the specific order in which
you carry out your systematic analysis depending on the
most apparent findings.

THE PLAIN CHEST RADIOGRAPH


Initial assessment
There are three vitally important things to do when first
presented with a chest radiograph:
1. Make a quick mental note of any technical inadequacies
that might influence further interpretation, e.g. suboptimal
exposure, rotation. Remember that in an exam situation
the examiner will have brought their best example of a
given case, so although it is worth noting the flaws in the
film, vocalizing these would not be advised unless they are
significantly hampering your ability to make a diagnosis.
2. Unforgivable misses: there are a few conditions that
you must simply never miss:
Pneumothorax in particular tension pneumothorax is
a medical emergency requiring an immediate chest
drain.
Free gas under the diaphragm, indicating perforation
of an abdominal viscus.
In real life the clinical history will often guide you to these,
but in a viva, it is worth making a rapid exclusion of such
conditions early on in your own mind before you become
immersed in detailed evaluation and discussion of the case.
Missing such serious abnormalities is unacceptable and only
spotting them after several minutes of evaluation doesnt
inspire confidence!
3. In a viva, you have about 1015 seconds to make your
initial evaluation before you need to start speaking any
information that can be gained about the patients sex, age
and ethnicity will be useful in making a diagnosis, so look
for this information on the film.

Lines
Comment on the additional lines that you can see on the
film these may include ECG leads, oxygen tubing, NG
tube, central venous catheters, chest drain and
pacemakers/wires. These are important, first because you
should assess that they are in the correct place and that
their insertion has had no complications, e.g. ensure the
NG tube is in the stomach, that there is no pneumothorax
associated with the jugular central line. Secondly,
identifying lines is important in assessing how unwell the
patient is your differential diagnosis is clearly going to be
different if the chest radiograph is from an intubated ITU
patient than if it is from an outpatient.

Lungs
First assess the chest to ensure that the general
opacification in both lungs is the same if not, then you
need to determine which is the abnormally hyper or hypo
transradiant lung and determine what the cause is.
Mastectomy is one obvious cause that must not be missed
as a history of breast cancer means the film must be
checked carefully for evidence of metastatic spread.
Once you have decided what/where the abnormality is
then describe the findings using a common vocabulary that
is understood by all and which leaves no room for
confusion. Accordingly, abnormalities can be described as:
Focal or diffuse.
Located in upper/middle/lower zones (avoids the
difficulty of assessing lobes at the outset).
Central or peripheral.
Single or multiple.
Exhibiting calcification or cavitation.
Broadly speaking lung abnormalities consist of:
Focal pulmonary lesion
You need to assess whether there are single or multiple
lesions. Is there calcification? Is there cavitation? The
differential diagnosis will depend on these patterns and
useful information that will help you to narrow down the
diagnosis can be sought from the examiner, such as a
history of pyrexia, weight loss, haemoptysis, etc.
Diffuse pulmonary opacity
First, make a distinction between alveolar (airspace) and
interstitial opacity. The former has a poorly defined, fluffy,
cottonwool-like appearance, while the latter consists of
reticular opacities, nodular opacities or a combination of
both. The differential diagnosis will often be very different,
as explained in subsequent cases in this chapter.

11

Chest Imaging

Hila
Assess the hila for position (left should be slightly higher
than the right), shape (a V-shape made by the angle of the
superior pulmonary artery and inferior pulmonary vein) and
size.
A change in the position of the hilum suggests volume
loss. Accompanying interstitial opacity suggests
fibrosis, whereas airspace opacity might suggest
atelectasis, for which there are many causes, in
particular obstructing masses.
If there is change in the hilar angle or size then this
would suggest a hilar mass such as enlarged nodes or
central tumour.

Mediastinum
Comment on the heart size is it enlarged or small? If
there is any doubt from observation alone, state that you
would like to formally measure it. Assess the mediastinal
contour, including the cardiac contour itself and the great
vessels. Ensure the trachea is central, that there is no
pneumomediastinum or pneumopericardium. Large mediastinal masses will be readily evident but check that there is
no loss of the paratracheal stripe due to smaller masses such
as lymphadenopathy.

General tips
Comparison with previous films is very useful in
everyday practice. A solitary pulmonary nodule
measuring 2 cm but not present on a film taken just a
few months ago may well be neoplastic, whereas a
stable appearance over many years makes this unlikely.
Once you have described the film and given a
differential diagnosis, only then ask for other studies
or more clinical information if it will help narrow
down the list of possibilities or aid management it
shows that you are able to make a differential list with
no clinical information and then narrow this down in
line with the clinical scenario. This is radiology in
practice!

CT THORAX
Just as for the plain radiograph, one must systematically
evaluate lungs, mediastinum, pleura, bones, peripheral soft
tissues, etc. In doing so, it is of course important to utilize
soft tissue, bone and lung windows if available. Make a
note of whether IV contrast has been given.

Pleura

Lungs

Pleural lesions form an obtuse angle with the chest wall, as


compared to a lung parenchymal lesion, which forms an
acute angle. A differential list for pleural lesions should be
easily brought to mind. Check for underlying rib
destruction as a sign of a malignant nature. Always consider
a pleural origin for opacities projected over the lungs but
with unusual appearance, e.g. the typical holly leaf pattern
of pleural plaques.

Focal lesions may well be obvious but dont forget to


check for more subtle abnormalities. Look carefully for
interstitial abnormalities, paying particular attention to any
relation to the secondary pulmonary lobule. Is the overall
lung density normal and homogeneous? Poor inspiration
will cause an artefactual increase in lung attenuation but
will be evident by inward bowing of the posterior wall of
the trachea. Dont forget to check the airways both large
and small. Modern multidetector CT scans provide
excellent visualization of the pulmonary vasculature when
IV contrast is given check for emboli.

Bones
Check the ribs, looking for:
Lytic/sclerotic lesions to suggest metastases.
Fractures if present look for a pneumothorax or
other signs of trauma. Certain fractures, such as those
of the upper three ribs, suggest significant trauma and
a high index of suspicion for other injuries is required.
Rib resection from a previous thoracotomy.
Check the thoracic spine, not forgetting to pay attention
to the paravertebral soft tissues that may point to
underlying bony abnormalities. The cervical spine may be
partly seen look for a cervical rib.
Bony abnormalities around the shoulders present some
favourite cases for vivas, so dont forget to check these areas
towards the edge of the film!

Review areas
Everyone will have particular areas on certain investigations
that they forget to evaluate well. These areas vary for each
individual, but there are certain recurring patterns of missed
abnormalities on the chest radiograph. So know your own
perceptual blind spots. In general, it is useful to check:
Behind the heart to look for a hidden mass or left
lower lobe collapse.
At the lung apices is there a Pancoast tumour there?
Beneath the diaphragm is there free gas/
splenomegaly/liver lesion?

12

At the edges of the film for a bone abnormality.

Mediastinum
Check for normal patency and anatomy of great vessels,
excluding aneurysm, dissection flaps, etc. Assess for
abnormal lymphadenopathy. Assess heart for size,
morphology, normal myocardial enhancement, filling
defects, etc. Modern CT scans often demonstrate the
proximal coronary arteries even without specific cardiac
gating techniques. Dont forget to follow the superior
mediastinum into the lower neck and supraclavicular
fossae.

Pleura
Small pleural nodules and plaques can be easily missed if
not specifically evaluated. Pneumothorax may well only be
evident on lung windows.

Bones
Bone windows are essential. Multiplanar reformats made
possible by multidetector CT scanners make assessment of
the spine and ribs much easier than axial images alone.

Soft tissues and the periphery


Just as with the chest radiograph, check the soft tissues of
the chest wall, breast, neck, axillae and the upper abdomen
on the lowest scans for hidden or incidental pathology.

Chest Imaging

CASE 1

Cases 1, 2

1a

History
A 34-year-old Caucasian male presented
to his GP with a dry cough and
shortness of breath for 3 months. He
was otherwise fit and well.

CASE 2

2a

History
A 32-year-old Asian male presented
with productive cough, shortness of
breath and night sweats.

13

Answer 1

Chest Imaging

ANSWER 1
Observations (1a)
This chest radiograph shows bilateral reticulonodular
shadowing predominantly affecting the mid and upper
zones with sparing of the absolute apices. The nodules are
small, ~34 mm in diameter, and appear ill defined, and
form confluent airspace opacities centrally. There is hilar
enlargement more obvious on the right, and also widening
of the right paratracheal soft tissues due to
lymphadenopathy. Given the clinical details, sarcoidosis is
the most likely diagnosis.

Chest radiograph changes have been classified from


04 as follows:
Stage 0 Normal film.
Stage 1 Lymphadenopathy only.
Stage 2 Lymphadenopathy and pulmonary infiltrate.
Stage 3 Pulmonary changes without adenopathy.
Stage 4 Pulmonary fibrosis.
Diagnosis is usually confirmed by HRCT of the thorax.
The classical finding is of very small pulmonary nodules

Diagnosis
Pulmonary sarcoidosis.

Differential diagnosis

1b

For hilar node egg shell calcification:


Sarcoid.
Silicosis.
Lymphoma following radiotherapy.
For bilateral hilar enlargement:
Sarcoid.
Lymphoma.
TB.
For perilymphatic nodules:
Sarcoid.
Silicosis.
Coal workers pneumoconiosis (CWP).
Lymphangitis carcinomatosa.
Lymphoma.

Discussion
Sarcoidosis is a systemic disorder characterized by the
presence of noncaseating granulomas within several organs.
It most commonly presents in young adults in the 3rd5th
decades, and is more common in women and in black
populations. Multisystem involvement can result in uveitis,
bilateral parotid enlargement, erythema nodosum, lupus
pernio, arthralgia, heart block, cardiomyopathy and bony
changes most commonly seen in the phalanges.
Radiological features of pulmonary sarcoidosis are as
follows:
Lymphadenopathy (7080%) is characterized by
bilateral, symmetrical hilar lymph node enlargement
and paratracheal lymphadenopathy. Egg shell
calcification of the nodes (1b) is seen in ~5% of
patients. Lymphadenopathy without pulmonary
changes has a more favourable outcome. Figure 1c
shows bilateral hilar lymphadenopathy with no
parenchymal disease.
The most common form of parenchymal change is
multiple small pulmonary nodules within the mid and
upper zones of both lungs the distribution described
in this case is classical. Presentation with larger
nodules measuring 15 cm, possibly with central
cavitation and calcification, account for ~5% of cases.
Airspace opacities are seen in 210%.
End stage disease results in upper zone fibrosis with
associated traction bronchiectasis.

14

1b Chest radiograph shows bilateral hilar


enlargement with hilar lymph node egg shell
calcification.

1c

1c Axial CT image shows bilateral hilar


lymphadenopathy.

Answers 1, 2

Chest Imaging

distributed in a perilymphatic manner, i.e. along the


bronchovascular bundles, the interlobular septae and
subpleural region (1d).

Elevated serum angiotensin converting enzyme


(SACE) levels may provide further supportive
evidence of the diagnosis and the degree of disease
activity.

Practical tips
Whenever there is evidence of adenopathy and
abnormal pulmonary opacities, always consider
sarcoidosis in the differential diagnosis.
The features described above are largely the classical
appearances. However, sarcoid can have almost any
appearance on a chest radiograph, so be wary of
categorically dismissing the diagnosis when the chest
film shows abnormal pulmonary parenchyma.
Conversely, it will often be a reasonable condition to
list in the differential diagnoses because of its protean
appearances.
HRCT is very useful for further evaluation. Look for
nodularity along the bronchovascular bundles,
interlobular septae and pleura. Pleural nodularity due
to subpleural lymphatic spread is often easiest to
appreciate at the interlobar fissures where multiple
layers of pleura make the changes more apparent.

1d

Further management
Gallium scanning is now rarely used in diagnosing the
condition and in assessing disease activity. Recognized
patterns of activity include the lamda sign
(paratracheal and bilateral hilar uptake) and the
panda sign (uptake in the parotid, salivary and
lacrimal glands gives appearance of a panda face).

1d HRCT axial image demonstrates multiple nodules


distributed along the bronchovascular bundles and
subpleural regions with nodular thickening of the
interlobular septum.

ANSWER 2
Observations (2a)

Discussion

The chest radiograph shows consolidation in the medial


segment of the right middle lobe. There are right hilar and
right paratracheal nodular, soft tissue density opacities
consistent with lymphadenopathy. No pleural effusions are
seen.
Given the clinical details and radiological appearances,
pulmonary TB is most likely.

Although primary TB usually presents in children, there is


now increasing incidence in adults. Primary TB now
accounts for 2334% of all adult cases of TB.
Unlike post-primary pulmonary TB that usually
manifests in the upper lobes, primary TB affects lower
lobes, middle lobes and anterior segment of the upper
lobes. Presentation is with:
Parenchymal airspace consolidation.
Lymphadenopathy the patterns of distribution are
usually unilateral hilar +/ right paratracheal
lymphadenopathy or isolated right paratracheal
lymphadenopathy. Bilateral lymphadenopathy is
unusual and when it does occur, is usually
asymmetric.
Miliary tuberculosis.
Pleural effusion seen in 2338%.
Tuberculoma.

Diagnosis
Primary tuberculosis (TB).

Differential diagnosis
For consolidation and lymphadenopathy:
TB.
Sarcoid.
Lymphoma.
Other infective organisms: histoplasmosis,
mycoplasma, varicella.

(co nt.)

15

Answer 2

Chest Imaging

Complications of TB include:
Progressive primary TB due to a failed immune
response with subsequent disease progression.
Miliary TB massive haematogenous dissemination.
Post-primary/reactive TB presents in adults, with
reactivation of dormant organisms after several
asymptomatic years. Usually the apical and posterior
segments of the upper lobes are involved (8595%),
the superior segment of the lower lobes being less
commonly affected. Radiological presentation is with
cavitation, fibrosis, empyema, miliary TB,
tuberculoma, mycetoma. Adenopathy is not a major
feature.
HRCT can be useful for further evaluation and TB presents
with nodules in a centrilobular distribution due to
endobronchial spread. This gives a tree in bud appearance.
The differential diagnosis for centrilobular nodules includes
TB, endobronchial metastases, allergic bronchopulmonary

aspergillosis (ABPA), obliterative bronchiolitis and hypersensitivity pneumonitis. Figures 2b and 2c show axial CT
images demonstrating multiple poorly defined centrilobular nodules giving a tree in bud appearance in a
young male Asian patient with TB. Note that there is
sparing of the subpleural areas with no nodules seen within
5 mm of the pleural surfaces or fissures.

Practical tips
TB is a multisystem disorder with haematogenous spread
to multiple other sites. On a chest radiograph look for TB
discitis and TB affecting shoulder joints. The most
commonly affected joint is the hip with features of
monoarticular involvement. Radiographic changes of loss
of joint space, peripheral bone erosions and juxta-articular
osteoporosis are seen, which can lead to ankylosis of the
joint and limb shortening (2d).

Further management
Respiratory referral with a view to sputum cytology/
bronchoscopy to identify the acid fast bacilli.

Further reading
2b

2c

2b, 2c Axial CT images of the chest showing


widespread centrilobular nodules giving a
tree in bud appearance.

16

Harisinghani MG, McLoud TC, Shepard JO, et al.


(2000). Tuberculosis from head to toe.
Radio Graphics 20: 449470.
Kim HY, Song KS, Goo JM, et al. (2001). Thoracic
sequelae and complications of tuberculosis.
Radio Graphics 21: 839858.

2d

2d There is marked erosion of the left femoral head


with ankylosis of the hip joint. Marked limb
shortening is the consequence.

Chest Imaging

CASE 3

Cases 3, 4

3a

History
A 65-year-old male presented with
progressive dyspnoea and weight loss.

CASE 4

History
A 68-year-old male presented with
haemoptysis.

17

Answer 3

Chest Imaging

ANSWER 3
Observations (3a)
CT image of the thorax shows diffuse, irregular and
nodular pleural thickening in the left hemithorax. This
measures upwards of 1 cm in thickness and also involves
mediastinal pleura. No focal intraparenchymal pulmonary
abnormality is seen on these soft tissue window settings and
there is no visible mediastinal mass. Taken together with
the clinical history, the features are suspicious of
mesothelioma, though metastases cannot be excluded.
With regard to the former, the rest of the CT scan should
be examined for features of asbestos-related pleural disease
and asbestosis. An occupational history should also be
taken.

Diagnosis
Malignant mesothelioma.

Differential diagnosis
For diffuse pleural thickening:
Malignant mesothelioma.
Pleural metastases usually from adenocarcinoma
primary, e.g. breast, lung.
Empyema.
Pleural fibrosis from infection tuberculosis, fungal.

Discussion
Malignant mesothelioma is a rare primary tumour of the
pleura, seen more commonly in men, with a peak incidence
in the 7th8th decades. Aetiology is strongly associated
with asbestos exposure, and underlying lung changes from
asbestos are often seen in association. Between 5 and 10%
of asbestos workers will develop mesothelioma with a latent
period of up to 45 years. The parietal pleura is usually
involved and spread is locally to the chest wall,
mediastinum, diaphragm and peritoneum; there is
lymphatic spread to the hilar and mediastinal nodes and

haematologically to lungs, liver and adrenals. Unilateral


pleural effusion is commonly seen, but with no mediastinal
shift due to the tumour creating a fixed or frozen
hemithorax.
Prognosis is poor, with life expectancy of 12 months
after diagnosis. Treatment including radiotherapy,
chemotherapy and radical surgical pleurectomy produces
universally poor results.

Practical tips
Pleural thickening involving the mediastinal pleura is
a very good sign for mesothelioma.
Bilateral, calcified pleural plaques are characteristic of
asbestos exposure. (Figure 3b is a single axial CT
image showing calcified pleural plaques, which are
characteristic of asbestos exposure.) When identified,
examine the CT closely for evidence of asbestosis as
this may have greater immediate implications for the
patient in terms of morbidity (and in the UK at least,
financial compensation).

Further management
Pleural biopsy is required to confirm the diagnosis
and to differentiate mesothelioma from pleural
metastatic disease, which can show some response to
chemotherapy.
Discussion within a lung cancer multidisciplinary team
is required.
A detailed occupational history needs to be taken
since asbestos-related disease may entitle the sufferer
to compensation.

Further reading
Wang ZJ, Reddy GP, Gotway MB, et al. (2004).
Malignant pleural mesothelioma: evaluation with CT,
MR imaging, and PET. Radio Graphics 24: 105119.

3b

3b Axial CT image shows bilateral anterior calcified pleural


plaques characteristic of previous asbestos exposure.

18

Answer 4

Chest Imaging

Case 5

ANSWER 4
Observations (4)

Discussion

This chest radiograph shows complete opacification of the


left hemithorax with mediastinal shift to the left as
evidenced by the displaced trachea and elevated left
hemidiaphragm. There is a defect in the posterior aspect
of the left 5th rib in keeping with a previous thoracotomy
and presumed left pneumonectomy. In the right lung,
there are multiple small miliary nodules seen throughout
the lung. With evidence of previous pneumonectomy,
these most likely represent miliary metastases from a
previously resected bronchogenic tumour.

Thyroid cancer is classically described as a primary


carcinoma likely to produce miliary metastases (others
include melanoma and sarcoma). However, breast and
lung cancer can less typically produce this pattern, but
because they are more prevalent, may well be seen more
often.

Diagnosis
Miliary metastases from previous bronchogenic carcinoma.

Differential diagnosis
For miliary nodules:
Miliary metastases.
TB.
Sarcoid.
Chronic extrinsic allergic alveolitis.
Coal workers pneumoconiosis (CWP).
Histoplasmosis.
Of a completely opaque hemithorax:
Total lung collapse.
Pneumonectomy looks like total collapse but there
will be evidence of thoracotomy.
Huge pleural effusion volume expansion rather than
volume loss, i.e. the mediastinum will be displaced
away from the side of opacity.

CASE 5

Practical tips
This fairly straightforward case illustrates how films
used to test candidates in postgraduate exams will
often require piecing together of the pieces rather
than being presented with miliary shadowing alone
and being asked for a differential.
As always, look for other clues on the film. A primary
malignancy may be evident as a mastectomy, a thyroid
mass in the neck, a bony sarcoma on the edge of the
film or signs of previous lung cancer, as in this case.
Check the patient data for likely ethnicity to help
predict likelihood of TB.

Further management
Comparison with previous chest radiographs to look
for new features is very useful and should be done in
both clinical and exam situations.
A chest CT will characterize these changes in greater
detail.

5a

History
A 40-year-old male presented with
haemoptysis and renal impairment.

19

Answer 5

Chest Imaging

ANSWER 5
Observations (5a)
This frontal chest radiograph shows multiple relatively thick
walled cavitating nodules throughout both lungs.
Mediastinal contours are normal with no evidence on the
film of hilar or mediastinal lymphadenopathy. No pleural
effusion can be seen. Appearance of the shoulder joints and
the lateral ends of the clavicles is normal.

Diagnosis
Wegeners granulomatosis.

Differential diagnosis
For multiple cavitating lung nodules:
Neoplasia: metastases in particular, squamous cell,
sarcoma, melanoma and colorectal tumours.
Infection: bacterial septic emboli, TB, aspergillosis and
other fungal organisms.
Collagen vascular disease: Wegeners granulomatosis,
rheumatoid nodules.
Granulomatous disease: histiocytosis X and
sarcoidosis.
Vascular: pulmonary emboli with infarction.

Discussion
This is a systemic condition characterized by necrotizing
granulomata and a necrotizing vasculitis affecting medium
to small vessels. Pulmonary Wegeners granulomatosis has
a variety of presentations, which include:
Widespread nodules that typically cavitate, have
varying sizes (up to several centimetres) and show no
zonal predilection. Figure 5b is a single axial CT
image showing left lower lobe pulmonary nodules that
are relatively thick walled and some of which are
demonstrating cavitation. A coronal reformat of the
same patient is also shown (5c).

5b

5b Axial CT image showing two thick walled nodules


in the left lower lobe consistent with but not specific
for Wegeners granulomatosis.

20

Patchy alveolar infiltrates/consolidation/ground


glass opacity.
Lymphadenopathy is a rare feature.
Pleural effusions can be seen.
Upper respiratory tract involvement is always seen in
Wegeners and features include destruction of nasal
cartilage and bone, nasal mucosal ulceration, paranasal
sinus mucous membrane thickening, tracheal inflammation
and sclerosis resulting in stridor.
Other organ features include: glomerulonephritis,
migratory polyarthropathy and skin nodules.

Practical tips
There is a long differential for cavitating lung nodules
so look for clues on the film to point towards an
underlying diagnosis and also be guided by the
history. Check the bones for evidence of rheumatoid
arthritis and primary/secondary bone malignant
lesions.
A history of haemoptysis and renal impairment with
pulmonary abnormalities on the chest radiograph
brings to mind the diagnoses of Wegeners
granulomatosis and Goodpastures syndrome. Renal
failure causing fluid overload may also result in
pulmonary oedema that may be blood tinged.

Further management
Initial management should be referral to a respiratory
physician to exclude malignant/infective causes for
cavitating nodules.

5c

5c Coronal reformatted CT image showing bilateral


cavitating nodules.

Chest Imaging

CASE 6

Cases 6, 7

6a

History
A 45-year-old female presented with
progressive
dyspnoea
and
an
inflammatory arthropathy.

CASE 7

History
A 70-year-old factory worker presented
with progressive dyspnoea.

21

Answer 6

Chest Imaging

ANSWER 6
Observations (6a)
This frontal chest radiograph demonstrates a well defined
pulmonary nodule in the right lower zone. This appears to
be a solitary lesion. Arthropathy is noted at the left shoulder
joint with associated erosion of the lateral end of the clavicle
this suggests background rheumatoid arthritis. The
nodule is therefore likely to be a rheumatoid pulmonary
nodule, though other pathology such as a malignant
nodule cannot be excluded and follow-up is therefore
required. There is no evidence of pulmonary fibrosis.

Look for effects of drugs used to treat rheumatoid,


e.g. steroid use resulting in avascular necrosis (AVN)
of the humeral head or vertebral collapse.

Further management
Follow-up of single pulmonary nodules is advised to ensure
that the nodule is not a developing primary bronchogenic
carcinoma. Fleischner Society guidelines (Table 1) suggest
dividing patients into high-risk (smokers and other risk
factors) and low-risk categories and then organizing
subsequent follow-up depending on the size of the nodule.

Diagnosis
Rheumatoid lung.

Further reading
Differential diagnosis
For lateral end of clavicle erosion:
Arthropathies rheumatoid arthritis, gout,
scleroderma.
Hyperparathyroidism.
Post-traumatic osteolysis (6b).
Infection osteomyelitis.
Neoplastic conditions myeloma, metastases.
Hereditary disorders cleidocranial dysostosis,
pyknodysostosis, HoltOram syndrome.

MacMahon H, Austin J, Gamsu G (2005). Guidelines


for management of small pulmonary nodules detected
on CT scans: a statement from the Fleischner Society.
Radio lo gy 237: 395400.

6b

Discussion
Rheumatoid lung manifestations can be seen in up to 50%
of RA patients. The lung changes are more commonly
found in men. CXR features of RA include:
Interstitial fibrosis that predominantly affects the
lower zones.
Rheumatoid lung nodules these are usually multiple,
and commonly located in the lung periphery or
pleurally based. They often cavitate but dont calcify.
Pleural effusion usually this is a unilateral exudate.
More common in men.
Caplans syndrome is a condition characterized by
pneumoconiosis and RA in coal workers.

Practical tips
Look for associated bone abnormalities if rheumatoid
is suspected, e.g. resorption of the lateral ends of
clavicles, erosion of the acromioclavicular,
sternoclavicular and shoulder joints.

6b Radiograph demonstrates erosion of the lateral


edge of the right clavicle following an injury 2 years
previously.

Table 1 Fo llo w-up guidelines fo r pulmo nary no dules


Nodule size (mm)
<4

Low risk
No follow-up

46

CT at 12 months, nil further if


unchanged
CT at 612 months, then 1824
months if unchanged
CT at 3, 9 and 24 months +/ PET CT
and biopsy

68
>8

22

High risk
CT at 12 months, nil further if
unchanged
CT at 612 months, then 1824
months if unchanged
CT at 6 months, then 912 months
and then 24 months if unchanged
CT at 3, 9 and 24 months +/ PET
CT and biopsy

Answer 7

Chest Imaging

Case 8

ANSWER 7
Observations (7)
There is widespread abnormal interstitial opacity
throughout both lungs with no zonal predominance. This
is predominantly of tiny nodules with some reticulation.
Poorly defined heart border and hemidiaphragms indicate
subpleural involvement. There are large, poorly defined
conglomerate masses in both upper zones with surrounding fibrotic changes. These findings are in keeping with
progressive massive fibrosis in a patient with underlying
pneumoconiosis.

Diagnosis
Progressive massive fibrosis (PMF).

Differential diagnosis
For mass lesion with background pneumoconiosis:
PMF.
Bronchogenic carcinoma (increased incidence of
adenocarcinoma in scarred lung).
Granuloma (TB, histoplasmosis).
Caplans syndrome (rheumatoid nodules in those
with coexisting rheumatoid).
Any other parenchymal lesion can incidentally cooccur.

Small nodules silicosis, siderosis, coal workers


pneumoconiosis (CWP).
Reticulations asbestosis.
Reticulonodular opacities carbon/petroleum
products.
Interstitial pneumonia cobalt, titanium, nickel,
chromium exposure.
Progressive massive fibrosis arises as a consequence of
pneumoconiosis and can develop and progress even after
dust exposure has ceased. Radiologically, presentation is
with large opacities in the mid/upper zones since they
usually involve the posterior segment of the upper lobe and
superior segment of the lower lobes. Lesions are initially
seen in the lung peripheries and extend towards the hila
over time. Cavitation and calcification can occur.
Cavitation can then lead to secondary infection with
Aspergillus.

Practical tips
When mass lesions appear in patients with background
pneumoconiosis, remember that this may be PMF but that
they are also at increased risk of adenocarcinoma.

Further management
Discussion
Pneumoconiosis is a parenchymal lung reaction to chronic
inorganic dust exposure. Some typical appearances on chest
radiography are:

CASE 8

CT chest is usually required for further characterization


and it is often difficult to definitively differentiate this from
bronchogenic carcinoma. In these cases, review within a
lung cancer multidisciplinary team setting is required with
a view to percutaneous lung biopsy.

8a

History
A 30-year-old male presented with
haemoptysis and shoulder pain.

23

Answer 8

Chest Imaging

ANSWER 8
Observations (8a)
There are several well defined soft tissue density nodules
throughout both lungs with no zonal predominance. The
right humerus is seen on the edge of the film and is
abnormal in appearance with mottled sclerotic density and
apparent periosteal reaction. Further imaging of it is
required but the suspicion from this film must be of a
sarcoma of the right humerus with associated pulmonary
metastases.

8b

Diagnosis
Lung metastases with underlying osteosarcoma.

Differential diagnosis
For multiple lung nodules:
Neoplastic:
Malignant metastases.
Benign arteriovenous malformation (AVM).
Infectious:
Granulomas TB, histoplasmosis,
coccidioidomycosis, cryptococcus.
Abscesses.
Septic emboli.
Noninfectious:
Wegeners granulomatosis.
Rheumatoid arthritis (RA).
Infarcts.
Sarcoid.
Amyloid.

Discussion
An AP view of the proximal right humerus from this
patient is shown (8b). It demonstrates a pathological
fracture of the right upper diaphysis with a poorly defined
underlying lesion involving the metadiaphysis. This has a
wide zone of transition with lytic mottled areas and
sclerosis. Periosteal reaction is evident and appearances
most likely indicate an osteosarcoma.
Metastases to the lung are common and are seen in up
to 30% of all patients with malignancy. Common primary
tumours are breast, prostate, colon, renal cell carcinoma,
melanoma and osteogenic sarcoma. Some clue to the
underlying primary can be suspected from the appearances
of the metastatic lesion. Squamous cell carcinomas, colon,
melanoma, osteosarcoma and cervix metastases are more
likely to cavitate. Breast, thyroid, osteosarcoma, testes and
ovarian metastases are more likely to calcify. Metastases are
commonly multiple and are found in a subpleural location.

8b AP view of the proximal right humerus from


this patient showing a poorly defined lesion in
the metadiaphysis of the right humerus with a
wide zone of transition, lytic and sclerotic
areas. Periosteal reaction is also seen. This is
likely to be an osteosarcoma.

8c

Practical tips
Carefully check that the nodules are truly pulmonary.
If no nodules project outside a region of lung covered
by ribs, could they be bony? Do nodules extend into
the subcutaneous tissues? Figure 8c shows multiple
skin nodules in neurofibromatosis masquerading as
pulmonary nodules!
8c Chest radiograph of a patient with
neurofibromatosis type 1 with multiple skin nodules.

24

Answer 8

Chest Imaging

Look carefully at the film for a primary malignancy (as


in this case) and also metastases elsewhere, such as
bone deposits or calcified liver deposits under the
right hemidiaphragm.
Look for cavitation and/or calcification in the
nodules to point towards underlying
primary/diagnosis.

Case 9

Where a primary malignancy cannot be identified


then percutaneous biopsy or a surgical video-assisted
thorascopic surgery (VATS) procedure can obtain
tissue from a nodule for histological characterization.

Further reading
Winer-Muram HT (2006). The solitary pulmonary
nodule. Radio lo gy 239: 3449.

Further management
CT chest can identify further nodules and can help to
characterize them, i.e. looking for central necrosis, fat,
calcification, cavitation.

CASE 9

9a

History
A 45-year-old male smoker presented
with a cough.

9b

25

Answer 9

Chest Imaging

ANSWER 9
Observations (9a, 9b)

Discussion

PA chest radiograph (9a) shows a well defined lesion in the


periphery of the right lung forming an obtuse angle with
the chest wall. This is consistent with a pleurally based
lesion. The single axial image from a CT thorax examination (9b) shows a lucent lesion of fat density that extends
through the chest wall to a subpleural location.

Imaging features on a plain radiograph can usually be used


with a good degree of confidence to differentiate pleural
lesions from a lung parenchymal lesion.
Important differentiating features that point to a pleural
location include:
Obtuse angle with the chest wall.
Smooth tapering edges.
Peripheral location.

Diagnosis
Chest wall lipoma.

Practical tips
Differential diagnosis
For pleural lesions:
Metastasis lung and breast are the most common
primaries, with other adenocarcinomas such as ovary,
uterus, pancreas.
Pleural plaques.
Loculated pleural effusion.
Haematoma.
Lipoma.
Neurofibroma/schwannoma.
Rib lesions such as tumour or even healing fracture
can be hard to differentiate from pleural lesions.
(Figure 9c is an axial CT image showing a rib-based
lesion causing complete destruction of the posterior
right 8th rib. The patient had myeloma and
appearances are those of a plasmacytoma.)

When a pleural soft tissue mass is identified, carefully


check the film for supporting evidence of background
malignancy.
An underlying rib fracture points to a subpleural
haematoma.
US can be helpful with the differentiation by
determining whether the pleural lesion is fluid or
solid.

Further management
No further treatment required for this benign lesion.

9c

9c Axial CT image shows a rib-based lesion causing


complete destruction of the posterior right 8th rib.

26

Chest Imaging

CASE 10

Cases 10, 11

10a

History
A 76-year-old male presented with
chest pain.

CASE 11

11a

History
A 67-year-old male presented with
shortness of breath.

27

Answer 10

Chest Imaging

ANSWER 10
Observations (10a)
This AP chest radiograph demonstrates a large mediastinal
mass on the left. The arch of the aorta and proximal
descending aorta are not seen separate from this mass,
which indicates that it arises from the posterior mediastinum. There is curvilinear calcification in the lateral aspect
of this mass, which would suggest that the lesion is in fact
a dilated aneurysmal thoracic aorta. Comparison with old
films would be useful to assess any change.

Diagnosis
Thoracic aortic aneurysm.

Differential diagnosis
Of posterior mediastinal mass:
Thoracic aortic aneurysm.
Dilated oesophagus.
Hernia hiatus, Bochdaleks.
Neurogenic tumour neurofibroma, schwannoma,
ganglioneuroma.
Spinal abscess.
Extramedullary haematopoiesis.
Of anterior mediastinal mass:
Thymoma.
Teratoma.
Thyroid enlargement with retrosternal extension.
Lymphoma.

Anterior mediastinal space is from the anterior aspect


of the heart to the sternum, and contains thymus and
lymph nodes.
Knowing the contents of each space and applying the
silhouette sign allows for better identification of what a
mediastinal mass may be. For example, in Figure 10c there
is a large, well defined, round mediastinal mass related to
the left heart border. The left heart border can be clearly
seen, therefore the mass cannot be within the middle
mediastinum. Although slightly difficult to assess the
region behind the heart, no clear separation can be made
between the rounded lesion and the descending aorta,
suggesting that the lesion is a posterior mediastinal mass.
This is confirmed by the lateral film (10d) in the same
patient and the lesion was proved to be a neurofibroma.

Practical tips
Lateral films can be very useful in better localizing
mediastinal masses.

Further management
If there is clinical concern then CT is usually the next
investigation of choice.

Of middle mediastinal mass:


Lymph nodes.
Bronchogenic/duplication cyst.
Ascending aortic aneurysm.
Carcinoma of bronchus/trachea.

Discussion
An axial CT image (10b) of the chest with IV contrast
shows the descending aortic aneurysm. Typical radiological
appearances are of a mediastinal mass the wide tortuous
aorta with peripheral curvilinear calcification. Normal CT
measurements for the thoracic aorta are <3.5 cm diameter
for the ascending aorta and <2.5 cm for the thoracic
descending aorta. Surgical repair is considered over 6 cm.
In the acute situation, where no old films are available,
differentiation from an acute aortic dissection may be
necessary. In acute aortic dissection there can be widening
of the mediastinum, an irregular fuzzy appearance to the
aortic outline and displacement of calcification with
additional soft tissue identified lateral to calcification (due
to false channel formation).
Characterizing the position of mediastinal masses is
useful on plain radiography in order to narrow the
differential diagnosis:
Posterior mediastinum is the space between the
posterior aspect of the heart and the thoracic spine,
and contains descending aorta, oesophagus, azygous
veins and thoracic duct.
Middle mediastinal space contains heart and great
vessels.

28

10b

10b Axial CT image of a large descending thoracic


aortic aneurysm with marked thrombus.

Answers 10, 11

Chest Imaging

10c

10d

10c, 10d AP and lateral chest radiographs show a


posterior mediastinal mass.

ANSWER 11
Observations (11a)

Pleural pseudotumour.

Non-infectious:
Wegeners granulomatosis.
Rheumatoid arthritis (RA).
Infarct.
Sarcoid.
Amyloid.
Congenital:
Bronchogenic cyst.
Sequestration.
Extrapulmonary:
Pleural mass/fluid.
Rib fracture/lesion.
Subcutaneous lesion.
External artefact.

Differential diagnosis

Discussion

For solitary lung nodule/mass:


Neoplastic:
Malignant primary bronchogenic carcinoma,
solitary metastasis.
Benign hamartoma, adenoma, arteriovenous
malformation (AVM).
Infectious:
Granuloma TB, histoplasmosis,
coccidioidomycosis, cryptococcus.
Abscess.
Septic embolus.

This is focal accumulation of fluid in the interlobar fissure


and is identified on the chest radiograph as having very well
defined inferior borders. There are often other features of
pleural fluid elsewhere, or features to indicate background
pathology such as heart failure that has led to pleural fluid
accumulation. The main importance of such a condition is
that there is potential to misinterpret it as a focal
pulmonary nodule/mass.

This PA chest radiograph shows a very well defined 5 cm


soft tissue density lesion in the right mid zone, in the
region of the horizontal fissure. The heart is enlarged with
cardiothoracic ratio of 16:26. There is upper zone venous
congestion and a slight increase in interstitial opacity.
These appearances are in keeping with left heart failure and
the right sided opacity could therefore indicate localized
pleural fluid in the oblique fissure. Comparison with any
recent films or, alternatively, a follow-up film after
treatment should confirm resolution of the opacity.

Diagnosis

(co nt.)

29

Answer 11

Chest Imaging

Practical tips
Important features in differentiating solitary lesions
include:
Growth rate lesions that dont change in size
over a period of 2 years can be considered benign.
Tumour doubling times (time taken for volume of
nodule to increase twofold) have been reported
between 1 and 18 months.
Margins irregularity, lobulation and spiculation
of the lesions edge are a good indicator of
malignancy. However, 30% of lesions with smooth
margins are not benign usually representing
metastases.
Presence of fat this is usually a sure sign of
benignity and suggests the diagnosis of a
hamartoma. Very occasionally renal cell carcinoma
and liposarcoma metastases can also contain fat.
Calcification this is seen in both benign and
malignant conditions. Benign lesions usually have
calcification with a central nidus, laminated or
popcorn appearance. Calcification in malignant
lesions usually appears stippled, amorphous or
diffuse. Stippled calcification is seen in mucin
secreting tumour metastases, e.g. colon, ovary.

Cavitation again this is seen in both benign and


malignant lesions. Irregular thick walled cavities
are more likely to be malignant than thin, smooth
walled ones.
When an apparent pulmonary opacity has an unusual
shape or density, always stop and ask could it be in
the pleura, chest wall, soft tissue or even external to
the patient? Figure 11b, for example, shows a very
sharply defined calcific density opacity in the right
lower zone. It is difficult to imagine what pulmonary
pathology this might represent and it is actually a
calcified breast fibroadenoma.

Further management
Lateral CXR can be helpful in these cases and if there is
still concern then CT chest will usually resolve the issue.
(See Case 6, Tab le 1 for further management of single
pulmonary nodule.)

11b

11b Chest radiograph shows a very well defined


calcific density in the right lower zone, which
actually represents a calcified breast fibroadenoma.

30

Chest Imaging

CASE 12

Case 12

12a

History
A 69-year-old male smoker with
ischaemic heart disease, presented with
increasing dyspnoea over the last 2
months.

12b

31

Answer 12

Chest Imaging

ANSWER 12
Observations (12a, 12b)
The heart looks enlarged but as this is an AP radiograph, it
is not possible to be sure. There is a diffuse, bilateral
increase in reticular interstitial opacity with Kerley B lines
in the right lower zone best seen on the close-up (12b).
The left hilum is enlarged, and there is a pulmonary nodule
in the left upper zone measuring approximately 1.5 cm. A
tiny left pleural reaction is also noted.
The most likely explanation is that the patient has a left
upper lobe tumour with left hilar adenopathy and
lymphangitis carcinomatosa. However, given the history
of heart disease and the rather subjective cardiomegaly, it
is possible that he could have lung cancer and coexisting
left heart failure. A repeat film after treatment for heart
failure might help clarify, but he is likely to need staging
of the suspected tumour with CT anyway, and this may
answer the question.

Diagnosis

thickening. In addition, there are small peripheral subpleural nodules. Heart failure produces similar appearances
on plain film and CT, though the thickening is more likely
to be smooth and there will not be the associated lymphadenopathy that is often present in lymphangitis. Like heart
failure, lymphangitis usually produces bilateral changes.

Practical tips
When lymphangitis is diagnosed, look for other signs
of malignant spread on available images, i.e. lung
metastases, bone deposits.
CT imaging findings of subpleural nodules and
reticulation with interlobular septal thickening are the
best diagnostic features. However, if it remains
impossible to differentiate interlobular septal
thickening of heart failure from lymphangitis on CT
scanning, repeat imaging (chest radiograph or perhaps
follow-up CT) after treatment for heart failure may
resolve the issue.

Left upper lobe tumour with lymphangitis carcinomatosa.

Further management
Differential diagnosis
For tumours causing lymphangitis (anatomically from top
down):
Larynx.
Thyroid.
Breast.
Stomach.
Pancreas.
Colon.
Cervix.

Oncological assessment is necessary. If deemed suitable for


chemotherapy, a search for the primary tumour is
appropriate if not already apparent as in this case. CT
scanning of the thorax, abdomen and pelvis is most
commonly undertaken.

Further reading
Connolly JE, Erasmus JJ, Patz EF (1999). Thoracic
manifestations of breast carcinoma: metastatic disease
and complications of treatment. Clinical Radio lo gy
54(8): 487494.

For septal (Kerley B) lines:


Pulmonary venous hypertension.
Lymphangitis.
Many other conditions can show septal lines, though the
two conditions above rank far above these. Examples
include: sarcoid, any chronic fibrosing lung condition,
lymphangiectasia, lymphangiomyomatosis, lymphoma and
viral pneumonia.
For subpleural nodules:
Sarcoid.
Lymphangitis carcinomatosa.
Silicosis.
Lymphoma.

Discussion
Certain tumours show a propensity to invade the lung
interstitium, both the connective tissue and the lymphatics.
Lymphatics become distended by the tumour itself, and
also because of congestion resulting from tumour
obstruction. Symptoms include dyspnoea and cough.
Plain film signs are of increased reticular/reticulonodular interstitial markings, Kerley A and Kerley B lines.
CT shows irregular thickening of the interlobular septae,
along the central bronchovascular bundles and subpleural

32

12b

12b Kerley B
septal line.

Chest Imaging

CASE 13

Cases 13, 14

13

History
A 36-year-old female, otherwise fit and
well, presented with acute dyspnoea and
left sided chest pain.

CASE 14

14a

History
A 55-year-old man presented with
dysphagia.

33

Answers 13, 14

Chest Imaging

ANSWER 13
Observations (13)
There is a large, well defined, solitary, unilocular thin
walled cyst in the medial aspect of the left middle zone.
This contains an air-fluid level. A moderate sized left sided
pneumothorax is evident with signs of mediastinal and
tracheal shift to the right suggesting that this is a tension
pneumothorax. Immediate treatment of this tension
pneumothorax is required.
The air-containing mass has presumably ruptured and
led to a spontaneous pneumothorax. The fact that this
patient is young and otherwise in good health makes most
of the causes of large cavitating/cystic masses unlikely.
After initial treatment, a search for previous films should
be made to see if this is a longstanding benign lesion.
Otherwise, further investigation, perhaps with CT, will be
required.

usually in the medial aspect. Typical appearances are of a


solitary, unilocular thin walled cyst of uniform density due
to thick mucoid fluid content. They can also contain airfluid levels. Calcification of the wall is rare. They are usually
asymptomatic but can be complicated by infection and
haemorrhage or can cause compression to adjacent
structures, i.e. trachea/airways/oesophagus.

Practical tips

Intrapulmonary bronchogenic cyst.

As always, clinical history is vital in producing a


sensible differential diagnosis.
Tension pneumothorax is an emergency and should
be immediately treated or the appropriate clinician
should be informed. It arises when air is able to enter
the pleural space on inspiration but not escape on
expiration. The accumulating air produces increasing
mass effect (mediastinum displaced and diaphragm
depressed) that compromises ventilation of the other
lung and also cardiovascular function.

Differential diagnosis

Further management

For solitary cavitating lesion:


Cavitating neoplasm squamous cell carcinoma of the
lung is the most likely lung tumour to cavitate.
Lung abscess.
Cavitating pneumonia.
Infarct/haematoma.

Tension pneumothorax is a medical emergency requiring


immediate treatment. The increasing pressure must be
relieved either with an intercostal chest drain or perhaps
even insertion of a cannula into the pleural space in the
acute situation.

Diagnosis

Further reading
Discussion
Bronchogenic cysts are usually located in the mediastinum
(85%) but can also be intrapulmonary in location (15%).
They are more commonly found in the lower lobes and

Matzinger M, Matzinger F, Sachs H (1992).


Intrapulmonary bronchogenic cyst: spontaneous
pneumothorax as the presenting symptom. American
Jo urnal o f Radio lo gy 158: 987988.

ANSWER 14

34

Observations (14a)

Diagnosis

This frontal chest radiograph shows a triangular opacity


behind the left side of the heart. It obscures the silhouette
of the left hemidiaphragm where they meet, and there is
depression of the left hilum indicating volume loss. This is
the sail sign of left lower lobe collapse.
At least two cavitating pulmonary nodules are present
in the right upper lobe and close inspection of the
mediastinum reveals residual barium in the upper thoracic
oesophagus, terminating abruptly in the mid mediastinum.
The findings suggest that the patient has recently
undergone a barium swallow, which has demonstrated a
mid-oesophageal tumour. The pulmonary lesions on the
right are likely to represent cavitating metastases and the
left lower lobe collapse is presumably due to tumour
obstruction of the left lower lobe bronchus.

Oesophageal tumour with left lower lobe collapse and


cavitating pulmonary metastases.

Differential diagnosis
Of a veil-like opacity over a hemithorax:
Left upper lobe collapse.
Pleural effusion in the supine position.
Rotated patient position.
Overlying chest wall abnormality, e.g. gynaecomastia.
Unilateral airspace opacity.
Normal, i.e. it is actually the other side that is
hypodense!
Of a completely opaque hemithorax:
Total lung collapse.
Pneumonectomy looks like total collapse but there
will be evidence of thoracotomy.

Answer 14

Chest Imaging

Huge pleural effusion volume expansion rather than


volume loss, i.e. the mediastinum will be displaced
away from the side of opacity.

14c

Discussion
The various lobar collapses have their own characteristic
appearances but evidence of volume loss is fundamental.
Signs to look for include:
Elevation or tenting of the hemidiaphragm.
Elevation or depression of the hilum note that the
left is normally higher than right.
Mediastinal shift in the direction of the collapse.
Elevation or depression of the horizontal fissure.
Increased lucency or splayed vessels in the remaining
hyperexpanded lobe.
In older adults, an obstructing central tumour must always
be excluded. When causing upper lobe collapse, this can
sometimes produce the characteristic S sign of Golden
(14b). The right upper lobe collapses into a triangular
upper zone opacity, limited inferiorly by the horizontal
fissure. However, the central mass produces an overall S
configuration with the fissure. Figures 14c and 14d are axial
CT images in a patient with subtotal collapse of the right
upper lobe with medial collapse of the lobe and significant
volume loss as demonstrated by anterior movement of the
fissures. A coronal CT reformatted image (14e) again
shows subtotal collapse of the right upper lobe with volume
loss.

14d

(co nt.)

14b

14c, 14d Axial CT images of the upper chest


showing collapse of the right upper lobe with
anterior and medial collapse. Anterior movement of
horizontal and oblique fissures is demonstrated.

14e

14b Right upper lobe collapse secondary to a central


bronchogenic tumour produces an S sign of
Golden.

14e Coronal CT reformatted image of the chest


showing the medial displacement of the collapsed
upper lobe.

35

Answer 14

Chest Imaging

The left upper lobe has a characteristic appearance when


it collapses. Unlike the right upper lobe, there is no
horizontal fissure and the whole lobe frequently collapses
anteriorly so that no discrete edge is seen. Instead, there
is a veil of opacity over the left lung (14f). Figure 14g
is a left lateral film that shows how the lobe collapses
anteriorly.
The right lower lobe collapses in a similar fashion to the
left, but is easier to appreciate because it is not hidden by
the heart. Right middle lobe collapse will produce increased opacity in the lower zone too, but unlike lower
lobe collapse that obscures the diaphragm silhouette, this
obscures the right heart border.

14f

Total lung collapse is illustrated in Figure 14h (left


sided). This causes total opacification of the hemithorax
with prominent volume loss on the side of the abnormality
note the grossly displaced mediastinum to the left (this
illustration also shows right upper lobe collapse).

Practical tips
Differentiating collapse and consolidation can
sometimes be difficult since they often coexist. Look
for signs of volume loss as described above.
Lateral radiographs are useful in suspected collapse.
Look for other evidence on the film of a primary
bronchogenic tumour, e.g. metastases to lung and
bone, previous thoracotomy and radiotherapy change.
Check the position of endotracheal tube if present it
may have passed too far, occluding a bronchus and
causing lung/lobar collapse. (Figure 14h shows left
total lung collapse and right upper lobe collapse due
to passage of the endotracheal tube into the bronchus
intermedius; the SengstakenBlakemore tube in situ is
unrelated.)

Further management
In the adult patient with no obvious underlying cause, a
central or endobronchial tumour needs to be excluded and
respiratory referral with a view to bronchoscopy should be
made.

14g

14f, 14g AP and lateral chest radiographs


demonstrate left upper lobe collapse, which
gives a veiling opacity over the left lung. The
left upper lobe collapses anteriorly as shown
on the lateral film.

36

14h

14h Complete collapse of the left lung with right


upper lobe collapse.

Chest Imaging

CASE 15

Cases 15, 16

15a

History
A 68-year-old male presented with
slowly progressive dyspnoea.

CASE 16

16

History
A 40-year-old male presented with a
long progressive history of dyspnoea
and more recent onset cyanosis.

37

Answer 15

Chest Imaging

ANSWER 15
Observations (15a)
This chest radiograph demonstrates bilateral lower zone
interstitial reticular opacity with evidence of basal volume
loss demonstrated by descent of both hila. The interstitial
opacity gives the heart an irregular shaggy border. These
appearances are in keeping with basal fibrosis. There is a
calcified pleural plaque related to the right hemidiaphragm.
The combination of basal fibrosis with pleural disease
would suggest asbestos exposure with pulmonary asbestosis
and pleural plaques.

Diagnosis
Pulmonary asbestosis.

Discussion
Pulmonary asbestosis is a chronic progressive fibrotic
condition secondary to chronic asbestos exposure.
Crocidolite (blue) asbestos fibres are most commonly
associated with malignant disease and pleural disease.
Radiological features are of a fibrosing alveolitis that
predominantly affects the bases and is indistinguishable
from other causes. Fibrosis can progress to result in
progressive massive fibrosis, but this again predominates
at the lung bases. Pulmonary asbestosis has a latency period
of ~40 years and therefore pleural changes are usually seen
prior to lung parenchymal changes.
Other features of asbestos exposure include:
Pleural effusion this is the earliest pleural
abnormality, with a latency of ~10 years.
Focal pleural plaques have a latency of 2040 years.
Diffuse pleural thickening.
Pleural calcification.
Rounded atelectasis this is also known as folded
lung and arises due to infolding of thickened pleura
with associated subsegmental atelectasis. Most
commonly seen in the lower lobes, it has the
appearance of a rounded subpleural mass abutting

15b

15b CT image of the chest demonstrating a large


intraparenchymal lung lesion that is abutting
thickened pleura. Vessels appear to be radiating
towards the lesion as though pulled towards it.

38

thickened pleura, with linear bands extending from


the mass into the lung (crows feet) (15b).
Malignant mesothelioma ~90% are related to
previous asbestos exposure.
Lung carcinoma there is a latency of ~30 years and
occurrence is related to the dose of asbestos exposure
and to cigarette smoking which can increase risk by
100-fold.

Practical tips
Multiple pleural plaques are characteristic for previous
asbestos exposure (15c).
Look for signs of malignancy in patients with asbestos
exposure remember the increased risk of pleural and
pulmonary malignancy. Pulmonary masses should be
investigated with CT characteristic findings may
permit a confident diagnosis of folded lung in some
cases.
Asbestosis is the odd one out among the inorganic
dusts causing pulmonary fibrosis. The other
fibrogenic dusts cause upper zo ne fibrosis.

Further management
Systemic symptoms, e.g. weight loss, should be
carefully investigated to exclude mesothelioma or
bronchogenic carcinoma, for which these patients are
at increased risk.
In cases where there is still clinical concern about an
area of possible folded lung despite imaging,
percutaneous biopsy may be required to exclude a
malignancy.

Further reading
Akira M, Yamamoto S, Yokoyama K, et al. (1990).
Asbestosis: high-resolution CT-pathologic
correlation. Radio lo gy 176: 389394.

15c

15c Axial CT image shows a right anterior calcified


pleural plaque consistent with previous asbestos
exposure. There is also subpleural reticulation
representing fibrosis and appearances would be of
asbestosis.

Answer 16

Chest Imaging

Case 17

ANSWER 16
Observations (16)
This frontal chest radiograph shows extreme cardiomegaly.
There is marked dilatation of the central and main pulmonary arteries with pruning of peripheral pulmonary
arteries. No diffuse lung abnormality is seen.
The findings are indicative of pulmonary hypertension.
Given the gross cardiomegaly, a left to right shunt is the
most likely cause. However, cyanosis should not occur and
its presence suggests the shunt has reversed, that is, the
patient has developed Eisenmengers syndrome.

Diagnosis
Pulmonary arterial hypertension from an undiagnosed
ventricular septal defect (VSD) progressing to
Eisenmengers syndrome.

cardiovascular disease, either from an increase in overall


pulmonary arterial resistance or from an increase in the
overall circulatory volume going through the pulmonary
circulation.
Increased resistance pulmonary veno-occlusive
disease, chronic pulmonary thromboembolism, any
chronic ventilatory disorder leading to chronic
hypoxia and resulting vasoconstriction in the
pulmonary arterial bed.
Increased flow left to right shunts, i.e. ASD (atrial
septal defect), VSD (ventricular septal defect), PDA
(patent ductus arteriosus).
In Eisenmengers syndrome the pulmonary arterial
pressure climbs until it eventually exceeds the pressure in
the left heart and the shunt reverses. It is seen in those with
pulmonary hypertension from a left to right shunt.

Discussion
Pulmonary arterial hypertension is diagnosed by a sustained
mean pressure >20 mmHg (systolic >30 mmHg, diastolic
>15 mmHg). Radiological features on a plain chest
radiograph that suggest the diagnosis are:
Increase in size of the main pulmonary artery.
Reduction in size of peripheral pulmonary arteries
known as peripheral pruning.
Right heart enlargement.
Calcification of the central pulmonary arteries a late
but characteristic sign.
Parenchymal mosaic attenuation pattern seen on
HRCT.
Primary pulmonary hypertension is idiopathic. The
condition can also arise secondary to pulmonary disease or

CASE 17

Practical tips
On a plain chest radiograph, hilar lymphadenopathy
can mimic pulmonary arterial hypertension. Clinical
history here is vital and CT should be subsequently
undertaken in the right clinical setting.
The diameter of the main pulmonary artery should be
less than that of the ascending thoracic aorta. Reversal
of this ratio is a sign of pulmonary hypertension.

Further management
Primary pulmonary hypertension has no cure and a dismal
prognosis. It is a diagnosis of exclusion so all underlying
causes of secondary pulmonary hypertension must be
investigated. Cardiology referral with a view to echocardiography would be required initially.

17a

History
A 50-year-old female presented
with progressive dyspnoea and
intermittent cyanosis of the
fingers.

39

Answer 17

Chest Imaging

ANSWER 17
Observations (17a)
This frontal chest film shows abnormal reticular interstitial
opacity at both lung bases, though there are no features to
indicate significant volume loss at the present time. A large
area of calcinosis is noted in the soft tissues around the
upper right humerus. The combination of findings and
clinical history suggests a diagnosis of systemic sclerosis
with lower zone pulmonary fibrosis and Raynauds
phenomenon.

Diagnosis
Lower zone pulmonary fibrosis due to systemic sclerosis.

Differential diagnosis
For lower zone pulmonary fibrosis:
Idiopathic pulmonary fibrosis (IPF) (cryptogenic
fibrosing alveolitis CFA).
Connective tissue disorders systemic sclerosis,
rheumatoid.
Asbestosis.
Drugs especially certain cytotoxics, e.g.
cyclophosphamide, bleomycin, busulphan, etc.

Discussion
This autoimmune disease has also been known as
scleroderma, with a subgroup known as CREST syndrome.
Current nomenclature is systemic sclerosis with diffuse or
limited scleroderma, the latter being the equivalent of
CREST syndrome. The condition is three times as
common in females and typically presents in the 4th5th
decades. A variety of autoantibodies may be present
including ANA and rheumatoid factor. Clinical features
are many and varied but include:
Musculoskeletal thickened skin, soft tissue
calcinosis, Raynauds, erosive arthritis (see Case 152).
Lungs lower zone pulmonary fibrosis, aspiration.
Oesophagus hypotonia results in dilatation and
dysphagia. Incompetence of the gastro-oesophageal
sphincter results in reflux and consequent peptic
stricture, aspiration, etc.
Small bowel dilatation and slow transit result in
bacterial overgrowth and malabsorption. Barium
studies show hidebound appearance due to fibrosis
pulling the valvulae closer together.
Pseudosacculations and pneumatosis in small and
large bowel.

Soft tissue calcification systemic sclerosis.


Signs of malignancy including bony sclerosis from
myeloproliferative disorders cytotoxic induced.
Sympathectomy clips systemic sclerosis (17b).
Remember which disorders cause upper and lower
zone fibrosis (refer to the differential diagnosis above,
and also that in Case 18 for upper zone fibrosis):
sarcoid is the classical upper zone disease, much as
IPF is the classical lower zone disease. Thereafter,
remember that the upper zones are better aerated and
the lower zones better perfused. So, diseases caused
by inhaled dust (inorganic or organic, e.g. silicosis
and extrinsic allergic alveolitis [EAA] respectively)
affect the upper zones, while the lower zones will be
affected by blood borne disorders, i.e. drugs and
autoimmune conditions. Unfortunately, asbestos is an
exception and does not obey this logic.
As with many fibrotic lung conditions, there is an
increased incidence of pulmonary malignancy in
systemic sclerosis associated pulmonary fibrosis
check for focal nodules/masses on the chest
radiograph. Alternatively, focal airspace opacities may
represent aspiration.

Further management
HRCT is the imaging choice in diagnosis and follow-up
of interstitial lung disease. CT imaging findings of fibrosis
include lung volume reduction, subpleural reticulation,
interlobular septal thickening and traction bronchiectasis.

17b

The CREST syndrome represents Calcinosis, Raynauds,


oEsophageal dysmotility, Sclerodactyly, Telangiectasia.

Practical tips
Once lower zone pulmonary fibrosis has been noted
on the chest radiograph, examine the film for the
following features that may indicate a specific
diagnosis:
Dilated oesophagus systemic sclerosis (see Case
152).
Erosions of the lateral ends of clavicles
rheumatoid.
Pleural plaques asbestosis.

40

17b Radiograph demonstrates bilateral


sympathectomy clips indicating that the patient has
undergone treatment for Raynauds phenomenon.

Chest Imaging

CASE 18

Cases 18, 19

18a

History
A 33-year-old male presented with back
pain and progressive dyspnoea.

CASE 19
History
A 59-year-old male presented with left arm
pain on activity.

19

41

Answer 18

Chest Imaging

ANSWER 18
Observations (18a)
Plain radiograph of the chest shows changes of upper zone
fibrosis with elevation of both hila and upper zone reticular
opacities. In addition, there is a cavity in the left upper
zone containing a soft tissue density mass with surrounding
air crescent. These appearances would be consistent with
a mycetoma. The patient also has a marked kyphosis with
the head obscuring the lung apices. Moreover, on close
inspection, there is a hint of syndesmophyte formation
along the right lateral aspect of thoracic spine.
The combination of findings is consistent with upper
zone fibrosis associated with ankylosing spondylitis. There
is mycetoma formation in the fibrotic cavity in the left
upper zone.

Apical bullae and cavitation.


Paraseptal emphysema.
Bronchiectasis.
Cardiac features include aortitis involving the ascending
aorta with aortic valve insufficiency.
Plain radiographic features of upper zone fibrosis
include:
Elevation of the hila.
Tenting of the hemidiaphragms.
Elevation of the horizontal fissure on the right (a
good indicator).
Increased lucency of the lower zone due to
hyperexpansion.
Reticular opacities in the upper zones.

Diagnosis
Practical tips

Ankylosing spondylitis.

Differential diagnosis
For upper zone fibrosis (mnemonic STRAD):
Sarcoidosis.
TB.
Radiation.
Ankylosing spondylitis.
Dust inhalation inorganic (e.g. silica) and organic
(i.e. chronic extrinsic allergic alveolitis).

Discussion
Ankylosing spondylitis is an autoimmune disease that most
commonly manifests as a seronegative arthropathy,
predominantly affecting the axial skeleton (initially
sacroiliac joints then thoracic and lumbar spine). It usually
presents in the 2nd4th decade and more frequently affects
men (sex ratio of ~5:1). As well as bone involvement, there
are respiratory and cardiac manifestations. Respiratory
manifestations are seen in ~1% of cases and features
include:
Upper lobe pulmonary fibrosis.
Reticular/reticulonodular opacities in lung apices.

Clues to help limit the differential diagnosis for upper


zone fibrosis include:
Kyphosis and bamboo spine indicate ankylosing
spondylitis.
Egg shell nodal calcification suggests silicosis or
sarcoid.
Associated calcified granulomata suggest TB.
Always look for signs of secondary
infection/mycetoma in fibrotic cavities (18b).
When pulmonary fibrosis due to ankylosing
spondylitis is suspected, look for signs of
complications of drug treatment on the film:
Avascular necrosis of humeral heads secondary to
steroids.
Atypical distribution of fibrosis may be secondary
to drug treatment.

Further management
Multidisciplinary management is required in this
multisystem disease.

18b

18b CT image in the same patient shows a cavitating


lesion in the left upper lobe apical segment
containing an Aspergillus fungus ball.

42

Answer 19

Chest Imaging

Case 20

ANSWER 19
Observations (19)
Two spot images from an angiogram investigation at the
level of the aortic arch are presented. Both images show the
left shoulder joint in an abducted position. The left hand
image shows contrast filling of the aortic arch with filling
of the brachiocephalic trunk and left common carotid
artery. There is filling of the proximal subclavian artery but
then there is a complete occlusion with no filling beyond
it. The right hand image shows a slightly delayed film with
contrast seen in the left vertebral artery (best seen at the
level of the C3/4) providing filling of the distal left
subclavian artery.

Subclavian insufficiency pain, numbness and


weakness in the arm that is brought on by exercising
the limb. Necrosis of the fingertips.
Vertebrobasilar insufficiency syncope can be
precipitated by exercising the arm due to the stealing
of blood. Headaches, ataxia, vertigo, diplopia,
homonymous hemianopia and hemiparesis have all
been reported.

Practical tips
Diagnosis can be made noninvasively by US by identifying
reversal of Doppler flow in the vertebral artery.

Diagnosis

Further management

Subclavian steal syndrome.

CT can be useful to identify/characterize calcified


atherosclerotic plaque in the subclavian artery
(uncontrasted CT) and also the site/degree of
stenosis (arterial phase CT).
Surgical referral is required for treatment with either
balloon angioplasty (+/ stent insertion) or surgical
bypass (common carotid to subclavian artery).

Discussion
This is a condition that is usually acquired and caused by
atherosclerotic disease. Stenosis of the subclavian artery
results in stealing of blood to the arm via retrograde flow
in the ipsilateral vertebral artery. Other acquired causes
include vasculitis (Takayasu), embolism, aortic dissection,
radiation fibrosis and chest trauma. Congenital causes are
uncommon. Clinical features include:
Left arm is more commonly involved than the right.
Reduced BP by up to 40 mmHg in the affected arm.
Delayed/weak pulse in the affected arm.

CASE 20

Further reading
Chung JW, Park JH, Im JG, et al. (1996). Spiral CT
angiography of the thoracic aorta. Radio Graphics 16:
811824.

20a

History
A 44-year-old male having a preemployment chest radiograph.

43

Answer 20

Chest Imaging

ANSWER 20
Observations (20a)
This chest radiograph shows multiple fine, sand-like, tiny
calcified lesions measuring less than 1 mm in diameter,
spread throughout both lungs. Both lungs are of normal
volume. No other abnormality is seen.

question to the clinician such as how unwell is this


patient? can be most helpful.

Further management
No further management is required in this benign
condition.

Diagnosis
Alveolar microlithiasis.

Further reading

Differential diagnosis

Brown K, Mund DF, Aberle DR, Batra P, et al. (1994).


Intrathoracic calcifications: radiographic features and
differential diagnoses. Radio Graphics 14: 12471261.

With pin-point high-density nodules, the possibilities are


fairly limited, as follows:
Inhaled inorganic dusts such as silicosis. Nodules tend
to be a little larger and are predominantly in the
middle and upper zones. Coalescence to form larger
lesions with cavitation and fibrosis occurs. Egg shell
calcification of nodes.
Other inorganic inhaled dusts such as tin oxide,
limestone and marble.
Slightly larger high-density opacities lead to a larger
differential in addition to the above:
Varicella pneumonia previous infection can appear
radiologically with multiple calcified nodules
measuring 12 mm in size. No lymph node
calcification is seen.
Histoplasmosis healed infection can also result in
multiple tiny calcifications throughout the lungs.
Associated with mediastinal lymph node, liver and
spleen calcification.
Metastatic calcinosis focal calcification within the
alveolar septae due to elevated serum calcium and
phosphate levels in conditions such as
hyperparathyroidism, multiple myeloma, sarcoidosis,
milk-alkali syndrome or hypervitaminosis D. There is
upper zone predominance and disease can progress to
form airspace opacities, consolidative appearances and
fibrosis (20b).
Pulmonary haemosiderosis due to mitral valve disease.
Barium aspiration hyperdense opacities in the lower
zones more common on the right (20c).

20b

20b Chest radiograph showing bilateral upper zone


airspace opacities of metastatic calcinosis.

20c

Discussion
This is a rare condition that affects adults in the 4th6th
decades, resulting in calcification within the alveoli. Usually
these patients are asymptomatic, however they can present
with dyspnoea on exertion. Radiological appearances can
be quite striking with diffuse tiny calcified nodules <1 mm
in diameter spread throughout both lungs. The middle and
lower zones are preferentially affected. Serum calcium and
phosphate are normal. Differentiation from the causes
below is usually made by the normal biochemistry,
characteristic radiological appearances and the paucity of
clinical symptoms relative to the marked radiological
changes.

Practical tips
Clinical history is of vital importance when narrowing down
a list of differential diagnoses. Alveolar microlithiasis is a
good example of where marked radiological changes are
associated with a relative lack of symptoms. A simple

44

20c Chest radiograph showing hyperdense airspace


opacities in the lower zones predominating on the
right in a patient who aspirated during a barium
swallow examination.

Chest Imaging

CASE 21

Cases 21, 22

21a

History
A 28-year-old patient presented with
deteriorating chronic dyspnoea.

CASE 22

22

History
A 45-year-old male presented with
cardiac arrhythmia and shortness of
breath.

45

Answer 21

Chest Imaging

ANSWER 21
Observations (21a)

Practical tips

This frontal chest radiograph shows V-shaped soft tissue


opacities around the left hilum that are likely to represent
mucous plugs in dilated central bronchi. No segmental
collapse is evident. This appearance is described as finger
in glove and is a classical appearance of allergic bronchopulmonary aspergillosis.

Some indication to the underlying cause of bronchiectasis


can be made from its distribution and type:
ABPA central, cystic/varicose, predominantly in the
upper lobes.
Cystic fibrosis pan-lobar, affects the upper lobes
more than lower lobes (21d).
Post-pertussis infection cystic, lingula initially
affected.
Hypogammaglobulinaemia cylindrical. Lower lobe
predominance.

Diagnosis
Allergic bronchopulmonary aspergillosis (ABPA).

Differential diagnosis
For a flitting pneumonia:
Eosinophilic pneumonia.
Aspiration pneumonia.
ABPA.
Cryptogenic organizing pneumonia (COP).

Discussion
This is a hypersensitivity condition seen in longstanding
asthmatics with sensitivity towards aspergilli. It occurs in
12% of asthmatics and more commonly (~10%) in
patients with cystic fibrosis. Pulmonary features of the
disease are:
Pulmonary infiltration with eosinophils, which
presents as a migratory/flitting pneumonitis of patchy
alveolar infiltrates more commonly seen in the upper
lobes.
Cystic bronchiectasis with an upper lobe
predominance that is usually central, producing ring
shadows on plain radiographs.
Mucoid impaction in dilated central bronchioles,
producing the finger in glove appearance. This can
result in segmental/lobar collapse.
Lobar consolidation.
Axial CT images of a young female patient with ABPA are
shown (21b, 21c). Figure 21b shows dilated mucousfilled/plugged upper lobe bronchi, which can be best
appreciated on coronal reformats. Figure 21c demonstrates
the typical cystic dilated bronchi with further evidence of
mucous-plugging.

21b

21b Axial CT image showing dilated


mucous-filled bronchi in the upper
lobes.

46

Further management
Referral to chest physician is appropriate with further
imaging follow-up with HRCT.

Further reading
Williams SM, Jones ET (1997). General case of the day.
Allergic (or hypersensitivity) bronchopulmonary
aspergillosis (ABPA). Radio Graphics 17: 15971600.

21c

21c Coronal reformat showing the


finger in glove mucous-plugging
appearance of the bronchi.

21d

21d Axial CT image showing cystic


bronchiectasis in a patient with cystic
fibrosis.

Answer 22

Chest Imaging

Case 23

ANSWER 22
Observations (22)
This is a single long axis view of the left ventricle at delayed
gadolinium enhancement phase. The image shows delayed
enhancement of the septal, apical and lateral myocardium
with sparing of the endocardium. This pattern of delayed
intramyocardial enhancement with a normal endocardium
suggests an infiltrative/inflammatory process.

Diagnosis

Imaging findings of cardiac sarcoid include:


Cardiac granulomas involving the interventricular
septum (hypointensity on T2 weighted images).
Myocardial enhancement on delayed gadolinium
enhancement corresponding to areas of fibrosis, with
sparing of the endocardium, allowing for
differentiation from the infarcted myocardium of
ischaemic heart disease.
Areas of hypokinesia of the myocardial wall.

Cardiac sarcoid.

Practical tips
Discussion
At autopsy cardiac sarcoid is seen in 2025% of patients
with sarcoidosis. However, as few as 5% exhibit any clinical
signs. Clinical presentation of sarcoid is with cardiac
arrhythmias, cardiomyopathy and heart failure.

CASE 23

Sparing of the endocardium suggests that coronary artery


disease is not the underlying diagnosis.

Further management
Look at CXR for respiratory changes of sarcoid.

23a

History
A 50-year-old male patient
presented with orthopnoea.

47

Answer 23

Chest Imaging

ANSWER 23
Observations (23a)
This chest radiograph shows evidence of previous surgery
there are sternotomy wires and a metallic mitral valve
replacement. The heart is enlarged with a cardiothoracic
ratio of 21:32. There is a double density seen through
the right heart a sign of left atrial enlargement. Multiple,
small (15 mm) calcium density nodules are seen in both
lungs with mid and lower zone predominance. Upper zone
venous diversion and a tiny right pleural effusion suggest
pulmonary venous hypertension.
Appearances indicate that the patient has had surgical
mitral valve replacement, but there are persisting features of
mitral valve disease and signs of left heart failure. He
should undergo further assessment of valve and left heart
function. The high-density pulmonary nodules indicate
pulmonary haemosiderosis, a consequence of elevated
pulmonary venous pressure over many years.

Left atrial appendage enlargement. The normal left


mediastinal contour has two convexities above the left
ventricle the aortic arch and the main pulmonary
artery. Enlargement of the left atrial appendage
produces a third bump below the pulmonary artery
(23b). This is sometimes called the third mogul sign
(the term mogul is one used by skiers to describe
bumps in the snow!).
Calcification of thrombus in the left atrium (23c).
Right ventricular hypertrophy.
Pulmonary venous hypertension, interstitial and
pulmonary oedema.
Pulmonary haemosiderosis longstanding elevation
of pulmonary venous pressure results in oozing of
serum into the interstitium. Blood products within
this will ultimately be broken down to haemosiderin.

Practical tips
Diagnosis
Mitral valve disease.

Discussion
The most common cause of mitral valve disease is
rheumatic heart disease, with presentation most commonly
seen in middle aged females. It results in left atrial
enlargement and pulmonary venous hypertension/heart
failure.
Radiographic signs of mitral valve disease are:
Double density behind the right heart border due to
left atrial enlargement.
Splaying of the carina by the large left atrium.
Oesophagus displaced to the right.

23b

23b Chest radiograph shows enlargement of the left


atrial appendage in a patient with mitral stenosis.

48

It is difficult to distinguish from plain film whether there


is mitral valve stenosis or regurgitation the radiological
consequences and signs are the same and the two can
coexist. However, if there is gross left atrial enlargement,
there must be a component of stenosis present.

Further management
Cardiac referral with a view to echocardiographic
assessment is required. Treatment involves drug therapy
for arrhythmias (atrial fibrillation AF) and to improve
cardiac function. Surgical intervention includes
percutaneous valve balloon dilatation (high recurrence
rates), valvotomy and valve replacement.

23c

23c Lateral chest radiograph shows curvilinear


calcification of the left atrial wall.

Chest Imaging

CASE 24

Case 24

24

History
A 70-year-old male was referred for
further investigation following initial
imaging showing a right lower lobe
mass lesion.

49

Answer 24

Chest Imaging

ANSWER 24
Observations (24)
These PET scan images from a PET CT demonstrate a
large right lower lobe highly FDG (18 fluoro-2-deoxyglucose) avid lesion. There is a smaller left upper lobe FDG
avid lesion. There is also right hilar uptake as well as uptake
in the subcarinal region. Uptake in the region of the
oesophagus is also demonstrated. There is no FDG avid
lesion in liver or adrenals. No bony lesions are
demonstrated.

Diagnosis
Metastatic right lower lobe bronchogenic carcinoma.

Differential diagnosis
The left upper lobe lesion represents either metastases or a
synchronous lesion.

Discussion
PET scanning relies on increased uptake of FDG in cell
populations with higher metabolic turnover. The main
application is oncological imaging; other, less utilized
applications include CNS and cardiac imaging. It is essential
that PET scans are compared with cross-sectional imaging
if no PET CT scan has been performed. Assessing only
PET images alone can lead to diagnostic errors, mainly
incorrect staging of malignant disease. The CT images (not
shown) of this PET/CT scan demonstrated a right sided
pleural effusion that turned out to be malignant. Pleural
effusions do not demonstrate increased FDG activity.
Among other areas, normal FDG uptake is
demonstrated in the brain, heart, salivary glands, liver,
spleen as well as upper renal tract and it is also excreted in
the urine. Normal bowel uptake is also often demonstrated.
Pitfalls include increased FDG uptake in fat (brown fatty
tissue); this can simulate malignant nodal disease. In the
current study, the increased oesophageal uptake is due to
a coexisting reflux oesophagitis.

PET scans can be false negative for small lung


metastases or solitary lung nodules. False positive diagnosis
on PET scanning also occurs due to infection and
inflammation conditions that are associated with increased glucose turnover.

Practical tips
In the UK, PET scanning is used mainly as a staging
tool. Neurological and cardiac applications are not
utilized widely. Among others, PET is used for initial
staging and restaging of bronchogenic or oesophageal
carcinoma.
Cancer networks are increasingly utilizing PET prior
to planning for curative surgery. A growing
application is the assessment of solitary nodules. This
is particularly useful in central lesions for which
histological confirmation is more challenging and the
complication rate is increased.
A well documented pitfall is a false negative scan for
small lung nodules or metastases. Lesions measuring
less than 78 mm do have a higher false negative rate.
Lymphoma staging is also widely undertaken. This is
very useful for assessment of activity in residual
lymphoma masses and also for assessment of early
response to chemotherapy. Inflammatory or infected
nodal mediastinal masses (histoplasmosis) can also
give rise to false positive scans. Correlation with crosssectional imaging is important in all cases where FDG
imaging is undertaken.

Further management
PET CT results should be discussed within a multidisciplinary team with the purpose of deciding whether the
patient would be a candidate for surgical disease clearance.

24

24a Right lower lobe bronchogenic


carcinoma (left) and likely metastasis (right).

50

Chest Imaging

CASE 25

Cases 25, 26

25a

History
A 45-year-old female, previously
resident in the USA, presented for a
pre-employment chest radiograph.

CASE 26

26a

History
A 24-year-old male presented with
dysphagia.

51

Answer 25

Chest Imaging

ANSWER 25
Observations (25a)
This frontal chest radiograph demonstrates widespread 13
mm diameter miliary nodules seen throughout both lungs
with no zonal predominance. The miliary nodules are of
calcific density. Popcorn calcification of the left hilar lymph
nodes is seen. In addition, there is amorphous calcification
seen under the left hemidiaphragm, which is likely to be in
the spleen.

Diagnosis
Histoplasmosis.

Differential diagnosis
For increased density miliary opacities:
Miliary metastases.
Pneumoconiosis silicosis, siderosis, baritosis.
Varicella-zoster (25b).
Haemosiderosis due to chronic pulmonary venous
hypertension, pulmonary haemorrhage, or idiopathic.
Histoplasmosis.

and haematogenously to the spleen. Acute infection usually


presents with few nonspecific symptoms, and radiological
findings include generalized lymphadenopathy, flitting
nonsegmental bronchopneumonia, multiple miliary
nodules, popcorn calcification of hilar/mediastinal lymph
nodes and splenic calcification. Chronic histoplasmosis is
seen in patients with chronic obstructive airways disease
and has radiological features of peripheral consolidation
and apical fibrosis. Disseminated infection can occur in
immunocompromised patients.
Nonpulmonary features of histoplasmosis include
pericarditis (510%) and rheumatologic syndromes (~6%),
e.g. arthralgia, erythema nodosum.

Practical tips
In cases where there are multiple radiological findings,
consider the differential diagnosis list for each finding and
identify an overlapping diagnosis (easier said than done in
a viva situation!).

Further management
For popcorn calcification of lymph nodes:
Sarcoidosis.
Silicosis.
Histoplasmosis.
Coal workers pneumoconiosis (CWP).
Lymphoma post radiotherapy.
For splenic calcification:
Tuberculosis.
Histoplasmosis.
Infarcts secondary to sickle cell disease.
Hydatid cysts.
Haematoma.

Discussion
Histo plasma capsulatum is a fungus usually found in
temperate climates and most commonly in the northern
USA. Infection is by inhalation of air borne fungal spores.
These germinate in the alveoli and then spread via the
pulmonary lymphatics to the hilar/mediastinal lymph nodes

25b

25b Multiple calcified tiny nodules at


the left lung base in a patient with old
varicella pneumonia.

52

Clinical/occupational history and HRCT can be useful to


differentiate the possible underlying diagnoses. Respiratory
referral with a view to antifungal treatment would be
required in the acute infection.

Further reading
Brown K, Mund DF, Aberle DR, et al. (1994).
Intrathoracic calcifications: radiographic features and
differential diagnoses. Radio Graphics 14: 12471261.
Wheat LJ, Wass J, Norton J (1984). Cavitary
histoplasmosis occurring during two large urban
outbreaks. Analysis of clinical, epidemiologic,
roentgenographic, and laboratory features. Medicine
(Baltimore) 63(4): 201209.
Wheat LJ, Connolly-Stringfield PA, Baker RL (1990).
Disseminated histoplasmosis in the acquired immune
deficiency syndrome: clinical findings, diagnosis and
treatment, and review of the literature. Medicine
(Baltimore) 69(6): 361.

Answer 26

Chest Imaging

Case 27

ANSWER 26
Observations (26a)

Practical tips

This frontal chest radiograph demonstrates a right sided


aortic arch. No left sided aortic knuckle is seen. The heart
is not enlarged. No focal lung parenchymal abnormality is
seen.

Right sided aortic arch can give a notch in the posterior


aspect of the upper oesophagus on contrast swallow
examination.

Further management
Diagnosis

No further management is required in this condition.

Right sided aortic arch.

Discussion
Diagnosis can be confirmed with arterial phase contrast
enhanced CT chest (26b). Right sided aortic arch can be
associated with several congenital cardiac abnormalities but
it can also be seen in patients without cardiac abnormalities. The latter group of patients usually have a right
sided aortic arch with an aberrant left subclavian artery
(arising as the most distal branch of the aortic arch), which
passes behind the oesophagus. This can be seen on a
barium swallow examination as a posterior indentation in
the barium column of the mid oesophagus. Patients with
right sided aortic arch with mirror image branching, such
that the left subclavian arises as a branch of the first vessel
of the aortic arch, are the group usually associated with
cyanotic heart disease. The aorta descends in the right
posterior mediastinum (although in a small proportion,
<5%, this is on the left).

26b

26b Axial CT image demonstrates the right sided


aortic arch.

CASE 27

27a

History
A 34-year-old smoker presents with
progressively worsening shortness of
breath.

53

Answer 27

Chest Imaging

ANSWER 27
Observations (27a)
The chest radiograph shows bilateral thick and thin walled
cysts throughout both lungs with preservation of lung
volumes. Some small nodules are also seen particularly
within the right upper zone. No pleural effusion or
pneumothorax is seen. No bony abnormality or soft tissue
nodules to suggest neurofibromatosis. In spite of the
patients sex, the widespread distribution and nodularity
would make Langerhans cell histiocytosis the most likely
diagnosis.

Diagnosis
Langerhans cell histiocytosis.

Differential diagnosis
For cystic lung disease with normal or increased lung
volumes:
Lymphangioleiomyomatosis affects women in
2nd6th decades. There are multiple cysts seen
throughout the lungs of relatively uniform size (~5
mm) and shape, with no nodule formation.
Commonly associated with chylous pleural effusions
and recurrent pneumothoraces.
Neurofibromatosis predominantly apical cysts.
Tuberous sclerosis.
Cystic bronchiectasis.

Discussion
Langerhans cell histiocytosis is a multisystem
granulomatous disease, which occurs more commonly in
young (3rd4th decades) adult males, and has an
association with cigarette smoking. Pulmonary changes on
CXR involve a sequence of changes initial signs are of
nodules that progressively cavitate to result in thick and
then thin walled cyst formation. Lung volumes are usually

normal or increased in up to 30%. Pneumothoraces are


common and can be recurrent. Pleural effusions are rarely
associated. HRCT appearances are of centrilobular nodules
and complex thin walled cysts that are of variable size and
shape and show relative sparing of the apices and
costophrenic recesses (27b).

Practical tips
On a CXR, cystic lung disease with normal/increased lung
volumes can be differentiated by looking at patients sex
and by identifying other radiological findings:
Lymphangioleiomyomatosis occurs exclusively in
women.
Neurofibromatosis changes on a CXR include ribbon
ribs; soft tissue masses (neurofibromas) seen in
posterior mediastinum and skin.
Tuberous sclerosis is easily identified by the history
being characterized by the triad of mental retardation,
seizures and adenoma sebaceum.
Bronchiectasis produces thicker walled cysts and the
thickened airway walls may also be visible along their
lengths as tramtracking.

Further management
HRCT can be useful in the follow-up of these patients
when looking for potential complications of
aspergilloma/mycetoma infection, cavitating nodules and
pneumothoraces.

Further reading
Moore AD, Godwin JD, Muller NL, et al. (1989).
Pulmonary histiocytosis X: comparison of
radiographic and CT findings. Radio lo gy 172:
249254.

27b

27b Single HRCT image demonstrates multiple


central thin walled cysts of varying sizes.

54

Chest Imaging

CASE 28

Cases 28, 29

28

History
A 29-year-old male presented with
intermittent pain in both arms. Images
28b and 28c were taken just 1 minute
apart what has happened between
them?

CASE 29

29a

History
A 65-year-old male presented with
progressively worsening shortness of
breath.

29b

55

Answers 28, 29

Chest Imaging

ANSWER 28
Observations (28a, b, c)

Discussion

Selected images from an arch aortogram. The first image


(28a) is a pre-contrast image which demonstrates bilateral
cervical ribs. The second image (28b) shows a pigtail
catheter in the arch with contrast injected and normal
appearance of the aortic arch and the proximal main
branches. The third image (28c) shows bilateral occlusion
of the subclavian arteries at the level of the thoracic outlet.
Given the presence of cervical ribs, it is suspected that the
second image was acquired with the arms down, and the
third image with the arms elevated.
The patient has thoracic outlet syndrome.

Diagnosis

Thoracic outlet syndrome is a clinical syndrome caused by


nerve, artery or vein compression in the root of the neck
upon elevation of the arms. The vast majority of symptoms
arise due to nerve compression and vascular symptoms are
found in as few as 2% of symptomatic patients. The
subclavian artery and/or vein are transiently occluded with
arm movement, but more permanent vascular problems
can arise due to the repeated trauma of compression focal
stenosis, poststenotic dilatation and aneurysm formation.
The result of this is upper limb ischaemic symptoms with
pain, a cold limb and Raynauds phenomenon. Further
complications of poststenotic thrombus formation and
subsequent embolism can occur.

Thoracic outlet syndrome secondary to bilateral cervical


ribs.

Practical tips

Differential diagnosis

Cervical ribs occur in ~34% of the population but are only


symptomatic in 10% of cases.

Of causes of thoracic outlet syndrome:


Congenital cervical rib, fibrous band or abnormal
1st rib.
Acquired first rib exostosis or fracture, body
habitus.

Further management
US is useful as a dynamic test for assessing arterial
Doppler signal with the arm in a neutral and elevated
position.
CT/MRI can be useful in identifying the cause of
compression if not apparent on plain radiography.

ANSWER 29
Observations (29a, 29b)
Two axial CT images of the chest show features of
cardiomegaly, bilateral pleural effusions and patchy ground
glass opacity with smooth interlobular septal thickening.
This combination of imaging findings would fit with a
diagnosis of pulmonary oedema.

Diagnosis

For smooth interlobular septal thickening:


Pulmonary oedema.
Alveolar proteinosis.
Pulmonary fibrosis.
For nodular interlobular septal thickening:
Lymphangitis carcinomatosa.
Sarcoidosis.

Pulmonary oedema due to heart failure.

Discussion
Differential diagnosis
For ground glass opacity:
Pulmonary oedema.
Infection.
Haemorrhage.
Interstitial pneumonias.
Extrinsic allergic alveolitis.
Alveolar proteinosis.
Sarcoidosis.
Focal areas of ground glass alveolar cell
carcinoma/lymphoma.

56

Cardiac failure is a diagnosis usually made clinically and


confirmed with CXR. The following findings tend to
progress in sequence as the condition deteriorates and
pulmonary venous pressure increases further:
Upper zone vascular predominance.
Interstitial oedema with peripheral reticulations and
Kerley B lines.
Alveolar oedema with a perihilar predominance (bats
wing appearance).
The following features provide supplementary evidence to
suggest the diagnosis:
Cardiomegaly with cardiothoracic ratio >50%.
Pleural effusion.

Answer 29

Chest Imaging

These same findings are visible on CT. Alveolar fluid often


presents as ground glass opacity, which is an increased
haziness/attenuation in the lung and which can be
patchy in distribution. Ground glass opacity is a relatively
nonspecific imaging finding due to many conditions that
cause an overall increase in density within the segment of
lung displayed as a pixel on the CT image. There is a long
differential diagnosis including any cause of alveolar
fluid/consolidation.

CASE 30

Case 30

Practical tips
A combination of cardiomegaly, pleural effusion and
airspace opacity suggests cardiac failure with pulmonary
oedema.

Further management
Medical management with CXR radiological follow-up as
appropriate.

30a

History
A 65-year-old male was referred for a
cardiac MRI to assess cardiac viability.

30b

57

Answer 30

Chest Imaging

ANSWER 30
Observations (30a, 30b)
The first image (30a) is a short axis scan through the left
ventricle during the first pass of contrast. There is no
contrast enhancement of the thinned left ventricular septal
and anterior walls. The visualized posterior, lateral and
inferior walls show normal enhancement and are of normal
thickness. The second image (30b) is a delayed four
chamber view showing delayed, prolonged enhancement
of the septal, apical and anterolateral wall of the left
ventricle. These appearances are of an extensive left
ventricular infarct with no evidence that involved areas are
viable.

Diagnosis
Left ventricular infarct involving the septal/apical/
anterolateral walls.

Discussion
Cardiac MRI has an expanding role and current uses
include assessment of:
Cardiac viability prior to revascularization.
Cardiac congenital heart defect.
Cardiac tumours.
Pericardial disease.
Cardiomyopathies.

Short axis cine images are acquired at first pass of a


bolus injection of gadolinium to determine perfusion.
Infarcted myocardium shows no enhancement on first-pass
imaging (as demonstrated in this case). In addition, firstpass imaging can show:
Whether the infarct is transmural or subendocardial.
Degree of hypo/akinesia.
Delayed enhancement sequences at approximately 5 min
show enhancement in infarcted tissue since clearance of
contrast from fibrotic tissue is slower than from normal
myocardium.
If there is any uncertainty regarding differentiation of
ischaemic from infarcted myocardium then cardiac MRI
stress testing is performed with first-pass images acquired at
stress with adenosine and then repeated after 20 min at
rest. Areas of hypoenhancement at stress that show
recovery at rest represent areas of ischaemia rather than
infarction.

Practical tips
Cardiac MRI is a dynamic test that requires assessment of
cine images to make a subjective and objective assessment
of left ventricular function (ejection fraction).

Further management
Assessing cardiac viability post myocardial infarction is
important since revascularization of live tissue reduces
morbidity and mortality. Previously cardiac perfusion was
assessed by cardiac nuclear medicine (MIBG metaiodobenzylguanidine [scintiscan]) stress testing but there is
now an increased role for cardiac MRI.

Coronary artery atherosclerotic disease is characterized


using coronary angiography or coronary artery CT.
Patients with ischaemic but viable myocardium may be
suitable for revascularization with angioplasty or bypass
grafting.

30b

30b Transmural delayed gadolinium enhancement.

58

Chest Imaging

CASE 31

Cases 31, 32

31a

History
A 45-year-old female patient presented
with chest pain.

CASE 32

32a

History
A 24-year-old male presented with
chest pain after minor trauma.

59

Answer 31

Chest Imaging

ANSWER 31
Observations (31a)

Discussion

This chest radiograph shows evidence of a previous right


mastectomy. Heart and mediastinal contours are normal.
No lung abnormality is seen. There is diffuse sclerosis of all
visible bones, most evident in the ribs. Appearances are of
diffuse sclerotic metastases from a breast carcinoma primary.

With breast cancer being such a common malignancy,


complications will present frequently on plain radiographs
so it is important to note a mastectomy. Another tell tale
sign of previous breast cancer is the presence of axillary
clips from node sampling.
The mastectomy may first be perceived as a disparity
between the overall densities of the two hemithoraces.
There are many other causes for this, though sometimes it
may not be easy deciding which side is normal.

Diagnosis
Mastectomy and sclerotic bone metastases.

Differential diagnosis
For causes of unilateral hypertransradiancy:
Chest wall abnormality mastectomy, pectoral muscle
atrophy (polio) or absence (Polands syndrome).
Pleural abnormality pneumothorax.
Lung abnormality SwyerJames syndrome
(consequence of bronchiolitis as a child resulting in a
hypoplastic lung with air trapping on expiration),
emphysema, bullae, pulmonary embolus (31b).
For causes of unilateral hyperdensity:
Chest wall abnormality gynaecomastia (unilateral in
40% of cases), breast implant.
Pleural abnormality pleural effusion on supine film,
pleural thickening.
Lung abnormality unilateral pulmonary oedema
from lying on one side (31c), consolidation, lobar
collapse (especially left upper lobe).

Practical tips
In any case with a history of breast cancer, the chest
radiograph should be scrutinized for features of recurrence.
These are classical viva films for exams and also present
frequently in everyday practice. Features to look for
include:
Sclerotic/lytic bone metastases (31d).
Lung metastases.
Lymphangitis carcinomatosa.
Pleural effusion.
Axillary lymphadenopathy.
Right hemidiaphragm elevation secondary to liver
metastases.
Pulmonary pneumonitis/fibrosis from radiotherapy.

Further management
In cases where clinical history and examination do not
reveal an obvious cause for the relative differences in
chest lucency CT chest would be appropriate.
Isotope bone scan may identify distant bone
metastases.

31b
31c

31b There is a large central pulmonary embolus on


the right with marked reduction in vascular markings
on this side.

60

31c Unilateral pulmonary oedema with pleural


effusion and airspace opacity on the right.

Answers 31, 32

Chest Imaging

31d Multiple poorly defined lytic lesions are seen


throughout the ribs bilaterally.

31d

ANSWER 32
Observations (32a)

Discussion

The chest radiograph shows no focal lung abnormality and


no rib fracture or pneumothorax from the recent trauma.
However, there is bilateral inferior rib notching involving
the 3rd8th ribs. The aortic knuckle is not well seen but
there is no other mediastinal contour abnormality. The
heart is not enlarged. There is no evidence of previous
cardiac surgery. No other specific bony or soft tissue
abnormalities are seen.
The most likely diagnosis is coarctation of the aorta
the patients blood pressure should be checked in both
arms and compared with that in the legs for confirmatory
evidence.

The most common cause of inferior rib notching is


coarctation of the aorta. This can be of two types:
Preductal the hypoplastic narrowed segment is long.
These patients present in infancy and early childhood
with congestive cardiac failure. Prognosis is worse
than for those who present with the postductal type.
Postductal this usually consists of a short narrowed
segment, immediately distal to the site of the
ligamentum arteriosum (32b). Presentation is usually
in later childhood, and is with hypertension,
differential blood pressures in the upper and lower
limbs and/or a heart murmur.
(co nt.)

Diagnosis
Inferior rib notching due to coarctation of the aorta.

32b

Differential diagnosis
For inferior rib notching:
Arterial:
Coarctation of the aorta.
Subclavian obstruction after BlalockTaussig shunt
for tetralogy of Fallot.
Venous: SVC obstruction.
Arteriovenous malformations (AVM).
Neurogenic, neurofibromatosis.
For superior rib notching:
Rheumatoid arthritis, scleroderma and systemic lupus
erythematosus (SLE).
Hyperparathyroidism.
Neurofibromatosis.
Marfans syndrome.
Restrictive lung disease.

32b Single angiographic image shows postductal


stenosis of the descending thoracic aorta, with
poststenotic dilatation.

61

Answer 32

Chest Imaging

On chest radiographs, the focal narrowing of the aorta is


classically made more evident because of pre- and
poststenotic dilatation producing a classic figure of 3 sign.
Obscuration of the arch, as in this case, is also recognized.
Rib notching on the inferior surface of the 3rd8th ribs can
usually be seen in untreated patients by 8 years. Rib
notching occurs due to dilatation of the posterior
intercostal arteries, which act as collateral vessels. Since the
1st and 2nd posterior intercostal arteries arise from the
costocervical trunk of subclavian artery rather than
descending aorta, they do not form a collateral path and
hence do not cause rib erosion.
Coarctation of the aorta is associated with several other
congenital anomalies such as bicuspid aortic valve, patent
ductus arteriosus (PDA), ventricular septal defect (VSD),
tricuspid atresia and transposition of the great vessels. There
is also an association with Turners syndrome.

When inferior rib notching is noted check:


The aortic contour for the figure of 3 sign.
Is there evidence of previous repair, e.g.
thoracotomy scar?
The heart for evidence of left ventricular
hypertrophy, i.e. elevation of the apex.
Are there features to indicate neurofibromatosis,
e.g. cutaneous soft tissue nodules?
When rib notching is unilateral, suspect an aberrant
subclavian artery origin on the unaffected side.

Further management
Surgical treatment for coarctation of the aorta involves
resection and end to end anastomosis, prosthetic patch
graft, subclavian flap aortoplasty or balloon angioplasty.

Practical tips
When suspected from plain films and clinical findings,
MR or CT angiography has now largely replaced
conventional angiography as the next investigation of
choice (32c).

32c

32c A maximum intensity projection (MIP) image from an


MRA examination of the thoracic aorta.

62

Chest Imaging

CASE 33

Cases 33, 34

33a

History
A 38-year-old man was being
investigated for transient neurological
episodes.

CASE 34

34

History
A 52-year-old woman with oesophageal
cancer presented complaining of chest
pain.

63

Answers 33, 34

Chest Imaging

ANSWER 33
Observations (33a)

Further management

There are some increased tubular soft tissue density


opacities in the right lower zone, suggestive of abnormal
vessels, leading to a faint pulmonary nodule. There is a
further nodular opacity in the right upper zone, again with
the suspicion of vessels running to it from the right hilum.
The left lung is clear and the mediastinal outline normal.
Given the clinical details, pulmonary arteriovenous
malformations giving rise to paradoxical emboli and TIAs
must be presumed. Contrast enhanced CT would confirm.

Angiographic assessment and treatment with embolization


or balloon occlusion is now the preferred management.

Further reading
Coley SC, Jackson JE (1998). Pulmonary arteriovenous
malformations. Clinical Radio lo gy 53(6): 396404.
Pick A, Deschamps C, Stanson AW (1999). Pulmonary
arteriovenous fistula: presentation, diagnosis, and
treatment. Wo rld Jo urnal o f Surgery 23(11):
11181122.

Diagnosis
Pulmonary arteriovenous malformation (AVM).

Differential diagnosis
Single or multiple pulmonary nodules from other causes.

33b

Discussion
Pulmonary AVMs are abnormal vascular communications
between pulmonary arteries and veins (95%) or systemic
arteries and pulmonary veins. Most commonly, they are of
the simple type, with a single artery feeding a focal
aneurysmal segment and a single draining vein. Complex
lesions have more than one artery and/or vein. Figure 33b
is a single angiographic image that demonstrates the large
feeding vessel to a solitary AVM.
Multiple pulmonary AVMs may be associated with
OslerWeberRendu syndrome. They are usually asymptomatic until the 3rd4th decade when they can present
with local effects haemoptysis, dyspnoea on exertion and
cyanosis with clubbing (due to right to left shunt); or with
distal effects cerebrovascular accident (CVA) or brain
abscess due to paradoxical emboli. Lesions enlarge with
age.

Practical tips
The finger in glove appearance from mucoid impaction
of central airways in allergic bronchopulmonary aspergillosis can look similar in some ways to the vessels
supplying an AVM. However, there is a nodule at the end
of these vessels in AVM.

33b Single image from an angiographic


investigation demonstrating a large feeding
vessel to the pulmonary AVM.

ANSWER 34
Observations (34)

Differential diagnosis

This chest radiograph shows a normal sized heart and clear


lungs. There is, however, evidence of air within the
mediastinum best seen around the left heart border and
aortic arch. Below the medial left diaphragm, a stent is just
about visible across the gastro-oesophageal junction.
It is likely that there has been oesophageal perforation
from the stented oesophageal tumour.

For causes of pneumomediastinum:


Alveolar rupture in acute asthma.
Oesophageal rupture due to malignancy, trauma,
violent vomiting (Boerhaaves syndrome usually
associated with a left sided pleural effusion).
Extension from peritoneum pneumoperitoneum.
Iatrogenic following oesophageal balloon
dilatation/stenting, bronchoscopy, mediastinoscopy,
positive pressure ventilation.

Diagnosis
Pneumomediastinum
oesophageal cancer.

64

from

perforation

of

stented

Answer 34

Chest Imaging

Discussion
Pneumomediastinum is an uncommon condition that is
usually asymptomatic but can present with neck or chest
pain. In itself it rarely leads to any complications but it can
be a useful diagnostic sign for an underlying medical
condition that does need treatment. Although a rare
condition in adults, it is seen in children and most
commonly in neonates.
Radiological features include:
Thin line following the mediastinal contours, as in
this case. This line represents the pleura, lifted away
from the mediastinal soft tissue by air.
Streaky air lucencies in the mediastinum.
Continuous diaphragm sign where lucency is seen
behind the heart, connecting the two domes of the
diaphragm.

Case 35

Look for an effusion to suggest oesophageal injury, or


oesophageal stent, as in this case.

Further management
Chest CT after the patient has drunk some water-soluble
contrast, or a water-soluble contrast swallow, can confirm
an oesophageal tear and leak and help to identify the site.

Further reading
Gerazounis M, Athanassiadi K, Kalantzi N, Moustardas
M (2003). Spontaneous pneumomediastinum: a rare
benign entity. Jo urnal o f Tho racic and Cardio vascular
Surgery 126(3): 774776.

Practical tips
Assess the ribs for any sign of trauma.
Assess the lung fields for a lesion that might have
warranted bronchoscopic/surgical investigation.
Look for air under the diaphragm to indicate
pneumoperitoneum.

CASE 35

35a

History
A 50-year-old male presented to A&E
with recurrent acute-on-chronic
dyspnoea.

65

Answer 35

Chest Imaging

ANSWER 35
Observations (35a)
There is a large area of increased lucency in the right mid
and upper zone with no visible lung markings. There is no
visible lung edge, however, to suggest this is a pneumothorax. Moreover, there is crowding of vessels in the lower
zone. The appearances are likely to indicate a large upper
lobe bulla. There are no focal pulmonary opacities to
indicate superadded infection. Comparison with any
available old films should help confirm this interpretation.
A general reduction in density in the left upper zone
suggests that similar pathology is developing here too.

Diagnosis
Large right sided bulla.

Discussion
A bulla is a large dilated airspace within the lung with a wall
less than 1 mm thick. They are usually produced by alveolar
destruction in emphysema. Impaired ventilation of the rest
of the lung can result in dyspnoea, as well as that due to
the background chronic lung disease.
Confusion can arise when the absence of lung markings
leads to the erroneous diagnosis of pneumothorax. This

can have dire consequences if an intercostal chest drain is


then mistakenly placed in the bulla! An expiratory film
makes a pneumothorax easier to detect (because it enhances the contrast differential between pleural air and
lung parenchyma) but probably wont help resolve diagnostic confusion with a bulla. Patients with chronic lung
disease may well have previous films that show a bulla to
be longstanding.

Practical tips
A pneumothorax leads to two findings: a visible lung edge
and hypodensity with absent lung markings lateral to this
(35b). A bulla has absent lung markings but not the
discrete lung edge. Conversely, skin folds (seen most
frequently in babies and the elderly) can produce a pseudo
lung edge but there will be lung markings lateral to it
(35c).

Further management
If diagnostic uncertainty continues after expert review of
the films, then occasionally CT of the thorax may be
needed to differentiate bullae from pneumothorax.

35b
35c

35b Image demonstrates a small pneumothorax


with no lung markings lateral to the lung edge.

66

35c Chest radiograph demonstrates a skin fold


giving an apparent lung edge but lung markings are
visible lateral to it.

Chest Imaging

Case 36

CASE 36
History
A 15-year-old male with diabetes presented
with recurrent chest infections.

36b

36a

36a, 36b Sequential chest radiographs taken 4 months apart.

36c

36c HRCT thorax of the same patient at a later date.

67

Answer 36

Chest Imaging

ANSWER 36
Observations (36a, 36b, 36c)
This frontal chest radiograph (36a) demonstrates widespread, fairly symmetrical lung parenchymal abnormality
that is central in distribution and characterized by bronchial
wall thickening and cystic bronchiectatic change. Some
larger soft tissue density nodules are seen in the lower
zones, which likely represent mucous plugs. Bilateral
lobulated hilar enlargement is seen with the right sided
lymphadenopathy being more evident. Appearances are of
cystic fibrosis.
Chest radiograph (36b) from the same patient taken at
a later date demonstrates more advanced disease with more
mucous plugging and hyperinflated lungs secondary to air
trapping. Peribronchial cuffing and thickened nontapering
bronchi are evident. A right subclavian central line is noted,
presumably for the administration of IV antibiotics.
A CT image (36c) of the chest on lung windows shows
typical features of CF with cylindrical bronchiectasis and a
central distribution, peribronchial cuffing, mucous plugging
and focal atelectasis. It also shows the most common
complication of a pneumothorax on the left side.

Diagnosis
Cystic fibrosis (CF).

Discussion
Cystic fibrosis is an autosomal recessive multisystem
condition that is characterized by exocrine gland
dysfunction due to mucous plugging arising secondary to a
fault in cell chloride transport. The condition affects whites
with a geographical distribution affecting Europeans and
Ashkenazi Jews. Diagnosis is usually made in children, with
the majority being diagnosed within the first year of life
with the clinical presentation of meconium ileus and
respiratory symptoms. Pulmonary complications are the
predominant cause of death and survival is limited to
~3040 years.

36c

36c HRCT thorax of the same patient at a later


date.

68

Radiological features of CF on a chest radiograph are:


Cystic/cylindrical bronchiectasis.
Peribronchial cuffing/thickening.
Mucous plugging with secondary atelectasis due to
obstruction.
Hilar lymphadenopathy.
Allergic bronchopulmonary aspergillosis (ABPA).
Respiratory complications of CF include pneumothorax,
haemoptysis and cor pulmonale. Chronic pulmonary
infection can lead to hypertrophic pulmonary osteoarthropathy (HPOA).
Cystic fibrosis is a multisystem disease: as well as causing
meconium ileus (in children) and meconium ileus
equivalent (in adults), it can also cause pancreatic insufficiency, biliary cirrhosis, portal hypertension, malabsorption
due to gallbladder disease, cholelithiasis and clubbing
(36d).

Practical tips
Typical presentation is of widespread pulmonary
disease in a young patient.
Tunnelled central lines in patients are used for long
term drug treatment with antibiotics or
chemotherapy. Again this is a clue to the underlying
diagnosis.
Look for complications of CF pneumothoraces and
secondary infections.
Though not a common site, HPOA can occur in the
humerus and may thus be visible on the edge of the
film.

Further management
Follow-up in these patients is best performed with HRCT,
which can answer the important questions when lung
transplant is considered is there coexistent Aspergillus
infection? Are there complicating features, e.g. pneumothoraces?

36d

36d Single axial CT image


demonstrates significant splenomegaly
due to portal hypertension and
pancreatic calcification secondary to
recurrent bouts of acute pancreatitis.

Chest Imaging

Case 37

CASE 37
History
A 34-year-old IV drug user presented with cough and
severe shortness of breath. He is noted to be significantly
hypoxic on presentation. This series of chest radiographs
were taken at presentation, day 3 and day 7.

37a

37b

37c

69

Answer 37

Chest Imaging

ANSWER 37
Observations (37a, 37b, 37c)
The first chest film taken at presentation (37a) shows no
obvious radiological abnormality. The next radiograph
from 3 days later (37b) shows bilateral mid and lower zone
reticular/granular interstitial infiltrate with no evidence of
volume loss. The third film from 7 days after presentation
(37c) shows bilateral consolidative change affecting both
lungs. There is a pneumothorax on the left with subtotal
collapse of the left lung. Surgical emphysema is noted.
It is notable that the patient was initially very
symptomatic with a relatively normal chest film and then
showed rapid pulmonary changes over several days. The
history of IV drug abuse raises the possibility of background
HIV infection and the sequence of radiological appearances
is typical for Pneumo cystis carinii pneumonia.

Complications of PCP are common, e.g. pneumothorax


and superimposed TB/fungal infections.
Appearances on CT include a patchy mosaic/ground
glass pattern with subsegmental sparing and coexistent
thin walled cysts and pneumatoceles. Appearance of
nodules on CT imaging is suggestive of a second disease
process being involved metastases, lymphoma, septic
emboli or Kaposis sarcoma.
Bilateral gallium uptake is seen prior to radiological
changes being evident.

Practical tips
A strong index of suspicion for PCP should be maintained
when an immunocompromised patient presents with
dyspnoea. As in this case, symptoms can be out of
proportion to the initial radiographic changes.

Diagnosis
Pneumo cystis carinii pneumonia (PCP).

Further management

Discussion

Diagnosis is confirmed with sputum cytology or


bronchoscopy and lavage. Treatment is with IV cotrimoxazole.

P. carinii pneumonia is the most common cause of


interstitial pneumonia in immunocompromised patients.
Radiological findings can be very variable but the typical
pattern of development is as follows:
Normal chest radiograph is seen in up to 40% of
patients, especially early on in the infection.
Bilateral diffuse perihilar airspace/granular/reticular
opacities is the typical appearance.
Progression to diffuse airspace consolidation with air
bronchograms.
Pleural effusions are seen in ~20% of cases.
Response to treatment occurs over a period of ~1
week.
Atypical features (seen in ~5%) include upper lobe thin
and thick walled cysts, lung nodules, mediastinal and
hilar lymphadenopathy.
Treatment with aerosolized pentamidine alone results
in the disease affecting upper lobes.

70

Further reading
Kuhlman JE, Kavuru M, Fishman EK, Siegelman SS
(1990). Pneumo cystis carinii pneumonia: spectrum of
parenchymal CT findings. Radio lo gy 175: 711714.

Chapter 2

ABDOMINAL IMAGING

PLAIN ABDOMINAL RADIOGRAPHS


As always, a systematic approach is required to be able to
extract the most information from a radiograph. The order
suggested is a guide and should be adapted to your own
preferences and adjusted for each individual case.

General assessment
Rapidly assess the quality of the film, ensuring that there
is adequate coverage are the hernial orifices covered in
the patient who appears to have small bowel obstruction?
In addition, make a quick assessment of the film to exclude
two important diagnoses that are surgical emergencies and
will require immediate management:
Perforation.
Toxic megacolon.

Lines
Identify any lines that are present, e.g. postsurgical drains,
NG tubes, urinary catheters and peritoneal dialysis
catheters. The presence of peritoneal dialysis catheters or
postsurgical drains may offer a simple explanation for free
intraperitoneal gas.

forget to check for bowel loops extending below the


inguinal ligament indicative of hernia.
Exclude bowel dilatation and wall thickening. If there
is dilatation, decide whether it is small or large bowel and
then try to establish the cut-off point. Later cases illustrate
specific features to check for when one suspects small or
large bowel obstruction.

Soft tissue organs


Check the size and outline of the liver, spleen and kidneys.
Also check the psoas outlines which may be obscured by
retroperitoneal pathology.

Calcification
Look at the rest of the film for calcified densities such as
gallstones (only 510% visible on plain radiographs) and
renal tract calculi (8085% visible on plain radiographs).
Common incidental calcific opacities include mesenteric
nodes, phleboliths, vascular calcification and uterine
fibroids. Calcification within the solid organs may be more
significant however. Unusual calcified opacities may allow
for a specific diagnosis such as a pelvic dermoid.

Gas pattern
Differentiating small from large bowel is very difficult in
young children and one can usually only say whether a
problem is in the proximal or distal bowel. In adults,
features used to distinguish the two include position in the
abdomen; diameter (large bowel <5 cm, small bowel <3
cm); valvulae conniventes traverse the whole width of small
bowel but this is not so with the colonic haustrations; solid
faeces only seen in colon.
The range of normal bowel gas patterns is rather wide,
but from experience, one should quickly know whether
the amount and distribution of bowel gas are very
abnormal. Unusually shaped collections of gas (e.g.
triangular) should raise suspicion of gas outside the GI
tract and lead you to examine the area very closely. Check
carefully for gas in areas of the abdomen where it is not
normally seen, e.g. over the liver (either as pockets of gas
or within the biliary tree) or in the retroperitoneum. Dont

Bones
Assess the bones of the vertebral column and pelvis for
incidental pathology or findings that might be relevant to
intra-abdominal disease such as sacroiliac joint changes
associated with inflammatory bowel disease.

Periphery of the film


Conclude by looking at peripheral structures. For example,
pathology at the lung bases that might mimic abdominal
pathology, incidental lesions in the abdominal wall.

ABDOMINAL CT
The first question to ask yourself when looking at an
abdominal CT is what type of contrast has the patient been
given, i.e. oral/IV/both and during what phase has
imaging been carried out. The typical phase for abdominal

71

Abdominal Imaging

imaging is portalvenous (6070 seconds delay) but other


phases, e.g. pre-contrast, arterial, delayed, are useful in
characterizing specific lesions.
Axial images will commonly be presented and you can
proceed in one of two ways either assess all structures on
each axial image then proceed to the next image, or,
alternatively, assess each organ in turn over a series of slices
before moving on to the next organ. In reality, a
combination of the two is often used but the latter ensures
nothing is missed.
As well as searching for focal lesions within the organs,
other features to check include the following:
Liver:
Overall attenuation (e.g. reduced in fatty change)
and enhancement homogeneity.
Size and contour (e.g. irregular contour in
cirrhosis).
Patency of normal vascular structures.
State of intrahepatic bile ducts.
Gallbladder check for stones, the density of bile,
wall thickening and pericholecystic fluid. Carefully
assess the extrahepatic bile ducts.

72

Pancreas check for calcification; the state of the


pancreatic duct, peripancreatic collections and
inflammatory changes.
Spleen check size and adjacent varices.
Kidneys is the attenuation of the kidneys correct for
that phase of scan? Is there collecting system
dilatation? Check the adrenals above.
Then systematically evaluate the non-solid organs:
Aorta as well as overall size, check for dissection and
patency of main branches. Check major venous
patency at the same time, excluding deep vein
thrombus.
Bladder, ovaries/uterus.
Lymphadenopathy.
Intra-abdominal fat peritoneal soft tissue deposits
are easily overlooked unless the abdominal fat is
carefully scrutinized.
Colon, ileum and stomach.
Free gas/fluid.
Bones and lung bases.

Abdominal Imaging

CASE 38

Cases 38, 39

38a

History
A 53-year-old male presented with a
history of gradually progressive
dysphagia.

CASE 39

39

History
A 78-year-old female presented with
rectal bleeding.

73

Answer 38

Abdominal Imaging

ANSWER 38
Observations (38a)

Practical tips

Single AP image from a double contrast barium swallow


examination shows a short, smoothly tapered narrowing
in the lower oesophagus just superior to the gastrooesophageal junction and a small hiatus hernia. The
oesophagus proximal to this has delicate transverse mucosal
folds, the so-called feline oesophagus. This appearance is
associated with gastro-oesophageal reflux and leads to the
conclusion that the short stricture is a peptic stricture.

Previous CXRs can be useful in identifying a cause for a


stricture look for a tumour that might have been
irradiated or features of aspiration pneumonia/hiatus
hernia.

Diagnosis

Further reading

Peptic stricture feline oesophagus.

Luedtke P, Levine MS, Rubesin SE, et al. (2003).


Radiologic diagnosis of benign esophageal strictures:
a pattern approach. Radio Graphics 23: 897909.

Differential diagnosis

Further management
Gastroenterology referral with a view to direct visualization
and confirmation of diagnosis with endoscopy.

For oesophageal strictures:


Lower oesophagus:
Peptic stricture secondary to gastro-oesophageal
reflux.
Scleroderma affected patients have an
incompetent lower oesophageal sphincter and
reduced peristalsis resulting in marked gastrooesophageal reflux.
NG intubation prevents closure of the lower
oesophageal sphincter.
ZollingerEllison syndrome.
Upper and mid oesophagus:
Barrett oesophagus acquired condition
characterized by columnar metaplasia secondary to
chronic gastro-oesophageal reflux/oesophagitis.
Premalignant condition with an increased risk of
adenocarcinoma of the oesophagus.
Caustic ingestion usually long, smooth
narrowing forms 13 months post ingestion.
Mediastinal radiotherapy usually long, smooth
narrowing forms 48 months post radiotherapy.
Skin diseases epidermolysis bullosa, pemphigoid,
erythema multiforme.

38b

Other less common causes of strictures include Crohns


disease, Candida oesophagitis and Behets disease.

Discussion
Peptic strictures have this typical appearance of short (14
cm), smooth, tapered, concentric narrowing in the lower
oesophagus. Associated radiological findings include
intramural pseudodiverticulosis (38b) and feline oesophagus (so called because this is the normal appearance in
cats). Longitudinal scarring can cause fixed transverse folds
but these can be differentiated from feline oesophagus
since they are only seen in the region of the stricture and
do not extend more than half way across the oesophagus,
giving a step ladder appearance.
38b Contrast barium swallow shows flask shaped
outpouchings with a narrow neck of intramural
pseudodiverticulosis.

74

Answer 39

Abdominal Imaging

Case 40

ANSWER 39
Observations (39)
Single image from a mesenteric angiogram examination
shows an abnormal cluster of vessels and contrast blush
in the caecum.

Diagnosis
Angiodysplasia.

Differential diagnosis
Of lower GI haemorrhage:
Diverticular disease.
Colonic carcinoma.
Colonic angiodysplasia.
Inflammatory colitis.
Mesenteric varices.

Discussion
Angiodysplasia is the most common cause of occult
bleeding in the large bowel, predominantly affecting the
elderly population. The condition is characterized by
vascular ectasia of the colonic circulation, most commonly
affecting the caecum and ascending colon. The condition
leads to chronic low-grade blood loss but can also lead to
episodes of severe lower GI bleeding. There is an association with valvular heart disease, specifically aortic stenosis.

CASE 40

Diagnosis can be made with selective mesenteric


angiography or CT angiography. Both are able to identify
bleeding when the rate is as little as 1 ml/min. Three levels
of abnormality are identified:
In early disease, a densely contrast filled dilated vein is
seen within the bowel wall.
As the disease progresses, a vascular tuft can be seen
at the lesion site.
Further progression shows an early filling vein during
the arterial phase of scanning.
Mesenteric angiography has the advantage of proceeding
directly to treatment with embolization.

Practical tips
CT is excellent at identifying the bleeding point when
there is active GI bleeding. Always perform a pre-contrast
scan prior to the arterial phase scan so that highcontrast intraluminal blood can be differentiated from
high-contrast bowel food content/debris.

Further management
Surgical resection is the definitive treatment when
endoscopic treatments have not controlled bleeding.

40a

History
A 35-year-old farmer presented with
mild ache in the right upper abdomen
for several months.

75

Answer 40

Abdominal Imaging

ANSWER 40
Observations (40a)
There is a large, well defined lesion in the right upper
quadrant, which has thin, curvilinear calcification of its wall.
This is projected over the liver and is probably intrahepatic,
though a calcified gallbladder cannot be excluded from this
film. This is a solitary lesion with no other abnormality
seen. Given the appearances and patients young age and
occupation, a hydatid cyst of the liver is most likely. Further
imaging with CT would help confirm the location of the
lesion and the likely diagnosis. Serological tests for hydatid
disease should also be undertaken.

There is a predilection for the lower lobes and disease is


more commonly seen on the right. Cysts are multiple in
20% and bilateral in 20% of cases. Figure 40b shows several
left sided, well defined round intrapulmonary lesions.
Calcification is rare. When air infiltrates between the layers
of the cyst wall it can give the appearance of a meniscus
sign, onion peel sign and finally the water lily sign, when
there is complete separation of the endocyst from the
pericyst. Rupture of the cyst can result in surrounding
consolidation.

Practical tips
Diagnosis

Of calcified liver lesion:


Metastasis especially colorectal cancer.
Primary liver tumour.
Infection hydatid, TB.

Benign liver cysts are common, but all cysts should be


closely inspected for atypical features, e.g.
hyperattenuating wall or wall calcification suggestive
of abscess/hydatid; poorly defined edges, which may
suggest the lesion is in fact a metastasis.
Cystic lesions involving liver and lungs should suggest
infective/malignant underlying diagnosis until proven
otherwise.

Discussion

Further management

Hydatid disease is acquired through infection by the


parasitic tapeworm Echino co ccus granulo sus. Dogs are the
definite host with the human acting as an accidental host
following accidental ingestion of eggs from canine faeces.
The liver is the most commonly involved organ and
presentation is with abdominal pain, jaundice, biliary colic
with eosinophilia in 2050%.
In the liver, infection results in the formation of a cyst,
which is more commonly found in the right lobe; the size
of the cyst ranges up to 50 cm but is ~5 cm on average and
multiple in 20% of cases. The cyst is composed of three
layers the outer pericyst, middle laminated ectocyst and
the inner endocyst.
Radiological features are:
Variable appearance ranging from a simple unilocular
cyst to a complex heterogeneous cystic mass.
Daughter cysts are characteristic but are a rare finding.
Their presence is noted by a racemose appearance.
Initially daughter cysts are well defined and round and
are seen at the periphery of the mother cysts. They
progress to form large, irregular shaped cysts filling
the mother cyst.
Can also contain debris (hydatid sand), internal
septations and wall calcification.
Calcification is seen in 2030% of hydatid cysts and is
usually curvilinear or ring like. Dense calcification is
seen as the cyst starts to heal.
On CT, the appearance is of a cyst with a high
attenuation wall on unenhanced CT even without
calcification.
MRI appearance is of a cyst with a low signal rim.
On CT/MRI, the wall and septae enhance with
contrast, which can aid in the differentiation between
hydatid cyst and a simple liver cyst.

Management can be either medical (two benzimidazoles


are commonly used, albendazole and mebendazole) or
surgical (cystectomy or partial organ resection).

Hydatid disease.

Differential diagnosis

Complications are of cyst rupture (5090% of cases) and


infection.
The lungs are the second most common site of
involvement in adults and the most common in children.

76

Further reading
Pedrosa I, Saz A, Arrazola J, et al. (2000). Hydatid
disease: radiologic and pathologic features and
complications. Radio Graphics 20: 795817.
Polat P, Kantarci M, Alper F, et al. (2003). Hydatid
disease from head to toe. Radio Graphics 23:
475494.

40b

40b Patient with pulmonary hydatid presented with


several large, well defined nodules in the left lung
with no calcification.

Abdominal Imaging

CASE 41

Case 41

41a

History
A 67-year-old, overweight female
patient, with no past medical history,
presented with vague abdominal pain,
nausea and vomiting.

41b

77

Answer 41

Abdominal Imaging

ANSWER 41
Observations (41a, 41b)
AP abdominal radiograph shows distended loops of gasfilled small bowel but absent colonic gas. Together with
the clinical history, appearances are consistent with small
bowel obstruction. There is no evidence of free gas on
these films but on the second image there is an abnormal
collection of air over the central liver that has a somewhat
linear/branching configuration. This is consistent with air
in the biliary tree. In the right side of pelvis, there is a
round opacity showing peripheral calcification this is likely
to indicate an obstructing gallstone.

Diagnosis
Gallstone ileus.

Discussion
Gallstone ileus is relatively rare, accounting for 12% of all
mechanical obstructions (though more in the elderly). The
most common scenario is of a stone eroding through from
gallbladder to duodenum the cholecystoduodenal fistula
leads to pneumobilia and the stone then impacts in the
small bowel. The fistula can also be from the common duct,
and can extend to the colon or stomach instead of the small
bowel.
Occasionally, the diagnosis can be made on plain films
with Riglers triad of small bowel obstruction, pneumobilia
and ectopic gallstones. Often, however, the gallstone is not
seen on plain film since the stones frequently have a
predominant composition of cholesterol with little

41c

41c Axial CT image shows a calcified gallstone


within the lumen of the distal ileum with dilated
loops of small bowel seen proximally.

78

calcification. An axial CT scan of the abdomen (41c)


confirmed a gallstone ileus with a 5 cm diameter laminated
gallstone found in the distal ileum. The bowel was
collapsed distal to the site of stone impaction. At
laparotomy, the stone was milked back to the jejunum and
removed.

Practical tips
Always check for air in the biliary tree on the small
bowel obstruction abdominal film.
Tiny locules of air in the biliary tree tend to be
centrally located in the liver (41d) compared with
portal vein gas, which is seen in the periphery.
Biliary tree gas can also be seen as a normal finding in
patients who have had a previous sphincterotomy or
following a recent ERCP (endoscopic retrograde
cholangiopancreatography).

Further management
Mechanical small bowel obstruction is a surgical
emergency.

Further reading
Gurleyik G, Gurleyik E (2001). Gallstone ileus:
demographic and clinical criteria supporting
preoperative diagnosis. Ulus Travma Derg 7(1):
3234.
Pangan JC, Estrada R, Rosales R (1984).
Cholecystoduodenocolic fistula with recurrent
gallstone ileus. Archives o f Surgery 119: 12011203.

41d

41d Axial CT image shows central locules of gas


within the biliary tree.

Abdominal Imaging

CASE 42

Cases 42, 43

42

History
A 68-year-old male is admitted with
hepatic encephalopathy.

CASE 43

43

History
An asymptomatic 22-year-old male
presented with deteriorating renal
function.

79

Answers 42, 43

Abdominal Imaging

ANSWER 42
Observations (42)
Image from a double contrast barium meal examination
shows multiple serpiginous filling defects in the lower
oesophagus. Normal appearances of the gastric fundus are
observed. Appearances are consistent with oesophageal
varices and the distribution suggests that these are uphill.

Diagnosis

to IVC/hepatic vein/splenic vein thrombosis or


obstruction.
Downhill varices (found in the mid and upper
oesophagus); these are characterized by collateral
blood flow from the SVC via the azygous vein into
the IVC, and arise due to SVC obstruction from
conditions such as lung tumour, lymphoma and
retrosternal goitre.

Oesophageal varices.

Differential diagnosis
For oesophageal varices:
Varicoid carcinoma of oesophagus.
For gastric varices (i.e. causes of thickened gastric folds):
Hypertrophic gastritis.
Mntriers disease.
Lymphoma.
Splenic vein thrombosis.

Discussion
Oesophageal varices have a very typical appearance on
contrast swallow examination of dilated, smooth,
serpiginous filling defects. Varices collapse in the erect
position and are best imaged with the patient prone. There
are two types:
Uphill varices (found in the lower oesophagus); these
are characterized by collateral blood flow from the
portal vein via the azygous vein to the superior vena
cava (SVC). These arise due to liver cirrhosis and due

Gastric varices are seen in combination with oesophageal


varices in patients with portal hypertension. When seen in
the absence of oesophageal varices, splenic vein thrombosis
should be suspected. Again, appearances are of smooth,
serpiginous or grape-like filling defects; most commonly
seen in the gastric fundus.

Practical tips
Best images are obtained with the patient in a prone
position.
Further investigation with an ultrasound of the
abdomen should be advised to look for cirrhosis and
portal hypertension.

Further management
Treatment is aimed at controlling portal hypertension with
medical and surgical (transjugular intrahepatic
portosystemic shunt TIPS) means. Treatment of bleeding
varices and preemptive treatment of nonbleeding varices is
achieved with endoscopic banding and sclerotherapy.

ANSWER 43
Observations (43)
This plain abdominal radiograph shows multiple small foci
of calcification over both renal areas in the region of the
renal medulla rather than renal cortex. No stones are seen
elsewhere along the course of the renal tracts.

Renal papillary necrosis.


Hypervitaminosis D.
Milk-alkali syndrome.
Malignancy bone metastases, multiple myeloma,
paraneoplastic syndrome.
Primary hyperoxaluria.

Diagnosis
Renal medullary nephrocalcinosis.

Discussion
Medullary nephrocalcinosis represents calcification in the
distal convoluted tubules, i.e. in the renal pyramids. There
are many causes and the underlying pathology can rarely
be determined on a plain radiograph clinical history is far
more important here.
The causes are:
Renal tubular acidosis (RTA).
Endocrine causes hyperparathyroidism,
hyperthyroidism, Cushings.
Medullary sponge kidney.
Idiopathic hypercalcuria.

80

Practical tips
The most common causes of symmetrical medullary
nephrocalcinosis are hyperparathyroidism and RTA.
The most common cause of asymmetrical medullary
nephrocalcinosis is medullary sponge kidney.

Further management
Underlying cause must be identified particularly treatable
causes.

Further reading
Dyer RB, Chen MY, Zagoria RJ (1998). Abnormal
calcifications in the urinary tract. Radio Graphics 18:
14051424.

Abdominal Imaging

CASE 44

Cases 44, 45

44

History
A 34-year-old male presented with
nausea and vomiting.

CASE 45

45a

History
A 68-year-old male presented with
postprandial bloating.

81

Answers 44, 45

Abdominal Imaging

ANSWER 44
Observations (44)
This is a single AP image from a double contrast small
bowel barium examination. There is focal, eccentric
narrowing of the second part of the duodenum, with
predominant notching of the lateral wall. Appearances of
the duodenum superior and inferior to this are completely
normal.

Clinical presentation can be with:


Polyhydramnios in utero .
Persistent vomiting, double bubble in neonates.
Nausea, vomiting and abdominal pain in adults.
The condition is complicated by an increased incidence of
acute and chronic pancreatitis and periampullary peptic
ulcer.

Diagnosis
Annular pancreas.

Differential diagnosis
For annular pancreas:
Sphincter of Oddi oedema (secondary to impacted
stone or pancreatitis)/carcinoma usually produces an
eccentric lesion but this is predominantly medially
located.
Duodenal adenocarcinoma usually presents with an
annular concentric lesion with shouldering and
ulceration. There is an association with Gardners
syndrome and coeliac disease.

Discussion
Annular pancreas is a disorder characterized by failure of
rotation of the ventral bud of the pancreas resulting in
pancreatic tissue encircling the duodenum. The second part
of the duodenum is involved in 85% of cases. Usually
patients are asymptomatic, but the condition can present
at any age, with 48% of cases presenting in adulthood.
There is an association with other congenital abnormalities
when the condition presents in childhood tracheooesophageal atresia, duodenal atresia, imperforate anus,
Downs syndrome.

Practical tips
Carefully examine the film for gallstones or small bowel
features of coeliac disease (small bowel dilatation,
flocculation of contrast, featureless smooth small bowel
lumen/folds, jejunization of ileal loops and poor
peristalsis) to suggest another diagnosis.

Further management
CT will confirm pancreatic tissue encircling the
duodenum.
ERCP (endoscopic retrograde
cholangiopancreatography) or MRCP (magnetic
resonance cholangiopancreatography) shows a
normally located main pancreatic duct in the body of
the pancreas, and a small duct in the head of the
pancreas encircling the duodenum.

Further reading
Rizzo RJ, Szucs RA, Turner MA (1995). Congenital
abnormalities of the pancreas and biliary tree in
adults. Radio Graphics 15(1): 4968.

ANSWER 45
Observations (45a)

Discussion

Single image from a percutaneous cholangiogram is shown.


The percutaneous needle is seen with the tip in a proximal
intrahepatic biliary duct. There is clear abnormality of the
common duct, which has several strictures with duct
dilatation and beading. No filling defects are seen to
indicate gallstones. Contrast is seen in the duodenum with
no obstructing lesion seen at the level of the sphincter of
Oddi.

Primary sclerosing cholangitis is a progressive fibrosing


inflammatory condition affecting both intrahepatic and
extrahepatic bile ducts. The condition is strongly associated
with inflammatory bowel disease (ulcerative colitis [UC]
found in 70%, Crohns in 15%). Other associations include
retroperitoneal and mediastinal fibrosis, chronic active
hepatitis, Riedels thyroiditis, pancreatitis and Sjgrens
syndrome. Presentation is with progressive fatigue, pruritus
and jaundice. Biochemical changes are found with elevated
serum bilirubin and alkaline phosphatase.
Imaging features on cholangiography
(MRCP/ERCP): there are multifocal strictures
affecting intra- and extrahepatic bile ducts with skip
lesions. The classic pattern is of a beaded appearance
with alternating segments of stenosis and dilatation
(45b).
Imaging features on CT: ducts have the appearance of
strictures, dilatation, beading and pruning.
Imaging on US: usually normal but may show duct
wall thickening.

Diagnosis
Primary sclerosing cholangitis (PSC).

Differential diagnosis
For PSC:
Sclerosing cholangiocarcinoma.
Acute ascending cholangitis.
Bile duct carcinoma this can rarely involve the biliary
system in a diffuse manner producing multiple tumour
strictures.

82

Answer 45

Abdominal Imaging

Complications of PSC include:


Gallstones.
Biliary cirrhosis.
Portal hypertension.
Cholangiocarcinoma.

Case 46

45b

Practical tips
MRCP is now the noninvasive diagnostic investigation of
choice.

Further management
The only curative treatment for this condition is liver
transplantation. Medical palliative care involves treatment
of the symptoms of cirrhosis, portal hypertension, chronic
cholestasis (pruritus and malabsorption) and ductal
complications such as strictures and ascending cholangitis.
A multidisciplinary approach is therefore adopted requiring
hepatologists, transplant surgeons and interventional
radiologists.
45b Single image from an ERCP examination
showing multiple strictures with poststenotic
dilatation giving a beaded appearance.

CASE 46

46a

History
A 67-year-old male presented with
abdominal pain and vomiting.

83

Answer 46

Abdominal Imaging

ANSWER 46
Observations (46a)
This plain abdominal radiograph shows a large, ahaustral
gas-filled viscus arising from the left side of the pelvis and
extending into the upper abdomen. The loop is projected
over the left side of pelvis and descending colon with its
apex under the left hemidiaphragm. The medial walls of
the loop form a summation line. There are several dilated
loops of descending colon evident with absence of gas in
the rectum. The features are typical of sigmoid volvulus and
there is no free intraperitoneal gas seen to indicate
perforation.

Diagnosis
Sigmoid volvulus.

valve and can usually be differentiated from sigmoid


volvulus by several features:
Caecal volvulus usually occurs in a younger age
group: 3050 years.
Dilated obstructed caecum often dilates to fill the left
upper quadrant (although in many cases vertical
rotation occurs with caecum still filling the right iliac
fossa). The main axis will be opposite that of sigmoid
volvulus however, extending from the right iliac fossa
towards the left upper quadrant.
Some haustral markings are still evident, unlike
sigmoid volvulus.
There may well be small bowel dilatation but the rest
of the colon will be undilated, unlike sigmoid
volvulus.

Differential diagnosis
For large bowel obstruction:
Colonic malignancy.
Inflammatory strictures: Crohns, ischaemia,
diverticulitis.
Volvulus.
Infectious processes: TB, amoebiasis.
Extrinsic lesions: abscess, bladder/prostate/uterine
tumour, endometriosis.

Further management
Urgent surgical referral with a view to insertion of a flatus
tube to decompress the bowel.

Further reading
Burrell HC, Baker DM, Wardrop P, Evans AJ (1994).
Significant plain film findings in sigmoid volvulus.
Clinical Radio lo gy 49: 317319.

Discussion
Volvulus account for ~10% of large bowel obstructions in
the UK, the most common type being sigmoid volvulus.
This occurs more commonly in the elderly. The twisting of
the sigmoid colon on its mesenteric axis is usually a chronic
problem with superimposed acute episodes, and represents
a closed loop obstruction. Radiologically, the features are
of large bowel obstruction with a markedly dilated loop of
colon seen arising from the left iliac fossa. The volvulus is
characterized by an ahaustral inverted U-shaped loop of
colon. The medial walls produce a summation line and
together with the lines of the lateral walls create the classic
coffee bean appearance. Several radiological features have
been documented as typical, though the most specific are:
Apex of the loop under the left hemidiaphragm.
Inferior convergence of the loop in the left side of the
pelvis the main axis of the loop therefore extends
from left iliac fossa towards right upper quadrant.
Left flank overlap sign loop overlaps descending
colon.
Medial wall summation line.

46b

Other features described include liver overlap and pelvic


overlap signs (where the loop overlaps liver and left iliac
bone, respectively); apex of loop above T10; an air to fluid
ratio >2:1.
Diagnostic confusion can be resolved with a barium
enema examination. This demonstrates a smooth, tapered
beak-like end of the barium column termed the birds
beak sign. Treatment involves the placing of a rectal flatus
tube.

Practical tips
Caecal volvulus (46b) can sometimes be a confusing
differential. It represents twisting just above the ileocaecal

84

46b Caecal volvulus with a dilated caecum


extending up into the right upper quadrant. Small
bowel is dilated secondary to this obstruction but
large bowel is collapsed helping to differentiate
caecal from sigmoid volvulus.

Abdominal Imaging

CASE 47

Cases 47, 48

47a

History
A 35-year-old female presented with
abdominal pain and vomiting.

CASE 48

48a

History
A 38-year-old woman underwent
contrast enhanced CT for further
evaluation of a lesion noted on
ultrasound.
(see page 88 fo r case answer)

85

Answer 47

Abdominal Imaging

ANSWER 47
Observations (47a)
Multiple dilated loops of gas-filled small bowel that
measure more than 3 cm in diameter are seen within the
central abdomen. No gas is seen within the large bowel and
appearances are consistent with small bowel obstruction.
Surgical clips are noted in the right side of pelvis along with
cholecystectomy clips in the right upper quadrant.
Adhesions are therefore the most likely cause of the
obstruction.

Small bowel can be differentiated from large bowel using


the following features:
Presence of valvulae conniventes which extend across
the width of the bowel. Colonic haustra do not
traverse the whole lumen.
Dilated bowel located in the central abdomen rather
than the periphery (47d).
Diameter of loops is <5 cm.
Absence of solid faeces.

Diagnosis

Practical tips

Small bowel obstruction from adhesions.

Always check the film to try to identify the underlying


cause of the obstruction:
Check hernial orifices at the groin there should be
no bowel gas extending below the position of the
inguinal ligament (a line from public tubercle to
anterior-superior iliac spine) (47e, 47f, 47g, 47h).
Look for evidence of previous surgery, as in this case.
Look for air in the biliary tree and radio-opaque
gallstones outside the territory of the gallbladder as
indicators of gallstone ileus.
Examine bones for metastatic lesions, which can point
to malignancy.
Always remember to check for free gas secondary to
perforation!

Differential diagnosis
For small bowel obstruction:
Adhesions account for up to 60% of small bowel
obstructions.
Hernia.
Gallstone ileus.
Small bowel or caecal malignancy.
Intussusception.
Malrotation and volvulus.

Discussion
Small bowel obstruction can have a varied presentation on
plain abdominal radiography. The classical appearance is of
central abdominal small bowel loops dilated to >3 cm in
diameter with a paucity of gas in the large bowel. Other
appearances can be of:
String of beads sign due to small air-fluid levels in
fluid-filled obstructed loops of small bowel (47b).
Absence of gas in the small bowel due to complete
obstruction and complete fluid filling of loops (47c).

Further management
CT is the investigation of choice for small bowel
obstruction and it can identify both the site and cause of
obstruction and also the complications. Mechanical small
bowel obstruction is a surgical emergency.

47b
47c

47b Plain abdominal radiograph


demonstrating the string of
beads sign.

86

47c Plain abdominal radiograph


demonstrating complete absence of
small bowel gas due to fluid filling.

Answer 47

Abdominal Imaging

47d

47e

47e Axial CT image shows that there has been a


previous attempted hernia repair with a mesh
noted in situ. There has, however, been a
recurrence with dilated bowel going into the
hernia and completely collapsed bowel emerging
from it. This shows that this is the level of
obstruction.
47d Classic distribution of dilated
small bowel: dilated loops are
centrally located within the
abdomen.

47f

47g

47f Pelvis radiograph shows loops of small bowel


below the inguinal ligament.
47g Coronal CT reformatted image in
the same patient demonstrates a right
sided inguinal hernia.

47h

47h Axial CT image of a patient with small


bowel obstruction.

87

Answer 48

Abdominal Imaging

ANSWER 48
Observations (48a)
A large central mass lesion is demonstrated in the left
kidney. This is slightly heterogeneous but has a predominantly fatty density. Appearances are consistent with left
renal angiomyolipoma.

Diagnosis

Appearance on CT: again, appearance is of a well


defined fat-containing lesion with some areas of
higher attenuation tissue. Identification of fat (HU
<20) within a renal lesion is highly specific for an
angiomyolipoma.
Appearances on MRI: a fat suppression sequence can
be very useful in confirming intralesional fat content.

Angiomyolipoma.

Discussion
Angiomyolipoma is a benign lesion containing fat, blood
vessels and smooth muscle. They tend to present in two
groups of patients:
Women in their 4th7th decades where lesions arise
spontaneously and tend to be so litary and unilateral.
Young patients with tuberous sclerosis where multiple
and bilateral lesions are seen in up to 75% of patients
(48b).
They are also seen rarely in autosomal dominant polycystic
kidney disease (ADPKD) and neurofibromatosis.
Appearance on US: typical appearance is of a well
defined echobright lesion due to a high fat content
(48c). There can be a variable degree of reduced
echogenicity depending on the amount of smooth
muscle and/or haemorrhage.

48b

The main complication of these lesions is haemorrhage and


the risk is related to size. Lesions greater than 4 cm in
diameter have a risk of spontaneous bleeding of
approximately 50%.

Practical tips
Identification of fat in a renal lesion is very specific for
angiomyolipoma.

Further management
Small lesions (<4 cm) are usually asymptomatic but lesions
>4 cm are almost always symptomatic with pain and a risk
of haemorrhage. Lesion resection or nephrectomy should
be considered in these patients. Transcatheter arterial
embolization is an alternative.

48c

48b Single axial CT image demonstrating multiple


low-attenuation lesions in both kidneys in a patient
with tuberous sclerosis.

48c US appearances of angiomyolipomas, which


appear as well defined hyperechoic lesions.

88

Abdominal Imaging

Case 49

CASE 49
History
A 30-year-old male presented with
abdominal swelling over several months.

49a

49a Selected images from a recent contrast enhanced CT study.

49b A barium enema done on


the same patient 3 years
previously.

49b

89

Answer 49

Abdominal Imaging

ANSWER 49
Observations (49a, 49b)
The two CT images (49a) demonstrate a large, well defined
soft tissue mass in the central abdomen. This displaces
adjacent bowel loops and most likely originates in the
mesentery.
The single image (49b) from a double contrast barium
enema examination shows multiple small, well defined
mucosal filling defects throughout the colon consistent
with widespread colonic polyposis. Multiple polyps
throughout the colon suggest an underlying genetic
condition.

Diagnosis
Familial adenomatous polyposis (FAP) with mesenteric
desmoid tumour.

Discussion
Familial adenomatous polyposis is an autosomal dominant
disease (chromosome 5) characterized by multiple colonic
adenomatous polyps that inevitably progress to colorectal
cancer within 20 years of diagnosis. Treatment involves
prophylactic total colectomy in early adult life and genetic
screening of family members from the second decade with
a view to prophylactic surgery. All patients have colonic
polyps but small bowel and gastric adenomas are also found
(periampullary cancer is the most common cause of death
once colectomy has been performed).
Other associated features include:
Desmoid tumours.
Mesenteric fibrosis.
Gastric hamartomas.
Hypertrophy of retinal pigment epithelium.
Gardner and Turcot syndromes are variants of FAP.
Gardner syndrome also includes:
Osteomas of the skull and mandible.
Dental abnormalities dentigerous cysts, odontoma,
hypercementoma, supernumerary teeth.
Soft tissue tumours such as fibroma, lipoma,
leiomyoma, neurofibroma.
Epidermal cysts.
Association with thyroid cancer.

49b

90

49b Multiple
polyps.

Turcot syndrome is FAP with associated CNS malignancy


such as medulloblastoma.
Other polyposis conditions are:
PeutzJeghers autosomal dominant (AD)
hamartomatous polyposis condition with features
of mucocutaneous pigmentation (usually brown
pigmented freckling on the mucous membranes
of lips and gums) and multiple polyps found
predominantly in the stomach and small bowel, with a
few also seen in the large bowel. Hamartomas have no
malignant potential but the condition is associated
with an increased risk of upper GI tract malignancies.
Complications of the condition include:
Malabsorption.
Transient intussusception.
Carcinoma of the GI tract.
Carcinoma of breast, pancreas, ovary, endometrium
and testes.
Cowdens syndrome AD condition characterized by
multiple hamartomatous polyps, breast and thyroid
malignancy and skin lesions.
CronkhiteCanada syndrome hamartomatous
colonic polyps are associated with alopecia, skin
pigmentation and nail atrophy.

Practical tips
Images should be carefully inspected for a coexistent
colonic tumour as well as extracolonic malignancies.
Intussusception in adults indicates an underlying
bowel pathology, whereas in children it can be
idiopathic.

Further management
Surgical referral is required for prophylactic colectomy, as
is referral for genetic screening of relatives.

Further reading
Galiatsatos P, Foulkes WD (2006). Familial
adenomatous polyposis. American Jo urnal o f
Gastro entero lo gy 101(2): 385398.

Abdominal Imaging

Cases 50, 51

50a

CASE 50
History
A 43-year-old male presented with
abnormal liver function tests.

CASE 51
History
A 19-year-old male presented with
multiple episodes of renal colic.

51a

51b

91

Answers 50, 51

Abdominal Imaging

ANSWER 50
Observations (50a)

Practical tips

This single T1 weighted coronal image shows a striking


reduction in signal intensity throughout the liver
parenchyma. This is likely to indicate iron overload. It is
notable that the spleen retains normal signal intensity so
the liver abnormality is most likely due to haemochromatosis.

Multiple transfusions for chronic haematological disorders


can lead to iron overload, i.e. transfusion siderosis. MRI
will show hypointensity in the liver and spleen in this
condition. This helps differentiate it from haemochromatosis where the spleen shows normal signal on
MRI.

Diagnosis

Further management

Haemochromatosis.

In the course of follow-up, ultrasound monitoring may be


useful due to the high risk of developing cirrhosis and
hepatocellular carcinoma.

Discussion
Primary haemochromatosis is an autosomal recessive
condition characterized by increased absorption and
deposition of iron within several organs including liver,
pancreas, heart and pituitary gland. Patients are usually
asymptomatic until the 2nd decade, then they present with
a varied clinical picture due to iron deposition in:
Skin hyperpigmentation.
Liver cirrhosis, hepatomegaly.
Pancreas diabetes.
Heart arrhythmias, dilated cardiomyopathy.
Musculoskeletal arthralgia.
Pituitary pituitary failure with signs of impotence,
testicular atrophy, hair loss.
Radiologically, imaging of the abdomen shows marked
abnormality of the liver. Accumulation of iron results in the
liver being of diffusely reduced signal on MRI. The degree
of iron deposition has been shown to correlate with the
MRI appearances. Unenhanced CT of the liver
demonstrates increased attenuation (>75 HU). Follow-up
in these patients is important due to the hepatic
complications of cirrhosis and the increased risk of
hepatocellular carcinoma.
Arthropathy of haemochromatosis is similar to that of
calcium pyrophosphate deposition disease. Chondrocalcinosis is a feature. Typically, appearances are of squaring
of the metacarpal heads due to flattening and peripheral
small, hook-like spurs (50b). Osteopenia is also common.

50b

50b Radiograph of both hands shows flattening of


the 2nd and 3rd metacarpal heads with loss of joint
space, giving a squared appearance. Early spur
formation is seen on the lateral aspect of the 2nd
metacarpal of left hand.

ANSWER 51
Observations (51a, 51b)

Discussion

Images are control and delayed prone abdominal


radiographs from an IVU series. The control film shows
medially located appearance of both kidneys and although
the superior poles of both are identifiable, inferior poles
are not. The delayed IVU image (51b) shows medial
location of the pelvicalyceal systems, which are anteriorly
orientated. No filling defects are identified.

This is the most common fusion abnormality of the


kidneys. It is more commonly found in male patients and
has an incidence of 1 in 300. The kidneys are joined at
their lower poles in 90% of cases, by a parenchymal/fibrous
isthmus band (51c). The long axis of the kidneys is
medially orientated with anterior rotation, such that the
renal pelvises are anteriorly located. The condition is
complicated by urinary stasis with renal stone formation,
infection and reflux. Vesicoureteric reflux and
hydronephrosis secondary to ureteropelvic junction
obstruction are common. There is a reported increase in

Diagnosis
Horseshoe kidney.

92

Answer 51

Abdominal Imaging

the incidence of renal adenocarcinoma, transitional cell


carcinoma and Wilms tumour. In addition the kidney is
more susceptible to injury following abdominal trauma.
Horseshoe kidney is associated with:
Genitourinary abnormalities: hypospadia,
cryptorchidism, ureteral duplication, bicornuate
uterus.
Cardiovascular abnormalities.
Skeletal abnormalities.
CNS abnormalities.
Trisomy 18.
Turners syndrome.

Case 52

51c

Practical tips
On an IVU, check for filling defects, which could
represent a renal calculus or transitional cell
carcinoma.
On US, look for a renal cell carcinoma.
On plain radiography, look for signs of bony
metastases anal stones.

51c Axial CT image demonstrates the horseshoe


kidney with a narrow isthmus of tissue extending
anterior to the aorta and IVC.

Further management
Follow-up may be considered due to increased incidence
of renal malignancy.

CASE 52

52a

History
A 64-year-old male with weight loss.

93

Answer 52

Abdominal Imaging

ANSWER 52
Observations (52a)

Practical tips

Single AP radiograph of the abdomen shows a metallic


stent within the central pelvis, which presumably lies in the
rectum or distal sigmoid colon. It is likely that this stent
has been inserted to relieve symptoms from a colorectal
tumour. The rest of the bowel gas pattern is unremarkable
with no evidence of obstruction.
There are, however, amorphous, poorly marginated
areas of calcification seen in the region of the liver and these
likely represent calcified liver metastases. Ultrasound or CT
should be undertaken, and a contrast enhanced CT (52b)
of this patient does confirm the presence of calcified liver
metastases.

As with all radiographs that show evidence of likely


primary malignancy, once this has been noted, look
carefully for metastatic disease elsewhere on the film.
Classically, colorectal cancers metastasize to the liver
due to the venous drainage of bowel via the portal
venous system. However, the venous drainage of the
rectum interfaces with the systemic venous drainage at
the anal canal and thus pulmonary metastases are said
to be more likely in rectal cancer than other colonic
tumours. In reality, pulmonary metastases are not an
uncommon finding in colon or rectal cancer.

Further management
Diagnosis
Stented rectal tumour with calcified liver metastases.

Differential diagnosis
For calcified liver metastases:
Mucinous adenocarcinomas colon, rectum, ovarian,
breast and stomach.
Osteosarcoma.
Endocrine pancreatic carcinoma.
Medullary carcinoma of thyroid.
Lung cancer.

Discussion
Colorectal carcinoma is the third most common cancer
diagnosed in the developed world. Rectum and sigmoid are
the most common sites of lesions. Where surgical resection
is not possible or appropriate, stents can provide
symptomatic relief and prevent obstruction.
The liver is the most common site for metastatic spread
after regional lymph nodes.

52b

52b Axial CT image confirms calcified liver


metastases.

94

TNM (tumournodemetastases) staging must be


accomplished as for most tumours. Tumour staging of
rectal cancer is done with MRI (52c) (along with local
nodal staging). The primary reason for MRI is to assess
proximity of tumour to the mesorectal fascia this is the
plane along which the surgeon dissects in a total mesorectal
excision (TME) procedure. This boundary is thus referred
to as the circumferential resection margin (CRM). If local
tumour spread extends close to it, the surgical margin may
well be contaminated with tumour with the attendant risk
of local recurrence. Identifying patients where the CRM is
threatened in this way means they can be selected for
preoperative radiotherapy or chemotherapy to reduce this
risk. Distal nodal disease and metastases can be assessed
with CT or MRI but if the chest is also to be imaged to
exclude pulmonary metastases, CT is required for this
component at least.

52c

52c Thin section T2 weighted image shows a


circumferential rectal tumour with wall breach at
the left anterolateral wall (122 oclock) consistent
with this being a T3 tumour.

Abdominal Imaging

CASE 53

Case 53

53a

History
A 35-year-old female presented with
abdominal pain and per rectal bleeding
for 3 months.

53b

95

Answer 53

Abdominal Imaging

ANSWER 53
Observations (53a, 53b)
Plain abdominal film of adult patient shows marked wall
thickening of the transverse colon with thumb-printing.
The sigmoid loops show no such abnormalities and the
rectum contains faeces. The double contrast barium enema
film confirms extensive mucosal ulceration and a somewhat
cobblestone appearance that extends from caecum to the
descending colon. The colon distal to this is normal. The
appearances are in keeping with colitis, and sparing of the
more distal colon makes Crohns disease more likely than
ulcerative colitis. It is notable that the sacroiliac joints are
normal.

Diagnosis
Crohns disease.

Differential diagnosis
For terminal ileal disease:
TB usually has more severe involvement of the
caecum (53c). There is often evidence of pulmonary
TB.
Radiation ileitis.
Yersinia.

53c

53c TB can also affect the bowel and appearances


can mimic those of Crohns disease. Caecal
involvement with features of stricturing and
ulceration is more common than terminal ileal
involvement.

96

For thumb-printing:
Inflammatory colitis Crohns, ulcerative colitis
(UC).
Ischaemic colitis.
Infectious colitis/pseudomembranous colitis.
Diverticulitis.
Other causes: endometriosis, amyloidosis, hereditary
angioneurotic oedema, lymphoma.

Discussion
Crohns disease is a chronic, inflammatory, granulomatous
disease that can affect any part of the bowel from
oesophagus to rectum. Small bowel is most commonly
involved and the terminal ileum is involved in over 95% of
cases (53d). Presentation is usually in the 2nd4th decades
with symptoms of abdominal pain, diarrhoea, per rectal
bleeding, weight loss and features of malabsorption.
Radiological features are:
Aphthous ulcers shallow ulcers with surrounding
oedema.
Fissures, sinuses and fistulae Crohns is the third
most common cause of fistulae after idiopathic causes
and diverticulitis. The fistulae can be between loops

53d

53d Decubitus film from a barium enema series


shows marked narrowing of the terminal ileum in a
patient with Crohns disease.

Answer 53

Abdominal Imaging

of bowel, between inflamed bowel loops and other


abdominal viscera, e.g. colovesical or from bowel to
the skin.
Cobblestone mucosa longitudinal and transverse
ulcers separated by oedematous mucosa.
Thickening of small bowel folds.
Separation of small bowel loops due to inflammation
and oedema of wall.
Mucosal granularity with <1 mm rounded mucosal
lesions.
Pseudopolyps inflammatory or hyperplastic mucosa.
Strictures often multiple.
Skip lesions with discontinuous disease are seen in
>90% of cases.
CT imaging features (53e) are demonstrated in this case of
a young male patient who has had a previous terminal ileal

resection for Crohns disease. There is now recurrence of


disease in the neoterminal ileum with features of:
Thickening of the bowel wall.
Marked stranding of the surrounding fat due to
inflammation.
Engorged and dilated mesenteric vessels referred to as
the comb sign due to the similarity in appearance to
the teeth of a comb.
Skip lesions with two involved segments shown on this
single axial image.
MR imaging (small bowel enterography) features are
demonstrated in Figures 53f53i. The fat suppressed
coronal/axial images (53f, 53g) show bowel wall
thickening in the proximal ileum.
(co nt.)

53e

53e CT image shows recurrence of disease in the


neoterminal ileum post surgical resection. There is
small bowel wall thickening and oedema with
inflammatory change in the surrounding tissues.

53f

53g

53f (coronal), 53g (axial) fat suppressed images


showing bowel wall thickening of a loop of proximal
ileum. Dynamic images are obtained and viewed in
cine mode to see how this focus of bowel contracts.

97

Answer 53

Abdominal Imaging

The pre- and post-contrast T1 weighted coronal images


(53h, 53i) show enhancement of an involved loop of small
bowel in the central lower abdomen.
Treatment is both medical and surgical, with a high rate
of recurrence even after resection (almost 40%), particularly
in the neoterminal ileum following distal ileal resections.

of disease on the film. Check for sacroiliac joint


disease, gallstones, hypertrophic osteoarthropathy.
Always look for signs of drug treatment of disease on
the film. Check femoral heads for evidence of
avascular necrosis from steroid treatment.

Further management
Practical tips
Terminal ileal involvement, skip lesions and multiple
strictures are the best signs for Crohns disease. When
assessing colonic disease, remember that UC almost
always involves the rectum and has a continuous
distribution without skip lesions. However, if a UC
patient has had steroid enemas, the rectum may look
normal.
Always look for signs of complications of Crohns, i.e.
adenocarcinoma (risk increased up to 20-fold),
lymphoma, toxic megacolon, perforation, abscess,
fistulae.
Always look for signs of extraintestinal manifestations

53h

53h (pre-contrast), 53i (post-contrast) coronal


images of the colon showing enhancement in the
thickened small bowel loops in the lower abdomen.

98

Initial diagnosis in suspected cases is often confirmed


with a small bowel barium study (follow through or
enteroclysis). Sometimes a more acute presentation
with abdominal pains may lead to the diagnosis first
being suggested by CT. Capsule endoscopy is a newer
investigation that may also first identify disease. Once
the diagnosis is made, follow-up imaging with MRI is
ideal as this incurs no radiation risk in what are
frequently young patients. CT imaging remains
equally useful, however, when complications such as
abscess formation are suspected.
Treatment includes medical and surgical disciplines.

53i

Abdominal Imaging

CASE 54

Cases 54, 55

54

History
A 54-year-old male presented
with dyspepsia.

CASE 55

55

History
A 29-year-old male presented with
progressive dysphagia.

99

Answers 54, 55

Abdominal Imaging

ANSWER 54
Observations (54)

Discussion

Single image from a double contrast barium meal


examination shows multiple, small dense foci of contrast
within the antrum and body of the stomach. These are
surrounded by a lucent halo representing oedema. There
is some irregular thickening of the gastric folds, with the
target lesions appearing to be orientated along these.

Gastritis often has this aphthoid appearance with


varioliform ulcers, consisting of a tiny dense focus of
barium surrounded by a radiolucent halo of oedema giving
a target lesion appearance. Lesions are usually multiple.
The antrum is preferentially affected with spread towards
the fundus, lesions appearing to be longitudinally
orientated along the rugal folds.
Causes:
In 50% of cases, no causative abnormality is identified.
Peptic disease.
Drugs aspirin, NSAIDs, steroids.
Alcohol.
Infection herpes simplex, cytomegalovirus (CMV),
Candida.
Crohns disease usually there are signs of Crohns
disease in other locations, most commonly the
terminal ileum.

Diagnosis
Erosive gastritis.

Differential diagnosis
For aphthous ulceration:
Erosive gastritis.
Crohns disease.
Barium precipitate artefacts.
For gastric fold thickening:
Erosive gastritis.
ZollingerEllison syndrome.
Crohns disease.
Malignancy lymphoma, carcinoma.
Benign reactive lymphoid hyperplasia.
Mntriers disease.

Practical tips
On the contrast examination look for features in the
oesophagus of an infective cause for the gastric appearances
or for oesophageal varices pointing to alcohol as an
underlying cause.

Further management
Gastroenterology referral with a view to endoscopy.

ANSWER 55
Observations (55)
Single image from a barium swallow examination shows a
lesion in the lower oesophagus, just superior to the gastrooesophageal junction. The lesion is well defined with a
smooth edge, indenting the oesophageal lumen. No
ulceration or infiltration is seen. Appearances suggest a
benign intramural mass, most likely a leiomyoma.

Diagnosis
Leiomyoma of the oesophagus.

Differential diagnosis
For smooth oesophageal mass lesion:
Neurofibroma.
Lipoma.
Haematoma, e.g. from instrumentation.
Duplication cyst can simulate an intramural mass.

presentation is with dysphagia, odynophagia and possibly


haematemesis.
Radiological features are:
Well defined, large, smooth intramural mass
extending into the oesophageal lumen.
Forms an obtuse angle with the adjacent mucosa
good sign of a benign lesion.
Usually found in the mid and lower oesophagus.
Calcification is sometimes seen and is virtually
diagnostic since it is the only oesophageal tumour to
calcify.
Ulceration is rare.
May be multiple in ~3%.
Uniform contrast enhancement on CT.

Practical tips
Smooth, well defined, slow growing oesophageal lesions
are likely to be benign.

Discussion
This is the most common benign tumour of the
oesophagus. It is usually found in young adults, being
slightly more common in males. Growth is slow and

100

Further management
Gastroenterology referral with a view to direct visualization
with endoscopy.

Abdominal Imaging

Case 56

CASE 56
History
A 24-year-old male patient presented with dysuria.

56a

56b

101

Answer 56

Abdominal Imaging

ANSWER 56
Observations (56a, 56b)
The AP postmicturition image (56b) from an IVU series
shows bilateral dilatation of the distal ureter with a cobra
head appearance. There is a surrounding thick halo of
lucency within the bladder, representing oedema. These
appearances are of bilateral ureteroceles. The control film
(56a) demonstrates bilateral calculi in the pelvis that lie
within these ureteroceles.

Diagnosis
Bilateral ureteroceles.

Differential diagnosis
For radiolucent bladder filling defects on IVU:
Ureterocele.
Bladder tumour.
Radiolucent calculus.

56c

Sloughed renal papilla.


Gas secondary to fistula, cystitis, idiopathic causes and
trauma.
Island prostate enlarged central zone can appear as a
central bladder lucency.

Discussion
A simple or orthotopic ureterocele is a congenital prolapse
of the distal ureter and its orifice into the bladder. It is
usually an incidental finding in adults and is bilateral in a
third of cases. Figure 56c shows how a ureterocele can
produce a less specific type of filling defect when the
bladder is full and Figure 56d shows the typical ultrasound
appearance. The main complication is of obstruction,
which can cause collecting system dilatation and renal
failure. There is also an increased risk of stone formation
and it is therefore imperative to check the control film in
every patient as in this case!

56d

56d US image of the bladder shows protrusion of


the distal ureter into the bladder, indicating that
these are orthotopic ureteroceles and not
pseudoureteroceles.

56c IVU film in a patient with a full bladder showing


a radiolucent filling defect.

102

Answer 56

Abdominal Imaging

Pseudoureteroceles can have a similar appearance and


are caused by obstruction of a normal ureter. Differentiation between the two types can be made using
ultrasound or oblique films, which show no protrusion of
the ureter into the bladder lumen with pseudoureteroceles.
Causes of pseudoureteroceles include:
Oedema of the distal ureter secondary to impacted
stone, infection, radiotherapy.
Bladder tumour (56e).

Case 57

56e

Practical tips
Always remember to check the control film of an IVU.

Further management
While a small asymptomatic ureterocele may not require
treatment, recurrent urinary tract infection (UTI), calculi,
pain and obstructive uropathy are indications for surgical
intervention.
56e Single axial T2 weighted image of the pelvis
shows a pseudoureterocele secondary to a large
bladder tumour.

CASE 57

57a

History
A 36-year-old male presented with
progressive dysphagia.

103

Answer 57

Abdominal Imaging

ANSWER 57
Observations (57a)
This frontal chest radiograph of an adult patient shows an
added convex soft tissue density along the right mediastinal
border and behind the heart. There is no normal gastric
air bubble beneath the left hemidiaphragm. The findings
suggest dilatation of the oesophagus secondary to chronic
distal obstruction, most likely due to achalasia. A barium
swallow would confirm.

Diagnosis
Achalasia.

Differential diagnosis
Secondary achalasia due to a stricture at the gastrooesophageal junction. There will be normal peristalsis
however.
Chagas disease is essentially the same as achalasia but
the neurenteric plexus damage is due to Trypano so ma
cruzi infection.

Presentation is with progressive dysphagia. Investigation


is with a contrast swallow examination (57b), which may
show features of:
Vigorous achalasia multiple tertiary contractions in
the distal oesophagus can be the earliest sign.
Birds beak deformity symmetrical stenotic
segment of oesophagus at the gastro-oesophageal
junction. Imaging the patient erect allows best
demonstration with contrast forcing its way through
the gastro-oesophageal sphincter when the
hydrostatic pressure of the barium column in the
oesophagus is sufficiently high.
Mega-oesophagus dilatation of the oesophagus that
can involve its entire length.
Relaxation of the lower oesophageal sphincter can be
induced with amyl nitrate inhalation.
Achalasia is complicated by an increased risk of developing
an oesophageal squamous cell carcinoma.

Discussion

Practical tips

Achalasia is a motility disorder that is idiopathic in


aetiology, characterized by degeneration of Auerbachs
plexus. This results in a failure of relaxation of the caudal
oesophagus at the gastro-oesophageal sphincter.

Check lungs for pulmonary changes of previous


aspiration.
Check for evidence of pulmonary metastases from
secondary oesophageal malignancy (or indeed if the
dilated oesophagus is due to a primary malignancy
rather than achalasia).

57b

Further management
The condition is further investigated with pressure
measurements (manometry) and endoscopy.
Treatment is most commonly by pneumatic dilatation
of the gastro-oesophageal sphincter. Surgical
myotomy is considered where there is disease
recurrence.

57b Two images from a barium swallow


examination show a dilated oesophagus containing
food debris, which narrows down to form the
classical birds beak appearance at the gastrooesophageal junction.

104

Abdominal Imaging

Case 58

CASE 58
History
An 18-year-old male presented with
headaches.

58a

58a, 58b, 58c IVU films taken at 0, 5


and 10 minutes following contrast
injection.

58b

58c

105

Answer 58

Abdominal Imaging

ANSWER 58
Observations (58a, 58b, 58c)
Three images from an IVU series are provided but no
control film (which would normally be assessed prior to
interpretation of the post-contrast films). Images show
unilateral increasingly dense and persistent nephrogram on
the right. The right kidney shows uniform smooth
reduction in size when compared to the left. There is
delayed excretion of contrast by the right kidney on the 10
minute film (58c). These appearances suggest unilateral
right renal artery stenosis. The young age of the patient
makes fibromuscular dysplasia more likely than
atherosclerosis as the underlying pathology.

Diagnosis
Renal artery stenosis.

Differential diagnosis
For persistent dense nephrogram:
Unilateral:
Obstruction acute obstruction is the most
common cause of this sign.
Renal artery stenosis/ischaemia.
Renal vein thrombosis.
Acute bacterial pyelonephritis.
Acute papillary necrosis.
Bilateral:
Hypotension/shock.
Acute tubular necrosis.
Acute glomerulonephritis.
Causes of unilateral change involving both kidneys.

Fibromuscular dysplasia (1020%) usually in the


mid and distal renal artery; affects young adults and
children; more common in women; bilateral in twothirds.
Less common causes include vasculitis, arterial dissection
and thromboembolic disease.
Hypertension in neurofibromatosis is a consequence of
phaeochromocytoma and/or renal artery stenosis, with a
smooth stenosis seen in proximal renal artery.
In this patient, a renal angiogram was performed and
showed a smooth narrow stenosis in the mid portion of
the right renal artery (58d) and this would be in keeping
with the underlying diagnosis of fibromuscular dysplasia
of the renal artery.
The consequence of renal artery stenosis is systemic
hypertension as a result of overactivity of the renin
angiotensin system. Treatment is aimed at medically
controlling hypertension and with renal artery angioplasty.

Practical tips
If there is bilateral delayed persistent nephrogram
with absent or decreased excretion, then the patient
needs to be immediately checked to ensure that
contrast anaphylactic shock has not occurred.
Magnetic resonance or CT angiography is the
preferred investigation for this condition in the
modern era.

Further management
Attempted renal artery angiography and angioplasty are
usually advised.

Discussion
There are two main causes of renal artery stenosis:
Atherosclerosis (8090%) usually in the proximal
2 cm of the renal artery; affects older population >50
years; more common in men; bilateral in one-third.

58d

58d Single image from a renal angiogram showing a


stenosis in the mid portion of the right renal artery.

106

Abdominal Imaging

CASE 59

Case 59

59a

History
A 37-year-old female presented with
acute abdominal pain.

59b

107

Answer 59

Abdominal Imaging

ANSWER 59
Observations (59a, 59b)
Single image from a barium follow-through examination
shows smooth thickened small bowel folds. Selected axial
images from an IV contrast enhanced CT scan of the
abdomen again demonstrates smooth thickening of bowel
folds in dilated loops of fluid-filled small bowel. In addition,
there is thrombus seen in the mid superior mesenteric vein.

Diagnosis
Small bowel ischaemia secondary to superior mesenteric
vein (SMV) thrombosis.

Differential diagnosis
For smooth thickened folds:
Haemorrhage.
Ischaemia:
Acute embolus, HenochSchnlein purpura
(HSP).
Chronic vasculitis, thromboangiitis obliterans,
radiotherapy.
Oedema:
Hypoproteinaemia cirrhosis, nephrotic syndrome,
protein-losing enteropathy.
Angioneurotic oedema.
Lymphatic obstruction lymphoma, mesenteric
fibrosis, intestinal lymphangiectasia.

Discussion
Small bowel/mesenteric ischaemia can present very acutely
with symptoms of acute abdominal pain, vomiting,
diarrhoea and rectal bleeding. This is usually due to arterial
thrombus, dissection or acute venous obstruction. Chronic
ischaemia usually due to chronic arterial thrombus has a

more indolent symptomatology with longstanding


grumbling abdominal cramps, postprandial pain, weight
loss and malabsorption. Other causes of ischaemia include
vasculitis, bowel obstruction, radiotherapy and acute
abdominal inflammation, e.g. pancreatitis, appendicitis.
Radiological features of acute small bowel ischaemia
include:
Bowel wall thickening/oedema.
Enhancement pattern of the bowel wall can be
increased or decreased (59c).
Stranding of the surrounding fat.
Pneumatosis intestinalis, which is a late sign and
indicative of necrotic bowel.
Free intra-abdominal gas due to perforation of
necrosed bowel.
Portal venous gas is seen in premorbid patients.
In chronic ischaemia, there can be additional complications
of strictures (which tend to be long, smooth and
symmetrical), ulceration and atrophic valvulae conniventes.
The splenic flexure and the proximal descending colon are
most commonly affected due to the transition from
superior to inferior mesenteric arterial supply at this point.

Practical tips
Look carefully for linear gas shadows within the
bowel wall indicative of intramural gas.
Portal vein gas is usually seen in the periphery of the
liver as well as centrally, in contrast to biliary gas,
which is usually only central. This is a premorbid sign
in adults.

Further management
Investigation with CT can both diagnose and identify the
complications of ischaemia.

59c

59c Axial CT image of the abdomen shows


dilated fluid-filled loops of non-enhancing
ischaemic small bowel.

108

Abdominal Imaging

CASE 60

Cases 60, 61

60

History
A 23-year-old male presented with a
history of renal tract calculi.

CASE 61

61

History
A 57-year-old male presented with
lower abdominal pain.

109

Answers 60, 61

Abdominal Imaging

ANSWER 60
Observations (60)

Discussion

Single oblique radiograph of the pelvis from an IVU shows


a small, shrunken, spastic trabeculated bladder with
multiple diverticula, with a superiorly pointed dome. This
is the so-called pine tree appearance of a neurogenic
bladder.

Bladder innervation is by the parasympathetic nerves


S2S4. Injury to these nerves causes denervation of the
detrusor muscle, giving this appearance of a shrunken,
heavily trabeculated bladder, with an irregular thickened
wall and multiple diverticula. Causes include:
Congenital anomalies myelomeningocele, spina
bifida.
Spinal trauma.
Diabetes mellitus.
Infection syphilis, herpes.
Spinal neoplasm.

Diagnosis
Neurogenic bladder.

Differential diagnosis
For small bladder:
Infection schistosomiasis/TB.
Iatrogenic postsurgery/radiotherapy.
Neurogenic.
Transitional cell carcinoma (TCC) asymmetric
bladder contraction with thick wall and filling defects.
Extrinsic compression usually gives a pear shaped
bladder appearance.

Practical tips
Inspect IVU films carefully to look for a spinal
abnormality, which can help to determine the underlying
cause of the bladder abnormality.

Further management
In the absence of an easily identifiable cause, lumbosacral
spine MRI can be useful.

ANSWER 61
Observations (61)

Colonic metastases.

Intraperitoneal seeding breast, ovarian, pancreatic


and gastric tumours are the most common primaries
responsible for intraperitoneal drop seeding. These
lesions involve the mesenteric borders. Appearances
are of a focal mass or diffuse stellate appearance on
CT, but on double contrast barium imaging,
appearances are of a focal extrinsic indentation of the
bowel. This involves the pouch of Douglas (50%) and
commonly the superior/anterior border of the
sigmoid. The distal ileum and medial border of the
caecum are also often involved.

Discussion

Practical tips

Tumours spread to the mesentery via four routes:


Direct invasion along the mesenteric vessels and fat
seen with gastric, pancreatic, colonic and biliary
cancers; 40% of patients with adenocarcinoma of the
pancreas have tumour extending along the mesenteric
root at diagnosis.
Extension via mesenteric lymphatics colonic,
ovarian, breast, lung, carcinoid and melanoma cancers
can spread to mesenteric lymph nodes, though
lymphoma is more common.
Haematogenous spread commonly from melanoma,
breast and lung primaries, metastases involve the
antimesenteric border of the bowel via small arteries.
They can act as the lead point for intussusception. Up
to 7.5% of melanoma patients show small bowel
involvement.

The most common underlying cause for intraperitoneal


malignant disease is ovarian cancer.

Single image from a barium enema examination


demonstrates a well defined, smooth, eccentric filling defect
in the distal sigmoid colon. This appears to be extraluminal
in origin and lies on the mesenteric aspect of the bowel. A
similar lesion is seen in the pouch of Douglas. Multiple
lesions centred in an intraperitoneal location suggest the
diagnosis of intraperitoneal metastases.

Diagnosis

110

Further management
Clinical review of patient to try and localize a likely primary
tumour. CT scanning will confirm peritoneal disease and
may well reveal the primary tumour.

Further reading
Sheth S, Horton KM, Garland MR, Fishman EK (2003).
Mesenteric neoplasms: CT appearances of primary
and secondary tumors and differential diagnosis.
Radio Graphics 23: 457473.

Abdominal Imaging

CASE 62

Cases 62, 63

62a

History
A 46-year-old female, recently
emigrated to the UK from Africa,
presented with symptoms of urinary
frequency and urgency.

CASE 63

63

History
A 45-year-old diabetic presented with
pyrexia and abdominal pain.

111

Answer 62

Abdominal Imaging

ANSWER 62
Observations (62a)

Practical tips

This single coned view of the pelvis shows curvilinear wall


calcification of a relatively normal capacity bladder. No
calcification of the lower ureters is seen. No discontinuity in
the calcification is seen.
The history of residence in Africa raises the possibilities
of bladder TB and schistosomiasis. The absence of gross
bladder contraction makes the latter more likely, but it
would also be helpful to review a full length abdominal film
to look for upper tract calcification. Transitional cell
tumour must also be excluded.

Schistosomiasis involves bladder and lower ureters


and results in a calcified, nonshrunken bladder. TB
involves the kidneys and spreads via the ureters to
involve the bladder. It is very unusual to have isolated
bladder involvement with TB and the degree of
bladder contraction is more marked than in
schistosomiasis.
Discontinuity of calcification in the bladder wall
should arouse suspicion of bladder cancer.

Further management
Diagnosis
Schistosomiasis.

Follow-up in these patients is required since the latency for


development of squamous cell carcinoma of the bladder
can be up to 3035 years.

Differential diagnosis
For calcified bladder wall:
Cancer primarily transitional cell carcinoma (TCC)
but also other rarer bladder tumours.
Radiotherapy.
Infection TB and schistosomiasis.

62b

Discussion
Schistosomiasis is one of the most common parasitic
infections, affecting 8% of the global population.
Schisto so ma haemato b ium is the female parasite which
affects the genitourinary system. Unlike TB, which tends
to affect the kidneys first and then spreads caudally,
schistosomiasis has a reverse involvement and usually is
confined to bladder and lower ureters. (Another case of
bladder schistosomiasis [62b] is shown note the absence
of upper tract calcification that one might see in TB.)
Classically, the patient presents with urinary frequency,
urgency and dysuria. Imaging findings are of:
Calcification of the bladder which results in reduced
bladder filling capacity and increased postmicturition
residual volume.
Calcification of the lower ureters.
Lower ureteric strictures.
Ureteritis cystica.
Complications of disease affecting the genitourinary system
include:
Cystitis.
Vesicoureteric reflux and subsequent pyelonephritis.
Increased risk of squamous cell carcinoma of the
bladder.
Other systems can be affected:
Liver oval migration results in portal hypertension
and subsequent oesophageal varices.
Respiratory system diffuse granulomatous lung
lesions.

112

62b Calcification of the bladder is seen in


schistosomiasis with absence of upper tract
calcification to help differentiate from TB.

Answer 63

Abdominal Imaging

Case 64

ANSWER 63
Observations (63)
Supine abdominal radiograph demonstrates gas within the
left pelvicalyceal system and upper ureter. No intraparenchymal renal gas is seen. No gas is seen in the right
renal tract or in the bladder.

Diagnosis
Emphysematous pyelitis.

Differential diagnosis
Of cause of gas in the urinary tract:
Emphysematous pyelonephritis/pyelitis/cystitis.
Gas forming perinephric abscess.
Trauma.
Iatrogenic urinary diversion procedures.
Urinary tract fistula to bowel due to inflammation,
e.g. Crohns, diverticulitis or spreading malignancy.

Discussion
Emphysematous pyelitis is a condition in which infective
organisms produce gas, which is confined within the renal
pelvicalyceal system. In emphysematous pyelonephritis, gas
also forms within the renal parenchyma a life-threatening
condition that requires prompt diagnosis and treatment.

CASE 64

There is an increased incidence of these conditions in


patients with diabetes mellitus and women are three times
more commonly affected than men. Escherichia co li is the
causative organism in 70% of cases with Klebsiella, Pro teus,
Candida and Pseudo mo nas organisms also being found.
In emphysematous pyelonephritis, small gas bubbles are
initially seen on plain radiographs involving the renal
parenchyma; this progresses to give a diffuse mottling in
more advanced disease and then progresses to produce a
crescent of perinephric gas when there is extension into
the perirenal fat.

Practical tips
CT is the best imaging modality for assessing extent and
location of gas.

Further management
Depends on cause but obviously infective causes require
prompt, appropriate antibiotic treatment.

Further reading
Joseph RC, Amendola MA, Artze ME, et al. (1996).
Genitourinary tract gas: imaging evaluation.
Radio Graphics 16: 295308.

64a

History
A 39-year-old female presented with
early satiety and epigastric pain.

113

Answer 64

Abdominal Imaging

ANSWER 64
Observations (64a)
Single image from a double contrast barium meal
examination shows a well defined smooth walled ovoid
mass lesion in the gastric antrum. A central smooth ulcer
is present and no calcification is seen.

Diagnosis
Leiomyoma of the stomach.

Differential diagnosis
For target lesions:
Neurofibroma.
Lipoma (64b).
Ectopic pancreatic rest.
Metastases commonly breast, lung, renal and
malignant melanoma.
Haemangioma.

they increase in size, when there may be epigastric pain and


bleeding. The gastric antrum and pylorus are the most
common sites affected. The majority of these lesions
extend intraluminally (60%) and form well defined ovoid
defects. They are more likely to ulcerate than oesophageal
leiomyoma with ulceration seen in up to 50%. Calcification
is rare.
Complications include:
Bleeding.
Obstruction.
Intussusception tumour can act as a lead point.
Malignant degeneration seen in up to 1520% of
cases.

Practical tips
Smooth, well defined, slow growing gastric lesions are
likely to be benign.

Discussion

Further management

This is the second most common benign gastric tumour


after gastric polyps. Like oesophageal leiomyoma, these are
slow growing lesions and are usually asymptomatic until

Although radiological appearances suggest this to be a


benign lesion, referral for endoscopy +/ biopsy should be
made.

64b

64b Axial CT image demonstrating a well defined,


smooth, rounded lesion in the stomach, which has
clearly the same attenuation as intra-/extra-abominal
fat. This has appearances of a gastric lipoma.

114

Abdominal Imaging

CASE 65

Cases 65, 66

65a

History
A 45-year-old female presented with per
rectal bleeding.

CASE 66

66a

History
A 42-year-old smoker with a family history
of bowel cancer.
(see page 118 fo r case answer)

115

Answer 65

Abdominal Imaging

ANSWER 65
Observations (65a)
Single image from a double contrast barium enema
examination shows abnormality of the colon that extends
from the rectum to the mid transverse colon. There are
features of luminal narrowing with mucosal irregularity,
granularity and shallow ulceration. The disease process
appears continuous along the affected segments with no
further lesions seen. Normal appearances of the ileocaecal
region. Normal sacroiliac joints. The appearances are in
keeping with a colitis, most likely ulcerative colitis.

Diagnosis
Ulcerative colitis (UC).

Discussion
Ulcerative colitis is an idiopathic inflammatory bowel
disease with involvement predominantly of the mucosa and
submucosa of the large bowel. There are two peaks of
presentation 3rd5th decades and 7th8th decades. The
most common presentation is with bloody diarrhoea and
abdominal pain.
The rectum is almost always involved (96% of cases)
with continuous, concentric and symmetric involvement of
the colon more proximally. The terminal ileum is involved
in 1025% due to backwash ileitis. In acute inflammation
there are findings of:

Thickening of bowel wall (65b).


Significant bowel wall thickening can lead to the
classical thumb-printing appearance.
Widening of the presacral space.
Fine mucosal granularity.
Superficial ulceration.
Pseudopolyps islands of oedematous mucosa.
Collar button ulcers (65c).
Appearances in the chronic stage:
Colon becomes rigid with luminal narrowing due to
chronic inflammation, and loss of haustrations
leadpipe colon.
Coarse granular mucosa.
Inflammatory polyps.
Backwash ileitis.

65c

65b

65b Single axial CT image shows continuous


thickening of the colonic wall involving sigmoid
colon. The surrounding fat is dirty (increased
attenuation) due to inflammatory change.
Appearances are of an acute active colitis though
the appearances here are not specific for an
underlying cause.

116

65c Single image from a barium enema examination


shows multiple shallow barium-filled ulcers in the
left sided colon.

Answer 65

Abdominal Imaging

The condition is complicated by:


Perforation from toxic megacolon in 510% the
most common cause of death (65d).
Colonic adenocarcinoma this complicates up to 5%
of UC patients with risk highest when there is
pancolitis or onset at a young age(<15 years) and
increases with chronicity of disease. The rectosigmoid
is the most common location for neoplastic
transformation.
Colonic strictures usually a single, short, smooth
stricture is found, most commonly in the
rectum/sigmoid.

transmural, skip lesions; preferential involvement


of terminal ileum. Fistulae and deep ulcers are
common features.
UC characteristically has a continuous, concentric,
symmetric involvement that extends proximally
from the rectum and only occasionally involves the
terminal ileum. Fistulae, fissures and deep
ulceration are not features.
Remember that although the rectum is always
involved in UC, it may appear spared if steroid
enemas have been used.

In addition, as with Crohns disease, there are a variety of


extracolonic complications which include iritis, pyoderma
gangrenosum, chronic active hepatitis, sclerosing
cholangitis and seronegative arthritis (Figure 65e
demonstrates sacroiliitis with early sclerosis of both
sacroiliac joints).

Look for intestinal complications of UC including


malignancy, toxic megacolon, pneumatosis intestinalis
and perforation.
Look for extraintestinal complications of UC such as
sacroiliitis.
Look for complications of treatment, e.g. steroids
causing avascular necrosis (AVN) or osteoporosis.

Practical tips

Further management

Differentiation of Crohns from UC is often possible


from the imaging findings:
Crohns characteristically has multiple, eccentric,

A combined medical/surgical approach to disease management should be taken.

65d

65e

65e Pelvic radiograph shows bilateral early sclerosis


of both sacroiliac joints.

65d Abdominal radiograph with features of toxic


megacolon and perforation.

117

Answer 66

Abdominal Imaging

ANSWER 66
Observations (66a)
Single image from a double contrast barium examination
is shown. There are multiple submucosal lesions seen
scattered throughout the colon with no regional
predominance. Close inspection shows that these are due
to gas-filled cysts in the bowel wall. No free intraabdominal gas is seen to suggest perforation. No linear gas
collections are seen. No portal vein gas is seen.

leading to pneumoperitoneum but with no symptoms of


peritonitis. This cystic pneumatosis is usually a benign,
innocuous condition and is associated with chronic
obstructive pulmonary disease, perhaps due to air tracking
from ruptured alveoli and along the mesentery via the
retroperitoneum. There is also an association with mucosal
disruption elsewhere in the GI tract, e.g. peptic ulcer
disease.

Diagnosis

Practical tips

Pneumatosis cystoides intestinalis.

Air in the bowel wall due to infarction typically appears


more linear (66b) and may be associated with portal vein
gas in premorbid cases.

Discussion
Pneumatosis cystoides intestinalis is usually a benign
condition of middle aged people who tend to be
asymptomatic but can present with symptoms of vague
abdominal pain, diarrhoea and mucous discharge.
Radiological findings are of multiple small 15 mm gasfilled cysts in a subserosal/submucosal distribution. They
are more commonly found on the mesenteric rather than
antimesenteric side of the colon. The cysts can rupture

66b

118

Further management
Pneumatosis of the colon is usually not clinically significant
the importance here is to treat the patient, not the x-ray.

Further reading
Pear BL (1998). Pneumatosis intestinalis: a review.
Radio lo gy 207: 1319.

66b Abdominal radiograph shows linear gas


opacity within the wall of the transverse colon in
a patient with ulcerative colitis.

Abdominal Imaging

Cases 67, 68

CASE 67
History
A 56-year-old male with lung cancer.

67

CASE 68

68

History
A 46-year-old male presented
recurrent urinary tract infections.

with

119

Answers 67, 68

Abdominal Imaging

ANSWER 67
Observations (67)
Selected in and out of phase T1 weighted axial images of
the abdomen are provided, the out of phase image lying to
the right. There is enlargement of the left adrenal gland
which appears of intermediate signal intensity on T1 in
phase and shows signal drop out on the T1 out of phase
image. Appearances on this chemical shift MRI are
consistent with fatty content and indicative of a benign
adenoma.

Diagnosis
Adrenal adenoma.

Discussion
Adrenal lesions are found in ~9% of the general population.
Adrenal adenomas are the most common adrenal lesion.
Imaging features can be used to differentiate benign
adenomas from primary malignant (carcinoma, phaeochromocytoma) or metastatic adrenal lesions.
Imaging features for differentiation of an adenoma from
metastasis are:
Adenomas are mostly <2.5 cm in size; lesions >4 cm in
size are more likely to be metastases or adrenal
carcinomas.
Lesions showing no growth over 6 months are usually
benign.
On unenhanced CT adenomas often contain
intracellular fat, lowering the overall density on CT.
When <10 Hounsfield units (HU), this is highly
specific for adenoma (96% specificity). Some
adenomas contain less fat and so a lesion with HU
>10 may represent adenoma or malignant lesion.
On IV contrast enhanced CT both adenomas and
metastases enhance, but adenomas washout more
rapidly. After a plain scan, acquire postcontrast scans
at 60 s (early) and 10 min (delayed).

% washout =

early-delayed
( early-unenhanced
) x 100

Washout >60% has sensitivity and specificity ~90% for


the diagnosis of adenoma.
On chemical shift MRI signal drop out on the out
of phase imaging is seen with adenomas. Lesions
without significant intracellular fat (e.g. metastases)
dont show any signal difference on in/out of phase
imaging. The sensitivity and specificity are similar to,
if not better than, those of contrast enhanced CT.
PET may be useful when CT and MRI fail to
characterize an adrenal mass. Malignant lesions show
increased uptake of FDG (18 fluoro-2-deoxyglucose)
(>liver activity) while benign lesions do not (<liver
activity).

Practical tips
An incidental enlarged adrenal picked up on US/CT/MRI
should be further investigated with a chest radiograph to
look for a lung neoplasm.

Further management
Adrenal adenoma should be followed up after 6 months.
Lesions that grow by over 1 cm in this time or those that
measure over 4 cm should be considered for surgery.
Hormonally active lesions may also be considered for
surgery.

Further reading
Mayo-Smith WW, Boland GW, Noto RB, Lee MJ
(2001). State-of-the-art adrenal imaging.
Radio Graphics 21: 9951012.

ANSWER 68
Observations (68)

Crossed fused renal ectopia.

Discussion

Practical tips

Crossed renal ectopia involves a kidney being located on


the opposite side of the midline from its ureteral orifice. It
is more common for the left kidney to have migrated to the
right, with the crossed kidney lying inferior to the normal
kidney. Usually the kidneys are fused and are associated
with aberrant renal arteries.

Try to identify complications of the condition such as renal


scarring from recurrent infections, or hydronephrosis due
to obstruction from stones/urethral valves.

Diagnosis

120

It is associated with:
Renal calculi.
Infection.
Reflux.
Megaureter.
Cryptorchidism.
Urethral valves.
Multicystic dysplasia.

Single image from an IVU study is provided without a


control film. It shows renal ectopia with both kidneys being
located on the right side. This is crossed fused renal ectopia.
There is no associated hydronephrosis.

Further management
No active management is required.

Abdominal Imaging

CASE 69

Cases 69, 70

69

History
A 25-year-old female presented with
fatigue and steatorrhoea for 12 months.

CASE 70

70

History
A 26-year-old male was involved in a
road traffic accident.

121

Answers 69, 70

Abdominal Imaging

ANSWER 69
Observations (69)
Single image from a small bowel enema examination is
shown. The tip of the small bowel enema catheter is seen
in the proximal duodenum. The examination demonstrates
a reversal of fold pattern with an increased number of folds
seen in the ileum and a reduction in the number of small
bowel folds seen in the proximal jejunum. These features
suggest malabsorption in the proximal small bowel such as
coeliac disease.

Diagnosis
Coeliac disease.

Discussion
Coeliac disease is a gluten-sensitive enteropathy
characterized histologically by villous atrophy. The
duodenum and jejunum are affected more than the ileum
because some proximal gluten digestion means that less
reaches the distal small bowel. It classically presents with
steatorrhoea and diarrhoea, but more commonly presents
in less specific ways such as fatigue, weight loss and
abdominal pain. Malabsorption can also result in anaemia,
osteomalacia, neuropathy and oedema. Age at presentation
can vary from early childhood to late middle age.
Imaging features of coeliac disease include:
Small bowel dilatation (due to reduced motility) is the
most common imaging finding.
Reduced number of folds in the jejunum.
Jejunalization of the ileum an increased number of
ileal folds. This is a response to the reduced mucosal
area in the jejunum.

Moulage sign dilated small bowel loops with fold


effacement, particularly in the jejunum and
duodenum.
Transient intussusception the cause of abdominal
pains.
Flocculation of the barium contrast due to
hypersecretion is not commonly seen with modern
barium preparations.
Autoantibodies (e.g. antiendomysial) may be suggestive,
but diagnosis is made by demonstration of villous atrophy
on jejunal biopsy. This abnormality resolves once gluten is
removed from the diet and recurs when it is reinstated. The
condition is complicated by an increased risk of lymphoma,
adenocarcinoma of the small bowel, oesophageal
carcinoma and pharyngeal carcinoma.

Practical tips
Folds in the jejunum are normally thicker and more
numerous than the ileum. Expect to see seven folds
per inch in the jejunum and three to four folds per
inch in the ileum. A reversal of this pattern should
raise suspicion of coeliac disease.
If the radiological features deteriorate while on a
gluten-free diet, raise the possibility of complications
such as lymphoma.

Further management
Confirmation of diagnosis is made with endoscopic small
bowel biopsy.

ANSWER 70
Observations (70)

Practical tips

Single image from an IVU series shows contrast in the


collecting systems and in the bladder. The bladder has an
abnormal elongated configuration extending up out of the
pelvis. There are bilateral pelvic fractures through the
acetabulum. No free contrast is seen leaking from the
bladder.

Look for an underlying cause for the abnormal pear shaped


bladder appearance pelvic fractures, curvilinear vascular
calcification in the walls of aneurysms, radiotherapy
changes in bone/small bowel to suggest underlying
malignancy and lymphadenopathy.

Further management
Diagnosis
Pear shaped urinary bladder secondary to pelvic
haematoma.

Differential diagnosis
For pear shaped bladder (mnemonic HELP):
Haematoma.
External iliac artery aneurysms.
Lymphadenopathy.
Pelvic lipomatosis.

122

Further treatment will depend on the underlying cause


this is a secondary finding pointing towards an underlying
problem.

Abdominal Imaging

Case 71

CASE 71
History
A 58-year-old female with an incidental
liver lesion detected on ultrasound.

71a

71a T2 weighted MRI scans with a normal and extended TE.

71b

71b T1 weighted scans, in and out of phase.

71c

71c T1 weighted scans post IV gadolinium at 30 s, 60 s and 3 min.

123

Answer 71

Abdominal Imaging

ANSWER 71
Observations (71a, 71b, 71c)

Discussion

and 90% measure less than 4 cm in diameter. Multiple


haemangioma may be associated with OslerWeberRendu
syndrome. Some enlargement can be seen during
pregnancy.
Imaging findings on ultrasound: well defined,
uniform, hyperechoic lesion is the typical ultrasound
appearance (71d). No Doppler signal is seen within
the lesion due to the low-velocity flow (71e). Larger
lesions can appear hypoechoic and show flow
however.
Imaging findings on CT: lobulated low attenuation
lesion, which is frequently peripheral in location.
Calcification is not common. Following contrast
injection, there is peripheral nodular enhancement
with centripetal filling. The lesion usually fills in
completely over 330 min to become isodense with
liver, though larger lesions may show persisting
central nonenhancement/scar and small lesions may
show immediate uniform enhancement.
Imaging findings on MRI: haemangiomas show high
signal on T2 weighted images that persist on more
heavily T2 weighted scans. This is called the light
bulb sign and is indicative of cyst or haemangioma.
The enhancement pattern with gadolinium is as
described for CT.
Imaging findings on sulphur colloid scan: cold defect.

Hepatic haemangioma is the most common benign tumour


of the liver, affecting up to 20% of the population. They
are usually asymptomatic, being incidental findings on
imaging studies, however large lesions can present with
acute haemorrhage and abdominal pain; 90% are solitary

Usually imaging is sufficient to make the diagnosis but


where atypical features are present, biopsy can be
undertaken provided there is normal liver between the
lesion and liver capsule to prevent haemorrhage.

T2 weighted images demonstrate a well defined, slightly


lobulated mass in the right lobe of liver, which is of
increased signal on the normal T2 and ultra-T2 weighted
scans (71a). This is the so-called light bulb sign. Axial T1
weighted images (71b) show the lesion to be of reduced
signal with no signal change in the liver on out of phase
scans to indicate that the lesion represents focal fatty
sparing. Dynamic post-contrast images (71c) show
peripheral nodular enhancement with centripetal filling in
over time. These appearances are diagnostic of cavernous
haemangioma.

Diagnosis
Cavernous haemangioma.

Differential diagnosis
For hyperechoic hepatic lesions on US:
Haemangioma.
Focal nodular hyperplasia (FNH).
Adenoma.
Metastasis.
Hepatocellular carcinoma.
Lipoma.

71d

71d Ultrasound appearances of haemangioma are of


a well defined, round, hyperechoic lesion.

124

71e

71e No flow is seen within the lesion on Doppler


ultrasound .

Answer 71

Abdominal Imaging

Case 72

Practical tips

Further management

MRI is the investigation of choice to characterize


suspected haemangioma. A positive light bulb sign
indicates that the lesion is either a cyst or a
haemangioma. However, depending on the specific
circumstances, post-contrast scans may still be
required as necrotic or cystic neoplasms can yield a
positive light bulb sign.
Use of IV contrast obtain pre-contrast scans, then
repeat at 30 s (arterial phase), 60 s (portal phase) and
35 min (equilibrium phase).
If the centripetal enhancement is intense and nodular,
this is highly specific for the diagnosis of
haemangioma.

These are essentially benign lesions and dont require any


further follow-up.

Further reading
Vilanova JC, Barcel J, Smirniotopoulos JG, et al.
(2004). Hemangioma from head to toe: MR imaging
with pathologic correlation. Radio Graphics 24:
367385.

CASE 72
History
A 43-year-old male presented with
central abdominal pain.

72a

72b

72c

72d

125

Answer 72

Abdominal Imaging

ANSWER 72
Observations (72a, 72b, 72c, 72d)
Several axial CT images of the abdomen with IV contrast
enhancement in the portal venous phase are shown. There
is significant stranding of the peripancreatic fat indicative
of acute inflammation, which is predominantly around the
pancreatic tail. Within the pancreatic tail, there is a
moderate sized fluid-filled cystic lesion, which would be
consistent with a pancreatic pseudocyst in a patient with
pancreatitis. The local inflammatory change appears to
extend to involve the distal transverse colon at the splenic
flexure. In addition, there is central low attenuation within
the portal vein extending into the proximal splenic vein in
keeping with portal/splenic vein thrombosis. The
pancreatic duct is dilated at 4 mm diameter. While the
whole gallbladder has not been imaged no gallstones are
seen on these images. The visualized liver edge is smooth
and liver has normal attenuation.

Diagnosis
Pancreatitis.

In chronic pancreatitis (following several bouts of acute


episodes), the pancreas becomes atrophic, there is
multifocal calcification, the pancreatic duct becomes dilated
and chronic pseudocysts can form (72e).

Practical tips
Look for the two main causes of pancreatitis alcohol
and gallstones. US is required to look for gallstones
and can be used to look for complications, e.g.
pseudocyst formation and portal vein thrombosis.
Look for the main complications of pancreatitis.
Assessing the pancreas is best done by three phase CT
imaging with a pre-contrast and pancreatic phase
(~40 s) scan of the pancreas, followed by portal
venous phase scans of the abdomen and pelvis. The
pre-contrast scan will allow for assessment of
calcification in the pancreas to determine whether this
is an acute-on-chronic episode. Pancreatic phase scans
best assess for areas of necrosis and focal mass lesions.
Portal venous imaging allows for assessment of local
disease and complications.

Discussion
Pancreatitis is an acute inflammatory condition. There are
two main types of pancreatitis oedematous and necrotic.
Oedematous pancreatitis is the much more common form,
characterized by gland oedema, whereas necrotic pancreatitis is rarer and is complicated by pancreatic haemorrhage
and necrosis.
Appearances of the pancreas in pancreatitis can therefore
be variable, with the pancreas appearing normal, having
areas of nonenhancement due to necrosis, having areas of
increased attenuation due to haemorrhage, or appearing
enlarged and oedematous. Universally, streaking into the
surrounding fat due to inflammation is observed.
Causes of pancreatitis include:
Alcoholism and gallstones are the two most common
causes of acute pancreatitis. Assess for signs of alcohol
excess by looking for fatty change in the liver (diffuse
low attenuation on CT, raised echogenicity on US),
cirrhosis (irregular nodular surface to the liver which is
atrophic) and portal hypertension (splenomegaly and
varices). Assess for gallstones with US initially, then
MRCP if there is still uncertainty.
Infections hepatitis, mumps.
Trauma.
Hypercalcaemia from multiple myeloma, sarcoidosis,
amyloidosis.
Drugs, e.g. steroids, diuretics (frusemide, thiazides),
azathioprine.
Malignancy.
Pancreatitis can be complicated by pseudocyst formation;
abscess formation (usually develops 2 weeks after acute
inflammation onset); spread of local inflammation to
involve bowel loops resulting in strictures, colitis and
perforation; portal vein, splenic vein, SMV thrombosis;
ascites and chest complications pleural effusion (usually
left sided), empyema, unilateral pulmonary oedema.

126

Further management
Treatment is usually conservative although necrotizing
pancreatitis may well need more aggressive management
involving surgery or percutaneous radiologically guided
drainage.

72e

72e Axial CT image showing features of chronic


pancreatitis with atrophic pancreas, dilated
pancreatic duct and marked calcification.

Abdominal Imaging

CASE 73

Cases 73, 74

73

History
A 34-year-old male presented with
scrotal tenderness.

CASE 74

74

History
A 56-year-old male presented with
haematuria and chronic renal failure.

127

Answers 73, 74

Abdominal Imaging

ANSWER 73
Observations (73)
Ultrasound longitudinal image of the right testicle
demonstrates multiple small hyperechoic foci, with no
posterior acoustic shadowing, distributed evenly
throughout the testicle. The testicle is otherwise of normal
size and shape. No other focal mass lesion is identified.

Associations include:
Cryptorchidism.
Infertility.
Klinefelters.
Downs syndrome.
Male pseudohermaphrodism.
Alveolar microlithiasis.

Diagnosis
Testicular microlithiasis.

Discussion
Testicular microlithiasis is a condition characterized by
failure of clearance of degenerated tubular epithelium from
the seminiferous tubules, which subsequently calcify. This
is an asymptomatic condition and is an uncommon
incidental finding. Image findings on US are of multiple
small 12 mm hyperechoic foci with no acoustic shadowing, distributed throughout both testes. Atypical
appearances of asymmetrical and unilateral foci are also
seen. There is an increased risk of testicular germ cell
tumour in 40% and therefore follow-up with 6 monthly US
is often advised.

Practical tips
Identifying the distribution of calcific foci is important,
since a cluster of calcification within a hypoechoic area
suggests tumour or chronic testicular infarction. Infarction
can be due to torsion, trauma or severe epididymitis.

Further management
Urological referral with follow-up testicular US at 6
monthly intervals to screen for malignancy.

ANSWER 74
Observations (74)
Single AP abdominal radiograph demonstrates amorphous,
putty-like calcification within a shrunken right kidney.

Diagnosis
Renal tuberculosis with autonephrectomy.

Ureteral TB is found in 50% of genitourinary TB cases.


Radiographic features of involvement include ureteral
filling defects, calcification and strictures. Bladder TB
presents with a shrunken, scarred, calcified bladder with
thickened wall and/or filling defects. Clinical presentation
is with frequency, urgency, dysuria, microscopic
haematuria and sterile pyuria.

Discussion
The urogenital tract is the second most common site to be
affected by TB, after the lungs. Renal TB is found in 510%
of patients with pulmonary TB, but there is only
radiographic evidence of pulmonary TB in fewer than 50%.
The kidney is usually affected first, via haematogenous
spread from a lung/bone/GI focus. There can then be
contiguous spread of infection to involve the ureters and
bladder. Renal TB is unilateral in 75% of cases and the most
common appearance is of a small, shrunken, scarred,
nonfunctioning kidney.
Less common appearances are of:
Cortical scarring.
Renal pelvis/infundibular strictures resulting in
hydrocalycosis and amputated calyx.
Nephrolithiasis.
Dystrophic parenchymal calcification.

128

Practical tips
Urinary tract TB spreads in an antegrade fashion.
Check chest radiograph for signs of pulmonary TB
but this is only seen in about 50% of cases.

Further management
Medical drug therapy for TB should be initiated.

Abdominal Imaging

CASE 75

Cases 75, 76

75

History
A 57-year-old male patient presented
with abdominal pain.

CASE 76

76

History
A 58-year-old female presented with
early satiety.

129

Answers 75, 76

Abdominal Imaging

ANSWER 75
Observations (75)
Supine abdominal radiograph shows a positive Rigler sign
suggestive of pneumoperitoneum. There is also outlining
of the falciform ligament and the left lateral umbilical
ligament with a large area of free gas seen overlying the
liver. These findings are consistent with pneumoperitoneum.

Diagnosis
Pneumoperitoneum.

Differential diagnosis
Of causes of pneumoperitoneum:
Trauma.
Iatrogenic causes, e.g. laparotomy usually gas should
have resolved within 3 days and its presence is
suspicious after this; although it can take up to 3
weeks to resolve in some cases. Faster absorption
occurs with CO2 and in obese patients.
Perforation of abdominal viscus.
Ruptured pneumatosis intestinalis.
Rupture of an abscess.
Extension from chest, i.e. pneumomediastinum,
bronchopleural fistula.
Through female genital tract, e.g. intercourse,
waterskiing.

Discussion
There are a variety of signs that aid in the detection of free
intraperitoneal gas.

Signs of free gas within the peritoneal cavity on a supine


radiograph are:
Football sign large round lucency in central
abdomen.
Rigler sign air on both sides of the bowel wall
makes it unusually visible.
Triangle sign gas within bowel doesnt normally
form edges therefore such unusually shaped
collections of air should arouse suspicion of free gas.
Urachus sign outline of median umbilical
ligament.
Outlining of falciform ligament as well as the medial
and lateral umbilical ligaments.

Practical tips
If there is any doubt about the diagnosis then an erect
chest radiograph or a left lateral decubitus film of the
abdomen should be acquired.

Further management
Free intraperitoneal gas is suggestive of a perforated viscus
and is a surgical emergency inform the surgical team
immediately.

Further reading
Levine MS, Scheiner JD, Rubesin SE, et al. (1991).
Diagnosis of pneumoperitoneum on supine
abdominal radiographs. American Jo urnal o f
Radio lo gy 156: 731735.

ANSWER 76
Observations (76)

Discussion

Single image from a barium meal examination shows there


is poor distension of the entire stomach, which is most
evident in the body and antrum, with relative sparing of
the cardia. Irregular gastric mucosa is seen within the
fundus and body. The duodenum is normal. The
appearances are those of linitis plastica and a gastric tumour
must be excluded in the first instance.

Gastric carcinoma is the third most common GI


malignancy and shows an increased prevalence in Japan.
The scirrhous type of the tumour, which produces linitis
plastica, accounts for 510% of gastric cancer. Diffuse
infiltration of tumour causes fibrosis and rigidity with
reduction/absence of peristalsis in affected areas. The
fundus and body of the stomach are most commonly
involved.

Diagnosis
Linitis plastica due to gastric carcinoma.

Practical tips

Differential diagnosis

Assessment of the stomach on CT is often very difficult


due to poor distension; however, contrast investigations
are much better due to their dynamic nature.

For linitis plastica appearance (mnemonic CALM


RAGE):
Cancer.
Amyloidosis.
Lymphoma usually non-Hodgkins type, most
common part of GI tract affected.
Metastases from lung, breast and melanoma.
Radiation.
Alkalis and other corrosives.
Granulomatous disorders, e.g. Crohns, TB.
Eosinophilic enteritis.

130

Further management
Referral to a gastroenterologist for endoscopy +/ biopsy
is appropriate.

Further reading
Ba-Ssalamah A, Prokop M, Uffmann M, et al. (2003).
Dedicated multidetector CT of the stomach:
spectrum of diseases. Radio Graphics 23: 625644.

Abdominal Imaging

Case 77

CASE 77
History
A 50-year-old female presented with
recurrent urinary tract infection.

77a

77b

131

Answer 77

Abdominal Imaging

ANSWER 77
Observations (77a, 77b)

Discussion

This full length film taken at 15 minutes in an IVU series


shows multiple small, well defined, smooth rounded filling
defects within both ureters.

This is a condition characterized by multiple small


subepithelial fluid-filled cysts in the wall of the renal pelvis
and ureters extending into the ureteric lumen. There is an
association with recurrent urinary tract infections and
obstruction, with a predisposition in diabetic patients. The
condition is asymptomatic and the lesions are not
premalignant. Occasionally it presents with haematuria.

Diagnosis
Pyeloureteritis cystica.

Differential diagnosis
For radiolucent filling defects in the ureters:
Radiolucent calculi.
Transitional cell carcinoma (TCC).
Pyeloureteritis cystica.
Blood clots.
Ureteric polyps.
Sloughed renal papillae.

Practical tips
Although any part of the ureter may be involved, there is
slight predilection for the upper third.

Further management
Characteristic IVU appearances may well permit a
confident diagnosis. However, if there is diagnostic
uncertainty then ureteroscopy with biopsy may sometimes
be required. Antibiotic treatment may be undertaken,
though the response is variable.

77b

77b Multiple round filling defects.

132

Abdominal Imaging

CASE 78

Case 78

78a

History
A 52-year-old female with diabetes
presented with renal failure.

78b

133

Answer 78

Abdominal Imaging

ANSWER 78
Observations (78a, 78b)
Two images from an IVU series are shown. There is
evidence of renal papillary swelling and enlargement. The
interpolar calyces on the right have a ball on tee
appearance with a collection of contrast material in the
centre of the papilla. A partial duplex system is also
demonstrated on the right. There is a well defined
intraluminal non-opaque filling defect in the upper ureter
on the right, which represents a sloughed papilla.

Diagnosis

of the renal pyramids. There is a progression in radiological


appearances as disease progresses. Phases of development:
Papillary swelling/enlargement.
Partial papillary sloughing tract formation
communicating with an irregular cavity. On an IVU
this is shown by the ball on tee appearance with
contrast extending into a centrally cavitated papilla.
Total papillary sloughing which gives a signet ring
appearance on IVU with the nonopacified sloughed
papilla being surrounded by contrast in the
calyx/ureter.

Renal papillary necrosis.

Practical tips
Differential diagnosis
Of causes of renal papillary necrosis (mnemonic SAD
ROPE):
Sickle cell disease.
Analgesics.
Diabetes.
Renal vein thrombosis.
Obstructive uropathy.
Pyelonephritis.
Ethanol abuse.

To differentiate between the various causes, identify


whether the involvement is unilateral (renal vein
thrombosis, obstructive uropathy) or bilateral
(systemic causes).
Diabetes is the most frequent cause (50%).
A classic exam type film to be aware of is the IVU
showing renal papillary necrosis and background bony
abnormalities of a condition that has been treated
with NSAIDs, e.g. ankylosing spondylitis, or bony
changes of sickle cell disease.

Discussion

Further management

Renal papillary necrosis is caused by a variety of disease


processes that result in ischaemia of the papillary portion

Treatment of the condition requires identification of the


underlying cause and control of this disease process.

78b

78b Sloughed papilla (left) and ball on tee appearance (right).

134

Abdominal Imaging

Cases 79, 80

CASE 79

CASE 80

History

History

A 36-year-old immunocompromised
patient presented with dysphagia.

A 28-year-old female presented with upper


abdominal pain, pyrexia and jaundice.

79

80a

80b

135

Answers 79, 80

Abdominal Imaging

ANSWER 79
Observations (79)
Single image from a barium swallow examination shows a
shaggy, ulcerated mucosal pattern in the thoracic
oesophagus typical of candidiasis.

Diagnosis
Candida oesophagitis.

Discussion
Candida oesophagitis is the most common cause of
infectious oesophagitis with C. alb icans being the most
commonly involved organism. Immunocompromised
patients have a predisposition to infection. Presentation is
with gradual dysphagia, odynophagia and retrosternal pain.
Association with oral infection is seen in up to 80%,
characterized by several patchy, white plaques covering the
mucosa. In the oesophagus there is predilection for the
upper half o f the o eso phagus with the appearance of
multiple, shaggy, longitudinal mucosal plaques. There is
reduction in primary peristalsis and some narrowing of the
lumen due to spasm and oedema. Stricture formation is a
rare complication.
Other infective causes of oesophagitis include:
Herpes oesophagitis the second most common cause
of infectious oesophagitis. Caused by herpes simplex
virus (HSV) type 1 and most commonly presenting in
males in 2nd4th decades. Usually involving the mid
oesophagus, there are multiple small discrete
superficial punctate ulcers. The intervening mucosa is
usually of normal appearance.

Cytomegalovirus (CMV) oesophagitis presents


radiologically with a single or multiple giant flat
superficial ulcer(s) near the gastro-oesophageal
junction. Less commonly, it can present as discrete
small superficial ulcers as are seen in herpes
oesophagitis.
HIV oesophagitis characterized by giant
oesophageal ulcers, difficult to differentiate from
CMV oesophagitis.
All affect immunocompromised patients and are rarely seen
in immunocompetents.

Practical tips
Do not mistake pseudodiverticulosis of the oesophagus for
deep ulceration. This is a benign condition where
oesophageal mucous glands become dilated and form flaskshaped outpouchings from the lumen. It is associated with
any severe oesophagitis (see Case 38).

Further management
Always consider immunocompromisation if not
already apparent.
Oropharyngeal candidiasis may be a pointer to the
nature of oesophageal ulceration but upper GI
endoscopy is still often needed to confirm via biopsies
and brushings.
Antifungal or antiviral drugs for treatment as
appropriate.

ANSWER 80
Observations (80a, 80b)

Diagnosis

Practical tips

Carolis disease.

Diagnosis is dependent on showing that the cysts


communicate with the biliary tree coronal and sagittal
reformats can be essential for this.

Discussion
Carolis disease is a rare autosomal recessive disorder
characterized by multifocal, cystic dilatation of the
intrahepatic bile ducts. The differential diagnosis for this
condition is polycystic liver disease, however the two can
be discriminated by identifying the communication
between cysts and bile ducts in Carolis.

136

Associations of Carolis disease:


Medullary sponge kidney (80%).
Polycystic kidney disease.
Congenital hepatic fibrosis.
Choledochal cyst.

Axial CT image and coronal reformats of the liver with IV


contrast in portal venous phase are shown. There are
multiple, well defined low attenuation cystic structures seen
throughout the liver. These appear to be communicating
with the bile ducts and are better demonstrated on the
coronal reformatted images (80b). No calculi are seen in
the ducts. The CT appearances are highly suggestive of
Carolis disease and the clinical details suggest cholangitis
has developed.

Complications include biliary stasis, stones, cholangitis and


hepatic abscess. There is an increased risk of cholangiocarcinoma in patients with Carolis disease. Biliary cirrhosis
and portal hypertension are no t complications.

Further management
If there is any concern as to whether the cysts are
communicating, MRCP (magnetic resonance cholangiopancreatography) is a noninvasive method of clarification.

Abdominal Imaging

Cases 81, 82

CASE 81
History
A 42-year-old female presented with
pelvic pain and menorrhagia.

81a

CASE 82

81b

82a

History
A 47-year-old presented with right
upper quadrant abdominal pain.

137

Answers 81, 82

Abdominal Imaging

ANSWER 81
Observation (81a, 81b)
Two MRI images are presented T2 weighted sagittal and
axial images through the pelvis. These images show bulky
enlargement of the uterus. The junctional zone is widened
and poorly defined and there are multiple high signal foci
within the myometrium in keeping with ectopic
endometrial tissue. These appearances are typical of
adenomyosis.

Diagnosis
Adenomyosis.

Discussion
Adenomyosis is a chronic condition affecting women
during the reproductive years. There is an increased
incidence in multiparous women, suggesting that uterine
trauma plays a role in the development of this disease. The
disease is characterized by ectopic endometrial tissue
spreading into the myometrium resulting in myometrial
hyperplasia. The condition is benign and although it is
associated with endometriosis, there is no increased risk of
developing endometrial/uterine malignancies. The disease
can be either focal or diffuse. It can sometimes be difficult

on US to differentiate focal forms of the condition from


uterine fibroids.
US appearances are of poorly defined hypoechoic areas
within the myometrium with multiple small, 24 mm,
ectopic endometrial cysts seen within the myometrium.
There is thickening of the anterior and posterior
myometrial walls.
MRI appearances are very similar myometrial wall
thickening; high signal (on T2 weighted images) ectopic
endometrial cysts in the myometrium and focal/diffuse
thickening of the innermost layer of myometrium known
as the junctional zone.
Symptomatically patients present with menorrhagia,
pelvic pain and dysmenorrhoea and the condition can lead
to infertility.

Practical tips
Widening of the junctional zone with ectopic endometrial
cysts are the best MRI features for diagnosis.

Further management
Hysterectomy is the only management option to relieve
symptoms.

ANSWER 82
Observations (82a)
Single AP radiograph of the abdomen demonstrates a large
mass lesion in the right upper quadrant, which shows
curvilinear calcification of the wall. Appearances would best
fit with this being a calcified wall of the gallbladder. No
discontinuous areas in calcification are seen.

Diagnosis
Porcelain gallbladder.

Differential diagnosis
Similar appearances on US can be due to:
Emphysematous cholecystitis.
Gallbladder filled with stones.
Normal gas-filled bowel.

Discussion
This is an idiopathic condition that was coined to describe
the blue discolouration and brittle nature of the gallbladder
at surgery. The condition is associated with gallstones in
90% of cases and is more frequently seen in women (sex
ratio of 5:1). Patients are usually asymptomatic and
diagnosis is made incidentally on plain films or US
imaging. Radiographic appearances mirror the histological
findings of dystrophic calcification in a chronically inflamed
gallbladder. The calcification can be of two types:
Curvilinear calcification in the muscularis (82b).
Punctate calcification in the mucosa/glandular spaces.

138

Imaging features on US are of curvilinear/punctate


calcification in the thickened wall of the gallbladder, which
can be focal or affecting the whole wall, with minimal
acoustic shadowing. On oral cholecystography, the
gallbladder is found to be nonfunctioning. The importance
of identifying the condition is the risk of developing
gallbladder carcinoma, which occurs in 1020% of cases.
Appearances of gallbladder carcinoma in this situation can
be of a luminal filling defect, infiltration causing focal wall
thickening or a localized fungating mass. Cholecystectomy
is therefore recommended even if the patient is
asymptomatic.

Practical tips
Ultrasound and CT scans should be scrutinized for
secondary gallbladder malignancy when porcelain
gallbladder is noted.
A discontinuity in the calcification when the whole
gallbladder wall appears to be calcified may be a clue
to gallbladder carcinoma.

Further management
Cholecystectomy is advised due to the high risk of tumour.

Further reading
Kane R, Jacobs R, Katz J, Costello P (1984). Porcelain
gallbladder: ultrasound and CT appearances.
Radio lo gy 152: 137141.

Answer 82

Abdominal Imaging

82b Axial CT image shows curvilinear


circumferential calcification of the gallbladder wall.

Case 83

82b

CASE 83
History
A 38-year-old female presented with
severe dysmenorrhoea and menorrhagia.

83a

83b

83c

139

Answer 83

Abdominal Imaging

ANSWER 83
Observations (83a, 83b, 83c)
Three MRI images are shown a sagittal T2 weighted
image (83a), axial T1 (83b) and axial T1 weighted fat
suppressed image (83c). There is a large 4 cm lesion
identified within the left ovary, which is of high signal on
both T1 and T2 weighted images and shows no fat
suppression. On the sagittal T2 weighted image, there is
evidence of layering of the contents of this mass. These
appearances would be entirely consistent with blood
products and the lesion represents an endometrioma.

Diagnosis
Endometrioma.

Discussion
Endometriosis is a chronic condition characterized by
ectopic deposits of endometrium outside the uterine cavity.
It affects women in the 4th6th decades and is
characterized by chronic pelvic pain, dysmenorrhoea,
menorrhagia and dyspareunia. Common sites for
endometrial deposits are the ovaries, fallopian tubes, pouch
of Douglas and uterosacral ligaments, however more distal
spread can occur to the bladder/bowel wall with even
further distal spread to the lungs and pleura. Spread to the
GI tract most commonly occurs to the distal sigmoid and
the rectosigmoid wall and deposits can result in peritoneal
adhesion and subsequent obstruction. Repeated bleeding
of the ectopic endometrial deposits results in fibrosis and
pain.

Endometrial deposits can be focal or diffuse. In this


case, the classical appearance of an endometrioma or
chocolate cyst is shown with a 15 cm cyst containing
layered blood products of high signal on T1 and T2
weighted images. There is no signal loss on fat suppression
sequences. The diffuse form is more common, however,
with multiple smaller scattered cysts seen in several
locations.
Complications depend on the site of the deposits and
can result in:
Infertility (due to involvement of the fallopian tubes
causing adhesions/fibrosis).
Bowel obstruction (due to fibrosis/stricturing of the
bowel wall).
Pneumothorax (due to spread to the pleura).

Practical tips
Endometriomas can be differentiated from dermoid cysts
by using a fat suppressed MRI sequence. Both will show
high signal on T1 and T2 weighted images but
endometriomas will still be high signal on fat suppressed
sequences.

Further management
This condition can be difficult to control. Medical
management with hormonal suppression of menstruation
is often attempted but surgical excision may be required.

83c

83c Lesion remains high signal on the T1 fat


suppressed image suggesting that it represents blood.

140

Abdominal Imaging

Case 84

CASE 84
History
A 34-year-old female presented
with vague abdominal pain.

84a

141

Answer 84

Abdominal Imaging

ANSWER 84
Observations (84a)
This single image from a double contrast barium enema
shows abnormality of the sigmoid colon with poor
distension and apparent external compression of the bowel
loops inferiorly. Three well defined calcific densities are seen
adjacent to the affected segment and have the appearance of
teeth.

Diagnosis
Ovarian dermoid.

Discussion
Ovarian dermoid cysts are benign lesions, which are
relatively common, accounting for ~20% of all ovarian
neoplasms. These mostly present during the reproductive
years, with a mean age of presentation of 30 years. They
contain tissue from all three germ cell layers ectodermal
(skin, brain), mesodermal (muscle, fat) and endodermal
(mucinous/ciliated epithelium). Tumours are bilateral in
~10%.
Typical appearance is of a unilocular cyst, lined by
squamous epithelium and containing sebaceous, fatty
material with hair follicles and skin glands arising from the
wall. A protuberance into the cavity known as a Rokitansky
nodule is usually the site of bone/teeth material.

84b

84b Plain radiograph of the pelvis shows lateral


displacement of the sigmoid colon due to the lucent
central pelvic mass.

142

Appearances on plain radiograph: teeth occur in 56%


of cases and are readily identifiable. Mass effect with
bowel loop displacement is often seen (84b). The
image shows a central lucent density representative of
fat in the right side of the pelvis, which is displacing
sigmoid colon.
Appearances on US/CT/MRI: a well defined high
echogenicity lesion is identified in the ovary due to
high fat content (84c). Again teeth are easily
identified with posterior acoustic shadowing, within
the solid Rokitansky nodule. Fat is readily identifiable
on CT and MRI by virtue of its low density and high
signal.
Complications are of torsion, rupture and malignant
degeneration (seen in 2% of cases).

Practical tips
Some films used in radiological exams rely on the candidate
noting an incidental lesion at the edge of the film. Teeth
in ovarian dermoids are a classical example of an edge of
the film case!

84c

84c Areas of increased echogenicity within the


lesion on US represent fat.

Answer 84

Abdominal Imaging

Further management

Further reading

Where there is still clinical concern after ultrasound,


MRI is useful in completely characterizing these
lesions. Figures 84df are axial T2 weighted (84d), T1
weighted (84e) and fat saturated (84f) axial images of
the pelvis showing dermoid cysts in both ovaries. The
right most cyst shows signal void posteriorly on all
image sequences in keeping with calcification and
complete fat suppression confirms the predominant fat
content of these lesions.
Referral to gynaecologist for surgical excision.

Outwater EK, Siegelman ES, Hunt JL (2001). Ovarian


teratomas: tumor types and imaging characteristics.
Radio Graphics 21: 475 490.

84d

84f

84e

84d84f Axial T2 weighted (84d), T1


weighted (84e) and fat saturated (84f)
axial images of the pelvis showing
dermoid cysts in both ovaries. Loss of
signal on the fat saturated image
confirms the high fat content.

143

Abdominal Imaging

CASE 85
History
Incidental liver lesion was noted on
abdominal ultrasound scan of a 26-yearold female.

85a

144

T2 weighted

T2 weighted (longer TE)

T1 weighted in phase

T1 weighted out of phase

Case 85

Abdominal Imaging

Case 85

85b

T1 weighted post Gad BOPTA 25 s

T1 weighted post Gad BOPTA 60 s

T1 weighted post Gad BOPTA 3 min

T1 weighted post Gad BOPTA 90 min

145

Answer 85

Abdominal Imaging

ANSWER 85
Observations (85a, 85b)
Selected pre- and post-contrast MRI images of the liver.
Pre-contrast images (85a) demonstrate a large, well defined
lesion in segment 4 of the liver, which is of slightly reduced
signal on T1 weighted images. Out of phase T1 weighted
images show no significant alteration of liver signal to
suggest the area is due to focal fatty sparing. The lesion is
slightly hyperintense on T2 weighted scans.
On dynamic post-contrast imaging (85b), the lesion
shows marked enhancement during early arterial (25 s)
phase with a central hypointense scar visible at this stage.
There is subsequent wash out of contrast in the portal (60
s) and equilibrium (3 min) phases to become isointense
with liver. The central scar shows enhancement in the
equilibrium phase.
The contrast agent used is gadolinium BOPTA (GdBOPTA) and the image taken at 90 min shows persisting
contrast uptake within the lesion indicative of hepatocyte
content. These appearances and pattern of enhancement
would best fit with the diagnosis of focal nodular
hyperplasia.

Diagnosis
Focal nodular hyperplasia (FNH).

Liver specific MRI contrast agents can help further.


In this case, a gadolinium agent with additional liver
specific properties (Gd-BOPTA) has been used. Most
gadolinium chelates in current use are extracellular
agents that are excreted via the kidneys. Gd-BOPTA
and Gd-EOB-DTPA are two agents currently in use
that also show a proportion of hepatic excretion. They
behave the same as conventional extracellular agents
during the arterial, portal and equilibrium phases of liver
MRI, but also illustrate functioning hepatocyte tissue
on delayed images. Since FNH contains hepatocytes
and abnormal bile ductules, such agents are retained
within FNH on delayed images (20 min to 2 h
depending on the agent). Other non-gadolinium based
liver specific MRI contrast agents can similarly help
characterize FNH on the basis of the functioning
hepatic tissue elements it contains.
Imaging features on angiography: these lesions
usually have a central supplying artery with centrifugal
filling giving a spoke wheel appearance; this can aid
differentiation from an adenoma, which has several
peripheral supply vessels.
Imaging features on sulphur colloid scan: usually
these lesions show normal activity due to the presence
of Kupffer cells.

Differential diagnosis
For hepatic lesion with central scar:
FNH.
Adenoma.
Hepatocellular carcinoma.
Haemangioma.
Fibrolamellar carcinoma.

Discussion
Focal nodular hyperplasia is a benign hepatic lesion that
occurs most commonly in young/middle aged women.
Usually these patients are asymptomatic, with diagnosis
made as an incidental finding, although some patients can
present with vague abdominal pain. They are solitary
(8595%) vascular lesions containing normal hepatic
elements Kupffer cells, hepatocytes and bile ducts.
Imaging features are very similar to those of hepatic
adenomas. Unlike hepatic adenomas, development is not
associated with the oral contraceptive pill, though
oestrogens can exert trophic effects.
Focal nodular hyperplasia is divided into typical and
atypical groups, with approximately 50% in each. Typical
lesions show the signal characteristics, enhancement pattern
and central scar as described below. Atypical lesions may
show signal heterogeneity, with more marked T1 and T2
hyperintensity, and no central scar.
Imaging features on CT and MRI: classical FNH is
isodense on CT, while MRI shows mild T1
hypointensity and mild T2 hyperintensity. There is
prominent enhancement in the arterial phase
following IV contrast, the central scar being
conspicuous at this point due to nonenhancement.
Contrast washes out rapidly in the portal and
equilibrium phases to become similar to normal liver,
though there may be scar enhancement in the
equilibrium phase.

146

Practical tips
Sulphur colloid scans have traditionally been used to
help confirm FNH suspected on CT and MRI by
virtue of the presence of Kupffer cells. Liver specific
contrast agents as described above may now obviate
this in many cases.
Hepatic adenoma is a common differential when
FNH is considered and there is evidence that agents
such as Gd-BOPTA can help differentiate FNH from
hepatic adenoma by demonstrating the presence of
functioning liver tissue in the former.

Further management
Differentiation from adenoma and hepatocellular
carcinoma can sometimes be difficult even with histological
information. Excision is then carried out.

Further reading
Schneider G, Grazioli L, Saini S (2005). MRI o f the
Liver. Springer, New York.

Chapter 3

CENTRAL NERVOUS
SYSTEM, HEAD AND
NECK IMAGING
CT AND MRI HEAD
As for all imaging, if an abnormality is not obvious at first
sight, check all areas systematically:
Establish if IV contrast has been given.
Ventricles check the configuration and size, ensuring
the size is appropriate for the other extra-axial CSF
spaces such as the sulci.
Cerebral hemispheres, brainstem and cerebellum on
MRI in particular, there is a lot of information to
evaluate for focal masses, signal abnormalities and
structural abnormalities! In examination vivas, it may
be unrealistic to expect the candidate to quickly detect
subtle abnormalities so you may well be given some
assistance or just presented with selected images. A
rapid evaluation of FLAIR (fluid attenuated inversion
recovery) or PD (proton density) sequences may
quickly lead you to pathology on MRI.
Extra-axial spaces carefully check for normal size,
absence of blood or other collections.
Vessels check for enhancement on CT and normal
flow voids on MRI (including vessels at the skull
base).
The periphery assess orbits, sinuses, temporal
bones/skull base and skull vault.
Everyone has their own blind spots but here are some
suggested areas to check when all else seems normal:
The temporal lobes and posterior fossa on CT
images of the temporal lobes are often degraded by
streak artefact and pathology easily missed. The same
applies to a lesser extent in the posterior fossa.
Skull base and petrous bone pathology can be easily
overlooked, especially on MRI where there is a lot to
assess.
Early signs of stroke the early changes of infarction
can be very subtle. Carefully check for subtle focal
swelling and reduced greywhite matter
differentiation. The latter is best detected by using
very narrow windows on CT.
Venous sinus thrombosis and subdural empyema are
two disorders with significant morbidity and mortality
that can be easily overlooked. When presented with an
apparently normal CT of a patient with a history of
convincing acute intracranial pathology, always double
check for increased density in the venous sinuses and
subtle subdural collections. Administration of IV
contrast may help confirm either.
Haemorrhage subtle subarachnoid or subdural
blood must not be missed. Check carefully for small

amounts of subarachnoid blood layering in the


occipital horns or around the medulla close to the
foramen magnum.
Pituitary this is well seen on sagittal MRI scans, so
do not forget to check it.
A note on mass lesions:
When confronted with an intracranial mass lesion, assess
the following:
Location certain tumours have a predilection for
certain areas of the brain.
Age this has a strong influence on the differential
diagnosis. Certain tumours are common in the
paediatric age group while metastases always have to
be considered in the older adult population.
Is the mass intra- or extra-axial that is, does it
originate within brain substance or not? The
distinction is very important in forming a differential
diagnosis and though it seems obvious, can
sometimes be difficult to establish on CT in
particular. An extra-axial mass causes crowding of the
subcortical white matter fronds that extend between
gyri. Conversely, an intra-axial mass stretches the grey
matter around it and consequently displaces the
fronds of white matter apart.
Mass effect and oedema check for midline shift and
effacement of normal CSF spaces such as sulci and
cisterns. If extra-axial CSF spaces are obliterated,
there is risk of death due to brain herniation through
dural apertures. If you see such features, then indicate
the degree of urgency required in seeking
neurosurgical evaluation and intervention.
Assess the enhancement pattern of the lesion. This is
often nonspecific but certain patterns such as the
intense, homogeneous enhancement of meningioma
may allow a confident diagnosis.

CT AND MRI NECK


Cross-sectional imaging of the neck requires excellent
anatomical knowledge! Familiarize yourself with the
anatomical divisions of the upper aerodigestive tract, the
fascial spaces of the neck along which tumour and infection
track, and the nodal stations of the neck. With this
knowledge, you can make a meaningful interpretation for
the ENT surgeon. T2 weighted scans with fat suppression
are particularly useful to evaluate at first inspection if these
are available, for detecting both primary pathology and

147

CNS, Head and Neck Imaging

nodal disease. Thereafter, simultaneous examination of the


T1 weighted scans can help to better delineate the precise
anatomical location of disease.

PLAIN RADIOGRAPHS
Skull
There are a very limited number of pathologies that are
likely to be shown on a skull radiograph:
Trauma
As well as skull fractures, check for indirect signs of fracture
such as fluid levels in sinuses and intracranial air. Remember
to look at all the bones on the film the fracture of
odontoid peg at the edge of the lateral skull film is a basic
exam type of case.
Calcification
Know the normal intracranial calcifications such as pineal
and choroid plexus so that you can differentiate from
pathological calcification such as that associated with
meningioma.
Pituitary
Size and shape of the pituitary fossa should be assessed.
Vault
Several diffuse processes have characteristic changes on the
skull radiograph and make for spot diagnoses, e.g.
acromegaly,
fibrous
dysplasia,
thalassaemia,
hyperparathyroidism and myeloma.

Spine
As always, there are many potential pathologies to see on
plain films of the spine but trauma, infection and neoplasia
are particularly common.
Trauma
When assessing cervical spine films:
First ensure that the complete cervical spine has been
imaged down to C7T1 level.
On the lateral film, look at the anterior spinal,
posterior spinal and spinolaminar lines to ensure
continuity (i.e. imaginary lines drawn along the
anterior and posterior vertebral body cortices and
along the anterior limits of the spinous processes).

148

Look at the soft tissues of the anterior cervical space,


which should be no greater than a third of the width
of a vertebral body for C13 and no more than the
width of a vertebral body for C47. Check the
interval between the anterior arch of atlas and the
odontoid peg this should normally be no greater
than 3 mm in adults and 5 mm in children.
On the AP film look for misalignment of the
vertebral spinous processes in a vertical plane, which
would suggest a facet joint dislocation. Ensure that
the AP peg view adequately excludes a C2 peg
fracture.
When you are suspicious of an unstable fracture, make
this clear so that adequate steps are taken to protect
the cervical spine until such time as this has been
confirmed or excluded with CT.
When assessing thoracic and lumbar films, carefully check
alignment and vertebral body height. On the AP film of
the thoracic spine, look for widening of the paravertebral
stripe suggestive of haematoma.
Malignancy
Bony involvement is one of the most common sites of
metastatic tumour spread along with the lungs and liver.
Look for focal lucency, sclerosis and destruction. On the
AP film, methodically check every pedicle, transverse and
spinous process for lytic deposits. As ever, pay attention to
the soft tissues a paravertebral soft tissue mass may guide
you to a subtle bony lesion. Do not forget that while
myeloma classically produces focal bony lucencies and
possible resulting vertebral collapse, a significant number
of cases show only diffuse osteopenia, which can mimic
osteoporosis.
Infection
Spinal infection is usually centred on the disc and therefore
there will be a reduction in disc height with involvement
of the superior endplate of the vertebral body below and
involvement of the inferior endplate of the vertebral body
above. Added soft tissue due to abscess is likely to be seen
and in the cervical spine will be seen as a prevertebral soft
tissue mass on the lateral cervical spine film; in the thoracic
spine this will be seen as a widening of the paravertebral
stripe; and in the lumbar region there may be a psoas
abscess.

CNS, Head and Neck Imaging

CASE 86

Cases 86, 87

86a

History
A 13-year-old presented with bilateral
progressive deafness.

CASE 87

87

History
A 10-year-old boy presented with
recurrent painful enlargement of the
parotid gland.

149

Answer 86

CNS, Head and Neck Imaging

ANSWER 86
Observations (86a)
Single high-resolution axial CT image of the skull at the
level of the petrous bone. There is bilateral enlargement of
the vestibular aqueduct. Precise measurements have not
been documented but the aqueducts are clearly significantly
wider than the horizontal semicircular canal seen on the
right.

Diagnosis
Enlarged vestibular aqueduct syndrome.

Discussion
Enlarged vestibular aqueduct syndrome presents clinically
with sensorineural hearing loss. Hearing is usually present
at birth and then deteriorates from age of ~3 years.
Deterioration is often in a stepwise manner, associated with
episodes of minor head trauma. There is a suggestion of an
inherited recessive genetic link and 50% of cases are
bilateral. A degree of cochlear dysplasia is present in 75%
of cases.
The normal endolymphatic duct originates from the
vestibule, via the common crus and extends posterolaterally
in the bony vestibular aqueduct to the endolymphatic sac.
The endolymphatic sac lies on the posterior aspect of the
petrous temporal bone. Enlarged vestibular aqueduct
syndrome is diagnosed when the diameter at its midpoint

exceeds 1.5 mm. Figure 86b indicates the dilated aqueduct


in this case.
Diagnosis of the condition is important due to the good
results achieved with cochlear implantation.

Practical tips
A quick assessment of the vestibular aqueducts can be
made by comparing the diameter at midpoint to the
diameter of the adjacent posterior semicircular canal
the aqueduct should not be larger.
When identified, check the cochlea for evidence of
dysplasia.

Further management
Advise on avoiding head trauma where possible, e.g.
no contact sports.
Hearing can be improved with cochlear implants.

Further reading
Dahlen RT, Harnsberger HR, Gray SD et al. (1997).
Overlapping thin-section fast spin-echo MR of the
large vestibular aqueduct syndrome. American Jo urnal
o f Neuro radio lo gy 18: 6775.
Valvassori GE (1983). The large vestibular aqueduct and
associated anomalies of the inner ear. Oto laryngo lo gic
Clinics o f No rth America 16: 95101.

86b

86b Axial CT image demonstrating an enlarged


vestibular aqueduct.

150

Answer 87

CNS, Head and Neck Imaging

Case 88

ANSWER 87
Observations (87)
Single oblique image from a parotid sialogram with no
control film provided for comparison. The parotid duct is
of normal calibre with no obstructing stones or stricture.
There is, however, florid punctate dilatation of the
intraglandular ductules and acini in keeping with punctate
sialectasis.

Although sialography is the primary method of


diagnosis, US is useful for follow-up as it avoids the
radiation burden and invasive nature of sialography. At US,
the gland is swollen with a heterogeneous appearance and
multiple hypoechoic foci within it. Intraglandular reactive
lymph nodes may be noted.

Practical tips
Diagnosis
Juvenile recurrent parotitis (also known as juvenile punctate
sialectasis).

Intraglandular lymph nodes are seen in the parotid but not


the submandibular gland, which becomes encapsulated
before lymph node development occurs embryologically.

Discussion

Further management

In adults, recurrent sialadenitis is usually associated with


stones or stricture causing duct obstruction. These causes,
however, are rare in children and chronic inflammation is
most commonly idiopathic in nature. The parotid is
affected more than the submandibular gland, perhaps due
to a comparatively smaller salivary output. Clinically,
episodes present with repeated attacks of swelling of the
gland and associated fever. Age of onset is typically 36
years, with symptoms usually resolving spontaneously after
puberty.

Although sialectasis most commonly idiopathic, Sjgrens


syndrome can also cause this appearance and should be
excluded with laboratory investigations.

Further reading
Yam KL, Lau C, Li CK (1997). Primary Sjgren
syndrome presenting as recurrent parotitis. Ho ng
Ko ng Jo urnal o f Paediatrics (New Series) 2: 4750.

CASE 88
History
A 35-year-old male presented with
diplopia.

88a

151

Answer 88

CNS, Head and Neck Imaging

ANSWER 88
Observations (88a)
Coronal T1 weighted MR images of the orbit show marked
bilateral swelling of the muscle bellies of the inferior, medial
and superior rectus muscles with sparing of the tendinous
insertions. This distribution of extraocular muscle swelling
is typical of thyroid ophthalmopathy.

Diagnosis
Thyroid ophthalmopathy.

Differential diagnosis
Orbital pseudotumour usually involves both muscle
belly and tendino us insertio n; is painful compared to
painless thyroid ophthalmopathy and is more commonly
unilateral.

Discussion
Thyroid ophthalmopathy is a disease of the orbit
characterized by deposition of mucopolysaccharides within
the muscle bellies of the intraorbital muscles. It usually
presents in adults and is more common in women. The
patient is usually hyperthyroid (although 10% of cases are
found in euthyroid patients) with ocular presentation
within 1 year of hyperthyroid symptom onset. Severity of
the eye disease, however, is not related to the severity of
the thyroid hormone imbalance.
Radiological features are as follows:
Majority of cases are bilateral although there is often
asymmetrical disease. About 10% are unilateral.
Inferior and medial rectus are most commonly
affected, with the lateral rectus being least likely to be
affected.

Muscle bellies are affected with swollen appearance


and relative sparing of the tendinous insertions. Axial
CT images (88b) show swollen medial recti with
typical sparing of the tendinous insertions.
Proptosis, lid retraction and reduced eye movements
are features.
Usually the disease resolves spontaneously but steroid
treatment is prescribed to treat the swelling and to reduce
intraorbital pressure, which can otherwise cause optic nerve
and ophthalmic vein compression. Surgical decompression
is sometimes required when there is acute swelling and
threatened visual loss.

Practical tips
The order of muscles involved in thyroid
ophthalmopathy is (from most to least frequent)
Inferior>Medial>Superior>Lateral>Oblique Mnemonic
= IM SLO(w).

Further management
Thyroid ophthalmopathy with visual disturbance requires
prompt evaluation and treatment given the potential risk
to visual function.

Further reading
Hosten N, Sander B, Cordes M, et al. (1989). Graves
ophthalmopathy: MR imaging of the orbits.
Radio lo gy 172: 759762.

88b

88b Selected axial CT images through the orbits show thickening of the medial rectus muscles with sparing of
their tendons.

152

CNS, Head and Neck Imaging

Case 89

CASE 89
History
A 21-year-old male presented with
positional headaches.

89a

89a Axial PD (top left), axial and sagittal T1 weighted (left), axial and coronal T2 weighted
(right) MR images.

153

Answer 89

CNS, Head and Neck Imaging

ANSWER 89
Observations (89a)

Practical tips

Selected MRI images demonstrate a round lesion in the


anterosuperior aspect of the 3rd ventricle. The lesion is
mildly hyperintense on T1 weighted images, with variable
signal on T2 weighted images and a graduated fluid level
appearance. It is likely that the contents are proteinaceous
fluid and the findings are consistent with a colloid cyst of
the 3rd ventricle.

A large basilar tip aneurysm can be mistaken for a


colloid cyst on unenhanced CT scan and this requires
early identification and treatment.
Need to assess whether or not there is hydrocephalus
look at the degree of dilatation of the temporal
horns of the lateral ventricles and look for depression
of the brainstem.

Diagnosis

Further management

Colloid cyst.

Although these are benign lesions they can present acutely


with CSF flow obstruction resulting in headache, loss of
consciousness and death, so a clinical review for evidence of
elevated intracranial pressure and neurosurgical review are
appropriate.

Differential diagnosis
For 3rd ventricular lesion:
Meningioma these are not usually hyperintense on
T1. Meningiomas commonly calcify and show diffuse
enhancement with contrast.
Arachnoid cyst these are usually isodense on CT and
isointense with CSF on MRI.
Dermoid cyst usually found in the midline. These
contain fat and are therefore usually hypodense on CT
and have mixed signal on MRI.
Ependymoma of the 3rd ventricle these are very
rare. Imaging features include cystic areas, necrosis
and calcification and they show diffuse uniform
enhancement.
Basilar tip aneurysm.

Discussion
Colloid cysts account for approximately 0.51% of CNS
tumours. They usually present in young adults and are
more commonly seen in males. The cysts arise from the
inferior aspect of the septum pellucidum and extend into
the 3rd ventricle. Presentation is usually secondary to
obstructed CSF flow at the level of the 3rd ventricle with
features of positional headache due to transient obstruction,
disturbances of gait, reduced consciousness level and
papilloedema. The majority of cysts (80%) contain
mucinous material while 20% contain fluid similar to CSF.
Typical features on CT: well defined thin walled cyst
in the 3rd ventricle of slightly increased attenuation
with no enhancement (89b). Can cause erosion of the
sella.
Typical features on MRI: mucinous material contained
within the cyst produces a well defined lesion of
increased signal intensity on T1 and T2 weighted
images.

89b

89b Typical CT appearances of a colloid cyst are of a


well defined, thin walled cyst in the 3rd ventricle
showing increased attenuation without
enhancement.

154

CNS, Head and Neck Imaging

CASE 90

Cases 90, 91

90a

History
A 68-year-old male presented with
pyrexia and fits.

CASE 91

91

History
A 54-year-old male presented with
bilateral visual field blurring.

155

Answer 90

CNS, Head and Neck Imaging

ANSWER 90
Observations (90a)
This contrast enhanced CT image of the brain
demonstrates multiple ring enhancing lesions with
surrounding vasogenic oedema in the left frontal lobe.
Though no lesions are seen in the right hemisphere on this
single image, apparent vasogenic oedema in the right
frontal lobe suggests that there may be further lesions on
this side that are not visualized. The differential diagnosis
for ring enhancing lesions is long, but the history of pyrexia
points towards cerebral abscesses being most likely.

Practical tips
Differentiation of the above list can be difficult and
history is very important to identify immune status,
foreign travel and clinical presentation.
Look for complications of abscesses such as rupture
causing meningitis or ventriculitis, radiologically
identified by enhancement of these structures or high
signal in the sulci on FLAIR MR images.
Wall enhancement of cerebral abscess is typically
uniform thick, irregular wall enhancement should
raise suspicion of cerebral metastases.
A peripheral ring on the unenhanced scan is more
common in metastasis than glioma.
Assess paranasal sinuses for a primary source of
infection and ensure a chest radiograph has been done
to look for sources of infection and potential right to
left shunts.

Diagnosis
Cerebral abscesses.

Differential diagnosis
For ring enhancing CNS lesions:
Cerebral abscess.
Metastases.
Toxoplasmosis.
Demyelination.
Lymphoma.
Multicentric glioma.
Multiple infarcts.
Resolving haematomas.

Discussion
Patients who are immunocompromised, on steroids or have
diabetes are more susceptible to CNS abscess formation.
The most common source of infection is haematogenous
spread, though direct spread from infected paranasal sinuses
can also occur. Clinical presentation is with headache,
seizure and pyrexia. Focal infection preceding abscess
formation is cerebritis, and appears as focal low-density
change on CT or high T2 signal on MRI (90b, 90c). This

90b

proceeds to capsulation/abscess formation at 1013 days,


with the ring enhancement illustrated. When arising via
haematogenous spread, the classical location is at the
greywhite matter interface, and most commonly in the
middle cerebral artery territory. The medial wall is often
thinnest (due to better perfusion of the grey matter
laterally than the white matter medially) and results in a
tendency to rupture on this aspect into the ventricle.

Further management
MRI with diffusion weighted imaging can sometimes help
differentiate tumour/metastases from abscess. Restricted
diffusion of pus in an abscess cavity means that an abscess
is high signal on DWI and low signal on ADC (apparent
diffusion coefficient) mapping. Opposite findings are seen
with necrotic tumours.

90c

90b, 90c Axial T1 weighted postcontrast and coronal T2 weighted MRI images
of left temporal lobe abscesses. The axial T1 postcontrast image shows an
enhancing rim with surrounding low signal oedema. The coronal image shows
a high signal abscess with significant high signal surrounding oedema causing
a moderate degree of mass effect and midline shift.

156

Answer 91

CNS, Head and Neck Imaging

Case 92

ANSWER 91
Observations (91)

Discussion

Axial CT image through the orbits at the level of the optic


nerves demonstrates bilateral focal, discrete calcification at
the point of union of the retina and optic nerve.

Optic drusen are focal accumulations of hyaline material


in the region of the optic nerve head, which commonly
calcify. Aetiology is thought to be either a developmental
abnormality or a degenerative process and it is
histopathologically separate from retinal drusen deposits
that can be a normal finding or associated with age related
macular degeneration. Clinically, patients are usually
asymptomatic but can present with reduced visual acuity,
migraine-like headaches and pseudopapilloedema. Diagnosis is made by the absence of adverse imaging features
such as abnormal enhancement, optic nerve thickening,
mass effect or posterior globe solid lesions. Ocular ultrasound can be particularly useful in evaluating, and
confirming this is not actually papilloedema. Optic disk
haemorrhage is a very rare complication. The majority of
cases (75%) are bilateral.

Diagnosis
Optic drusen.

Differential diagnosis
For ocular calcification:
Neoplasic causes:
Retinoblastoma (most common cause, accounting
for >50% of cases).
Astrocytic hamartoma.
Choroidal osteoma.
Infection:
Toxoplasmosis.
Rubella.
Cytomegalovirus (CMV).
Herpes simplex.
Other:
Optic drusen.
Phthisis bulbi.
Retinal detachment.
Retinopathy of prematurity.
Hypercalcaemic states hyperparathyroidism,
sarcoidosis, chronic renal failure.

CASE 92

Practical tips
Bilateral calcification does not necessarily suggest a
benign disease process. The nonheritable form of
retinoblastoma (66%) presents at ~24 months with
usually unilateral disease; but the heritable form (33%)
presents at ~12 months with often (66%) bilateral
disease.

Further management
Ophthalmological assessment is required to confirm the
diagnosis and check visual fields and acuity.

92

History
A young adult male presented with
headaches and paralysis of upward gaze.

157

Answer 92

CNS, Head and Neck Imaging

ANSWER 92
Observations (92)
This midline sagittal T1 post-contrast MR image
demonstrates a large, slightly lobulated but well defined
mass in the pineal region. The mass enhances homogeneously, compresses the superior colliculus and causes
dilatation of the 3rd ventricle. The most likely diagnosis is
a pineal germinoma causing Parinauds syndrome and
obstructive hydrocephalus.

Diagnosis
Pineal germinoma.

Differential diagnosis
Of pineal region masses:
Germ cell tumours (>50%):
Germinoma.
Teratoma extremely heterogeneous mass. Occurs
in young children.
Choriocarcinoma.
Embryonal cell tumour.
Pineal parenchymal tumours (25%):
Pineocytoma well demarcated, calcified, slow
growing tumour in middle aged adults.
Pineoblastoma similar to medulloblastoma.
Affects young children. Enhances avidly and is not
usually well circumscribed.
Others:
Meningioma.
Epidermoid or dermoid.
Arachnoid cyst CSF density/signal.
Pineal cyst common; fluid density or signal
though contents can be proteinaceous on MRI.
Lipoma.

Discussion
The pineal gland is a midline structure situated behind the
3rd ventricle and responsible for biorhythm. It is calcified in
most people over the age of 15 years and in almost all

92

92 Pineal germinoma.

158

elderly. Germ cell tumours are the most common tumours


of the pineal region, accounting for over 50% of all pineal
masses. The most common subtype is germinoma, which is
histologically similar to testicular seminoma. Pineal
germinomas are well defined midline masses that are much
more common in males than females. They are normally
seen in children and young adults. The lesion enhances
avidly. Due to the anatomical location of the pineal gland
these lesions compress the aqueduct of Sylvius producing
hydrocephalus. The superior colliculus of the brainstem
may also be compressed producing Parinauds syndrome:
paralysis of upward gaze. Less common germ cell tumours
of the pineal gland include teratomas, which are mostly
seen in young children. Choriocarcinomas and embryonal
cell tumours are even less common and are highly
malignant.

Practical tips
The pineal region is in the midline and masses in this
region may compress the aqueduct of Sylvius, so
always look for associated hydrocephalus.
When the normal calcification in the pineal gland
exceeds 1 cm in diameter a pathological pineal process
should be suspected.
Calcification in germinoma when present is central, in
pineoblastoma peripheral.
In young children think of teratoma (which are very
heterogeneous masses), pineoblastoma and a vein of
Galen aneurysm.
In young adults, the most common solid lesion is a
germinoma.

Further management
Surgery is difficult due to the central location in the brain.
The main role for surgery is in obtaining a biopsy and
possibly debulking of tumour to relieve obstructive
symptoms. However, germinomas are extremely
radiosensitive and therefore radiotherapy is the mainstay
of treatment.

CNS, Head and Neck Imaging

Case 93

CASE 93
History
A 35-year-old male patient presented
with headache.

93a

93a PD and T2 weighted axial (top), T2 weighted coronal and T1 weighted sagittal (bottom) images.

159

Answer 93

CNS, Head and Neck Imaging

ANSWER 93
Observations (93a)
Selected MR images of the brain demonstrate
hyperintensity in the right transverse sinus on the T1 and
T2 weighted images with an absence of the normal venous
sinus flow voids. There is high-signal abnormality seen
within the right mastoid air cells on T2 weighted images,
which is intermediate signal on T1 imaging this is likely to
indicate infection/inflammatory change. Appearances are
consistent with a diagnosis of venous sinus thrombosis
secondary to mastoiditis. No intracerebral/cerebellar
haemorrhage or infarct is demonstrated on these images.

Diagnosis
Venous sinus thrombosis.

Discussion
There are a variety of underlying causes of venous sinus
thrombosis, which include trauma, infection, idiopathic and
hypercoagulable states, i.e. oral contraceptive pill,
antiphospholipid syndrome, paraneoplastic tumour
syndromes, antithrombin III deficiency. Presenting
symptoms can often be very nonspecific, meaning that
diagnosis is often only made radiologically. Symptoms
include headache, nausea, vomiting and drowsiness. The
superior sagittal sinus is most commonly affected, followed
by transverse and sigmoid sinuses.

Radiological appearances are:


Uncontrasted CT may show a hyperdense venous
sinus due to thrombus (Figure 93b shows a
thrombosed superior sagittal sinus). Contrast
enhanced CT shows a filling defect within the
triangular lumen outlined by a small rim of contrast
the empty delta sign.
MRI demonstrates an absence of flow void within the
venous sinus, with local effects of oedema, subcortical
infarction, sulcal effacement and haemorrhage.
In the acute phase, thrombus can appear as
isointense on T1 weighted imaging and
hypointense on T2 weighted imaging. This
hypointensity on T2 can sometimes be mistaken
for flow void and therefore phase-contrast MRA,
which shows flow, is better at identification.
In a chronic thrombosis, the venous sinus appears
hyperintense on T1 and T2 weighted images due
to extracellular methaemoglobin.
Venous infarcts are identifiable by their
nonconformity with arterial territories and
haemorrhagic tendency. Figure 93b demonstrates
a venous infarct that does not conform to the
anatomical area supplied by the vessels of posterior
circulation.

93b

93b Selected axial CT images demonstrate a focal area of low attenuation involving grey and white matter
with appearances consistent with an infarct. The distribution, however, does not conform to arterial territories
since this represents a venous infarct.

160

Answer 93

CNS, Head and Neck Imaging

Practical tips
Venous sinus thrombosis can present in a very
nonspecific manner and has significant morbidity and
mortality untreated. When assessing scans of acutely
neurologically unwell patients, always keep it in mind
and check the scan carefully. Similarly, beware of
dismissing infarcts that show features as described
above without questioning the possibility of an
underlying venous sinus thrombosis.
The appearances of thrombus on MRI are complex
and vary with the age of thrombus. Moreover, patent
venous sinuses can show absence of flow void when
imaged in certain planes. Making the diagnosis of
venous sinus thrombosis from MRI can therefore be
complicated. Modern multidetector CT scanners have
sufficient speed and spatial resolution to image the
venous sinuses and provide an alternative that can be
easier to interpret, thrombus appearing as a filling
defect in the lumen of the otherwise enhanced venous
sinus. Figure 93c is a coronal reformat of a CT
venogram showing thrombus in the left transverse
sinus as an empty delta sign.

Further management

Case 94

93c

93c Coronal reformat CT image shows thrombus in


the left transverse sinus.

Usual treatment involves anticoagulation.

CASE 94

94a

History
A 57-year-old male presented
with tinnitus and hearing loss.

161

Answer 94

CNS, Head and Neck Imaging

ANSWER 94
Observations (94a)
Single T2 weighted MR image of the brain at the level of
the internal auditory meatus. There is a large, well defined
soft tissue mass lesion in the right cerebellopontine (CP)
angle with extension into the internal auditory canal.
Widening of the canal is seen. No dural tail is evident.
Acoustic neuroma is most likely.

Diagnosis
Acoustic neuroma.

Differential diagnosis
Of CP angle lesions:
Acoustic neuroma accounts for 75% of CP angle
masses.
Meningioma is the most likely differential at 10%
(94b). Lesions are extra-axial and tend to be
extracanalicular and commonly calcify. Dural tails are
seen in up to 60% of tumours on MRI.

94b

162

Epidermoid 5% of CP angle lesions.


Arachnoid cyst.
Posterior circulation aneurysm.
Metastases.

Discussion
Acoustic neuromas are the most common tumours of the
CP angle and internal auditory canal. They typically arise
from the vestibular division of the 8th nerve and should
perhaps be more accurately termed vestibular schwannomas. These lesions present in the 4th7th decades and
are more frequently seen in females. Presentation occurs
at a younger age in patients with type 2 neurofibromatosis,
with presentation in the 2nd decade. Bilateral acoustic
neuromas are virtually pathognomonic of neurofibromatosis type 2, while solitary tumours are seen in up
to 25% of cases. Presentation is with symptoms of
sensorineural hearing loss, tinnitus, vertigo and dizziness.

94b Axial PD, T2 and T1 weighted


images with three post-contrast
coronal images demonstrate a well
defined, extracanalicular, uniformly
enhancing lesion in the left CP
angle representing a meningioma.

Answer 94

CNS, Head and Neck Imaging

Imaging features:
80% arise in the internal auditory canal (IAC).
80% cause enlargement/erosion of the IAC.
Lesions extend into the CP angle.
Larger tumours (>3 cm) have central areas of necrosis
and haemorrhage.
Calcification is not a feature.
On CT lesions are usually isodense with uniform
enhancement.
On MRI, lesions are low signal on T1, high signal on
T2 and show uniform enhancement with gadolinium.

Practical tips
Distinguishing the main CP angle masses from each
other:
Both acoustic neuroma and meningioma show
uniform enhancement so this is unhelpful.
Acoustic neuroma expands the IAC, causing
flaring of the porus acousticus. May be bright on
T2 unlike meningioma. Makes an acute angle with
petrous bone.
Meningioma dural tail of enhancement, obtuse
angle with petrous bone. Relatively little tissue in
the IAC compared to acoustic neuroma.
Epidermoid vs arachnoid cyst both appear to
follow the density and signal of CSF on CT and T1
and T2 weighted MRI. However, the epidermoid
shows increased signal on PD, FLAIR and diffusion
weighted MRI.

CASE 95

Case 95

Detection of acoustic neuroma is largely done with


MRI when possible thin section T2 weighted
sequences are used in the main, with contrast reserved
for difficult cases. When contraindicated, use contrast
enhanced CT.
A subgroup of acoustic neuroma show relatively rapid
growth that necessitates treatment while others show
extremely slow growth and can be managed more
conservatively. Following initial diagnosis, a repeat
scan after approximately 6 months may help identify
the subgroup that is going to grow rapidly and those
that can be managed more conservatively.
Neurofibromatosis type 2: bilateral acoustic neuroma
is a recognized diagnostic criterion for this condition,
also known as the MISME syndrome (multiple
inherited schwannoma, meningioma and
ependymoma). In the real world most acoustic
neuromas are unrelated to neurofibromatosis type 2.
However, in exam vivas (and certainly when bilateral),
have a high index of suspicion for the meningioma
elsewhere on the scan.

Further management
Therapeutic options involve surgical excision, stereotactic
radiation and conservative management. Which one
depends on the particular circumstances of each patient,
but also whether the tumour is fast growing or not. A
repeat scan a few months after the first can help identify
the group of faster growing lesions that require a more
aggressive approach.

95a

History
A 30-year-old male:
orthopantomogram
(OPG) done for dental
assessment.

163

Answer 95

CNS, Head and Neck Imaging

ANSWER 95
Observations (95a)
The OPG shows multiple missing teeth in the upper jaw
and to a lesser extent the lower jaw too. In the right body
of the mandible, there is a well circumscribed, thin walled,
unilocular lucency measuring several centimetres in
diameter. There is no associated bony destruction. The cyst
is associated with the crown of an unerupted molar and
appearances are consistent with a dentigerous cyst.

Diagnosis
Dentigerous cyst.

Differential diagnosis
Of cystic mandibular lesion:
Periapical/radicular cyst.
Dentigerous cyst.
Odontogenic keratocyst.
Ameloblastoma.
Aneurysmal bone cyst.
Simple bone cyst.
Metastasis/myeloma.
Brown tumour.
Fibrous dysplasia.
Many other small print lesions nasopalatine cyst,
Pindborgs tumour, Stafnes bone cyst, median
mandibular cyst, etc.

Discussion
Dentigerous cysts arise from the crown of an unerupted
tooth. The typically affected age group is 1030 years, with
a male predominance. The cysts are slow growing and
usually an incidental finding, but occasionally cause pain,
swelling or become secondarily infected. The majority
occur in the posterior mandible. They are variable in size,
and have a thin walled, unilocular appearance. The
asso ciatio n with an unerupted cro wn is the key to diagno sis.
The differential diagnosis of cystic lesions of the
mandible is very long but most of the causes are so rare that
they can be dismissed most of the time. The emphasis will
consequently be on common lesions and important rare
ones. It is best to assess lesions according to visible
association with dentition, unilocular vs multilocular nature
and age of the patient. Note that although not visibly
dentition related, lesions such as ameloblastoma and
odontogenic keratocyst do arise from dental-related tissue.

95b

164

Dentition-associated cysts:
Periapical/radicular cyst secondary to pulp necrosis
of a carious tooth. Unilo cular cyst asso ciated with the
ro o t o f a to o th. Can be destructive when large, but
not expansile. Most common lesion seen. Likely to be
painful.
Dentigerous cyst unilo cular cyst asso ciated with the
cro wn o f an unerupted to o th.
No visible association with teeth:
Ameloblastoma/adamantinoma rare, locally
aggressive (but non-metastasizing) lytic tumour that
is slow growing and often painless. This is an
expansile, multilo culated cyst with bubbly
appearance (though 20% unilocular). Can be
associated with unerupted molar tooth. Thinned
cortex and no matrix mineralization. Contrast
enhanced CT/MRI may show enhancement of soft
tissue elements and possibly an enhancing mural
nodule. Five times more common in mandible than
maxilla. Typical age group 3050 years. Figure 95b is
an OPG in an adult patient with ameloblastoma
showing a lytic expansile lesion of the right side of the
mandible with areas of internal septation producing a
soap bubble appearance. The lesion is locally
aggressive and has destroyed several right lower teeth
roots.
Odontogenic keratocyst (OKC) multilo culated o r
unilo cular cyst o ften near site o f 3rd lo wer mo lar;
expansile extending along the mandible; sclerotic rim;
may displace teeth. Same age and sex group as
dentigerous cyst; 50% are symptomatic with swelling,
growth rate is rapid and recurrence rate after
curettage is high. Multiple OKCs are a feature of
GorlinGoltz (basal cell naevus) syndrome, an
autosomal dominant condition characterized by
multiple cutaneous basal cell carcinomas.
Aneurysmal bone cyst (ABC) multilo cular cyst with
so ap bubble appearance in po sterio r mandible.
Appearances are therefore very similar to
ameloblastoma but ABC occurs primarily in those
under 20 years of age.
Fibrous dysplasia ground glass matrix with
calcificatio n.
Metastases and myeloma dont forget these in the
older patient.

95b OPG shows large lytic ameloblastoma in the


right mandible with areas of internal septation
producing a soap bubble appearance.

Answer 95

CNS, Head and Neck Imaging

Practical tips
The cystic mandibular lesion ask yourself:
Any relation to tooth?
Multilocular or unilocular?
Age of patient?

Case 96

If brown tumour is suspected look for other signs of


hyperparathyroidism such as resorption of the lamina dura
of the teeth producing floating teeth and generalized
demineralization of the mandible and maxilla.

Further management
Multilocular cyst:
<30 years = ABC or OKC.
Multiple = OKC with Gorlin syndrome.
>30 years = ameloblastoma.
Ground glass contents OKC or fibrous dysplasia
(but matrix calcification in the latter).
Unilocular cyst with dental association:
Associated with tooth apex and pain = periapical cyst.
Associated with unerupted tooth = dentigerous cyst
(<30 years and associated with crown) or
ameloblastoma (>30 years).

Though often asymptomatic, dentigerous cyst can cause


pain and swelling and may predispose to pathological
fracture. Surgical excision (including the unerupted tooth)
may therefore be undertaken.

Further reading
Scholl R, Kellett H, Neumann D, Lurie A (1999). Cysts
and cystic lesions of the mandible: clinical and
radiologic-histopathologic review. Radio Graphics 19:
11071124.

Unilocular cyst with no dental association:


<30 years = OKC, simple bone cyst.
>40 years = ameloblastoma, metastases or myeloma.

CASE 96

96a

History
A 30-year-old patient presented with
headaches and suspected sinusitis.

165

Answer 96

CNS, Head and Neck Imaging

ANSWER 96
Observations (96a)

Practical tips

This occipitomental radiograph shows a large calcified


lesion in the right frontal sinus. The lesion has very well
defined margins and a slightly lower density centre. No
fluid is seen within the sinus and there is no evidence of
local bony destruction. The appearances are those of a
frontal sinus ivory osteoma.
Ivory osteoma.

When multiple osteomas are seen always consider


Gardners syndrome and investigate the family history
and for the presence of colonic polyposis.
Paranasal sinus osteomas may cause local pressure
erosion. A coronal CT reconstruction (96c)
demonstrates a left ethmoid sinus osteoma eroding
the lamina papyracea and extending into the left
orbit. Background sinus inflammatory changes are
present.

Discussion

Further management

Osteomas are benign tumours of membranous bone. These


are round, well defined lesions of bony density that are
usually found incidentally. They are commonly found in
the paranasal sinuses, particularly the frontal and ethmoid
sinuses. Other common sites include the calvarium and
mandible. A CT coronal reconstruction (96b) in the same
patient confirms that the lesion lies within the right frontal
sinus.

Incidental finding requiring no further management.

Diagnosis

96b

96c

96c Coronal CT reconstruction showing an osteoma


of the left ethmoid sinus extending into the left orbit.

96b Coronal CT reconstruction demonstrating a right


frontal sinus osteoma.

166

CNS, Head and Neck Imaging

CASE 97

Case 97

97a

History
A 39-year-old female presented with
transient visual loss; and an episode of
ataxia 6 months ago.

97b

167

Answer 97

CNS, Head and Neck Imaging

ANSWER 97
Observations (97a, 97b)
Selected MR images of the brain FLAIR axial and T2
weighted sagittal are presented. These scans show
multiple focal ovoid signal abnormalities in the
periventricular white matter, which are orientated
perpendicular to the long axis of the ventricles. Further
lesions involve the corpus callosum. No associated oedema.
No evidence of hydrocephalus. Given the clinical details, it
is likely that the patient has multiple sclerosis and is
currently experiencing optic neuritis.

findings on MRI with several aetiologies. Making a


specific diagnosis of demyelination is often not
possible from imaging alone though multiple lesions
involving the corpus callosum are characteristic for
MS. Coexisting lesions in the brainstem, cerebellum
and spinal cord also increase specificity.
Be wary of making a conclusive diagnosis of MS from
MRI unless features are highly specific and there is a
supporting history the consequences for the patient
are significant. Often, one can only offer a differential
diagnosis for UBOs that includes MS.

Diagnosis
Multiple sclerosis (MS).

Differential diagnosis
Of white matter lesions on MRI:
Acute disseminated encephalomyelitis (ADEM).
Vasculitis.
Ischaemic disease (97c).
Migraine.
Neurosarcoid.

Discussion
This is the most common chronic demyelinating disease
and is characterized by multiple lesions spread in time and
space. Typically, it has a remitting /relapsing course. The
onset of symptoms is usually in the 3rd4th decades and
there is a slightly increased predominance in females.
Increased prevalence is noted in areas of temperate climate.
Clinical presentation is with focal neurological signs,
commonly including optic neuritis.
Imaging features are:
Lesions are classically periventricular in location, oval
in shape, with their long axis perpendicular to the
lateral ventricle walls Dawsons fingers.
Common locations for plaques include periventricular
white matter, corpus callosum, internal capsule,
centrum semiovale, optic nerve/tracts, cerebellum.
Lesions on the inferior aspect of the corpus callosum
are characteristic.
Acute lesions can have mild surrounding oedema and
can enhance with contrast.
Chronic lesions have no mass effect/oedema and
dont enhance.
Lesions are hyperintense on T2, hypo/isointense on
T1.

Further management
MRI of the spinal cord is frequently also undertaken to
complete assessment. Though MRI findings may be highly
suggestive, careful neurological assessment is required to
correlate the imaging findings with clinical and laboratory
findings before making the diagnosis.

Further reading
Runge VM, Price AC, Kirshner HS, et al. (1986). The
evaluation of multiple sclerosis by magnetic resonance
imaging. Radio Graphics 6: 203212.

97c

Multiple sclerosis is also the most common demyelinating


process affecting the spine the cervical spine being most
frequently affected. It is characterized by plaques orientated
along the axis of the spinal cord.
Further clinical evaluation involves:
Lumbar puncture with CSF analysis for oligoclonal
bands.
Electrophysiological studies.

Practical tips
Focal white matter signal abnormalities in cerebral
white matter (sometimes referred to as unidentified
bright objects UBOs) are common incidental

168

97c Single axial MRI FLAIR image shows


periventricular high signal with scattered high
signal white matter foci. Appearances are of agerelated small vessel disease.

CNS, Head and Neck Imaging

CASE 98

Case 98

98a

History
A 51-year-old male presented with
chronic headaches.

98b

169

Answer 98

CNS, Head and Neck Imaging

ANSWER 98
Observations (98a, 98b)
This lateral image of the skull (98a) shows elongation of
the jaw (prognathism), frontal bossing, enlargement of the
frontal sinuses and thickening of the calvaria. There is
marked enlargement of the pituitary fossa with evidence of
expansion but no erosion.
Selected sagittal T1 weighted images (98b) of the brain
confirm the plain film findings of enlargement of the frontal
sinuses, frontal bossing and thickening of the calvaria.
There is a mass lesion within the pituitary fossa with
appearances consistent with pituitary macroadenoma.
Enlargement of the tongue is also noted.

Plain radiographic image findings are:


Enlargement of the sella (98c).
Mandibular enlargement.
Increase in size of the frontal sinuses with prominence
of the supraorbital ridge.
Enlarged hands with spade-like appearances of the
terminal phalanges (98d).
Thickening of the calvaria.
Dural ectasia and posterior vertebral body scalloping.
Increased heel pad thickness( >25 mm) (98e).
Premature osteoarthritis (OA).

Practical tips
Diagnosis
Acromegaly.

Normal dimensions of the pituitary fossa on lateral skull


films are a length of <15 mm and height of <12 mm.

Discussion

Further management

Excess growth hormone secretion by the anterior lobe of


the pituitary gland results in a variety of musculoskeletal
abnormalities.

Management of this condition is both medical (somatostatin/bromocriptine) and surgical (transsphenoidal


hypophysectomy).

98c

98d

98c Magnified image from plain skull radiograph


shows enlargement of the sella.

98e

98e Soft tissue lateral radiograph demonstrates


thickening of the heel pad, which measures >25 mm
in thickness.

170

98d Radiograph
of the hand
shows marked
soft tissue
enlargement
giving it a spadelike appearance.
In addition there
is widening of the
terminal tufts.

CNS, Head and Neck Imaging

Case 99

CASE 99
History
A 22-year-old female presented with
visual field defects.

99

99 T1 weighted axial (top left), thin section T2 weighted axial orbits (top right), axial and coronal T2 weighted
(bottom) MR images.

171

Answer 99

CNS, Head and Neck Imaging

ANSWER 99
Observations (99)
The axial images of the orbit show fusiform enlargement
of the posterior right optic nerve, which also involves both
sides of the optic chiasm. The coronal scan confirms
thickening of the posterior aspect of both optic nerves.
Axial T2 weighted images also show a round focus of high
signal in the right cerebellar hemisphere and a second lesion
in the posterior aspect of right midbrain that produces
convexity to the margins of the cerebral peduncle. The
appearances are likely to indicate optic nerve glioma
involving the chiasm and both optic nerves along with
hamartomas in the cerebellum and midbrain due to
neurofibromatosis type 1. No cutaneous neurofibromas are
seen on these images.

Diagnosis
Optic chiasm/nerve glioma due to neurofibromatosis type
1 (NF1).

Differential diagnosis
For optic nerve thickening:
Optic nerve glioma 80% under 20 years, variable
enhancement, calcification rare, buckling of nerve,
often asymptomatic.
Meningioma of optic nerve middle aged women,
tramtrack enhancement, calcification in 2050%,
straight nerve, visual impairment early.
Sarcoidosis.
Multiple sclerosis.
Lymphoma, leukaemia and metastatic disease.
Intracranial hypertension enlarges the perineural
CSF space.

Discussion
Optic nerve glioma typically presents in childhood, only
20% manifesting beyond the age of 20. Relatively slow
growing and benign in children, lesions presenting in adults
often show more rapid malignant growth with intracranial
spread. Though often asymptomatic, presentation can be
with visual loss and strabismus. Bilateral tumours herald
NF1. The tumour causes fusiform or tubular enlargement
of the optic nerve sheath complex and shows variable
enhancement with IV contrast. The majority of lesions
occurring in the orbital portion of the nerve do not extend

172

intracranially. Some optic nerve gliomas have extensive


associated thickening of the surrounding meninges, termed
arachnoidal hyperplasia, which is often seen in patients with
NF on T2W MRI, this is seen as a central low-signal
tumour surrounded by a higher-signal rim that can look
like a dilated perineural CSF space.
Approximately 25% of patients with optic glioma have
NF1 and it is one of the diagnostic criteria listed for the
condition, of which two or more are required: six or more
caf-au-lait patches; two or more Lisch nodules; two or
more neurofibromas or one plexiform neurofibroma;
axillary freckling; optic glioma; bone dysplasia or
pseudarthrosis; first degree relative with NF1. In this case,
the coexisting lesions in the cerebellum and midbrain lead
one to the overall diagnosis of NF1. These high T2 signal
foci are seen in the brainstem, basal ganglia, cerebral
peduncles, cerebellum and the supratentorial white matter.
They are usually thought of as representing hamartomas
and often decrease in size with age. NF1 also predisposes
to astrocytoma in the cerebrum, brainstem and cerebellum
and telling the difference may be difficult. If a lesion
enlarges over time or shows enhancement, then the
possibility of astrocytoma must be considered. Prominent
choroid plexus calcification and hydrocephalus due to
aqueduct stenosis are other intracranial features.
NF1 is sporadic in 50% and autosomal dominant in 50%
(chromosome 17).

Practical tips
T1 weighted post-contrast scans of the orbit should
be performed with fat suppression in view of the
adjacent orbital fat.
Meningioma is the main differential diagnosis for
optic nerve thickening and the features listed above
may help distinguish the two.
Whenever NF1 enters the differential for any
radiological study, always check for evidence of
cutaneous nodules that clinch the diagnosis.

Further management
Treatment depends on the size of tumour along with the
age and general condition of the patient. Options include
surgery, radiotherapy and chemotherapy.

CNS, Head and Neck Imaging

Case 100

CASE 100
History 1
A 73-year-old male presented with
acute onset of dysphagia, right sided
Horners syndrome and ataxia, with left
sided sensory disturbance.

100a

History 2
A 69-year-old female with diabetes and
hypertension presented with sudden
onset right oculomotor palsy and left
sided ataxia and tremor.

100b

173

Answer 100

CNS, Head and Neck Imaging

ANSWER 100
Observations 1 (100a)
Axial T2 weighted image demonstrates high signal in the
lateral aspect of right medulla. This is bright on the DWI
indicating restricted water molecule diffusion probably from
cell swelling due to acute infarction. Together with the
clinical details, findings are consistent with lateral medullary
syndrome due to infarction in the territory of the right
posterior inferior cerebellar artery (PICA).

Observations 2 (100b)
These axial and coronal T2 weighted images show a focus
of increased signal in the right side of the midbrain. There
is no associated mass effect and the lesion lies in the region
of the red nucleus. The clinical symptoms are compatible
with an infarct affecting the red nucleus and causing
Claudes syndrome.

Diagnosis 1
Right PICA infarction (Wallenbergs syndrome).

Diagnosis 2
Right midbrain infarction affecting the red nucleus
(Claudes syndrome).

Discussion
There are various specific patterns of brainstem infarction,
often having unusual eponyms. Others include Webers,
Nothnagels, MillardGubler and Fovilles syndromes.
Wallenbergs syndrome is due to PICA occlusion and
presents with ipsilateral ataxia, dysphagia, facial pain and
temperature sensory impairment and Horners syndrome,
with contralateral impairment of pain and temperature
sensation in the body and limbs. The PICA is the first
major intracranial branch of the vertebrobasilar system and
supplies the dorsolateral medulla, cerebellar vermis and
posterolateral cerebellar hemisphere. It arises from the distal
vertebral artery just below the basilar artery origin. As well

174

as thrombosis, infarction can result from dissection of the


vertebral artery.
Claudes syndrome is due to a lesion of the red nucleus,
of which infarction is one example. The consequences are
ipsilateral oculomotor palsy and contralateral tremor and
ataxia.

Practical tips
These are examples of comparatively rare and very
specific infarcts but they do make for an interesting
test of neuroanatomy understanding!
DWI depicts reduction in Brownian motion of water
molecules. Cytotoxic oedema in acute infarction will
produce this and present high signal on the DWI
scan. It is very sensitive, depicting infarcts just 30 min
or so from onset.
The DWI image also has inherent T2 weighting (T2
shine through), that is, the signal is a combination
of T2 weighted and reduced diffusion. As such, it
may not distinguish an older infarct (T2 hyperintense)
from a recent one (reduced diffusion) as both will
appear bright. An alternative depiction that can
differentiate acute and nonacute infarcts is the
apparent diffusion coefficient (ADC) map. This has
no T2 component and shows reduced diffusion (i.e.
acute infarct) as low signal. An older infarct that is
also bright on DWI by virtue of T2 shine through
will be bright on ADC.
Ischaemia is not the only cause of reduced diffusion.
Others include seizure, trauma, hypoglycaemia,
abscess.

Further management
DWI is more sensitive in the early detection of infarction
when compared to CT. However, when thrombolysis is
being considered, CT is more than adequate to exclude a
haemorrhage alone.

100a

100b

100a High signal in lateral aspect of


right medulla.

100b Right midbrain high signal.

CNS, Head and Neck Imaging

CASE 101

Case 101

101a

History
A 32-year-old male presented with
poorly controlled epilepsy.

101b

175

Answer 101

CNS, Head and Neck Imaging

ANSWER 101
Observations (101a, 101b)
The axial CT image of the brain (101a) shows several small
calcified lesions in the subependymal region of the body of
the lateral ventricles. The T2 weighted MR image of the
brain (101b) confirms several small subependymal nodules
with associated low signal representing calcification. These
appearances are consistent with subependymal hamartomas.
There are abnormal widened gyri in the left parietal lobe
and right frontal lobe seen on the MR image, which are
likely to indicate cortical hamartomas.

Diagnosis
Tuberous sclerosis.

Distinguishing tubers/hamartomas from giant cell


astrocytomas: tubers and subependymal hamartomas
can show some enhancement on MRI but CT is not
usually sensitive enough to show this. Because giant
cell astrocytomas show more enhancement, this may
be appreciable on CT and should arouse suspicion.

Further management
Tuberous sclerosis has a high mortality with 70% dying
before the age of 24 years. A multidisciplinary team
approach is required with follow-up imaging involving
MRI brain, renal ultrasonography (monitoring angiomyolipomas and looking for renal cell carcinomas) and
echocardiography (50% of patients have rhabdomyomas).

Discussion
Tuberous sclerosis is an inherited autosomal dominant
disorder of the neuroectoderm that is characterized by
multisystem abnormalities. The classical triad of features
are mental retardation, seizures and adenoma sebaceum.
CNS features:
Subependymal hamartomas most commonly seen
along the ventricular surface of the caudate nucleus.
Multiple small subependymal lesions which calcify in
80%. Figure 101c shows another case where nodules
have not calcified, but produce a wavy border to the
ventricle walls.
Cortical tubers appearances are of large widened
atypical gyri with reduced attenuation centres on CT.
They are usually multiple and can show rim
calcification in 50%.
Heterotopic grey matter islands these appear as large
hypodense focal islands of tissue within the cerebral
white matter.
Giant cell astrocytomas occur around the foramen of
Monro, and can cause hydrocephalus. Malignant
potential is low.

Further reading
Altman NR, Purser RK, Post MJ (1988). Tuberous
sclerosis: characteristics at CT and MR imaging.
Radio lo gy 167: 527532.

101c

Multisystem involvement:
Ocular ocular phakomas, optic nerve gliomas.
Renal angiomyolipoma, cysts, increased risk of renal
cell carcinoma.
Respiratory lymphangiomyomatosis-like features
with cystic lung disease, spontaneous pneumothoraces
and chylothorax.
Cardiovascular rhabdomyoma, aortic aneurysms.
Skin adenoma sebaceum (red/brown small flat skin
lesions distributed symmetrically over nose and
cheeks), shagreen patches, ash leaf lesions, subungual
fibromas.

Practical tips
The phakomatoses (neurocutaneous disorders), of
which tuberous sclerosis is an example, make great
exam cases because of the multitude of radiological
signs to piece together know them well!

176

101c Subependymal nodules have not calcified in


this case.

CNS, Head and Neck Imaging

Case 102

CASE 102
History
Adult patient presented with recent
onset of seizures.

102a

177

Answer 102

CNS, Head and Neck Imaging

ANSWER 102
Observations (102a)
Axial T2 (left) and gradient echo T2 (right) images
demonstrate a lesion in the medial right temporal lobe with
a very low signal rim representing haemosiderin. This is
characteristically more prominent on the gradient echo T2
sequence with blooming artefact. The centre of the lesion
shows T2 hyperintensity and the overall shape is round.
The features are consistent with a cavernoma.

Diagnosis
Cavernoma (cavernous angioma or cavernous malformation).

Discussion
Vascular malformations are a common cause of
parenchymal brain haemorrhage and should be excluded
when young patients present with spontaneous
haemorrhage. They develop from congenitally abnormal
vascular connections, which may increase in size with time.
Radiological features of vascular malformations are:
Arteriovenous malformations (AVMs) are the most
common type and are essentially an abnormal
collection of arteries connected directly to veins with
no intervening capillaries (102b). The vast majority
are supratentorial. On CT they are of mixed density
and may have calcifications. Enhancement is also seen.
However, the classical appearance is found on MRI

102b

where flow voids with complex flow patterns are seen.


Figure 102c demonstrates an example on an axial T2
weighted image with an AVM in the right frontal lobe
producing the characteristic tangle of flow voids.
Cavernous malformations (cavernomas) are thin
walled sinusoidal vessels representing congenital
hamartomas. Unlike other vascular malformations,
there is no brain parenchyma between the vascular
spaces. The appearances on CT are rather nonspecific
but MRI shows very characteristic features, as
illustrated. The central high signal represents
methaemoglobin and the outer low signal rim
haemosiderin. They present with focal seizures or
small parenchymal haemorrhages.
Venous angiomas are usually asymptomatic and are
anomalous veins that drain the normal brain.
Classically, an enhancing stellate venous malformation
is seen extending to the ventricular or cortical surface.

Practical tips
AVMs typically have a tangle of low-signal flow voids
on MRI best seen on T2 and PD weighted imaging.
10% of AVMs develop an associated aneurysm so
always look for this.
Gradient echo T2 is an excellent technique for
identifying haemosiderin on MRI, which is often seen
in vascular malformations.

102c

102b Large right cerebral AVM


demonstrated on a CT cerebral angiogram.

102c Axial T2 MRI demonstrates right frontal AVM


with characteristic flow voids.

178

Answer 102

CNS, Head and Neck Imaging

Haemosiderin will also be seen in an old haemorrhage


not necessarily involving an underlying vascular
malformation. Figure 102d demonstrates an old
haemorrhage in the left internal capsule/thalamus
and it would be easy to confuse this with cavernoma
from the axial image. However, an old haemorrhage
collapses to form a slit-like cavity unlike the rounded
shape of cavernoma, and the coronal scan illustrates
this difference. Note also the local volume loss due to

102d Coronal T2 and axial gradient echo


T2 MRI demonstrating old haemorrhage
in left internal capsule.

Case 103

gliosis resulting in some enlargement of the adjacent


left lateral ventricle.

Further management
Often these are asymptomatic and can be monitored with
imaging to assess change in growth. There is, however, an
up to 2% risk of bleeding and surgery/stereotactic
radiosurgery are treatment options.

102d

CASE 103
History
A 65-year-old male presented with
swelling and impaired vision of the left
eye.

103

103 Axial and coronal T2 weighted scans of orbits (left/middle) and a time of flight MR angiogram of the
intracranial circulation (right).

179

Answer 103

CNS, Head and Neck Imaging

ANSWER 103
Observations (103)
T2 weighted images demonstrate dilatation of the left
superior ophthalmic vein with presence of flow void and
no focal compressive mass lesion. The time of flight MRA
demonstrates normal arterial intracranial anatomy with
signal in the dilated left ophthalmic vein indicating fast
flow. The findings are in keeping with a left caroticocavernous fistula.

Diagnosis
Caroticocavernous fistula.

Differential diagnosis
For superior ophthalmic vein distension:
Cavernous sinus thrombosis.
Superior ophthalmic vein thrombosis.
Pseudotumour.
Graves disease.
Obstructive orbital mass.

Cranial nerve palsies most commonly 6th and 3rd


nerves. This is thought to be due to either impaired
venous drainage of the nerve or direct compression of
nerves by distended veins.
Contralateral symptoms are seen in ~10% due to the
presence of connections between the two cavernous
sinuses.
Imaging features on MRI are:
Dilated superior ophthalmic vein with flow void.
Enlargement of the cavernous sinus.
Swelling of the extraocular muscles, which can result
in limited eye movements.
Angiography shows early opacification of the superior
ophthalmic vein when contrast is injected into internal
carotid artery. Early opacification of the veins communicating with cavernous sinus may also be noted.

Practical tips
Discussion
Fistulous communication between the internal carotid
artery and the cavernous sinus can arise secondary to head
trauma or rupture of an internal carotid artery aneurysm.
The condition often occurs spontaneously, however, due
to atherosclerosis. In addition, fistulae can occur with dural
branches of the external carotid artery. Orbital bruit is
found in ~50% of patients due to turbulent arterial blood
flow. The increased arterial pressure in the venous system
and orbital vein congestion result in the symptoms of:
Pulsatile proptosis.
Chemosis.
Reduced visual acuity due to impaired retinal
perfusion severe/rapid visual loss requires
angiographic investigation and fistula closure as an
emergency to preserve function.

103

103 Dilated left superior ophthalmic vein.

180

Early radiological changes are of enlarged oedematous


extraocular muscles and dilatation of the superior
ophthalmic vein.
More chronic changes are of enlargement of the
superior orbital fissure and sellar erosion.
Clinically, pulsatile exophthalmos suggests this
condition.

Further management
When imaging features of a caroticocavernous fistula are
associated with decreased visual acuity, emergency treatment is required to relieve intraocular pressure.

CNS, Head and Neck Imaging

Case 104

CASE 104
History
A 32-year-old male presented with
headaches and new onset epilepsy.

104

104 MRI T1 and T2 weighted axials (top) and T1 weighted post IV gadolinium axial and sagittal (bottom).

181

Answer 104

CNS, Head and Neck Imaging

ANSWER 104
Observations (104)
Selected MR images of the brain show a large lesion in the
cerebellum, which is predominantly cystic in nature
(hyperintense on T2 and hypointense on T1 weighted
images). Following IV contrast there is enhancement of the
wall of the cystic component and an associated solid nodule
posteriorly. Sagittal image shows some fullness of the lateral
ventricles and obstructive hydrocephalus is likely with
compression at the level of the 4th ventricle.

Diagnosis
Haemangioblastoma.

Differential diagnosis
Juvenile pilocytic astrocytoma can be very difficult to
differentiate from a haemangioblastoma. This is the
most common infratentorial tumour in children.
There is an association with neurofibromatosis type 1.
Lesions are of an identical appearance, with a
predominant cystic component and enhancing solid
peripheral nodule.
Metastasis.
Atypical medulloblastoma.

Discussion
Haemangioblastoma is a benign vascular tumour affecting
the CNS. It is the most common primary infratentorial
tumour in adults. The majority (80%) present in adults in
the 3rd6th decades, although there is an association with
von HippelLindau (VHL), which sees these tumours
presenting in childhood and in this case may be multiple.

Typical appearance is of a well defined cystic lesion


containing fluid of CSF density with a solid peripheral
nodule, showing uniform avid enhancement. There is
commonly haemorrhage or necrosis in the solid nodule
but it rarely calcifies. In up to 30% of cases the lesions can
be entirely solid with no cystic component.

Practical tips
When forming a differential for the posterior fossa cystic
mass with enhancing peripheral nodule, the following
generalizations apply:
Children pilocytic astrocytoma >
haemangioblastoma.
Younger adults haemangioblastoma most common.
Older adults consider cystic metastasis.

Further management
When haemangioblastoma is suspected, dont forget to
raise the possibility of VHL (420% of haemangioblastoma
occur in the context of VHL and multiple lesions are
diagnostic of VHL). VHL is an autosomal dominant (AD)
inherited condition characterized by a predisposition to
develop a spectrum of tumours including haemangioblastomas, cardiac rhabdomyomas, renal cell carcinomas,
pheochromocytomas, pancreatic cystadenocarcinomas, islet
cell tumours and haemangioblastomas, liver haemangiomas
and retinal angiomas.

104

104 Enhancement of the solid posterior nodule on post-gadolinium images.

182

CNS, Head and Neck Imaging

Case 105

CASE 105
History
A 28-year-old male of no fixed abode,
presented with a short history of
confusion and limb weakness.

105

105 T1 sagittal (top left) and T2 weighted images (top right and bottom).

183

Answer 105

CNS, Head and Neck Imaging

ANSWER 105
Observations (105)
The selected images demonstrate diffuse cerebral atrophy
exceeding that expected for the patients young age. There
is a large focal area of signal abnormality in the central pons
that is hyperintense on T2 weighted and hypointense on
T1 weighted scans. There is no associated mass effect. The
signal changes would fit with central pontine myelinolysis
and the clinical details and cerebral atrophy suggest that
chronic alcohol abuse may be the underlying cause.

Clinically this results in pseudobulbar palsy, tetraplegia,


convulsions, acute confusion and progression to coma. It
is most commonly confined to the pons but changes can
also be seen in the basal ganglia, caudate, thalamus and
subcortical white matter. Radiological presentation is with
fairly well defined low attenuation lesions on CT in the
central pons. On MRI, lesions are of low intensity on T1
and high signal intensity on T2 imaging.

Practical tips
Diagnosis
Differential diagnosis

Standard MR/CT imaging can be normal for up to 14


days after symptom onset, lagging considerably behind the
clinical presentation. Restricted diffusion on DWI,
however, is found within 24 hours.

Pontine glioma.
Infarction.

Further management

Central pontine myelinolysis.

Discussion
Central pontine myelinolysis results from the destruction
of myelin sheaths, classically in patients with rapidly
corrected hyponatraemia. Cases are reported in patients
with:
Chronic alcohol abuse.
Chronic liver disease.
Severe malnutrition.
Wilsons disease, chronic renal failure, diabetes, acute
myelogenous leukaemia.
In the case demonstrated there is diffuse global cerebral
atrophic change suggestive of chronic alcohol abuse.

Early MRI appearances can be similar for a basilar


infarct and MRA can be useful to look for arterial
thrombus.
Search for underlying cause if not readily apparent.

Further reading
Ruzek KA, Campeau NG, Miller GM (2003). Early
diagnosis of central pontine myelinolysis with
diffusion-weighted imaging. American Jo urnal o f
Neuro radio lo gy 25: 210213.
Stadnik TW, Demaerel P, Luypaert RR, et al. (2003).
Imaging tutorial: differential diagnosis of bright
lesions on diffusion-weighted MR images.
Radio Graphics 23(1): e7.

105

105 Focal area of reduced signal on T1 weighted (left) and increased signal on T2
weighted (right) images.

184

CNS, Head and Neck Imaging

CASE 106

Cases 106, 107

106a

History
A 49-year-old female patient presented
with headaches.

CASE 107
History
A 2-year-old female presented with
developmental delay.
(see page 188 fo r case answer)

107a

107a T1 weighted sagittal (left) and T2 weighted axial and coronal (middle/right) images.

185

Answer 106

CNS, Head and Neck Imaging

ANSWER 106
Observations (106a)
This non contrast enhanced CT image demonstrates a large
midline mass extending into the 3rd ventricle, which has
both solid and cystic components with central calcification.
There is dilatation of the temporal and anterior horns of
the ventricles but normal appearance to the 4th ventricle
findings are in keeping with obstructive hydrocephalus at
the level of the 3rd ventricle secondary to a mass lesion.
The mixed density and calcification are very suggestive of
a craniopharyngioma and MRI is suggested to further
evaluate.

Diagnosis
Craniopharyngioma.

Differential diagnosis
Of suprasellar/intrasellar mass lesion:
Pituitary macroadenoma.
Craniopharyngioma.
Rathke cleft cyst.
Other masses in the sellar region include meningioma,
metastases, chordoma and internal carotid artery aneurysm.
Pituitary macroadenoma is the most common lesion
involving the sellar and suprasellar regions. Microadenomas
(20%) are defined as being less than 10 mm in size and
usually present with symptoms due to hormonal secretion.
Macroadenomas (80%) measure over 10 mm in size and
are usually endocrinologically inactive. They present with
symptoms secondary to mass effect such as hydrocephalus,
bitemporal hemianopia from optic chiasm compression,
involvement of cranial nerves travelling in the adjacent

cavernous sinus (3, 4, 5a and 6) and hypopituitarism


resulting from compression of normal pituitary tissue.
Compared to craniopharyngioma, pituitary macroadenomas are predominantly solid and show more intense
uniform enhancement. Calcification is rare, unlike
craniopharyngioma. Suprasellar extension produces the
snowman configuration due to waisting at the level of
the diaphragm sellae. Figure 106b shows pre- and postcontrast T1 weighted images of a pituitary macroadenoma
with such features.
Rathke cleft cyst is the other main differential diagnosis,
with 70% involving sellar and suprasellar regions. It is a
benign cyst arising from remnants of Rathkes pouch and
can be differentiated by the following features: more
regular and ovoid shape, smaller size (<2 ml), cystic with
no, or minimal enhancement of the thin wall.

Discussion
Craniopharyngioma account for ~4% of intracranial
neoplasms, with a slight predominance in males, and two
peaks of increased incidence:
Firstly in children in 1st2nd decades, where they
account for 50% of suprasellar tumours.
Secondly in adults in 5th 6th decades.
These are benign tumours arising from remnants of
Rathkes pouch. They grow from a suprasellar origin into
the base of the 3rd ventricle. Intrasellar extension occurs
in 21%. Involvement of the bony sella can be seen on plain
radiographs and on CT bone window images. These
demonstrate appearances of J-shaped sella, enlarged sella
and then ultimately erosion and destruction.

106b

106b Pre- and post-contrast T1 weighted MR sagittal images demonstrate a large pituitary
adenoma with uniform enhancement with contrast and with no cystic elements or signal
voids to suggest calcification.

186

Answer 106

CNS, Head and Neck Imaging

Typical CT appearances are shown in this case with


solid and cystic elements and calcification (which is
seen in ~90% of cases). Following contrast there is
enhancement of the solid components and the cyst
wall.
Typical MRI findings reflect CT features with a cystic
component following fluid signal (although high
signal may be seen on T1 weighted images depending
on protein/blood content). The solid component is
isointense on T1 and shows enhancement with
gadolinium contrast.

Practical tips
Look for the complications of mass lesions in this area:
Hydrocephalus and optic chiasm compression.
Lateral extension to involve the cavernous sinus (seen
in up to 10% of pituitary macroadenomas), which can
lead to thrombosis and cranial nerve palsy. Lateral
extension beyond the lateral wall of the internal
carotid is rare with Rathke cleft cyst and may help
differentiate it from the other two conditions listed.

Further management
Treatment is surgical +/ postoperative radiotherapy.

Figure 106c shows T1 sagittal pre- and post-IV contrast


(right) and T2 axial and sagittal images (left). These
illustrate a large suprasellar lesion with intrasellar extension.
There are solid and cystic elements with enhancement of
the solid components and cyst wall. Complications of
obstructive hydrocephalus, pituitary stalk and optic chiasm
compression can be appreciated.

Further reading
Choi SH, Kwon BJ, Na DG, et al. (2007). Pituitary
adenoma, craniopharyngioma, and Rathke cleft cyst
involving both intrasellar and suprasellar regions:
differentiation using MRI. Clinical Radio lo gy 62:
453462.

106c

106c T1 sagittal pre- and post-IV contrast and T2 axial and sagittal images demonstrate a
large suprasellar lesion with solid and cystic elements with enhancement of the solid
components and cyst wall. The lesion is causing obstructive hydrocephalus, pituitary stalk
and optic chiasm compression.

187

Answer 107

CNS, Head and Neck Imaging

ANSWER 107
Observations (107a)
T1 sagittal and T2 axial and coronal images of the brain
demonstrate complete agenesis of the corpus callosum with
no callosal tissue identified. There is increased separation
of the lateral ventricles on the axial images creating a bats
wing appearance. Coronal image demonstrates elevation
of the 3rd ventricle.

Prenatal detection can be made by US investigation.


Diagnosis cannot usually be made before 22 weeks
gestation. Equivalent image findings are of:
Absence of the septum pellucidum.
Dilated/elevated 3rd ventricle.
Enlargement of the occipital horns of the lateral
ventricles.

Diagnosis

Practical tips

Agenesis of the corpus callosum.

Differentiating between dysgenesis and ischaemic damage


is done by determining which parts of the corpus callosum
are abnormal. With ischaemic injury the rostrum will be
present but atrophic.

Discussion
The corpus callosum usually develops by 20 weeks
gestation. The genu and the body of the corpus callosum
develop first and the posterior body and splenium develop
later. The rostrum is the last part of the corpus callosum to
develop. Identifying the parts of the corpus callosum
present can help to differentiate between dysgenesis (absent
genu/splenium) and destruction, most commonly due to
ischaemia (genu present but may be atrophic). Corpus
callosal agenesis is usually associated with reduced
intellectual function and can be associated with a variety of
CNS abnormalities including hydrocephalus, midline
lipoma, DandyWalker cysts, interhemispheric arachnoid
cysts, neuronal migration disorders and ArnoldChiari
malformation.
Classical imaging appearances on CT/MRI are with:
Bats wing appearance of the lateral ventricles due to
parallel lateral ventricles with marked separation
(107b).
High riding 3rd ventricle can be seen at the level of
the lateral ventricles.
Enlarged foramen of Monro.

Further management
Sometimes this finding is made incidentally on brain
imaging in later life and no further investigation is
required.

Further reading
Babcock DS (1984). The normal, absent, and abnormal
corpus callosum: sonographic findings. Radio lo gy
151: 449453.
Davidson HD, Abraham R, Steiner RE (1985). Agenesis
of the corpus callosum: magnetic resonance imaging.
Radio lo gy 155: 371373.

107b

107b Axial CT images of the brain demonstrating bats wing appearance of the lateral ventricles in a patient
with agenesis of the corpus callosum.

188

CNS, Head and Neck Imaging

Case 108

CASE 108
History
A 38-year-old female presented with
left sided tinnitus and hearing loss.

108a

108b

189

Answer 108

CNS, Head and Neck Imaging

ANSWER 108
Observations (108a, 108b)
Axial CT images (108a axial CT scan at skull base with
IV contrast, seen on bone and soft tissue windows) of the
brain demonstrate a poorly defined enhancing lesion in the
jugular foramen, which is extending superiorly to involve
the middle ear and is causing permeative erosion of the
petrous temporal bone at the jugular foramen. The T2
weighted MR image (108b) confirms this mass lesion,
which appears hyperintense with serpiginous flow voids
indicating vascular flow.

Diagnosis

destruction seen with skull base metastases. Vascular flow


voids and foci of haemorrhage are better demonstrated on
MRI flow produces black signal voids and haemorrhage
produces hyperintense foci on T1 weighted images. This
combination of black holes and white dots leads to the socalled salt and pepper appearance commonly described.
Angiography is sometimes required for smaller tumours
that dont demonstrate the classical imaging appearances,
and reveals a hypervascular lesion with a dense reticular
tumour stain.
There is a small risk of malignant transformation seen
in ~2% of cases.

Glomus jugulotympanicum tumour.

Practical tips
Description
Glomus jugulare tumours are paragangliomas and are the
most common jugular foramen lesion. Typically, these
lesions are solitary with a peak incidence in the 5th6th
decades and increased incidence in females. There are
familial associations in which multiple paragangliomas
present in conditions such as multiple endocrine neoplasia
(MEN).
Glomus jugulare tumours usually extend intracranially
and can involve the middle ear, as demonstrated here.
Clinical presentation may be related to involvement of the
9th, 10th and 11th cranial nerves. When there is extension
to involve the middle ear (glomus jugulotympanicum) then
pulsatile tinnitus is the classical presentation.
These tumours are highly vascular and enhance avidly
with contrast on CT and MRI. On CT, permeative bone
erosion is classical, differing from the coarser lytic

When suspected on MRI, CT can still help make a more


conclusive diagnosis in many cases by demonstrating the
classical permeative pattern of erosion at the jugular
foramen.

Further management
Depending on the size of the tumour and the intracranial
extension, a combined ENT/neurosurgical procedure is
performed.

Further reading
Caldemeyer KS, Mathews VP, Azzarelli B, Smith RR
(1997). The jugular foramen: a review of anatomy,
masses, and imaging characteristics. Radio Graphics
17: 11231139.

108b

108b Hyperintense mass lesion with flow voids.

190

CNS, Head and Neck Imaging

Case 109

CASE 109
History
A 24-year-old female presented with
chronic headaches and visual loss.

109

109 T1 weighted sagittal and coronal (top) and T2 weighted coronal (bottom) MR images.

191

Answer 109

CNS, Head and Neck Imaging

ANSWER 109
Observations (109)
These selected MR images show an expanded sphenoid
sinus filled with abnormal signal rather than air. The
contents show mild uniform hyperintensity on T1 weighted
and T2 weighted images. The walls of the sinus appear
smooth and intact with no obvious evidence of destruction.
The findings are consistent with a mucocele of the
sphenoid sinus, the signal indicative of proteinaceous fluid
contents. Sagittal T1 and coronal T2 images demonstrate
displacement of the optic nerves by the expanded sinus,
presumably causing compression of the anterior optic
pathway given the history of visual disturbance.

Diagnosis
Sphenoid sinus mucocele.

Discussion
A mucocele arises due to chronic obstruction of a sinus,
which then becomes filled with mucus. This collection acts
like a slow growing mass causing expansion of sinus bony
walls without frank bony destruction. Vessels and other
structures are displaced rather than being encased, as with
a tumour. Usually these lesions are asymptomatic until they
become large when they can cause optic nerve compression,
proptosis and headache. Secondary infection of the
mucocele can supervene, but this is a rare complication.
Ninety per cent of lesions are found in the frontal and
ethmoid sinuses with sphenoid sinus mucoceles being only
rarely seen.

Appearances on MRI are variable depending on the


fluid/protein content of the mucocele but important
imaging findings are of a nondestructive, slow growing
lesion.

Practical tips
CT is good for showing bony expansion with the
absence of bone erosion.
MRI is good for identifying the extent/size of the
lesion and looking for complications of optic nerve
compression (usually from posterior ethmoid lesions)
and proptosis (usually from frontal and anterior
ethmoid sinuses).
T1 hyperintensity within a lesion on any MRI study
often helps to rapidly limit the differential diagnosis as
there are a limited number of things that are bright
on T1 imaging, i.e. fat, blood, proteinaceous fluid,
paramagnetic contrast agents.

Further management
Referral to ENT for consideration of endoscopic sinus
surgery.

109

109 Superior displacement of optic nerve (left); lateral displacement of cavernous sinus
by large sinus mucocele (right).

192

CNS, Head and Neck Imaging

CASE 110

Case 110

110a

History
A 68-year-old female
presented with worsening
headaches.

110b

193

Answer 110

CNS, Head and Neck Imaging

ANSWER 110
Observations (110a, 110b)
The selected plain CT images (110a) show a poorly
defined, hyperdense midline lesion extending across the
body of the corpus callosum. There is no significant
surrounding oedema.
Pre- and post-contrast T1 weighted MR images (110b)
from the same patient show a multilobulated lesion with
uniform enhancement crossing the midline in the body of
the corpus callosum. No further lesions are seen within the
brain parenchyma. Incidental note is made of cavum
septum pellucidum. The findings are of a butterfly tumour
in the corpus callosum.

Diagnosis
Butterfly glioblastoma multiforme (GBM).

Typical imaging appearances of a butterfly glioma are


of a poorly defined lesion that enhances uniformly. Cyst
formation, necrosis and haemorrhage are seen in about 5%
of cases and calcification is rare and usually associated with
chemo/radiotherapy. The corpus callosum is made up of
dense, tightly packed white matter tracts and therefore
mass effect and vasogenic oedema are minimal.
Hemispheric glioblastomas conversely have quite marked
mass effect and surrounding oedema.

Practical tips
GBM and lymphoma are the two likeliest diagnoses for
butterfly lesions involving the corpus callosum. Although
difficult to differentiate radiologically, cavitation and
necrosis are relatively uncommon in lymphoma (except in
AIDS patients).

Differential diagnosis
For lesions crossing the midline in the corpus callosum:
GBM.
Lymphoma.
Demyelinating disease multiple sclerosis.

Discussion
This is the most common and the most malignant primary
brain tumour. It accounts for >50% of brain tumours with
a wide age distribution peaking at the 7th8th decades. The
most common location is within the white matter of
centrum semiovale with increased incidence in the frontal
lobes. Other patterns of distribution include callosal
extension giving this appearance of a butterfly glioma,
posterior fossa lesions and multifocal distribution (seen in
24%).
Tumours spread by direct extension involving white
matter tracts, such as the corpus callosum and cerebral
peduncles or via the CSF (<2% of cases). Haematogenous
spread can also very rarely occur.

Further management
T2 hypointensity is sometimes present with CNS
lymphoma and therefore this image sequence can
sometimes help to differentiate lymphoma from
glioblastoma.
Neurosurgical evaluation.

Further reading
Bourekas EC, Varakis K, Bruns D, et al. (2002). Lesions
of the corpus callosum: MR imaging and differential
considerations in adults and children. American
Jo urnal o f Radio lo gy 179: 251257.
Rees JH, Smirniotopoulos JG, Jones RV, Wong K
(1996). Glioblastoma multiforme: radiologicpathologic correlation. Radio Graphics 16: 6.

110a

110a Lobulated lesion crossing the midline in the body of the corpus callosum.

194

CNS, Head and Neck Imaging

CASE 111

111a

Case 111

111b

History
A 33-year-old male presented with
lower back pain.

111c

195

Answer 111

CNS, Head and Neck Imaging

ANSWER 111
Observations (111a, 111b, 111c)
These STIR sagittal (111a), STIR coronal (111b) and T2
weighted axial (111c) images of the spine demonstrate
posterior vertebral scalloping, particularly at the T11, T12
and L2 levels. There is also a thoracic scoliosis. At
approximately T10 level, coronal images show a right sided
high-signal lesion projecting laterally from the spinal canal,
presumably through the intervertebral foramen. This
follows CSF signal and is probably a lateral thoracic
meningocele. The axial T2 weighted image shows a second
lateral thoracic meningocele on the left side, in a similar
region of the spine. The combination of findings is strongly
suggestive of neurofibromatosis type 1 and multiple highsignal cutaneous nodules on the sagittal images confirm this
diagnosis.

Diagnosis
Neurofibromatosis type 1 (NF1).

Discussion
Neurofibromatosis type 1 is a neurocutaneous disorder, of
which 50% is inherited in an autosomal dominant manner
and 50% is sporadic. Classical features in the spine include:
Lateral thoracic meningocele (dysplasia of the
meninges resulting in diverticula of the thecal sac,
which extends through the neural foramina).
Posterior vertebral scalloping also due to dural
ectasia.

111a

111a Posterior
vertebral
scalloping.

Neurofibromas enhancing dumbbell-shaped lesions.


Scoliosis and kyphosis.
Enlarged vertebral foraminae due to neurofibroma
or lateral meningocele.
Hypoplasia of the pedicles, transverse and spinous
processes.

Practical tips
Always look for the presence of subcutaneous nodules
on the image when NF1 is suspected.
It may be difficult to differentiate between a lateral
thoracic meningocele and a dumbbell neurofibroma.
However, on MRI the former will be of CSF density
on all sequences, i.e. high signal on T2, whereas the
latter will have a hyperintense periphery on T2 with a
hypointense core.
In a case such as this, one might also comment that
there are no obvious adrenal masses or renal
asymmetry (implying association with
phaeochromocytoma and renal artery stenosis,
respectively).

Further management
Neurofibromas can undergo malignant transformation in
23%; therefore a rapid rate of growth or new onset pain
should be thoroughly investigated.

111b

111b Lateral thoracic meningocele.

196

CNS, Head and Neck Imaging

Case 112

CASE 112
History
A 22-year-old female had had persistent
headache for several days. There was no
history of trauma.

112a

197

Answer 112

CNS, Head and Neck Imaging

ANSWER 112
Observations (112a)
Selected unenhanced axial CT images of the brain show a
well defined low attenuation lesion in the left frontal lobe
causing minimal mass effect. The contents are of lower
attenuation than CSF but given the absence of trauma it is
unlikely that this is due to intracranial air. It is likely that
instead, it represents fat, which could be easily confirmed
by adjusting the CT windows or taking a direct density
measurement. A calcified nodule is seen at the posterior
aspect of the lesion with further mural calcifications
elsewhere. Further small locules of fat are seen in the
anterior horn of the left lateral ventricle and several sulci.

Typical appearances on MRI: well defined lesion


which is high signal intensity on T1 weighted imaging
with signal void at points of calcification. No
enhancement with contrast.
Cyst rupture leading to chemical meningitis is an
occasional complication, as in this case. Release of fat
globules can produce fat-fluid levels with CSF. Figure
112b demonstrates typical appearance of a dermoid with
a fat-containing lesion of high signal on T1 weighted
imaging. Some strands of mesodermal tissue are noted
within it. A fat-fluid level is seen in the lateral ventricle due
to rupture into the subarachnoid space.

Diagnosis
Practical tips

Ruptured dermoid cyst.

Discussion
Dermoid cysts are fairly uncommon CNS lesions, usually
presenting before the 4th decade. They arise due to
inclusion of epithelial elements at the time of closure of the
neural tube and therefore contain ectodermal and
mesodermal components, i.e. hair, sebaceous glands and
skin. Dermoids are usually located near the midline and can
be found within brain parenchyma, ventricles, CSF spaces
or within bone. Development and growth are very slow and
presentation is therefore late.
Typical appearances on CT: low-density lesions
containing fat with no contrast enhancement. Mural
or focal central calcification can be seen.

Posterior fossa dermoids are usually found in the


midline and show no contrast enhancement on
CT/MRI.
On T1 weighted MRI high signal is produced by:
Fat.
Blood products (methaemoglobin).
Proteinaceous fluid.
Contrast.

Further management
MRI is better than CT for identifying rupture of a dermoid
cyst and the consequent chemical meningitis.

112b

112b Sagittal T1 weighted image of the brain shows


rupture of the midline dermoid cysts with a fat-CSF
level seen in the lateral ventricle.

198

CNS, Head and Neck Imaging

Case 113

CASE 113
History
A 45-year-old patient presented with
confusion developing over 1 week.

113a

199

Answer 113

CNS, Head and Neck Imaging

ANSWER 113
Observations (113a)
These T2 weighted axial brain images show a diffuse
increase in signal affecting the right temporal lobe. This
involves grey and white matter, extending to the cortical
surface. Sulci in the affected area are effaced. Similar signal
changes extend along the cingulate gyrus and also affect
the contralateral temporal lobe to a lesser extent. The
duration of the history and distribution of involvement are
in keeping with encephalitis, most likely due to herpes
simplex virus.

Diagnosis
Herpes simplex (HSV) encephalitis.

Differential diagnosis
Right middle cerebral territory infarct.

Discussion
Encephalitis is the term generally used to describe a diffuse
cerebral inflammatory process of viral aetiology. Clinical
presentation is with confusion, headache and seizures
progressing to coma. Fever is almost always present. The
most common organism is HSV, infection being either
primary or due to virus reactivation. The resulting cytotoxic
oedema manifests as high signal on T2 weighted, FLAIR
and DWI MRI. The temporal lobe is typically affected,
often with the inferior frontal lobe and cingulate gyrus.
Unlike other viral infections, the basal ganglia are usually
spared.

Bilateral temporal lobe involvement is virtually


pathognomonic of HSV infection. Features that are
atypical and should arouse suspicion of other
organisms include basal ganglia involvement and
isolated involvement of other lobes without temporal
lobe involvement.
Cytotoxic oedema (seen in stroke and encephalitis)
extends to the periphery of the brain whereas
vasogenic oedema (seen with tumours, abscess, etc.)
affects white matter only.
Without clinical history, one might mistake the MRI
changes of HSV encephalitis for a middle cerebral
artery (MCA) infarct. However, note that the
posteromedial temporal lobe affected by HSV is
spared in an MCA infarct as this territory is supplied
by the posterior cerebral artery (PCA). Figure 113b
shows a typical MCA infarct with low attenuation
change in the right temporal lobe but sparing the area
described posteromedially (this scan also happens to
show a hyperdense MCA due to thrombus). For
comparison, Figure 113c shows a PCA infarct with
the posteromedial temporal lobe affected.
Neonatal herpes-related encephalitis is due to HSV
type 2, probably acquired via the genital tract
perinatally. The imaging features are different from
those due to HSV type 1 described above.

Further management
Urgent treatment with antiviral drugs mortality in
untreated HSV encephalitis can be as high as 65%.

Practical tips
HSV encephalitis has high mortality and morbidity
rates but is treatable with acyclovir so a high index of
suspicion is needed, especially on CT, which may well
be the first imaging done signs of oedema and
swelling are likely to be comparatively subtle
compared to MRI (if present at all).

113b

200

113b Axial plain CT


brain shows a right
middle cerebral artery
territory infarct with
hyperdense artery
sign.

Answer 113

CNS, Head and Neck Imaging

Case 114

113c Axial plain CT


brain shows a PCA
infarct with low
attenuation in the
posteromedial
temporal lobe.

113c

CASE 114

114a

History
A 36-year-old male was brought to
A&E in cardiorespiratory arrest. No
other history is available.

201

Answer 114

CNS, Head and Neck Imaging

ANSWER 114
Observations (114a)
This single CT image shows diffuse low-attenuation change
in the frontal and temporal lobes and brainstem with loss
of the greywhite matter differentiation. The cerebellum is
better preserved. Generalized sulcal effacement indicates
cerebral swelling.
This widespread reduction in brain density makes the
normal cerebral vessels and dural reflections appear
spuriously conspicuous. Close inspection confirms that this
is a perceptual abnormality the basal cisterns are of
normal CSF density so there is not acute subarachnoid
haemorrhage. The findings are suggestive of global cerebral
ischaemia and oedema.

Diagnosis
Global cerebral anoxia.
This particular case turned out to be secondary to
asthma-induced cardiorespiratory arrest.

Discussion
Global ischaemia can occur following prolonged hypoxia,
which may be secondary to fitting, aspiration, smothering,
strangulation, etc. The CT findings are distinctive, with loss
of greywhite matter differentiation and changes due to
cerebral oedema including sulcal effacement. Generalized
low density of the cerebral cortex develops. The cerebellum

114b

202

is more resistant to hypoxia and therefore appears of high


density in comparison with the low-density cerebral cortex;
the so-called reversal sign (see also Case 163).
The conspicuity of the vessels and dura may lead one
to the erroneous diagnosis of acute subarachnoid
haemorrhage (SAH) on first inspection. An example of a
real SAH is shown (114b) note high density in the
suprasellar cistern representing acute haemorrhage and the
normal density cerebral cortex with preserved greywhite
matter differentiation. There is also dilatation of the
temporal horns of the lateral ventricles indicating early
obstructive hydrocephalus. Trauma is the most common
cause of SAH. CT is 90% sensitive in the detection of SAH
in the first 24 hours following presentation. However, this
decreases to 50% at 1 week and continues to fall thereafter.
Small SAH may not be seen on CT and will only be
detected on lumbar puncture (LP) as xanthochromia. The
cause of 75% of spontaneous SAH is berry aneurysms and
these are mostly found sprouting from the circle of Willis.
An example of a large SAH in a patient with a ruptured
aneurysm is shown (114c). Note the extensive amount of
blood filling the sulci and Sylvian fissures. Blood has also
entered the ventricular system and can be seen in the 3rd
and 4th ventricles. The large aneurysm appears to arise
from the origin of the left middle cerebral artery.

114b Axial plain CT of brain showing SAH with


blood in suprasellar and basal cisterns.

Answer 114

CNS, Head and Neck Imaging

Practical tips
Dont mistake cerebral anoxic change for SAH in the
unconscious patient check that the hyperdensity
really is in the subarachnoid space and not a
perceptual illusion from cerebral anoxia.
Always remember that a normal CT scan does not
exclude SAH, especially as the time interval from onset
of symptoms increases. If the CT is normal, LP
looking for xanthochromia is required (this may be
evident in CSF after 24 hr but can only be
conclusively excluded if the LP is done at least 12 hr
after onset).
Remember that SAH can be secondary to trauma as
well as leaking aneurysm, etc.
When SAH is present, the site of aneurysm is likely to
be where there is most blood!

114c Axial plain CT brain


demonstrating large SAH with
large aneurysm arising from the
left middle cerebral artery.

CT remains the primary imaging modality for


suspected SAH. Conventional T1 and T2 MRI scans
are unhelpful. FLAIR MRI may show high signal in
the CSF spaces but such changes are also seen in
meningitis, i.e. it is not specific for SAH. However,
MRI may be useful in detecting evidence of chronic
small subarachnoid bleeds as low signal along the
leptomeninges on gradient recalled T2 weighted
scans.

Further management
Regarding suspected SAH, a negative CT alone is
insufficient to exclude the diagnosis. LP (looking for
xanthochromia) is required in cases where CT is normal.
This should be delayed for at least 12 hr from the onset of
headache as false negatives may occur before this time.

114c

203

This page intentionally left blank

Chapter 4

MUSCULOSKELETAL
IMAGING
Much of the advice given here relates to the plain
radiograph, though the fundamental principles can often
be applied to CT and MRI too. When analysing a musculoskeletal film, check the bones, the joints and the soft tissues.
There may be obvious abnormalities at first inspection but,
as always, a systematic check of all three areas will ensure
nothing is missed.
In the real world, the radiologist is usually made aware
of a history of trauma but this may not be provided in a
viva. There are certain diagnoses that must not be missed
on the chest radiograph (e.g. tension pneumothorax) and
abdominal film (e.g. viscus perforation) due to the gravity
of the consequences for the patient (and the viva candidate,
who will fail as a result!). Similarly in MSK imaging,
fractures must not be missed. One should therefore always
be on guard for subtle fractures when nothing else is
apparent, especially those such as the unstable cervical spine
injury where the consequences of nonrecognition are
serious.
Musculoskeletal imaging is one area in particular where
there are many Aunt Minnie diagnoses disorders with
classical appearances that rely on recall from previous
experience rather than a systematic analysis. In the real
world, some such conditions are important to recognize so
as to avoid unnecessary further investigation. In a viva, they
represent easy points for those who have spent time
preparing well!

Thus, not all aggressive lesions are malignant!


Aggressive bone lesions often represent tumour or
infection and show features such as:
Poor definition of edges suggesting rapid growth and
an aggressive nature. The radiological term for poorly
defined borders is a wide zone of transition.
Moth eaten/permeative bone destruction.
Cortical destruction.
Associated periosteal reaction particular patterns of
periosteal reaction point to an aggressive underlying
lesion. The following examples are those classically
described but like most things in medicine, are not
absolute associations!
Multilamellated reaction, e.g. active osteomyelitis,
Ewings sarcoma and osteosarcoma.
Spiculated perpendicular hair on end reaction, e.g.
Ewings sarcoma.
Divergent sunburst appearance, e.g.
osteosarcoma, metastases, Ewings sarcoma, TB.
Associated soft tissue swelling/extension.

BONES

Assessment of periosteal reaction is one feature that may


help assess whether a focal lesion is benign or aggressive,
but it is worth noting that periosteal reaction does not
automatically mean there is an underlying bone lesion.
There are a few systemic conditions that cause periosteal
reaction on normal underlying bone. The key here is to
note features such as normal underlying bone, multiple
bones affected or symmetrical distribution.

When a bony abnormality is identified, the first thing to


establish is whether you are dealing with a focal lesion or a
diffuse abnormality. Fundamentally, a diffuse process
affecting several bones implies a systemic problem be that
metabolic, inflammatory, congenital, etc. A focal bone
lesion implies a local disease process with a very different
differential diagnosis.
The focal bone lesion is a common problem in
radiology, especially the radiology viva. There are several
features to assess and pieces of information to assimilate to
produce a differential diagnosis; the main three are as
follows.

Aggressive vs benign
This is a very important decision with regards to clinical
management of focal bone lesions. Note the use of the
term aggressive rather than malignant while malignant
bone lesions are often aggressive, nonmalignant pathology
such as infection can also behave in an aggressive manner.

Benign lesions typically show features such as:


Well defined borders/narrow zone of transition,
perhaps with marginal sclerosis.
Intact cortex (unless there has been a pathological
fracture).
No associated soft tissue mass.
Periosteal reaction absent or of benign pattern.

Location
As well as the obvious question of which bone is affected,
the part of the bone affected (epiphysis, metaphysis or
diaphysis) also helps one to narrow down the differential
diagnosis. For example, an aggressive lesion with a
permeative appearance in the diaphysis of a young patient
might be a Ewings sarcoma but the same type of appearance in the metaphysis would raise the possibility of
osteosarcoma instead. An expansile lesion seen within the
pelvis would have a different differential to an expansile
lesion seen in the axial skeleton.

205

Musculoskeletal Imaging

Age
Age is vitally important in forming a differential diagnosis
for focal bone lesions, as the probability of many lesions
varies dramatically between young and older people. For
the purposes of radiological assessment of focal bone
lesions, one might consider less than 30 years of age as
being young and over 40 as older! For example, many
benign bony lesions such as simple bone cyst and
aneurysmal bone cyst are really only seen in the younger
age group. Similarly, there are certain primary bone
malignancies also seen primarily in this age group, e.g.
Ewings sarcoma. Conversely, metastases are relatively
common in the older age group and consequently often
enter the differential diagnosis.

JOINTS
Arthropathy is a very common radiological problem in the
real world and in examinations. Pattern recognition of the
more common varieties is aided by exposure to as many
films as possible but thereafter, first principles can be used
when in doubt.
The do-not-miss lesion involving the joints is septic
arthritis be suspicious of this when presented with a case
in which there is a single joint showing features of joint
space narrowing, erosion of bone on both sides of the joint
and soft tissue swelling.
Subchondral sclerosis, geodes and osteophytosis are
features of degenerative disease, while osteopenia, soft
tissue swelling and erosions are common features of
inflammatory arthropathy. Joint space loss is seen in both,
so is not usually helpful in discriminating the two.
Other features to assess are the distribution axial vs
appendicular skeleton, small vs large joints, distal vs

206

proximal, synovial vs nonsynovial joints, symmetrical vs


nonsymmetrical. Age and gender may also be very helpful
to know. The distribution and nature of erosions are often
typical for various disorders and the radiologist must be
familiar with these aspects of the various erosive
arthropathies. Sometimes there is no definite best fit
diagnosis and one can only state the features supporting
the various possibilities and the importance of correlation
with clinical and laboratory features.
Other points to consider:
Syndesmophytes in the spine indicates a seronegative
arthropathy, commonly associated with the HLA-B27
antigen. These include ankylosing spondylitis,
psoriatic arthropathy, Reiters syndrome and
inflammatory bowel-related arthropathy.
Rheumatoid arthritis has classical features of
bilaterally symmetrical, proximal arthropathy with
marginal erosions and deformity but it is often
difficult to make a definite diagnosis from radiographs
alone.
Dont forget the crystal arthropathies well defined
erosions with sclerotic borders are typical of gout.
The 5 Ds of neuropathic joint destruction,
disorganization, dislocation, debris, density
(sclerosis).

SOFT TISSUES
Though more readily assessed on MRI, it is easy to forget
the soft tissues on plain films. However, swelling around
a bone may guide you to underlying fracture or
osteomyelitis, while swelling around a joint may point to
arthropathy or intra-articular fracture for example.

Musculoskeletal Imaging

CASE 115

Case 115

115a

History
A 13-year-old male presented with pain
in his left arm following a fall.

207

Answer 115

Musculoskeletal Imaging

ANSWER 115
Observations (115a)
This radiograph demonstrates a well defined lucent lesion
of the proximal metaphysis of the left humerus. The lesion
is centrally located with slight thinning of the cortex, mild
expansion and several internal septae. It has a narrow zone
of transition. There is a pathological fracture through the
lesion with associated periosteal reaction and a fallen
fragment sign. The features are consistent with a benign
lesion, specifically a simple bone cyst.

Diagnosis
Simple bone cyst (SBC).

Differential diagnosis
The fallen fragment sign is supposedly very specific for
SBC, but otherwise, possibilities include:
Aneurysmal bone cyst (ABC) 1030 year olds.
Occurs in the metaphysis +/ epiphysis. Like a simple
bone cyst it is a well defined lucency with an intact
cortex. This is typically an expansile lucent lesion. An
example is shown in the pelvis of a child (115b).
Giant cell tumour (GCT) seen in 2035 year olds.
Subarticular, eccentric location within epiphysis
(metaphysis if growth plate open). Cortical
destruction can occur and rarely metastases to the
lungs.
Brown tumour.
Fibrous dysplasia.
Enchondroma.

Discussion
Simple (unicameral) bone cysts are benign lesions usually
(75%) located in the proximal humerus and femur. They
occur in patients aged 515 years and are more commonly
found in males (ratio 3:1). Typical imaging features are
demonstrated with the lesion seen in the metaphysis,
however they gradually migrate to the diaphysis as the
bones grow (115c). They are asymptomatic unless
pathological fracture occurs and a small fragment of bone is

115b

115b AP pelvis in a child with an


aneurysmal bone cyst of the left iliac
bone characteristically appearing as a
lucent expansile lesion with a narrow
zone of transition.

208

115c

identifiable within the cyst the fallen fragment sign.


Periosteal reaction is not seen unless there is a fracture. A
fluid-fluid level may be seen on CT/MRI. Most
spontaneously regress.

Practical tips
A detailed discussion of all the possible focal lucent bone
lesions is beyond the scope of this case discussion and the
reader is advised to consult the relevant chapter in one of
the many excellent textbooks available. However, a few
words of advice can be given on how to approach the
problem and some basic guidelines regarding this case:
Know the age of the patient (this may be obvious
from the radiograph, for example if there are unfused
epiphyses). Over the age of 40, metastases and
myeloma become common and can show benign
features so often end up high on the list of
possibilities. Conversely, a young age immediately
skews the differential towards benign pathology.
Does the lesion have aggressive or nonaggressive
features? Bear in mind that aggressive does not
necessarily mean malignant as certain pathologies
such as infection can look aggressive. Aggressive
features include cortical destruction/breakthrough,
the presence of lamellated, spiculated or hair on end
periosteal reaction and a wide zone of transition (i.e.
poorly defined margins). Permeative lesions (multiple
small holes with no perceptible border and a wide
zone of transition) also tend to indicate malignancy.
Shown as an example is the pelvic radiograph (115d)
in an elderly female with metastases from breast
carcinoma; these lytic deposits are affecting the
majority of the right hemipelvis, the left ilium and
both femoral necks. In fact, there is a pathological
fracture of the left femoral neck. Note how the lesions
have a wide zone of transition and evidence of cortical
destruction with aggressive periosteal reaction, i.e. all
the features of an aggressive pathology. This contrasts
markedly with the nonaggressive features of the ABC
(115b).
115c Sagittal T2
weighted MRI image
shows a well defined
high signal cystic
lesion in the
proximal left femur.

Answer 115

Musculoskeletal Imaging

Which bone is affected and is it epiphyseal,


metaphyseal or diaphyseal? Certain pathologies tend
to have characteristic locations.
Differentiating SBC from ABC both affect a similar
age group and location and show benign features with
possible expansion, cortical thinning and fluid levels
on MRI. However, the SBC has a more central
location within the affected bone, is less expansile and
may show a pathognomonic fallen fragment.
GCT in the fused skeleton is subarticular and
eccentric with no marginal sclerosis when all these
features are present in the correct age group, the
diagnosis is clear.
Infection can look aggressive or benign so is always
worth bearing in mind depending on clinical history.
Similarly, fibrous dysplasia has such varied appearances
that it frequently enters the differential diagnosis for
the focal benign bone lesion!

Further management
None is necessary. Most spontaneously regress with age.
Pathological fractures commonly occur and may need
fixation.

Case 116

115d

115d AP pelvis in a female with


widespread lytic metastases and a
pathological fracture of the left femoral
neck. Note the cortical destruction and
the wide zone of transition of the
majority of the lesions.

CASE 116
History
A 40-year-old male patient with left hip pain.

116a

209

Answer 116

Musculoskeletal Imaging

ANSWER 116
Observations (116a)
There is right femoral head resurfacing in this patient.
Arthropathy of the left hip is seen with loss of joint space
and subchondral cysts. Sacroiliac joints are not well
demonstrated but there is the suspicion of fusion along
with ossification in the midline along the interspinous
ligaments. There is also arthropathy at the pubic symphysis
and whiskering at the ischial tuberosities in keeping with
enthesopathy. The features are characteristic of ankylosing
spondylitis.

Diagnosis
Ankylosing spondylitis.

Differential diagnosis
Of sacroiliitis on plain film:
Ankylosing spondylitis.
Inflammatory bowel disease.
Hyperparathyroidism tends to cause sacroiliac joint
widening due to bone resorption.
Rheumatoid.
Gout.
Psoriatic arthropathy.
Reiters syndrome.
Osteoarthritis (OA).
Infection TB.

seen and enthesopathy causes calcification at sites of


ligamentous insertion, described on plain films as
whiskering. An early sign of the disease is inflammation
at the site of attachment of the anterior longitudinal
ligament to the vertebral bodies and discs giving rise to
erosions at the anterior disco-vertebral margin. This finding
is termed a Romanus lesion, examples of which are shown
affecting the 3rd and 4th lumbar vertebrae in a young
adult patient (116b).
The peripheral skeleton is affected in 20% of patients
and commonly involves the hips. The disease is associated
with ulcerative colitis, aortic insufficiency and apical lung
fibrosis.
There are several causes of sacroiliitis, which can
sometimes be differentiated by looking for ancillary signs
and the degree of symmetry if bilateral. Generally, most
causes are bilateral except for infection and OA. However,
Reiters and psoriasis in particular are more likely to be
asymmetrically bilateral.
One final condition worth mentioning is osteitis
condensans ilii, a chronic stress reaction occurring in young
multiparous females, presumably due to pelvic instability.
This produces focal sclerosis along the inferior margin of
the joint on the iliac side only. The joint space itself is
spared.

Practical tips
Discussion
Ankylosing spondylitis is an autoimmune seronegative
arthropathy primarily affecting young males and
predominantly involving the spine. Syndesmophyte
formation and ossification of spinal ligaments leads to
ankylosis and stiffness. Bilateral symmetrical sacroiliitis is

116b

116b Lateral lumbar


spine radiograph
demonstrates lucent
lesions of the
anterosuperior aspects
of the L3 and L4
vertebrae. These
represent Romanus
lesions in a young adult
with early onset
ankylosing spondylitis.

On the pelvic radiograph showing sacroiliitis:


Unilateral disease infection or OA.
Bilateral but asymmetrical disease consider Reiters
and psoriasis. Figure 116c shows a case of Reiters
with sacroiliitis worse on the left.

116c

116c AP and lateral lumbar spine radiographs


showing sacroiliitis worse on the left side with
erosions and sclerosis.

210

Answer 116

Musculoskeletal Imaging

Bilateral symmetrical disease:


Spondylitis, syndesmophytes and periosteal
whiskering point to ankylosing spondylitis.
Look for bowel wall thickening or a stoma as a
marker of inflammatory bowel disease (Figure 116d
shows bilateral sacroiliitis and thickened distal
sigmoid colon in a patient with ulcerative colitis).
A femoral haemodialysis line or peritoneal dialysis
catheter may be present in patients with
hyperparathyroidism secondary to renal failure.

Case 117

116d

Iliac involvement only at the inferior joint margin in a


female patient is osteitis condensans.

Further management
Treatment of ankylosing spondylitis is supportive, one in
five patients progressing to significant disability. Ulcerative
colitis and aortitis are associations. One per cent of patients
develop upper zone lung fibrosis, which can be complicated
by superinfection with aspergillus. Death may occur from
cervical spine fracture or aortitis.

116d AP pelvis of a patient with ulcerative colitis.


There is bilateral sacroiliitis with an oedematous
thick walled sigmoid colon.

Further reading
Levine D, Forbat S, Saifuddin A (2004). MRI of the axial
skeletal manifestations of ankylosing spondylitis.
Clinical Radio lo gy 59: 400413.

CASE 117

117a

History
A 13-year-old male presented
with pain in his left hip.

211

Answer 117

Musculoskeletal Imaging

ANSWER 117
Observations (117a)
This AP radiograph in an unfused skeleton shows widening
of the growth plate of the left hip with a slight decrease in
height of the left femoral epiphysis compared to that on
the right. Also, Kleins line does not intersect the femoral
epiphysis on the left. The appearances are consistent with
a slipped left upper femoral epiphysis. A frog lateral should
be done to check if there is a subtle slip on the right.

to correct any rotational deformity. Prophylactic pinning


of the contralateral hip is often performed due to the risk
of bilateral disease.

Further reading
Boles C, El-Khoury G (1997). Slipped capital femoral
epiphysis. Radio Graphics 17(4): 809823.

Diagnosis
Slipped upper femoral epiphysis (SUFE).

Discussion
Atraumatic fracture through the hypertrophic zone of the
physeal plate results in a slipped capital femoral epiphysis.
It is the most common abnormality of the hip in
adolescence with an incidence of 2 in 100,000. It is
thought to occur due to widening of the epiphyseal plate
during growth spurt with change in the orientation of the
physis leading to an increase in shear forces. Affected
patients tend to be overweight teenaged males. The mean
age affected is 13 years for boys and 11 years for girls,
corresponding to the growth spurts. The condition is
associated with delayed skeletal maturation after
adolescence and is bilateral in 20%. Hip pain is the most
common presenting symptom with knee pain affecting a
quarter of all cases.
Radiographs show:
Apparent reduction in height of the femoral epiphysis.
Irregularity of the growth plate.
Kleins line does not intersect the femoral epiphysis.
The apparent reduction in epiphyseal height on the AP
radiograph is due to the direction in which it slips, most
commonly posteromedial. The line of Klein is drawn along
the superior edge of the femoral neck and should intersect
part of the epiphysis in a normal hip. Figure 117b shows
Kleins lines drawn on this childs film note how the left
line fails to intersect the epiphysis. These features may all
be very subtle on the AP radiograph and therefore a frog
lateral should always be performed when the diagnosis is
suspected. The frog lateral will often accentuate the
findings and may also confirm a subtle slip in the
contralateral hip. The frog lateral in this case (117c)
confirms the slip, which is more obvious than on the
anteroposterior film.

117b

117b Pelvis radiograph shows Kleins lines drawn on


it. The left line fails to intersect the epiphysis
suggesting slip.

117c

Practical tips
When assessing radiographs of childrens hips, the likely
pathologies can be easily predicted from the childs age:
48 years Perthes; 817 years SUFE.

Further management
If untreated, slipped capital femoral epiphysis can result in
avascular necrosis (AVN) of the femoral head with
consequent osteoarthritis, a debilitating condition when
acquired at such a young age. The risk of AVN increases
the larger the degree of slip and the longer the delay to
surgery. It is therefore of paramount importance that it is
not missed on imaging. Treatment involves pinning the
epiphysis to the femoral neck, sometimes with osteotomy

212

117c Frog lateral in the same patient accentuates the


slipped left femoral epiphysis.

Musculoskeletal Imaging

CASE 118

Case 118

118a

History
A male patient presented with pain in
amputation stump.

213

Answer 118

Musculoskeletal Imaging

ANSWER 118
Observations (118a)

Discussion

A below knee amputation has previously been performed.


There is a permeative lucency with a wide zone of transition
affecting the tibial stump. Periosteal reaction is visible along
the posterior cortex and there is also a hint of this along
the fibula. Furthermore, faint lucencies are suspected along
the anterior cortex of the distal femur. These are the
appearances of an aggressive lesion.
The reason for amputation and the time interval since
have not been stated this would help clarify the likely
diagnosis. From the radiological findings alone, the main
possibilities are that the amputation was for a distal
malignancy such as bone lymphoma that has now recurred,
or that the current findings are due to osteomyelitis in the
stump following amputation for an unrelated reason.
However, suspected involvement of the distal femur
without joint destruction favours the former option.

Another example of a lymphoma is seen affecting the right


tibia of a patient in Figure 118b; a permeative lesion is
present with evidence of cortical destruction.
Permeative bone destruction on radiography indicates
the presence of aggressive pathology with rapid growth
potential. The permeative lesion has a diffuse lytic moth
eaten appearance (118c). The zone of transition is wide, in
other words the lesion is poorly demarcated and
imperceptibly merges with uninvolved bone, unlike benign
lesions where the zone of transition to normal bone is
narrow. Periosteal reaction may or may not be present. The
finding of a permeative lesion on imaging usually indicates
malignant pathology; however, infection can produce the
same appearance.

Diagnosis
Lymphoma recurrence following amputation.

Differential diagnosis
Of permeative bone lesions:
Metastases.
Myeloma.
Lymphoma.
Leukaemia.
Osteomyelitis.

118b

Practical tips
This is an example of a viva type film where there is no
definite or spot diagnosis to be made it is simply a case
of presenting a reasoned approach to a sensible differential
diagnosis, then stating how this could be narrowed down
using clinical information or further investigations.

Further management
Recurrence of lymphoma may entail further surgery
and/or radiotherapy. MRI may be helpful to assess the
local extent more accurately.

118c

118b AP and lateral images of the tibia show a


permeative lesion is present with evidence of cortical
destruction.
118c Fat Saturated coronal MRI
image from the same patient as
118b shows an aggressive lytic
bone lesion with a wide zone of
transition and permeative bone
destruction breaching the cortex
of the tibia.

214

Musculoskeletal Imaging

Cases 119, 120

CASE 119
History
A 4-year-old boy presented with a
suspected chest infection.

119a

CASE 120
History
A 62-year-old female presented
with neck pain.

120a

215

Answer 119

Musculoskeletal Imaging

ANSWER 119
Observations (119a)

Discussion

There is diffuse osteosclerosis of the visible skeleton. Subtle


widening of the dia-metaphysis of the proximal humeri are
noted, suggestive of Erlenmeyer flask deformity. There is
no lung abnormality. The findings are consistent with
osteopetrosis.

Osteopetrosis (known as marble bone disease) is a rare


hereditary disorder of defective osteoclast function with
failure of proper reabsorption and remodelling leading to
sclerotic and structurally brittle bones. In adults, the
disease has an autosomal dominant inheritance and is
asymptomatic in half of patients. Symptomatic patients
have recurrent fractures and occasionally cranial nerve palsy
due to narrowing of the neural foramina at the skull base
(the calvaria is often spared). Erlenmeyer flask deformity
of the long bones may be seen due to lack of tubulization.
Erlenmeyer flask deformity is a descriptive term used to
describe the distal expansion of long bones, particularly the
femora, that is seen in a number of skeletal diseases (named
after the wide-necked laboratory flask bearing the name of
this German chemist). A radiograph of the right femur of
the same 4-year-old boy (119b) shows the characteristic
Erlenmeyer flask deformity with diffuse osteosclerosis.
Other associated features on plain radiography include
bone within bone appearance and sandwich vertebrae
(sclerotic endplates). The infantile autosomal recessive
form is the severe type associated with stillbirth whereas
the adult type is associated with normal life expectancy.

Diagnosis
Osteopetrosis.

Differential diagnosis
Of osteosclerosis in children:
Osteopetrosis.
Pyknodysostosis.
Renal osteodystrophy.
Hypervitaminosis D.
Hypervitaminosis A.
Fluorosis.
Of Erlenmeyer flask deformity (mnemonic Lead
GNOME):
Lead.
Gauchers.
NiemannPick disease.
Osteopetrosis.
Metaphyseal dysplasia (Pyles) and craniometaphyseal
dysplasia (same as Pyles disease but there is a history
of cranial nerve palsies).
Ematological!! thalassaemia.

119b

216

119b Radiograph
of femur in same
child shows
diffuse
osteosclerosis
with widening of
the distal femoral
diametaphysis, i.e.
Erlenmeyer flask
deformity.

Practical tips
For osteosclerosis in children:
Erlenmeyer flask deformity, sandwich vertebrae and
bone within bone appearance point to osteopetrosis.
Pyknodysostosis is associated with multiple wormian
bones. As with osteopetrosis, pathological fractures
may be seen (119c). A hand radiograph in another

119c

119c Diffuse
osteosclerosis
with a
pathological
fracture of the
tibia in a patient
with
pyknodysostosis.

Answers 119, 120

Musculoskeletal Imaging

patient with pyknodysostosis (119d) demonstrates


diffuse sclerosis with classical pointed chalk terminal
phalanges.
Look for a haemodialysis line in renal osteodystrophy.
Fluorosis (119e) is associated with ligamentous
insertion calcification.
For Erlenmeyer flask deformity:
Diffuse sclerosis indicates osteopetrosis.
Pyles disease will demonstrate sclerosis of the
diaphysis and lucency of the widened metaphysis.
Gauchers disease will be associated with generalized
osteopenia and pencil-thin cortices. There may also be
signs of avascular necrosis of the femoral or humeral
heads and massive hepatosplenomegaly may be seen
on abdominal radiographs.
Thalassaemia is associated with coarsened
trabeculation producing a cobweb appearance.
Lead poisoning causes dense metaphyseal bands.

Further management
The only treatment available for osteopetrosis is bone
marrow transplantation. Patients are more prone to
fractures than the normal population.

119d
Pyknodysostosis
hand showing
osteosclerosis with
pointing of the
distal phalanges
producing a
pointed chalk
appearance.

119d

119e Forearm of a
patient with
fluorosis showing
diffuse
osteosclerosis and
prominent
ligamentous
insertion
calcification.

119e

ANSWER 120
Observations (120a)
Lateral flexion and extension views of the cervical spine are
shown. On flexion there is significant atlantoaxial joint
subluxation. The odontoid peg is not clearly demarcated
and is likely to be partially eroded. The remainder of the
cervical spine is quite well preserved. The most likely
diagnosis in a patient of this age is rheumatoid arthritis.

Diagnosis
Atlantoaxial subluxation in a patient with rheumatoid
arthritis (RA).

Differential diagnosis
Of atlantoaxial subluxation:
RA.
Psoriatic arthropathy.
Juvenile idiopathic arthritis.

Ankylosing spondylitis.
Systemic lupus erythematosus (SLE).
Downs syndrome.
Morquios syndrome.
Retropharyngeal abscess in a child.

Discussion
Atlantoaxial subluxation occurs when the distance between
the posterior aspect of the arch of the atlas and the anterior
aspect of the odontoid peg exceeds 3 mm in adults and 5
mm in children. Erosion and destruction of the odontoid
peg may also be seen, particularly when the process is
caused by an inflammatory arthropathy. Several causes are
described in the differential diagnosis list but RA is the
most common cause in adults. Synovitis with pannus
formation causes erosion of the odontoid peg and
atlantoaxial ligaments and consequent subluxation. This
(co nt.)

217

Answer 120

Musculoskeletal Imaging

can be more readily seen on MRI: a T1 weighted sagittal


image (120b) shows synovial destruction of the odontoid
peg and subluxation at midcervical spine level causing
compromise to the spinal cord. Eventually cranial settling
can occur whereby the odontoid process can project into
the skull base due to significant disease of the atlantooccipital and atlantoaxial joints.
The hallmark of RA is bilateral symmetric arthropathy
of more than three joints. Typically, the second and third
metacarpophalangeal and the third proximal interphalangeal joints are involved early in the course of the
disease. Bilateral and symmetric involvement of foot joints
is another typical manifestation of RA (120c). The
radiological features seen in RA affecting the extremities
include:
Periarticular soft tissue swelling.
Periarticular osteoporosis.
Marginal erosions.
Ankylosis.
Subluxation and dislocation.
Subchondral cysts.
Bilateral symmetrical distribution.
Widening of joint space early on, with narrowing later
in disease.
Arthritis mutilans late in disease.

Practical tips
Look for other signs of RA in the neck.
Bamboo spine suggests ankylosing spondylitis.
Posterior vertebral scalloping and anterior vertebral
beaks in Morquios.
Soft tissue swelling in a child consider abscess.

Further management
Clinical assessment and measurement of serum markers
such as rheumatoid factor are required. To confirm the
findings on plain radiography one might consider MRI of
the cervical spine to assess the soft tissue component of the
disease. Atlantoaxial subluxation may be managed
surgically or conservatively with a stiff collar depending on
the particular circumstances. The anaesthetist is often
particularly interested in excluding this complication in
rheumatoid patients as it presents an obvious hazard
during preoperative airway intubation.

Further reading
Sommer O, Kladosek A, Weiler V, et al. (2005).
Rheumatoid arthritis: a practical guide to state-of-theart imaging, image interpretation, and clinical
implications. Radio Graphics 25: 381398.

120b

120c

120b Sagittal T1 weighted image of


the cervical spine showing erosion
of the odontoid peg and subsequent
atlantoaxial subluxation.

218

120c Radiograph of both feet of a rheumatoid


patient.

Musculoskeletal Imaging

CASE 121

Case 121

121a

History
A young Afro-Caribbean patient
presented with left knee pain.

121b

219

Answer 121

Musculoskeletal Imaging

ANSWER 121
Observations (121a, 121b)

Tumoral calcinosis.

cause pain and limitation of movement, with overlying skin


ulceration and the development of a sinus tract draining
chalky fluid. In fact, the masses may contain fluid-fluid
levels on imaging that have a milk of calcium consistency.
The periarticular region of the hip is the most commonly
affected site. The masses can grow to a very large size and
there is a tendency for recurrence if the lesions are not
completely excised.

Differential diagnosis

Practical tips

Of periarticular soft tissue calcification:


Haematoma.
Myositis ossificans.
Crystal arthropathy.
Scleroderma (121c note the characteristic distal tuft
resorption in the index finger).
Dermatomyositis.
Synovial osteochondromatosis.
Tumoral calcinosis.
Synovial sarcoma (121d, 121e, 121f, 121g).

The radiological appearances of the varying causes of


periarticular calcification can be quite different:
Tumoral calcinosis lobulated dense masses, fluidfluid levels.
Myositis ossificans and synovial sarcoma may look
similar but without fluid levels.
Dermatomyositis sheets of calcification.
Synovial osteochondromatosis several foci of
calcification with lucent centres that are essentially
loose bodies within the joint.

Discussion

Further management

Tumoral calcinosis is a rare, benign condition characterized


by the presence of progressively enlarging periarticular
calcified soft tissue masses. It usually affects young black
patients and there is a familial tendency. The masses may

Patients with tumoral calcinosis are usually treated with


phosphate depletion. Surgical excision is also a treatment
option but needs to be meticulous as recurrence often
occurs with incomplete excision.

AP and lateral radiographs of the left knee show lobulated


calcified masses in the soft tissues anterior to the left knee.
Taking into consideration the age and ethnicity of the
patient, tumoral calcinosis is the most likely diagnosis.

Diagnosis

121c

121e

121c Radiograph of the


hand in a female adult
with scleroderma shows
the typical soft tissue
calcification. Note the
distal tuft resorption
affecting the index
finger, which is
characteristically seen in
this condition.

121f

121d

121d
Radiograph of
the left hip of a
child with
periarticular
calcification
arising from a
synovial
sarcoma.

121g

121e, 121f, 121g Axial T1, T2 and fat saturated images of the synovial sarcoma shows a fairly well defined
soft tissue lesion which is of low signal on T1 and inhomogeneously high signal on T2.

220

Musculoskeletal Imaging

CASE 122

Cases 122, 123

122a

History
A 40-year-old presented with knee
discomfort.

CASE 123

123

History
Male patient presented with rigidity and
kyphosis.

221

Answer 122

Musculoskeletal Imaging

ANSWER 122
Observations (122a)

Practical tips

AP and lateral radiographs of the right knee and upper tibia


and fibula demonstrate marked cortical thickening of the
diaphyses of the tibia and fibula, which has the appearance
of dripping candle wax. This is typical of melorheostosis.
There are also multiple small sclerotic foci within the
metadiaphysis of the distal femur and the proximal tibia,
which have the appearance of osteopoikilosis.

Osteopoikilosis tends to be distributed around joints,


whereas multiple sclerotic metastases will not be so
confined.

Diagnosis

Further management
No further management is normally necessary. Both these
conditions are usually found incidentally. The main thing
is to ensure that a patient with multiple sclerotic lesions
has osteopoikilosis and not sclerotic metastases.

Melorheostosis and osteopoikilosis.

Further reading
Differential diagnosis
Sclerotic metastases in a patient with melorheostosis; this
is much less likely. However in patients with multiple
sclerotic foci alone, sclerotic metastases must always be
considered.

Levine S, Lambiase R, Petchprapa C (2003). Cortical


lesions of the tibia: characteristic appearances at
conventional radiography. Radio Graphics 23:
157177.

Discussion
Melorheostosis is a nonhereditary disease of unknown
aetiology that often presents as an incidental finding. It is
usually discovered in childhood where it has a rapid
progression, but it occasionally presents in adults where it
has a slow chronic course. The limb involved often
demonstrates joint pain, swelling and limitation of
movement. Males and females are equally affected.
Radiological signs of melorheostosis are as follows:
Cortical hyperostosis in one or multiple tubular bones
with streaks of sclerosis beginning at the proximal end
of the bone and extending distally. This produces the
characteristic dripping candle wax appearance. This is
shown particularly well in Figure 122b, where the
right humerus and scapula are affected in another
patient.
Predominantly affects the diaphysis of the bone.
The lower extremities are more commonly affected
than the upper.
Although a single bone may be involved, contiguous
bones of an extremity are more often affected.
Bilateral signs are extremely rare and should prompt
consideration of other causes of sclerosis.
Limb length discrepancy is also a feature and the
sclerosis may cross the joint and result in joint fusion.
The skull, spine and ribs are rarely involved. Melorheostosis
is associated with osteopoikilosis, osteopathia striata
(asymptomatic disease consisting of longitudinal striations
along the metaphyses of long bones) and arteriovenous
malformations.
Osteopoikilosis is an autosomal dominant disorder that
is more common in males. It is asymptomatic and consists
of multiple ovoid bone islands parallel to the long axis of
the bone. These bone islands normally measure 210 mm
and are found at the metaphysis and epiphysis, rarely
extending into the midshaft. It usually affects the pelvis,
wrist and ankle and rarely affects the skull, ribs and
mandible. The differential diagnosis of this condition,
which often leads to clinical confusion and concern, is
disseminated sclerotic metastases.

222

122b

122b AP radiograph of
melorheostosis affecting the
humerus and scapula
demonstrating the typical
dripping candle wax appearance.

Answer 123

Musculoskeletal Imaging

Case 124

ANSWER 123
Observations (123)
This AP radiograph of the pelvis reveals thick linear
columns of calcification and ossification around both hips
and in the paravertebral regions bilaterally. The hip joints,
sacroiliac joints and spine are fused. The patient has had
previous internal fixation of the left hip. The appearances
are in keeping with myositis ossificans progressiva.

Diagnosis
Myositis ossificans progressiva.

Discussion
This is an Aunt Minnie. Myositis ossificans progressiva is
also known as fibrodysplasia ossificans progressiva. It is a
rare slowly progressive disease characterized by
exacerbations and remissions of fibroblastic proliferation
leading to ossification and calcification of skeletal muscle,
subcutaneous fat, tendons and ligaments.
Radiological features are:
Linear columns of ossification and calcification.
Ossified bridges between different bones.
Ankylosis.
Kyphosis due to rigidity of the muscles of the spine
and upper limbs.

CASE 124

Half of patients present by the age of 2 years. Initially


subcutaneous painful masses develop in the neck and upper
limbs, which may ulcerate and bleed. Eventually there is
progressive involvement of the remaining musculature,
including that of the pelvis and lower extremities.
Torticollis also occurs due to restriction of the sternocleidomastoid muscles.
Ankylosis and limitation of movement progress and the
patient gradually becomes a stone person. Eventually,
respiratory failure develops due to calcification of the
thoracic muscles. Skeletal anomalies are associated with this
condition including microdactyly of the big toes and
thumbs and shortening of the first metatarsal with hallux
valgus deformity. Some patients may have a shallow
acetabulum with shortening and widening of the femoral
necks. Fusion of the middle ear ossicles may lead to
conductive hearing loss.

Practical tips
Ossified bars and bridges between bones are the hallmark
of this rare condition.

Further management
Treatment is supportive. Attempts at surgery to relieve
rigidity have led to accelerated ossification at the surgical
site.

124a

History
A child presented with retarded
growth.

223

Answer 124

Musculoskeletal Imaging

ANSWER 124
Observations (124a)

Further management

There is diffuse, coarse trabeculation of the visible bones


causing a cobweb-like appearance. Erlenmeyer flask
deformity of the distal femurs is present. The features are
in keeping with thalassaemia major.

Death usually occurs within the first decade. Treatment is


by multiple transfusions, however as explained in the
discussion this too leads to skeletal abnormalities.

Further reading
Diagnosis
Thalassaemia major.

Tyler P, Madani G, Chaudhuri R, et al. (2006). The


radiological appearances of thalassaemia. Clinical
Radio lo gy 61(1): 4052.

Discussion
Thalassaemia is an inherited disorder of haemoglobin
synthesis characteristically seen in Mediterranean patients.
The homozygous form, thalassaemia major, is more severe.
The radiological signs result from marrow hyperplasia and
expansion due to extramedullary haematopoiesis. Every part
of the skeleton may be affected in patients with untreated
disease. Radiological features of the complications of
treatment such as recurrent transfusions and iron chelation
therapy may also be seen. In fact, abnormalities secondary
to iron chelation therapy are now more common than
those due to marrow hyperplasia.
The radiological signs to look for depend on the site of
the body imaged:
Peripheral skeleton:
Coarse trabeculation causing cobweb appearance
(124a).
Loss of concavity of tubular bones (124a).
Erlenmeyer flask deformity of metaphyses of long
bones (124a).
Arthropathy and chondrocalcinosis as a result of
haemochromatosis secondary to hypertransfusion.
Fraying of metaphyses and dense metaphyseal
bands secondary to iron chelation therapy.
Skull:
Hair on end appearance.
Frontal bossing (124b) due to diploic expansion.
Obliteration of paranasal sinuses (except for
ethmoid sinuses) due to marrow hyperplasia
(124b).

124b

124b Lateral radiograph of a child with thalassaemia;


there is frontal bossing with expansion of the diploic
space and obliteration of the maxillary sinuses.

124c

Axial skeleton:
Coarse trabeculation causing cobweb appearance
(124c).
Biconcave vertebrae.
Bone within bone appearance of spine and ribs.
Paraspinal masses (due to extramedullary
haematopoiesis).
Expansion of the ribs posteriorly, particularly at the
costochondral junctions, due to marrow hyperplasia
(124c).

Practical tips
The only sign may be a diffuse but subtle coarsening
of the bony trabeculae.
On an AXR look for evidence of hepatosplenomegaly
(a result of extramedullary haematopoiesis) and
gallstones.
On a CXR look for cardiomegaly secondary to
anaemia, as well as a paraspinal mass due to
extramedullary haematopoiesis.

224

124c Radiograph of chest demonstrates diffuse


coarse trabeculation producing a cobweb
appearance with expansion of the ribs due to
marrow hyperplasia.

Musculoskeletal Imaging

CASE 125

Case 125

125a

History
A 10-year-old child presented with leg
pain and fever.

225

Answer 125

Musculoskeletal Imaging

ANSWER 125
Observations (125a)
There is a mostly sclerotic lesion arising in the diaphysis of
the proximal fibula in this unfused skeleton. This has a wide
zone of transition and demonstrates a florid hair on end
periosteal reaction. The features are in keeping with an
aggressive lesion. Ewings sarcoma and osteosarcoma are
the two main differential diagnoses and the patient should
be referred urgently to a specialist unit.

Diagnosis
Ewings sarcoma.

Differential diagnosis
Osteosarcoma.

Discussion
Ewings sarcoma is the most common malignant bone
tumour in children, with a peak age of 15 years. Most
patients (96%) are Caucasian with a male to female ratio of
1:2. These are round cell tumours and are clinically,
radiologically and histologically very similar to primitive
neuroectodermal tumours (PNET). Patients present
clinically with severe pain that may be associated with a soft
tissue mass. Fever and leukocytosis are also features. Long
bones are affected in the majority of cases and this usually
affects the diaphysis of the bone rather than the metaphysis
(which is the more common site of involvement for
osteosarcoma). Flat bones such as the pelvis and ribs are
affected in 40% of cases and this is predominantly seen in
older patients, whereas younger patients tend to present
with long bone lesions. In most cases the lesion is lytic
rather than dense and there is often a permeative moth
eaten appearance. Invasion into local soft tissues is seen in
up to a half of cases and there is usually an aggressive
periosteal reaction causing a sunburst or hair on end
appearance. A Codmans triangle may also be seen.
These features can all be seen in osteosarcoma and
sometimes it can be difficult to differentiate the two
conditions radiologically. A radiograph of the right femur
and knee in a 12-year-old boy with osteosarcoma is shown
(125b). This can be seen as a sclerotic density with a wide
zone of transition in the metadiaphysis of the distal femur,
with aggressive periosteal reaction. Overall, the appearances
are quite similar to those seen in Figure 125a and both
diseases have a similar 6080% 5 year survival rate. The
practical tips section below lists some features that may help
differentiate the two.

Practical tips
In assessing focal bone lesions, the first concern is
whether the lesion is benign or aggressive (note that
aggressive lesions are not necessarily malignant
infection, for example, can cause aggressive
appearances). One of the most important factors is the
clarity and extent of the margin between the lesion
and adjacent normal bone, the so-called zone of
transition. A wide zone of transition indicates an
indistinct and extended junction between lesion and
normal bone, and is a hallmark of the aggressive

226

lesion. Conversely, benign lesions show a narrow


zone of transition. The presence and type of periosteal
reaction are other important features to assess.
Aggressive lesions more commonly show periosteal
reaction, which may be irregular, lamellated or
spiculated. Codmans triangle is the term applied to
the elevated corner of periosteum at the margins of
the periosteal reaction, and is frequently seen in
malignancy. Cortical destruction is another sign of the
aggressive lesion, though one can see cortical
thinning and pathological fracture through benign
bone lesions.
The other two important factors in establishing a
diagnosis in the focal bone lesion are the patients age
and the location of the lesion. Location includes not
only which bone, but whether the lesion is epiphyseal,
metaphyseal or diaphyseal. A comprehensive
discussion of the distribution of benign and aggressive
lesions by age and location is beyond the scope of this
case discussion, but an appreciation is vital in
establishing a meaningful differential diagnosis.

125b

125b AP knee
radiograph in a
12-year-old child
demonstrates a
sclerotic lesion of the
femoral metaphysis
that has a wide zone
of transition and
aggressive periosteal
reaction. This proved
to be an
osteosarcoma.

Answer 125

Musculoskeletal Imaging

To summarize the approach to the focal bone lesion,


ask yourself:
Is it benign or aggressive?
Diaphysis, metaphysis or epiphysis?
How old is the patient?
In the young patient with an aggressive lesion such as
those illustrated, the two main primary tumours to
consider are Ewings and osteosarcoma. The two can
be difficult to differentiate radiologically, and can also
cause similar clinical features such as fever and
leukocytosis. The features shown in Table 2 may help,
though none are pathognomonic.

Case 126

Further management
MRI is helpful in assessing local spread and aids planning
of surgical resection.

Further reading
Murphey M, Robbin M, McRae G, et al. (1997). The
many faces of osteosarcoma. Radio Graphics 17:
12051231.

Table 2 Differentiating features o f Ewings sarco ma and o steo sarco ma


Age
Location
Appearance
Metastases

Ewings sarcoma
Children
More common in diaphysis
More commonly lucent or permeative
To bone

CASE 126

Osteosarcoma
Older children and young adults
More common in metaphysis
More commonly sclerotic
To lung

126a

History
A 35-year-old male presented with
shoulder pain.

227

Answer 126

Musculoskeletal Imaging

ANSWER 126
Observations (126a)

Practical tips

There is a transverse fracture at the medial third of the


clavicle, which is likely to be an insufficiency fracture as
there are Looser zones at the lateral aspect of the clavicle
and the lateral border of the body of the scapula. The bones
are generally osteopenic with marked thinning of the
cortices. The findings are consistent with a diagnosis of
osteomalacia.

Osteomalacia is related to renal failure and secondary


hyperparathyroidism therefore:
On an abdominal radiograph look for a peritoneal
dialysis catheter or a tunnelled femoral haemodialysis
line.
On a chest radiograph look for a central haemodialysis
line and eroded lateral clavicles.
Prominent vascular and soft tissue calcification may be
seen.
Brown tumours may be present and appear as lytic
lesions, which may also be associated with
pathological fracture.

Diagnosis
Osteomalacia.

Discussion
Osteomalacia is a disorder of insufficient osteoid
mineralization causing bone softening. Aetiology can be
due to dietary deficiency, decreased absorption or deficient
metabolism of vitamin D.
Radiological features of osteomalacia include:
Generalized osteopenia.
Cortical thinning.
Bowing of long bones.
Protrusio acetabuli.
Coarse trabecular pattern (126b).
Looser zones:
Pseudofractures that consist of transverse lucent
clefts with sclerotic margins.
These are mostly seen in the pelvis, femoral necks
(126c) and scapula.

Further management
Treatment involves reversing the cause of vitamin D
deficiency.

Insufficiency fractures.

126b

126c

126b Radiograph of both hands in a patient with


osteomalacia demonstrates diffusely coarsened
trabecular pattern.

126c AP pelvic radiograph with Looser zones in both


proximal femora.

228

Musculoskeletal Imaging

Cases 127, 128

CASE 127
History
A 5-year-old child presented with
marked bowing deformity of right leg.

127a

CASE 128

128

History
A young adult with nail
abnormalities.

229

Answer 127

Musculoskeletal Imaging

ANSWER 127
Observations (127a)
There is marked angulation deformity of the diaphyses of
the lower tibia and fibula with the formation of
pseudarthroses. The bones are also generally osteopenic.
The most likely diagnoses are nonunion of previous
fractures, neurofibromatosis or osteogenesis imperfecta.

Diagnosis
Neurofibromatosis.

Differential diagnosis

Nonunion of a fracture.
Osteogenesis imperfecta.
Fibrous dysplasia.
Congenital.

Discussion
Congenital affects the middle to lower third of the tibia
and fibula. Half of congenital pseudarthroses present in the
first year of life and later on there may be cupping of the
proximal bone end and pointing of the distal bone end.
Neurofibromatosis type 1 (NF1) is a common genetic
disorder and in addition to cutaneous and neurological
abnormalities, osseous lesions are also seen. There may be
anterolateral bowing of the tibia with or without a
hypoplastic fibula. Focal narrowing and intramedullary
sclerosis or cystic change at the apex of the angulation is
due to hamartomatous fibrous tissue, typically at the
junction of the middle and distal third of tibia. Pathological
fracture with nonunion often results in pseudarthrosis of
the tibia and fibula, with pencil pointing of the bone
fragments. Prophylactic bracing of limbs with bowing
deformity may prevent the development of pseudarthrosis.
Osteotomy with bone grafting and pinning (127b) is the
treatment of choice if the pseudarthrosis has already
occurred.

127b

230

Osteogenesis imperfecta all four types can result in


bowing of the long bones due to bone softening and
multiple fractures. Bowing typically involves all the long
bones and can result in pseudarthroses. Again osteotomies
and pinning are the preferred treatment although
bisphosphonates have been shown to produce some
success.
Cleidocranial dysplasia is associated with congenital
pseudarthrosis of the femur.
Ankylosing spondylitis can lead to pseudarthrosis in the
spine.

Practical tips
Severe osteopenia and multiple fractures of differing
ages that have exuberant callus formation suggest
osteogenesis imperfecta. Remember that some
osteopenia may result from disuse, however, e.g. in
fracture nonunion.
Look for soft tissue nodules indicative of
neurofibromatosis.
A ground glass density lesion associated with the
pseudarthrosis is suggestive of fibrous dysplasia.

Further management
As in this case, osteotomy with bone grafting and pinning
is the treatment of choice.

Further reading
Cheema J, Grissom L, Harcke H (2003). Radiographic
characteristics of lower-extremity bowing in children.
Radio Graphics 23: 871880.

127b Radiograph of the same patient postfixation


with pins.

Answer 128

Musculoskeletal Imaging

Case 129

ANSWER 128
Observations (128)
There are bilateral posterior iliac horns consistent with
Fongs disease.

Diagnosis

dislocations. Hypoplasia of the capitellum and radial head


may also be present and this can lead to an increase in the
carrying angle at the elbow. Similarly, genu valgus can
occur due to asymmetrical development of the femoral
condyles. Some patients may have a short 5th metacarpal.

Fongs disease.

Practical tips
Discussion
This case is an Aunt Minnie. Fongs disease, also known as
nailpatella syndrome and osteo-onychodysplasia, is a rare
autosomal dominant disorder characterized by symmetrical
ectodermal and mesodermal anomalies. Patients tend to
present with abnormalities of nail dysplasia that can
manifest as spooning and splitting of the fingernails or even
hypoplasia or aplasia. This particularly affects the thumb
and index fingernails. Patients may also have abnormal
pigmentation of the iris.
The presence of bilateral posterior iliac horns is seen in
80% of cases and is diagnostic of the condition. Hypoplasia
of the anterior half of the ilia can result in drooping of the
iliac crests. The other major finding is aplasia or hypoplasia
of the patellae, which frequently results in recurrent lateral

CASE 129

Look for a peritoneal dialysis catheter or femoral


tunnelled dialysis line on the pelvic radiograph, as
these patients may have renal failure.
If the knees are included on the radiograph, look for
hypoplastic or absent patellae and genu valgus
deformity.

Further management
The most serious association of this condition is renal
dysfunction, which occurs secondary to abnormality of the
glomerular basement membrane leading to proteinuria,
haematuria and renal failure. The exact mechanism for this
is unknown, but renal failure tends to occur in later life.
This is an important point to note when diagnosis of this
condition is made, often incidentally.

129a

History
A patient presented with head injury.

231

Answer 129

Musculoskeletal Imaging

ANSWER 129
Observations (129a)

Practical tips

There is a hair on end appearance to the skull vault and


the diploic space is widened. This does not affect the
calvaria below the level of the internal occipital protuberance. The maxillary sinuses are obliterated. The most likely
cause is thalassaemia major.

Thalassaemia major and sickle cell disease are the


most common causes of hair on end skull. To
differentiate the two look at the maxillary sinuses if
they are obliterated this suggests thalassaemia. This
does not occur in sickle cell disease.
Is the appearance diffuse or localized? If localized
consider neoplastic causes such as haemangioma
(129b).
Haemangioma tends to have a corduroy appearance,
i.e. thickened and coarsened rather than thin vertical
trabeculation due to vascular channels. This is well
demonstrated in Figure 129b.

Diagnosis
Thalassaemia major.

Differential diagnosis
Of hair on end appearance of skull:
The mnemonic is STAN and is easy to remember when
you think of Stan Laurels hair!
Sickle cell disease.
Thalassaemia major.
Anaemia (other anaemias):
Hereditary spherocytosis.
Glucose-6-phosphate dehydrogenase deficiency.
Severe iron deficiency anaemia.
Neoplastic:
Haemangioma (129b).
Neuroblastoma metastases in children.

Further management
Thalassaemia major has a poor prognosis with most
affected children not surviving past the first decade.
Treatment is by repeated transfusions.

Further reading
Hollar M (2001). The hair-on-end sign. Radio lo gy 221:
347348.

Discussion
The hair on end sign is a finding that can be seen in the
diploic space of the skull on radiographs, CT and MRI, and
has the appearance of long, thin, vertical striations. On plain
radiographs and CT the appearance is caused by alternating
thickened trabeculae and radiolucent marrow hyperplasia.
On MRI, the alternating bands of hypointense trabeculae
and hyperintense marrow produce the distinct striated
pattern. Essentially the effect is due to marrow hyperplasia.
The diploic space widens and the outer table thins and can
become obliterated. With regard to anaemic causes, the
marrow hyperplasia begins in the frontal region and can
affect the entire calvaria excluding that which is below the
internal occipital protuberance, since there is no marrow in
this area. Marrow hyperplasia in thalassaemia major is more
marked than in any other anaemia and may cause
hyperplasia of the facial bones resulting in obliteration of
the paranasal sinuses. However the ethmoid sinuses are
spared as they do not contain marrow. The hair on end
appearance can also be seen in severe childhood cases of
iron deficiency anaemia.
The medical literature is split as to whether the hair on
end appearance may be reversed following treatment of
the anaemia. Some authors have reported that resolution
of the appearance occurs with treatment, although the
diploic space may remain wider than normal. However
others have reported that the appearances persist without
regression even over a follow-up period of approximately
20 years.

232

129b

129b The trabeculations here are thickened and


coarsened rather than thin, vertical trabeculations.
Haemangioma tends to have this appearance.

Musculoskeletal Imaging

CASE 130

Cases 130, 131

130a

History
A 45-year-old male presented with left
sided chest pain for several weeks. He
had suffered minor trauma to the arm.

CASE 131

131a

History
A 20-year-old male presented
with knee pain.

233

Answer 130

Musculoskeletal Imaging

ANSWER 130
Observations (130a)
Multiple bony exostoses are seen arising from the ribs on
both sides of the chest. On the left, there is a large
associated soft tissue mass projected over the lateral thorax.
Given the history of chest pain, sarcomatous transformation
must be suspected. Moreover, there is a small left pleural
reaction, a large pulmonary nodule in the left upper zone
and possible pulmonary nodules in the left lower zone and
right costophrenic recess.
The combination of findings suggests sarcomatous
transformation in diaphyseal aclasis with pulmonary
metastases.

Diagnosis
Diaphyseal aclasis with sarcomatous transformation.

Discussion
Diaphyseal aclasis is an autosomal dominant condition
characterized by multiple exostoses (osteochondromas).
Osteochondromas are benign cartilaginous tumours.
Diaphyseal aclasis is usually discovered in childhood and
short stature may occur due to the development of
exostoses at the expense of normal bone growth. The
exostoses are usually multiple and bilateral and mostly affect
the limbs though ribs can be affected, as in this case. The

234

exostoses point away from the nearest joint, as illustrated in


image 130b. A pseudo-Madelung deformity may develop
(130b), where there is ulnar shortening with bowing of
the radius, and ulnar tilt of the distal radial articular surface.
In fewer than 5% of patients, malignant transformation
into chondrosarcoma or osteosarcoma can occur. A further
example is shown (130c) where a patient with diaphyseal
aclasis developed a chondrosarcoma affecting the femur.
The exostoses may also cause neurological compromise
due to nerve compression or entrapment.

Practical tips
Diaphyseal aclasis is a misnomer as the exostoses
arise from the metaphyses.
They are often multiple and bilateral and point away
from the nearest joint.
The cartilage cap of the exostosis may be calcified.

Further management
In cases of nerve entrapment, surgical excision may be
possible. The crucial factor is to identify malignant
degeneration when it occurs; suspicious signs include pain
and growth of an exostosis after physeal closure and
thickening of the cartilaginous cap by greater than 1.5 cm.
This is best delineated on MRI.

130b

130c

130b AP and lateral radiographs of the ulna


demonstrate multiple exostoses. Note how
they point away from the joint.

130c AP of right femur demonstrates


extensive flocculent calcification in a
huge chondrosarcoma, which arose
from the exostosis at the medial
femoral condyle.

Answer 131

Musculoskeletal Imaging

Case 132

ANSWER 131
Observations (131a)

Practical tips

AP and lateral radiographs of the left knee reveal a well


defined defect in the lateral aspect of the medial femoral
condyle. A separate bony fragment lies within the defect.
Findings are those of osteochondritis dissecans.
Osteochondritis dissecans.

In the adolescent knee with symptoms but no obvious


abnormality on first inspection, check for the subtle
osteochondral defect. The LAME mnemonic
identifies the classical location.
When identified, check for radiological evidence of
joint effusion and loose bodies.
Suggest MRI for further evaluation.

Differential diagnosis

Further management

Spontaneous osteonecrosis.

Identification and treatment are important to prevent the


development of osteoarthritis. Arthroscopy and removal
or pinning of the detached fragment is the treatment of
choice when conservative management with rest and
NSAIDs fails.

Diagnosis

Discussion
Osteochondritis dissecans is synonymous with
osteochondrosis dissecans and osteochondral fracture
(131b). The cardinal feature is fragmentation of a portion
of the articular cartilage and underlying bone, which may
separate to form a loose body within the joint. It is thought
to occur due to subchondral fatigue fracture as a result of
shearing from rotatory impaction forces. Though
sometimes asymptomatic, presentation is often with pain
aggravated by movement and/or limited movement.
Patients are most commonly affected in adolescence with
males affected more than females. The typical location of
osteochondritis dissecans at the knee is the Lateral Aspect
of Medial femoral Epicondyle (usefully remembered by the
LAME mnemonic). The condition can be bilateral in up
to 30% of cases. Other commonly affected sites include the
humeral head, capitellum and talus. MRI is useful in
determining whether the osteochondral fragment is loose
as evidenced by a rim of fluid around it on T2 weighted
images or a rim of contrast around it on MRI arthrography.

CASE 132

131b

131b T1 and T2 weighted sagittal MRI images


of the knee show a traumatic osteochondral
fracture in the anterior aspect of the lateral
femoral condyle

132a

History
A middle aged female presented with
painful hands.

235

Answer 132

Musculoskeletal Imaging

ANSWER 132
Observations (132a)
This radiograph of both hands reveals soft tissue swelling
at the metacarpophalangeal joints bilaterally and to a lesser
extent at the proximal interphalangeal joints. This is not
associated with any erosions, however there is subluxation
of several joints including the metacarpophalangeal joint of
the right index finger and the right first carpometacarpal
joint. Degenerative changes are also noted at both wrists.
In summary, there is a bilateral nonerosive arthropathy with
evidence of joint subluxation. The differential diagnosis
includes collagen vascular disease. Early rheumatoid arthritis
should also be considered.

distribution is often bilateral and symmetrical but the main


factor excluding rheumatoid arthritis is of course the
absence of erosions. Several different diseases may cause
similar appearances as outlined in the differential diagnosis.
Soft tissue calcification may be seen with scleroderma and
EhlersDanlos syndrome. Jaccouds arthritis is a rare,
nonerosive arthropathy affecting the hands and feet of
patients following rheumatic valve disease. Figure 132b
shows an example of this demonstrating characteristic
subluxation at the metacarpophalangeal joints producing
ulnar deviation of the digits. There is also subluxation at
the first carpometacarpal joint. Note the absence of
erosions.

Diagnosis
Systemic lupus erythematosus (SLE).

Jaccouds arthropathy.
Hypogammaglobulinaemia.

The only finding with the nonerosive arthropathies


may be periarticular soft tissue swelling in the early
stages.
Subluxation of joints is the major finding and this
mostly affects the metacarpophalangeal and first
carpometacarpal joints bilaterally.
The appearances tend to be bilateral and symmetrical.
Scleroderma (132c) and EhlersDanlos syndrome
may be associated with soft tissue calcification on the
hand radiograph.

Discussion

Further management

Over 90% of patients with SLE develop arthralgia.


Nonerosive arthropathy is characteristically present, the
main features of which are soft tissue swelling and joint
subluxation. In particular, the subluxation affects the first
carpometacarpal joints and also the metacarpophalangeal
joints leading to ulnar deviation of the fingers. The

SLE is, as its name suggests, a systemic disease and


although patients can expect a long life death eventually
occurs from renal failure or cardiomyopathy. Treatment is
supportive with the aim of suppressing the autoimmune
element of the disease.

Differential diagnosis
Of nonerosive deforming arthropathy:
Collagen vascular disease.
SLE.
EhlersDanlos syndrome.
Scleroderma.

132b

132b Radiograph of both hands in a patient with


Jaccouds arthropathy demonstrating bilateral
metacarpophalangeal joint subluxation.

236

Practical tips

132c

132c Radiograph of both hands in a patient with


scleroderma demonstrating florid soft tissue
calcification.

Musculoskeletal Imaging

CASE 133

Case 133

133a

History
A 15-year-old male presented with left
shoulder pain.

237

Answer 133

Musculoskeletal Imaging

ANSWER 133
Observations (133a)
AP and axial radiographs of the left shoulder demonstrate
a well defined lesion in the upper humeral epiphysis. This
has a thick sclerotic border and central lucency. The lesion
has nonaggressive features including a narrow zone of
transition and the likely diagnosis considering the age of
the patient and the location of the lesion is
chondroblastoma.

important to check whether the patient has unfused


epiphyses as GCT will not normally be seen in this
group.
If doubt still exists as to the nature of an epiphyseal
lesion, MRI may help. There is typically marked bony
oedema seen surrounding a chondroblastoma but
rarely around a GCT.

Further management
Diagnosis
Chondroblastoma.

Chondroblastoma is a benign tumour that may become


locally aggressive. Treatment is by curettage and bone chip
grafting.

Differential diagnosis
Of epiphyseal lesions:
Chondroblastoma well defined sclerotic border,
calcification in 50%, may have periosteal reaction.
Giant cell tumour (GCT) closed epiphyses, abuts
articular surface, eccentric, no marginal sclerosis. No
periosteal reaction unless fracture present.
Geode will be other signs of arthropathy; older
patients.
Metastases and myeloma.
Infection usually metaphyseal rather than epiphyseal.

133b

133c

Discussion
Chondroblastoma is a rare, cartilage containing tumour
that almost always occurs in the epiphyses of long bones.
Patients affected are under 30 years of age and tend to
present with localized pain. The lesion is usually well
defined with a sclerotic border and occurs most commonly
about the knee joint. Calcification within the lesion is seen
in approximately 50%. The tumour rarely metastasizes but
may be locally aggressive (133b, 133c).
As with all focal bone lesions, an assessment of benign
vs aggressive features, the patients age and the location of
the lesion within the bone are critical in forming a
meaningful differential diagnosis. Fortunately, the
differential for epiphyseal lesions is fairly short! One
condition that deserves brief discussion is GCT as this can
sometimes look similar. The vast majority occur in long
bones, most commonly at the knee. Age at presentation is
2040 years, often with pain. Classical features of GCT are
presence in a fused skeleton, epiphyseal subarticular
location, eccentric position and absence of marginal
sclerosis. Only when all these features are present can one
confidently predict GCT from the plain radiograph. What
is impossible to say, however, is whether the lesion is
benign or malignant.
Figure 133d is a radiograph of the knee in a patient with
a GCT. Note how the lesion abuts the articular surface but
has no marginal sclerosis unlike the chondroblastoma
illustrated (133a). The distinction may not always be this
clear however, and further advice is offered in the practical
tips section.

133b, 133c Plain radiograph of a lucent tibial


epiphyseal lesion and axial T2 weighted MRI image
of the same case, which shows a well defined
hypointense lesion in the tibial epiphysis consistent
with a chondroblastoma. The high signal thoughout
the tibial plateau is in keeping with marrow oedema.
This reactive oedema is a common feature of
chondroblastoma and can lead to overestimation of
the aggressiveness of the lesion.

133d

Practical tips
With regard to epiphyseal lesions in young patients,
the two main possibilities are chondroblastoma and
GCT. However, many chondroblastomas will not have
the prominent internal calcifications and sclerotic
border seen in Figure 133a. In such cases it is

238

133d Radiograph of knee


demonstrating a lucent subarticular
lesion typical of a GCT.

Musculoskeletal Imaging

CASE 134

Cases 134, 135

134

History
A 25-year-old male presented with knee
pain and fever.

CASE 135

135

History
A 28-year-old male with bilateral hip
pain.

239

Answers 134, 135

Musculoskeletal Imaging

ANSWER 134
Observations (134)
AP and lateral radiographs of the right knee reveal a well
demarcated area of lucency in the metaphysis of the
proximal tibia. This is surrounded by sclerosis and there is
no evidence of extension through the growth plate into the
epiphysis. A small amount of smooth periosteal reaction is
seen medially. Together with the history of pyrexia, the
findings best fit with the diagnosis of Brodies abscess.

Diagnosis
Brodies abscess.

Differential diagnosis
Osteosarcoma.
Giant cell tumour (GCT) (in child).

Discussion
Brodies abscess is the term used to describe a subacute
pyogenic osteomyelitis. It is a smouldering, indolent
infection that is most commonly caused by Staphylo co ccus
aureus. Children and males are more commonly affected.
It has a predilection for the ends of tubular bones and is
characteristically located in the metaphysis. The proximal
and distal metaphyses of the tibia are the most common
sites involved. The carpal and tarsal bones are other
commonly affected sites. The characteristic appearance is
of a central area of lucency surrounded by a dense rim of
reactive sclerosis. A lucent, tortuous channel extending
towards the growth plate is pathognomonic when seen.
Periosteal reaction and new bone formation are features
and there may be adjacent soft tissue swelling. On MRI, a

double line effect may be seen on T2 weighted images.


This refers to the high signal intensity of granulation tissue
surrounded by low signal intensity of bone sclerosis. The
abscess may also be more conspicuous on MRI after IV
gadolinium injection. The disease is generally indolent and
may persist for several months.

Practical tips
Giant cell tumour in children is often located in the
metaphysis rather than its characteristic epiphyseal
location in adults. Therefore GCT in children may
look like a Brodies abscess when marginal sclerosis is
limited.
In more subtle cases where the abscess is present near
the cortex of a bone, the lesion may mimic an osteoid
osteoma.
In difficult cases where the abscess is not well defined,
the appearances may look like those of a malignant
process and when considering the metaphyseal
location of the disease the findings may look
suspicious of an osteosarcoma. This is where local
periosteal reaction is important and if smooth rather
than aggressive, will suggest benign disease.

Further management
Treatment is surgical drainage.

Further reading
Rosenberg Z, Beltran J, Bencardino J (2000). MR
imaging of the ankle and foot. Radio Graphics 20:
S153S179.

ANSWER 135
Observations (135)

Differential diagnosis

Pelvic radiograph shows bilateral irregularity of the femoral


heads with decreased joint space, sclerosis and large
subchondral cysts. This is associated with multiple calcified
foci adjacent to both hips. These foci have lucent centres
and appearances are consistent with synovial
osteochondromatosis.
In summary, there is irregularity of the femoral heads
from an underlying condition that has led to premature
osteoarthritis and secondary synovial osteochondromatosis.
The main possibilities are epiphyseal dysplasia (including
Meyers dysplasia) and previous avascular necrosis.

Of irregular epiphyses:
Avascular necrosis.
Multiple epiphyseal dysplasia.
Meyers dysplasia.
Morquios syndrome.
Congenital hypothyroidism.
Chondrodysplasia punctata.

Diagnosis
Meyers dysplasia with secondary synovial osteochondromatosis.

240

Discussion
Meyers dysplasia, also known as congenital multicentric
ossification of the femoral heads, is an epiphyseal dysplasia
that is confined to the upper femoral epiphyses. The
femoral heads become irregular, fragmented and collapsed,
leading to premature degenerative joint disease. It can be
an incidental finding in asymptomatic hips of young

Answer 135

Musculoskeletal Imaging

children. The differential diagnoses for irregular (or


stippled) epiphyses are listed and all may lead to premature
osteoarthritis (OA).
The radiological features are:
Irregularity of contour.
Sclerosis.
Collapse.
Irregular epiphyseal calcifications.
Secondary degenerative changes.
Multiple epiphyseal dysplasia normally arises in childhood
and resembles Meyers dysplasia, although unlike the latter
it is not confined to the femoral heads and may affect the
tarsal bones, knees and ankles.
Synovial osteochondromatosis is due to metaplasia of
subsynovial connective tissue to form cartilage nodules that
go on to calcify and eventually migrate into the joint to
form loose bodies. In this case, secondary synovial osteochondromatosis has developed. Compared to primary
synovial osteochondromatosis, the intra-articular loose
bodies tend to be larger and more varied in size.

CASE 136

Case 136

Practical tips
Multiple epiphyseal dysplasia will be seen in other
joints and there may be a family history. Meyers is
confined to the hips.
When irregular/stippled epiphyses identified:
Look for ancillary signs of avascular necrosis:
Signs of sickle cell anaemia, e.g. endplate
infarctions of the vertebral bodies, and
gallstones on an AXR.
Associations with steroid therapy on a pelvic xray/AXR, e.g. thumb-printing in colon, stoma
bag or sacroiliitis.
Associations with condition of
immunosuppression on a pelvic x-ray, e.g. pelvic
transplant kidney.
Look for ancillary signs of Morquios syndrome:
Posterior vertebral scalloping.
Anterior vertebral body beaks.

Further management
Treatment is supportive. Joint replacement may be needed
in the long term.

136a

History
A 32-year-old male presented with
thigh pain.

241

Answer 136

Musculoskeletal Imaging

ANSWER 136
Observations (136a)

Practical tips

At the midshaft of the femur there is an ossific mass with


mixed sclerosis and lysis lying lateral to the femoral shaft.
A radiolucent line separates the mass from the cortex except
at its attachment superiorly. Periosteal reaction is also
noted. The features are characteristic for a parosteal
osteosarcoma.

The give away appearance of this tumour is that it looks


like a cauliflower on a stalk attached to the cortex.

Diagnosis

Further reading

Parosteal osteosarcoma.

Murphey M, Robbin M, McRae G, et al. (1997). The


many faces of osteosarcoma. Radio Graphics 17:
12051231.

Differential diagnosis

Further management
Surgical resection can result in an 8090% 10 year survival
rate, the best prognosis of all types of osteosarcoma.

Osteochondroma.
Extraosseous osteosarcoma.
Juxtacortical haematoma.
Myositis ossificans.

Discussion
Parosteal osteosarcoma accounts for 4% of all
osteosarcomas. The tumour originates in the outer layer of
periosteum and is slow growing. Eventually, invasion of the
medullary canal may occur. Fifty per cent of patients are
more than 30 years of age when affected, with the peak at
38 years. This contrasts with conventional osteosarcoma,
where 75% are younger than 30 years of age. Females are
more commonly afflicted at a ratio of 3:2. The lesion most
commonly affects the posterior aspect of the distal femur. It
also affects either end of the tibia, the proximal humerus
and the fibula, but is rare in other long bones. The
metaphysis is the most common part of the long bone that
is affected and the patient may present with a palpable mass.
The typical radiological appearances are of a lobulated
cauliflower-like ossific mass extending away from the
cortex. In over a third of cases a fine radiolucent line
separates the tumour mass from the cortex and this is
known as the string sign. The attachment to the cortex is
described as the tumour stalk, and when considering
differential diagnoses this will not be seen with myositis
ossificans. Non-aggressive periosteal reaction is also a
feature. Another example is shown in a child in Figure
136b.
A medullary canal mass with aggressive periosteal
reaction (e.g. sunburst or hair on end) points to
conventional osteosarcoma. These features are demonstrated in the typical metaphyseal location in a knee
radiograph of a child with osteosarcoma (136c).
Conventional osteosarcoma affects males more commonly
than females and there is a bimodal age distribution with
elderly patients being affected due to malignant transformation in Pagets disease. Parosteal osteosarcoma has the
best prognosis of all osteosarcomas with an 8090% 10 year
survival rate (6080% for conventional osteosarcoma).

242

136b

136b Parosteal osteosarcoma of the


distal radius of a child.

Answer 136

Musculoskeletal Imaging

Case 137

136c Conventional osteosarcoma affecting the metaphysis of the femur in a child.

136c

CASE 137

137a

History
A 58-year-old female presented with
back pain.

243

Answer 137

Musculoskeletal Imaging

ANSWER 137
Observations (137a)
There is homogeneous sclerosis of the L4 vertebral body
producing the appearance of an ivory vertebra. No
significant expansion or trabeculation of the vertebral body
is seen. The remaining vertebrae and bony skeleton are
normal in appearance. The most likely diagnosis in a female
patient is sclerotic metastases from a breast carcinoma
primary. The differential diagnosis includes lymphoma.

Diagnosis

Sclerotic metastases from breast carcinoma.

Differential diagnosis
(Mnemonic MetsLP HIM):
Lymphoma.
Pagets disease.
Haemangioma.
Infection.
Mastocytosis.

Discussion
The ivory vertebra sign refers to an increase in opacity of a
vertebral body that retains its size and contours.
In adults:
Osteoblastic metastases elicit a sclerotic response that
results in patchy replacement of the vertebral body
spongiosa with dense new bone that may be confluent
(137b, 137c). Sclerotic metastases from prostate
carcinoma in men and breast carcinoma in women are
the most common primaries. Occasionally
osteosarcoma and carcinoid are responsible.
Lymphomatous deposits can also elicit a marked
osteoblastic response resulting in diffuse sclerosis.
When considering lymphoma as a cause, Hodgkins

137b

137c

137c Sagittal fat


saturated MRI
image shows
multiple
sclerotic and
lytic metastases
throughout the
visuallized
spine.

disease is more frequent than the other reticuloses.


However lymphoma is generally more likely to result
in destructive lytic lesions than osteosclerosis.
Pagets disease tends to cause expansion of the
vertebral body with coarsening of the vertical
trabeculae. In fact, the sclerosis of Pagets tends to be
mostly at the periphery with relative lucency of the
centre owing to atrophy of the spongiosa. This can
produce a picture frame or windowed double
contour appearance.
As with Pagets disease, haemangioma causes
increased vertebral trabeculations and expansion in a
sclerotic vertebral body. The younger age group will
help to differentiate it from Pagets.
Infection in the healing phase may cause sclerosis in a
vertebra, however this rarely involves a single vertebra
and endplate destruction with decrease in disc space
height will point to the correct diagnosis.
Mastocytosis is a systemic disease characterized by
mast cell proliferation in skin and the
reticuloendothelial system. Release of histamine by
mast cells in bone promotes osteoblastic activity
leading to sclerotic skeletal foci particularly in the
spine.
Vertebroplasty, the introduction of cement into a
collapsed vertebral body under imaging guidance,
results in high density within the vertebra mimicking
an ivory vertebra. This procedure is performed in
patients with vertebral collapse who have severe pain
unresponsive to medication, and is highly effective.
An example is shown (137d) of an elderly patient who
had painful osteoporotic collapse at two lumbar levels
and was successfully treated with resolution of pain.

137d

137d AP and lateral radiographs of the


thoracolumbar spine following vertebroplasty
in a patient with osteoporotic collapse.
137b Sagittal fat
saturated MRI image
shows a sclerotic
metastasis in the T5
vertebral body.

244

Answer 137

Musculoskeletal Imaging

In children:
The ivory vertebra sign is less common in children and
frequently the result of lymphoma.
Less commonly, osteoblastoma, neuroblastoma,
osteosarcoma or medulloblastoma deposits can cause
the appearance.

Practical tips
Increased vertical trabeculation and expansion suggest
Pagets or haemangioma. The latter affects a younger
age group, whereas the former is seen in the elderly
and may be polyostotic.
A paraspinal mass may be seen with lymphoma due to
adenopathy, which may also cause anterior scalloping
of the vertebral bodies.
With mastocytosis, look for involvement of several
vertebrae and background small bowel

CASE 138

Case 138

thickening/dilatation and hepatosplenomegaly, which


may be seen on the spine radiograph.
Decreased disc space height and endplate changes
point to infection.

Further management
The underlying cause can be determined from the age of
the patient, ancillary signs on the radiograph (as described
above) and the history and examination. An isotope bone
scan may be required to locate further deposits in a patient
with metastases.

Further reading
Graham TS (2005). The ivory vertebra sign. Radio lo gy
235: 614615.

138a

History
A 67-year-old male presented with
a swollen foot.

245

Answer 138

Musculoskeletal Imaging

ANSWER 138
Observations (138a)

Radiological features of a Charcot joint include:


Dense (i.e. sclerotic) bones.
Destruction and fragmentation of articular surfaces.
Degeneration.
Debris (loose bodies).
Dislocation.
Deformity.
Joint effusion.
Excessive callus formation.

Diabetic neuropathic foot.

Differential diagnosis

Practical tips

Of causes of neuropathic joint:


Shoulder and upper limb joints:
Syringomyelia.
Congenital insensitivity to pain.
Leprosy.
Syphilis.

Look for vascular calcification on the radiograph as this


will often be present in patients with diabetes mellitus and
hence point to the underlying cause.

There is marked abnormality at the midtarsal joint with


dislocation, sclerosis and debris formation. Prominent
vascular calcification is noted. The appearances are those of
a Charcot joint and the most likely cause in view of the
vascular calcification is diabetes mellitus.

Diagnosis

Further management
Treatment is supportive. Amputation may be necessary in
severe progression.

Spine:
Trauma.
Tabes dorsalis.
Hip and knee:
Tabes dorsalis.
Steroids.
Ankle and foot:
Diabetes mellitus.
Alcoholism.
Myelomeningocele.
Congenital insensitivity to pain.

Discussion
Neuropathic arthropathy is a traumatic arthritis associated
with loss of sensation and proprioception of an affected
limb. When encountered clinically it is also known as a
Charcot joint. The decreased pain sensation produces
repetitive trauma leading to eventual destruction of the
joint. There is often no history of trauma and the patient
may present with a swollen warm joint with normal
inflammatory markers. A third have pain at presentation
although there is usually a decreased response to deep pain
and proprioception at this stage. Because the patient is still
using the limb there is no juxta-articular osteoporosis, in
fact the bones are sclerotic. The exception to this rule is in
patients with superadded infection, which is not
uncommon in diabetics. Repetitive trauma leads to
destruction, dislocation and deformity with multiple loose
bodies within the joint. The likely underlying pathology
depends upon the site of the joint and the age of the
patient. When considering the ankle and foot, the most
common causes in adults are diabetes mellitus and
alcoholism, whereas in children the most common causes
are myelomeningocele and congenital insensitivity to pain.
Examples are shown of a Charcot joint secondary to
diabetes mellitus (138b) and a Charcot elbow in a patient
with syringomyelia (138c).

246

138b

138b Charcot foot


in a diabetic
showing sclerosis
and loose body
formation at the
midtarsal joint with
subluxation and
destruction of the
2nd metatarsophalangeal
joint.

Answer 138

Musculoskeletal Imaging

Case 139

138c Charcot elbow showing


destruction, sclerosis and loose
body formation.

138c

CASE 139

139a

History
A 30-year-old male patient
presented with right hip pain and
limitation of movement.

247

Answer 139

Musculoskeletal Imaging

ANSWER 139
Observations (139a)
Irregularity, sclerosis and loss of height of the right femoral
head are seen on the pelvic radiograph. There is no
significant abnormality of the acetabulum and the left hip is
unremarkable. The findings are consistent with avascular
necrosis of the right hip. Bilateral sacroiliac joint fusion is
also present, though more prominent on the right side.
This suggests a background seronegative arthropathy. It is
therefore likely that the avascular necrosis is drug induced
by treatment for the seronegative arthropathy, or perhaps
associated inflammatory bowel disease. No bowel
abnormality is seen on this plain film to confirm the latter
hypothesis.

139b

Diagnosis
Avascular necrosis (AVN) in a patient on steroids for
seronegative arthropathy.

Differential diagnosis
For causes of avascular necrosis (mnemonic DRIED
HIP):
Diabetes and other metabolic conditions, e.g.
hyperlipidaemia, gout, pancreatitis.
Radiotherapy.
Inflammatory disorders, e.g. rheumatoid, SLE,
scleroderma.
Endocrine disorders, e.g. Cushings.
Drugs, e.g. steroids, anti-inflammatory and
immunosuppressive drugs, alcohol.
Haematological disorders, e.g. sickle cell, haemophilia,
polycythaemia, Gauchers.
Infection and injury, e.g. fractures, burns and fat
embolism.
Perthes disease (idiopathic AVN in children).

139b AP pelvis in a child. A subtle lucent crescent is


present in the subchondral surface of the left
femoral head indicating early AVN.

139c

Discussion
Avascular necrosis is a consequence of interrupted blood
supply to bone with death of cellular elements. The many
causes are listed in the differential diagnosis and follow the
mnemonic DRIED HIP. The femoral head is the most
common site affected. Other common locations include the
humeral head and femoral condyles. The earliest
radiological sign is subtle relative sclerosis secondary to
resorption of surrounding bone. A radiolucent crescent
parallel to the articular surface may appear. Flattening,
fragmentation and sclerosis then ensue. Subchondral cysts
and collapse lead to early osteoarthritis of the affected joint.
An AP of the pelvis (139b) shows the subtle crescent sign
in the left hip of a child with early avascular necrosis. This
is more clearly seen on the frog lateral view of the same
patient (139c).

248

139c The lucent crescent is more clearly visible on


this frog lateral view of the same patient as in
Figure 139b.

Answer 139

Musculoskeletal Imaging

Practical tips
The list of potential causes for AVN is long and, of course,
it may just be idiopathic Perthes disease in children. The
following radiological features are worth checking for in
the search for a cause, but clinical history may be required
thereafter. For example, the child with AVN shown in
Figure 139b had leukaemia, and steroid treatment was the
cause.
Check for:
Signs of sickle cell anaemia, e.g. vertebral endplate
infarctions producing H-shaped vertebrae, altered
bony trabecular pattern, gallstones and splenic
calcification.
Associations with steroid therapy, e.g. thumb-printing
in colon, presence of a stoma or sacroiliitis.

CASE 140

Case 140

Rheumatoid-type arthropathy or changes of


scleroderma.
Vascular calcification, which may point to underlying
diabetes.
Associations with immunosuppressives, e.g. pelvic
transplant kidney.

Further management
The underlying cause should be sought and treated. Many
patients will develop debilitating secondary arthritis and
go on to require replacement of the affected joint.

140a

History
A 64-year-old patient presented
with abdominal pain.

249

Answer 140

Musculoskeletal Imaging

ANSWER 140
Observations (140a)

Discussion

There is diffuse osteosclerosis affecting the visible skeleton.


There is also massive splenomegaly making the likely
diagnosis that of myelofibrosis. Lymphoma is another
possibility and less likely, mastocytosis.

This myeloproliferative disorder results in progressive


marrow replacement by fibrosis and consequent anaemia,
extramedullary haematopoiesis and splenomegaly (often
massive). Typical age of onset is over 50 years.

Diagnosis

Practical tips

Myelofibrosis.

Osteosclerosis is often missed when the appearance is


blamed on the quality of the film, i.e. when it is
thought to be due to the radiograph being
underpenetrated. Assessment of the intervertebral
discs should be made if they are visible then the
appearance is likely to be real. This is well
demonstrated in Figure 140c: there is diffuse
osteosclerosis affecting all the bones on this CXR of a
patient with osteopetrosis. Note how the
intervertebral discs are clearly seen, reinforcing the
fact that the radiograph is not underpenetrated.
Patients with renal osteodystrophy may have a
haemodialysis line on CXR or peritoneal dialysis line
on AXR. There may be evidence of subperiosteal
bone resorption, soft tissue and vascular calcification,
and rugger jersey spine.
When sclerotic metastases are suspected on a CXR,
check the breast shadows: in female patients there
may be a mastectomy, while in males enlargement of
breast tissue may be seen due to hormone therapy for
prostatic carcinoma. Figure 140d shows such features
sclerotic prostate metastases and gynaecomastia.

Differential diagnosis
Of diffuse bony sclerosis and splenomegaly:
Lymphoma.
Mastocytosis.
Of generalized osteosclerosis in adults:
Sclerotic metastases (especially breast or prostate
carcinoma).
Lymphoma.
Myelofibrosis.
Pagets disease (140b).
Renal osteodystrophy.
Sickle cell disease.
Mastocytosis.
Osteopetrosis.
Pyknodysostosis.
Fluorosis.
Osteopetrosis and pyknodysostosis have onset in the
paediatric age group.

140b

140c

140b Axial CT image of the


pelvis shows sclerosis of the
right hemipelvis with cortical
thickening and thickened internal
trabeculations typical for Pagets
disease.
140c CXR of an adult with
osteopetrosis demonstrates
diffuse dense osteosclerosis.

250

140d

140d CXR of a male patient with


diffuse osteosclerotic metastases
from prostatic carcinoma. Note
the gynaecomastia secondary to
hormone therapy.

Answer 140

Musculoskeletal Imaging

Case 141

Massive splenomegaly on the AXR suggests


myelofibrosis.
Splenic atrophy (possibly with calcification) suggests
sickle cell disease, and there may be other signs of this
such as gallstones, avascular necrosis of the femoral
heads and endplate infarctions causing H-shaped
vertebral bodies.
Splenomegaly and small bowel thickening suggest
mastocytosis.
On the AXR of a patient with osteopetrosis a
generalized bone within bone appearance may be
seen and the vertebral bodies may have densely
sclerotic endplates producing sandwich vertebrae.
Erlenmeyer flask deformity of the femurs should also
be looked for. Figure 140e is an orthopantomogram
in an adult patient with osteopetrosis demonstrating
diffuse osteosclerosis and supernumerary teeth.
Pagets disease can also produce a bone within bone
appearance, however coarse trabeculation and cortical
thickening normally differentiate this from other
causes.
Fluorosis is associated with ligamentous insertion
calcification.

for splenic enlargement that causes recurrent painful


episodes, severe thrombocytopenia or an unacceptably high
red blood cell transfusion requirement. Median survival
from time of diagnosis is approximately 5 years. End stage
myelofibrosis is a wasting illness characterized by general
disability, liver failure and bleeding from thrombocytopenia.

Further management

140e OPG of an adult with osteopetrosis; this


demonstrates diffuse osteosclerosis and
supernumerary teeth, which is an associated finding.

There is no specific treatment for myelofibrosis.


Splenectomy is not routinely performed, but is indicated

CASE 141

140e

141a

History
A
34-year-old
male
presented with right hip
pain.

251

Answer 141

Musculoskeletal Imaging

ANSWER 141
Observations (141a)
There is a large, expansile, mixed lytic/sclerotic destructive
lesion involving the right anterior hemipelvis. There is
marked cortical destruction with a soft tissue component.
The lesion has a wide zone of transition. This is an
aggressive lesion and the most likely diagnosis in a patient
of this age is a primary malignancy such as lymphoma or
chondrosarcoma. Metastasis should also be considered.

Diagnosis

greater than 1.5 cm then malignant transformation should


be suspected. Again, flocculent chondroid calcification is
characteristic for malignant degeneration.

Practical tips
It can often be difficult to differentiate an enchondroma
from an intramedullary chondrosarcoma, however the
latter is more likely to present with pain and unlike
enchondroma will often demonstrate periosteal reaction
and cortical breakthrough.

Chondrosarcoma.

Further management
Differential diagnosis
Lymphoma.
Metastasis.

As with most primary bone tumours, a suspected


chondrosarcoma should only be biopsied in a specialist
bone tumour centre so as not to seed tumour or
contaminate the surgical field.

Discussion
Chondrosarcoma is the third most common primary bone
malignancy with multiple myeloma being the most
common and osteosarcoma following second. Chondrosarcoma can be primary, or secondary following malignant
transformation in a pre-existing skeletal lesion such as an
osteochondroma, enchondroma or a parosteal chondroma.
Chondrosarcoma most commonly presents in the 4th or
5th decade of life with a male predilection of 2:1.
Radiographs typically reveal a mixed lytic and sclerotic
appearance. The sclerotic areas represent chondroid matrix
mineralization and are seen in 6078% of lesions. The
characteristic appearance of mineralized chondroid matrix
is a ring and arc pattern of calcification that can coalesce to
form snowflake-type calcification. This characteristic
chondroid calcification usually allows confident radiological
diagnosis of a cartilaginous lesion.
Radiological signs:
Mixed lytic/sclerotic lesion.
Wide zone of transition.
Ring and arc and snowflake calcification.
Cortical destruction.
Soft tissue mass.
Chondrosarcomas are also characterized as central or
peripheral. Central chondrosarcomas make up the majority
and are intramedullary in origin, although they may erode
through the cortex into the soft tissues. Central
chondrosarcomas usually arise in the pelvis or femur and
are often expansile with the characteristic calcification
described. Endosteal scalloping is often seen and can help
differentiate low-grade chondrosarcomas from enchondromas. Clinical symptoms are nonspecific, with pain being
the most frequent symptom. Peripheral chondrosarcoma is
also termed exostotic chondrosarcoma and refers to
malignant degeneration in an exostosis (i.e. is a secondary
chondrosarcoma). An example is shown in Figure 141b
note the flocculent calcification of the chondrosarcoma,
which had arisen from the exostosis at the lateral femoral
condyle. This should always be suspected when there is
growth of an exostosis after skeletal maturity or if an
exostosis becomes painful. The cartilage cap of a suspicious
exostosis can be measured on MRI, and if the thickness is

252

Further reading
Murphey M, Walker E, Wilson A, et al. (2003). Imaging
of primary chondrosarcoma: radiologic-pathologic
correlation. Radio Graphics 23: 12451278.

141b

141b Flocculent calcification


from a chondrosarcoma that has
arisen from the exostosis at the
lateral femoral condyle.

Musculoskeletal Imaging

Case 142

CASE 142
History
A 45-year-old male presented with
severe back pain for several weeks.

142a

142b

253

Answer 142

Musculoskeletal Imaging

ANSWER 142
Observations (142a, 142b)
AP and lateral images (142a) of the thoracolumbar spine
show marked deformity at the T12/L1 level with erosive
destruction of the opposing vertebral endplates. There is
anterior slip at this level with anterior angulation (gibbus
deformity). Appearances are in keeping with a discitis.
Axial and coronal T2 weighted MR images (142b)
demonstrate high signal in the T12/L1 intervertebral disc.
Signal change extends throughout the adjacent vertebrae,
and there is endplate destruction and partial vertebral
collapse at T12. T11 vertebra has increased marrow signal
suggesting it is also involved. The axial image demonstrates
high-signal fluid collections in both psoas muscles
consistent with bilateral psoas abscesses.

Diagnosis
Tuberculous spondylitis (Potts disease) with associated
psoas abscess.

Discussion
The spine is the most common bony location to be
involved in TB. Presentation is often late since initial
symptoms are of vague back pain and stiffness. The most
common location is the upper lumbar/lower thoracic
region, particularly around L1 level. The anterior aspect of
the vertebral body is most typically affected. The disk space
then becomes involved via extension along the anterior or
posterior longitudinal ligament or directly through the
endplate.

Radiological features of discitis include:


Reduction in height of the intervertebral disc, which
is usually the first sign of a discitis.
Erosion of the vertebral endplates.
Involvement of the vertebral bodies leads to collapse
and resulting gibbus deformity.
Spread of infection into adjacent soft tissues resulting
in psoas/paraspinal abscesses.
Neurological involvement arises due to intraspinal
spread of infection and vertebral body collapse. This
occurs much more commonly when the infective
organism is TB.

Practical tips
Discitis will cause bony abnormality of the superior
and inferior endplates of the adjacent vertebral bodies,
i.e. abnormality will be centred at the level of the disc.
The vertebral disc space is maintained longer in TB
spondylitis than in pyogenic discitis.
Look at the paraspinal regions to look for spread of
infection. Infection can present as leg pain/swelling
due to tracking of collections down the iliopsoas
muscle.
Look at, and around the spinal cord for extradural
collections and spinal involvement.

Further management
Treatment is with antituberculous medication. Drainage
of associated psoas or epidural abscesses may be required.

142b

142b (left) Vertebral body collapse with abnormal marrow signal


in the vertebral bodies above and below it. Bilateral psoas
abscesses (right).

254

Musculoskeletal Imaging

CASE 143

Cases 143, 144

143a

History
A 40-year-old male patient
presented with painful fingers.

CASE 144

144a

History
A young male presented with
pain in left hand following a
fight.
(see page 258 fo r case answer)

255

Answer 143

Musculoskeletal Imaging

ANSWER 143
Observations (143a)

Practical tips

There is a bilateral symmetrical erosive arthropathy affecting


the distal interphalangeal joints. Bone density is preserved
and there is pencil in cup deformity with bony
proliferation. At the left little finger there are signs of
ankylosis of the distal interphalangeal joint. The
appearances are consistent with psoriatic arthropathy.

When presented with a hand radiograph with evidence


of erosive arthropathy, certain differentiating features
can help identify the most likely aetiology. The
radiological differentiating features of the more
common erosive arthropathies are as follows:
Psoriatic arthropathy:
Usually asymmetrical erosive.
Interphalangeal joints, particularly the distal
interphalangeal joints, are affected.
Bo ny pro liferatio n/ perio steal reactio n and
preserved bo ne density characteristic.
Pencil in cup deformity.
Ankylosis.

Diagnosis
Psoriatic arthropathy.

Differential diagnosis
Ankylosing spondylitis.
Rheumatoid arthritis (RA).

Discussion
This is a seronegative arthropathy affecting synovium and
ligamentous attachments, and affects fewer than 5% of
psoriasis patients. In approximately 15%, the arthropathy
can predate development of skin changes by several years.
The pattern of disease is variable, though the case illustrated
is classical, i.e. asymmetric erosive oligoarthritis affecting
the distal joints of hands and feet. New bone formation is
characteristic, and may result in ankylosis of interphalangeal
joints, as in this case. Resorption of the distal phalangeal
tufts may be seen with accompanying nail changes.
Spondyloarthropathy is another pattern of disease, often
with sacroiliitis and paravertebral ossifications. Other disease
patterns include symmetrical polyarthritis mimicking
rheumatoid, monoarthritis and arthritis mutilans, a grossly
destructive pattern that may progress to form opera glass
hand.

256

RA:
Bilateral symmetrical ero sive arthropathy.
Metacarpophalangeal and proximal
interphalangeal joints affected, i.e. pro ximal
small jo ints o f digits.
Marginal erosions.
Subluxation with ulnar deviation of digits.
Decreased bo ne density.
Ankylosis.
Figure 143b is a radiograph of both hands in a
patient with rheumatoid arthritis demonstrating
bilateral symmetrical erosive destruction of the
metacarpophalangeal joints. There is also
generalized decreased bone density and
arthropathy at the carpal joints and radiocarpal
joints. Figure 143c shows similar changes in the
feet of a rheumatoid patient with marginal
erosions and symmetrical subluxation at the
metatarsophalangeal joints bilaterally.

143b

143c

143b Radiograph of both hands of a patient with


rheumatoid arthritis. Bilateral symmetrical erosive
destruction is present affecting the
metacarpophalangeal joints and the wrists. Note the
telescoping of bone ends and the periarticular
osteoporosis.

143c Rheumatoid arthritis of the feet with marginal


erosions and lateral subluxation of the digits.

Answer 143

Musculoskeletal Imaging

Gouty arthropathy:
Asymmetrical erosive arthropathy.
Erosions are juxta-articular with o verhanging
edges.
Preserved bone density and joint space.
Bony proliferation/periosteal reaction.
Go uty to phi causing soft tissue masses in 50%.
Figures 143d and 143e are radiographs of gout
demonstrating an asymmetrical erosive
arthropathy with punched out erosions. These
erosions have overhanging edges, which are best
seen at the proximal interphalangeal joints of the
index fingers bilaterally and the
metacarpophalangeal joint of the right thumb.
Further large punched out juxta-articular
erosions with associated tophi are seen around
the base of the right big toe in particular. Note
the preservation of bone density.

It is impossible to distinguish the


spondyloarthropathy of psoriasis from Reiters
syndrome though it is notable that in psoriasis the
hand is most affected and in Reiters, the foot.
Spondyloarthropathy of psoriasis is different
radiographically from ankylosing spondylitis the
paravertebral ossification is asymmetrical and not due
to true syndesmophytes.

143d

143e

Further management
Given the inflammatory nature of the disease, treatment
consists of anti-inflammatory and immunosuppressive
drugs as for other inflammatory arthropathies.

143d Radiograph of both hands demonstrates


several punched out erosions with overhanging
edges and associated soft tissue swelling in a patient
with gout.
143e Radiograph of both feet in a patient with gout
with characteristic punched out erosions affecting
the big toes.

257

Answer 144

Musculoskeletal Imaging

ANSWER 144
Observations (144a)

Practical tips

Radiographs of the left hand reveal a lucent expansile lesion


of the diaphysis of the 5th metacarpal. There is thinning of
the cortex but no cortical breakthrough or periosteal
reaction. No pathological fracture has occurred and no
fracture is seen elsewhere. The lesion has a narrow zone of
transition and nonaggressive features and is consistent with
an enchondroma.

Chondroid pattern calcification, often seen in


enchondroma, is however frequently absent in lesions
of the tubular bones of the hands and feet.
A painful enchondroma without pathological fracture
should be suspected of having undergone malignant
transformation until proven otherwise.

Further management
Diagnosis
Enchondroma.

There is no specific treatment for enchondroma as it is a


benign bony lesion that is often picked up incidentally or
when a pathological fracture occurs.

Differential diagnosis
For lytic lesion in the digits (mnemonic SEGA GAME
F):
Simple bone cyst rare in the hand.
Enchondroma is by far the most likely lesion with this
appearance at this site.
Giant cell tumour of tendon sheath.
Aneurysmal bone cyst rare in the hand.
Glomus tumour painful.
Abscess/osteomyelitis.
Metastases and myeloma.
Epidermal inclusion cyst (intraosseous) distal
phalanx; history of trauma, erythema and swelling may
be present.
Fibrous dysplasia rare in the hand.

Discussion
Enchondroma is a common benign cartilaginous tumour
most commonly seen in the tubular bones (50% in hands
and feet) of patients under the age of 30 years.
The cardinal radiological features are listed below:
Central lesion within the medullary canal.
Lucent expansion of bone.
Narrow zone of transition.
Thinning of the cortex but no cortical breakthrough.
No periosteal reaction unless pathological fracture.
Stippled calcification may be present.
Cortical endosteal scalloping.
Affects small tubular bones.
Olliers disease occurs when multiple enchondromas are
present (144b). The enchondromas are most commonly
seen in the femur, tibia and hands in an asymmetrical
distribution. In the long bones, presentation is with
asymmetric limb shortening due to impairment of epiphyseal fusion. The lesions do not increase in size after
cessation of normal growth. Malignant transformation to
chondrosarcoma can occur in up to 25% of patients with
Olliers disease by the age of 40. Malignant change is more
common in central lesions and should be suspected when
there is abnormal continued growth with pain and swelling
and/or imaging findings of growth in a previously stable
lesion, bony erosion and new or increasing calcification.
Maffuccis syndrome describes multiple enchondromas
with multiple soft tissue haemangiomas, which present on
imaging as multiple phleboliths in the soft tissues. It
presents later, usually after puberty, and has a much higher
incidence of malignant transformation.

258

144b

144b Multiple lucent expansile


lesions of the tubular bones of the
hand represent enchondromas in
this patient with Olliers disease.
Note that the absence of phleboliths
excludes the main differential
diagnosis, Maffuccis syndrome.

Musculoskeletal Imaging

CASE 145

Cases 145, 146

145

History
Follow-up hand x-ray in an
older child.

CASE 146

146

History
Adult patient presented with heel pain.

259

Answers 145, 146

Musculoskeletal Imaging

ANSWER 145
Observations (145)

Practical tips

There is diffuse osteosclerosis with a bone within bone


appearance. The most likely diagnosis is osteopetrosis.

The differential diagnosis of bone within bone appearance


is large but there are certain features of the underlying
disease process that may point to the specific cause:
Osteopetrosis may have Erlenmeyer flask deformity of
the ends of long bones and sclerotic vertebral
endplates producing sandwich vertebrae.
There may be ancillary signs of sickle cell disease on
the radiograph such as avascular necrosis of femoral
and humeral heads, gallstones, H-shaped vertebrae
due to endplate infarction and splenic atrophy or
calcification.
Acromegaly is associated with rectangular-shaped
vertebrae and chondrocalcinosis.
In children, consider rickets and look for splaying and
fraying of the metaphyses.
If the patient is elderly, consider Pagets and look for
increased trabeculation and cortical thickening.

Diagnosis
Osteopetrosis causing a bone within bone appearance.

Differential diagnosis
Of bone within bone appearance (mnemonic SHARPS
POOL):
Sickle cell disease.
Hypervitaminosis D.
Acromegaly.
Rickets.
Pagets disease.
Scurvy.
Post-radiation therapy/thorotrast/heavy metal
poisoning.
Osteopetrosis and oxalosis.
Osteomyelitis.
Leukaemia.

Discussion
Bone within bone describes the radiological appearance
where one bone appears to arise within another. It can be
seen as part of normal development, especially in the spine.
Growth arrest may also cause the appearance but there are
several pathological causes as listed. The diffuse
osteosclerosis and age of the patient in this case (145) point
to the correct diagnosis.

Further management
The only treatment available for osteopetrosis is bone
marrow transplantation. Patients are more prone to
fractures than the normal population.

Further reading
Williams H, Davies A, Chapman S (2004). Bone within
a bone. Clinical Radio lo gy 59: 132144.

ANSWER 146
Observations (146)
Within the body of the calcaneus there is a well-defined
lucent lesion with a narrow zone of transition. The lesion
has a very thin sclerotic border and centrally within the
lesion is a clump of calcification. There is no periosteal
reaction or overlying soft tissue mass. The lesion has a
nonaggressive appearance and the findings are typical of
an intraosseous lipoma.

Diagnosis
Intraosseous lipoma.

Differential diagnosis

Unicameral bone cyst.


Post-traumatic cyst.
Giant cell tumour (GCT).
Desmoplastic fibroma.

Discussion
The calcaneus is the most common location for an
intraosseous lipoma. Other sites of involvement include
the proximal femur, tibia, humerus, pelvis, mandible and
vertebrae. When involving tubular bones, it is usually
located at the metaphysis. Patients of any age may develop

260

an intraosseous lipoma, with no gender predilection. There


is an association with hyperlipoproteinaemia. The typical
radiological features are an expansile, nonaggressive
radiolucent lesion with a thin, well defined sclerotic border.
There is no periosteal reaction or cortical destruction.
Sometimes the lesion may be septated. A central clump of
calcification is virtually diagnostic and represents dystrophic
calcification from fat necrosis. Patients may be
asymptomatic or have localized bone pain. The radiographic appearances are similar to a unicameral bone cyst.
Other differential diagnoses are listed for completeness but
the central calcification and characteristic appearances of
the lesion (146) make it virtually diagnostic of an
intraosseous lipoma.

Practical tips
Intraosseous lipoma (lipoma of bone) can occur at
any age.
A central clump of calcification is virtually diagnostic.

Further management
This is a benign bony lesion and no further management
is necessary.

Musculoskeletal Imaging

CASE 147

Case 147

147a

History
A 30-year-old female presented after a
fall onto outstretched hand.

261

Answer 147

Musculoskeletal Imaging

ANSWER 147
Observations (147a)
There is shortening of the 4th metacarpal. No acute bony
injury is seen. The findings are most likely to be idiopathic
or due to previous trauma. However, less common causes
such as Turners syndrome, pseudohypoparathyroidism and
pseudopseudohypoparathyroidism should also be
considered.

Diagnosis
Idiopathic shortening of the 4th metacarpal.

Pseudopseudohypoparathyroidism has identical clinical


and radiological features but patients have normal calcium.
Turners syndrome results from nondisjunction of the
sex chromosomes, with affected females having 45 XO
chromosomes. Patients have absent secondary sexual
characteristics with short stature, webbed neck and small
iliac wings. Aortic coarctation is seen in 10% of patients
and horseshoe kidney is a common finding. Madelung
deformity and shortening of the 4th metacarpal are also
seen, sometimes also with accompanying shortening of the
3rd and 5th metacarpals.

Differential diagnosis
Practical tips

For short 4th metacarpal/metatarsal:


Post-traumatic.
Postinfarction (e.g. from sickle cell disease).
Turners syndrome.
Pseudohypoparathyroidism.
Pseudopseudohypoparathyroidism.

Discussion
Shortening of the 4th metacarpal can result from previous
trauma, particularly that involving the growth plate during
childhood. Pseudohypoparathyroidism is a congenital Xlinked dominant abnormality of renal and skeletal resistance
to parathyroid hormone due to end-organ resistance and
defective hormone. Patients therefore have the same
characteristics as those with hypoparathyroidism but also
short obese stature, round face, mental retardation,
abnormal dentition and hypocalcaemia. Shortening of the
3rd metatarsal occurs along with shortening of the 4th and
5th metacarpals and is seen in up to 75% (147b).

147b

262

The short 4th metacarpal is an Aunt Minnie case but


dont be put off by the same finding in the feet!
Figure 147c shows short 3rd5th metatarsals in the
same patient as seen in Figure 147b with pseudohypoparathyroidism.
If the wrist is also visualized, check for the presence of
Madelung deformity. This would suggest Turners
syndrome is more likely.
Involvement of multiple metacarpals is not specific to
any of the differential diagnoses but involvement of
the 4th and 5th metacarpals is particularly
characteristic of pseudohypoparathyroidism.

Further management
There is no further treatment. This finding is most
commonly idiopathic or related to previous trauma and is
often incidental.

147b Hand radiograph of a patient with


pseudohypoparathyroidism demonstrates
shortening of the 3rd to 5th metacarpals.

Answer 147

Musculoskeletal Imaging

Case 148

147c Radiograph of the foot in the same patient as in


Figure 147b demonstrates shortening of the 3rd to
5th metatarsals.

147c

CASE 148

148a

History
A 14-year-old boy presented with thigh
pain, which was worse during the night.

263

Answer 148

Musculoskeletal Imaging

ANSWER 148
Observations (148a)
There is an area of dense sclerosis affecting the shaft of the
femur with associated cortical thickening. Within this is an
area of central lucency. This likely represents a nidus and
the features are therefore consistent with an osteoid
osteoma. Depending on other clinical features, infection
might be considered as a differential diagnosis.

Diagnosis
Osteoid osteoma.

Differential diagnosis
Infection.
Eosinophilic granuloma.

Discussion
Osteoid osteoma is a relatively common, benign skeletal
neoplasm composed of woven and osteoid bone, with loose
intervening fibrovascular tissue. The lesion itself rarely
exceeds 1.5 cm in maximum dimension and there are three
main types: cortical, cancellous and subperiosteal.
Cortical lesions are the most common and have
characteristic findings. Radiographs show dense reactive
sclerosis that affects the shaft of a long bone, especially the
tibia and femur. The lesion itself, however, is a radiolucent
area within this area of osteosclerosis known as the nidus.
This is more clearly seen on the CT images of the same
patient (148b).
Cancellous osteoid osteomas have a site predilection for
the femoral neck, posterior elements of the spine and the
small bones of the hands and feet. By comparison, the
sclerosis associated with a cancellous lesion is usually mild

or moderate and may be distant from the lesion. Unlike


the classical cortical osteoid osteoma, the cancellous lesion
may not necessarily be situated at the centre of the
sclerosis, making treatment more difficult.
The subperiosteal type is rarest, and typically located at
the medial aspect of the femoral neck or in the hands and
feet. It produces a soft tissue mass immediately adjacent to
the affected bone rather than osteosclerosis. Cancellous
and subperiosteal osteoid osteomas typically arise in an
intra-articular or juxta-articular location.
Osteoid osteomas most commonly occur in the femur
and tibia, where they are usually diaphyseal or
metadiaphyseal. The most commonly affected area in the
spine is the neural arch of the lumbar spine. Atypical
locations include the skull, ribs, mandible and patella.
Intra-articular lesions are most commonly found in the hip.
The affected population is young, with about half
presenting between the ages of 10 and 20 years. Almost
all patients are Caucasian with a male predominance. Pain
is the usual mode of presentation and is worse at night. It
is thought to be related to the vascularity of the lesion.
Symptoms may be present before the lesion is radiologically visible and approximately 75% of patients report relief
of pain after salicylates. CT can aid plain radiography in
identifying the nidus. When the history is atypical, the
location unusual or radiographs unexpectedly normal,
bone scintigraphy can be very helpful. The characteristic
finding is a double density sign, in which there is a small
area of intense radionuclide activity corresponding to the
nidus, superimposed on a second larger area of increased
tracer accumulation due to reactive sclerosis.

148b

148b CT reconstruction in the same patient clearly


shows the lucent lesion with surrounding sclerosis
and central nidus.

264

Answer 148

Musculoskeletal Imaging

Case 149

Practical tips

Further reading

In young patients with bony sclerosis in the tibia or


femur, look for a radiolucent nidus to confirm this
diagnosis.
When suspected, suggest a CT scan to confirm and
accurately localize the nidus.
When the history and patients age are strongly
suggestive, consider an isotope bone scan if
radiographs and CT are unrevealing.

Gangi A, Guth S, Dietemann J, Roy C (2001).


Interventional musculoskeletal procedures.
Radio Graphics 21(3): 1.
Kransdorf M, Stull M, Gilkey F, Moser R (1991).
Osteoid osteoma. Radio Graphics 11(4): 671696.

Further management
Treatment consists of complete removal of the nidus.
Conventional treatment is surgical, however many cases are
now treated radiologically with CT-guided laser
photocoagulation. Greater than 90% success rates are
reported with this technique.

CASE 149

149a

History
A young adult presented after
falling onto left arm.

265

Answer 149

Musculoskeletal Imaging

ANSWER 149
Observations (149a)
There is an extensive, expansile lucent lesion involving the
length of the left humerus, predominantly the metaphyses
and diaphysis. Remodelling of the bone is evident and the
lesion has ground glass density in some places. There is,
however, no cortical break or periosteal reaction and the
lesion has nonaggressive features. The most likely diagnosis
is fibrous dysplasia.

Diagnosis
Fibrous dysplasia.

Discussion
Fibrous dysplasia is a benign fibro-osseous developmental
anomaly of bone whereby the medullary cavity becomes
replaced by an immature matrix of collagen and woven
bone. Males and females are equally affected and 75% of
patients develop the disease before the age of 30 years. The
monostotic form, which most commonly affects the ribs,
proximal femur and craniofacial bones, accounts for 80%
of cases.
The polyostotic form (i.e. involving more than one site)
is seen in the remaining 20% and predominantly affects the
femur, tibia, pelvis and facial bones. A subtype of the
polyostotic form is McCuneAlbright syndrome, where
polyostotic unilateral fibrous dysplasia is associated with
caf-au-lait spots and precocious puberty in young girls.
Radiologically, a ground glass density lesion in the
medullary cavity is the characteristic feature. The metaphysis

149b

149b Occipitomental radiograph of skull in a


patient with leontiasis ossea demonstrating
hemicranial sclerosis and expansion.

266

is the primary site of involvement with extension into the


diaphysis and the lesion may undergo calcification.
The craniofacial form is termed leontiasis ossea and is
hemicranial, unlike Pagets disease. The frontal and
sphenoid bones are most commonly involved. Sclerotic
overgrowth of the facial bones and calvaria results in facial
deformity, exophthalmos and visual impairment and
obliteration of the sinuses. Sclerosis of the skull base may
narrow the neural foramina causing cranial nerve
symptoms. Frontal and lateral skull radiographs (149b,
149c) of a patient with leontiasis ossea show these features.
Complications of fibrous dysplasia include pathological
fracture, and malignant transformation into osteosarcoma,
fibrosarcoma or malignant fibrous histiocytoma in up to
1% of cases.
Osteofibrous dysplasia is an entity that was mistaken in
the past for fibrous dysplasia. It is almost exclusively
confined to the diaphysis of the tibia and is seen in young
children. The appearances are very similar to fibrous
dysplasia with a nonaggressive expansile, lucent/ ground
glass lesion. Enlargement of the tibia occurs with anterior
bowing. As with fibrous dysplasia, pathological fractures
commonly occur. The lesion often regresses spontaneously
with age. An example is shown (149d) that illustrates the
typical appearances and mid-diaphyseal location in a young
child.
The radiological features of fibrous dysplasia are:
Lucent/ground glass lesion in medullary cavity.
Expansile.

149c

149c Lateral radiograph of the skull in the same


patient as in Figure 149b showing sclerosis and
expansion of the calvaria.

Answer 149

Musculoskeletal Imaging

Metadiaphyseal location.
Remodelling/alteration in bone architecture.
Endosteal scalloping.
Shepherds crook deformity when affecting proximal
femur.
Limb length discrepancy.
Pathological fracture.
No cortical destruction.
Nonaggressive features.

Practical tips

149d Radiographs
of the leg in a child
demonstrate a
lucent/ground
glass lesion in the
diaphysis, which
has nonaggressive
features and is
typical of
osteofibrous
dysplasia.

Case 150

149d

The appearances of fibrous dysplasia overlap with those of


many benign bone lesions and it is reasonable to include it
on the differential diagnosis list in many cases. As a result,
it is often the first differential provided by radiology trainees
in a viva even when the lesion in question has characteristic
features of something else! Only if a lesion has the
characteristic features should fibrous dysplasia be the first
diagnosis mentioned more often, it should be further
down the list.

Further management
There is no specific treatment for fibrous dysplasia.
Pathological fractures often occur and may necessitate
surgical fixation.

CASE 150
History
A 44-year-old female presented with
wrist pain.

150a

150b

267

Answer 150

Musculoskeletal Imaging

ANSWER 150
Observations (150a, 150b)
Axial images of the wrist with T1 weighting (150a) and T2
fat saturation (150b) are shown. Inflamed tendon sheaths
are of high signal on T2 fat saturated images and low signal
on T1 weighted images. In addition, there is palmar
bowing of the flexor retinaculum. The median nerve is of
increased signal on the T2 weighted image. This
combination of features is consistent with a diagnosis of
carpal tunnel syndrome.

Diagnosis
Carpal tunnel syndrome.

On MRI these same findings can be appreciated:


Median nerve swelling proximal to the carpal tunnel is
termed pseudoneuroma.
Palmar bowing of the flexor retinaculum.
Features of tenosynovitis which appear as high
signal on T2 weighted images around the flexor
tendons.
Increased signal in the median nerve on T2 weighted
imaging.
Median nerve enhancement following contrast
injection is variable and there can be marked
enhancement due to oedema or absence of
enhancement due to ischaemia.

Discussion
Carpal tunnel syndrome is a disease that arises due to
chronic pressure on the median nerve as it passes within the
carpal tunnel, usually associated with repetitive wrist
movements. It is more commonly seen in females with a
ratio of 4:1 and is bilateral in 50% of cases. Usually
diagnosis is made by clinicians with positive findings on
clinical examination, electromyelography and nerve
conduction studies. Radiological examinations have a role
in the few cases where these investigations are inconclusive.
Ultrasound is usually the first examination undertaken
although MRI has been shown to be more specific and
sensitive. US findings are of:
Median nerve swelling in the proximal tunnel/level of
the distal radius.
Nerve flattening in the distal tunnel.
Bowing of the flexor retinaculum.

Practical tips
Assessment of nerve swelling can be made by comparing
the size of the median nerve at the level of the distal
radius and at the hamate.

Further management
Methods to decrease oedema and swelling within the
carpal tunnel such as treatment of hypothyroidism or
restriction of fluid intake can help relieve symptoms.
However the gold standard treatment for carpal tunnel
syndrome is surgical decompression.

150b

150b High signal of median nerve on T2


weighted image (upper arrow); palmar
bowing of flexor retinaculum (lower arrow).

268

Musculoskeletal Imaging

Case 151

CASE 151
History
An adult female patient with back pain.

151a

151b

269

Answer 151

Musculoskeletal Imaging

ANSWER 151
Observations (151a, 151b)
AP and lateral radiographs of the lumbar spine show
posterior vertebral scalloping of the lower lumbar vertebrae
with a narrowed anteroposterior canal diameter. The AP
film (151a) shows that the interpedicular distance is
abnormally narrowed at L5 level and there is squaring of
the iliac wings. The features are consistent with
achondroplasia.

Diagnosis
Achondroplasia.

Differential diagnosis
Of posterior vertebral scalloping:
Pressure effect tumours in spinal canal,
syringomyelia and communicating hydrocephalus.
Dural ectasia. This occurs in neurofibromatosis,
Marfans and EhlersDanlos.
Acromegaly.
Achondroplasia.
Congenital disorders such as mucopolysaccharidoses,
e.g. Morquios syndrome.

Discussion

paternal age. Those with the homozygous form are usually


stillborn or die in the neonatal period. Those with
heterozygous achondroplasia can have a long life and have
normal intelligence.
Radiological features to look for include the following:
Macrocephaly with bulging forehead (151c). A
narrow foramen magnum may be associated with
hydrocephalus.
Small J-shaped sella due to flattening of tuberculum
sellae.
Short flared ribs.
Posterior vertebral scalloping.
Anterior-inferior vertebral body beaks.
Short pedicles and caudal narrowing of interpedicular
distance (this should normally increase in the caudal
direction). These abnormalities can lead to spinal
stenosis.
Squared iliac wings.
Champagne glass pelvic inlet (151d).
Horizontal sacrum.
Shortening and bowing of long bones with a
trumpet appearance due to disproportionate
metaphyseal flaring (151e).
Brachydactyly leading to short stubby fingers (151f).

Achondroplasia is an autosomal dominant disease of


defective enchondral bone formation related to advanced

270

151c

151d

151c Macrocephaly and bulging forehead on lateral


skull radiograph.

151d AP pelvis in an achondroplastic showing the


typical champagne glass pelvis with squaring of the
iliac wings. The interpedicular distance is narrowed
caudally and the sacrum is horizontal in orientation
and difficult to visualize.

Answer 151

Musculoskeletal Imaging

Practical tips
Achondroplasia is an Aunt Minnie familiarity with
the features makes it a straightforward viva case.
When the sacrum is not seen, possibilities include
agenesis, destruction and a horizontal position as in
achondroplasia check for other supporting evidence
of the latter, such as narrowing of the interpedicular
distance in the lower lumbar spine.
Posterior vertebral scalloping on plain film check for:
Narrowing of the interpedicular distance caudally in
achondroplasia.
Cutaneous nodules of neurofibromatosis.
Rectangular vertebrae with chondrocalcinosis in
acromegaly.

Case 152

151e Shortening of
both legs is seen in this
achondroplastic with
flaring of the
metaphyses producing
a trumpet appearance.

151e

151f Radiograph of the


hand demonstrating
short, stubby tubular
bones.

151f

Further management
There is no cure for the condition but affected individuals
can normally expect a long life.

Further reading
Cheema J, Grissom L, Harcke H (2003). Radiographic
characteristics of lower-extremity bowing in children.
Radio Graphics 23: 871880.

CASE 152

152a

History
A middle aged female
presented with dysphagia.

271

Answer 152

Musculoskeletal Imaging

ANSWER 152
Observations (152a)

Practical tips

Destruction of the distal right index finger is seen with


almost complete absence of the distal phalanx. Resorption
of the distal phalangeal tufts affects several other fingers,
most notably the left index finger. There is also widespread
soft tissue calcification. The features are characteristic of
scleroderma.

Soft tissue calcification is characteristic in scleroderma


but can also be seen in other causes of acro-osteolysis
such as hyperparathyroidism and Raynauds. When
present, check for erosive arthritis, tapered fingers
(sclerodactyly) or swollen fingers (sausage fingers) in
scleroderma. Hyperparathyroidism may also show
classical subperiosteal resorption along the radial
aspect of the middle phalanges.
With regard to the three different patterns of acroosteolysis:
Hyperparathyroidism can cause any of the three
patterns but is the only commonly occurring
condition that causes resorption of the midportion
of the phalanx.
Scleroderma can cause erosion of the distal tuft (by
pressure erosion from tight skin) or at the base of
the phalanx (by virtue of erosive arthropathy).
Resorption of the proximal phalanx is, not
surprisingly, caused by the erosive arthropathies, in
particular psoriatic arthropathy look for erosive
arthropathy of the interphalangeal joints with
ankylosis and pencil in cup deformity.
Most of the other conditions listed affect the distal
tuft.
Scleroderma is a multisystem disease so look for other
changes e.g. lung fibrosis, oesophageal dilatation
(152b) see Case 17.

Diagnosis
Scleroderma.

Differential diagnosis
Of acro-osteolysis (mnemonic SHARTEN):
Scleroderma.
Hyperparathyroidism.
Arthropathy psoriasis, erosive osteoarthritis,
rheumatoid, etc.
Raynauds.
Trauma and thermal injury.
Epidermolysis bullosa.
Neuropathy, e.g. syringomyelia and diabetes.
Of fingertip calcification:
Scleroderma.
Raynauds.
Hyperparathyroidism.
Dermatomyositis.

Discussion
Acro-osteolysis is the term used to describe resorption or
destruction of the distal phalangeal tufts. There are many
causes but only the more relevant and common ones are
listed. The three main patterns of involvement are: distal
tuft, midportion and periarticular (i.e. at the base).
Periosteal reaction is not usually seen at the site of
resorption.

Further management
There is no cure for scleroderma. Treatment is supportive.

152b

152b Single axial CT image of the same patient


with scleroderma showing lung parenchymal
changes of fibrosis with subpleural reticulation.
There is also a dilated oesophagus.

272

Musculoskeletal Imaging

Cases 153, 154

CASE 153

CASE 154

History

History

A child presented having had a fall on


outstretched hand.

An 15-year-old male presented with


back pain.

153

154

273

Answers 153, 154

Musculoskeletal Imaging

ANSWER 153
Observations (153)
There is relative shortening and ulnar curve of the distal
radius with wedging of the carpus between the distal radius
and ulna. The features are consistent with Madelung
deformity. There are no exostoses to indicate diaphyseal
aclasis, and a more specific cause cannot be established
from this film.

Diagnosis
Madelung deformity.

Differential diagnosis
Of causes of Madelung deformity (mnemonic TILDti):
Turners syndrome.
Idiopathic.
LeriWeil disease (dyschondrosteosis).
Diaphyseal aclasis.
Trauma and Infection can lead to pseudo-Madelung
deformity.

Discussion
Madelung deformity comprises a short distal radius which
has abnormal dorsal and ulnar curvature resulting in ulnar
tilt of the distal radial articular surface. The distal ulnar is
also subluxed dorsally, and the triangular-shaped carpus is
wedged into the reduced carpal angle created at the wrist.
There are several conditions associated with Madelung

deformity, shown in the differential diagnosis list below.


When idiopathic, Madelung deformity is seen mostly in
adolescent or young adult women and tends to be bilateral
and asymmetrical. LeriWeil disease is an autosomal
dominant condition where sufferers have bilateral
Madelung deformity and mesomelic long-bone shortening
with limited motion of the elbow and wrist.
Turners syndrome is associated with Madelung
deformity as well as short stature, webbed neck, horseshoe
kidney, aortic coarctation and shortening of the 4th
metacarpal.
Trauma and infection may lead to premature fusion of
the distal radial growth plate and if this occurs only on the
ulnar aspect, there will be resulting ulnar tilt of the distal
radius as the radial aspect of the physis allows continued
growth on this side. This results in a reduced carpal angle
and a pseudo-Madelung deformity.

Practical tips
Look for metaphyseal exostoses pointing away from
the joints indicating diaphyseal aclasis.
Look for shortening of the 4th metacarpal, which is
associated with Turners syndrome.

Further management
There is no specific treatment for this condition.

ANSWER 154
Observations (154)

Discussion

There is uniform collapse of the L4 vertebral body causing


the appearance of a vertebra plana. The most likely cause
in a child for a solitary collapsed vertebra is eosinophilic
granuloma.

Vertebra plana is the term used to describe uniform


collapse of a vertebral body into a thin, flat disk. The most
common cause in children is eosinophilic granuloma, with
the thoracic vertebrae most frequently affected. The
vertebral body is described as having a coin on edge
appearance. The disc spaces are preserved and can in fact
appear slightly widened. Usually there is no kyphotic
deformity associated. The posterior elements of the
vertebra are normally spared. Early on, the vertebra will
appear lytic and preserved in height before gradually
uniform collapse develops. Soft tissue oedema or a
paraspinal soft tissue mass are sometimes seen. With
healing there is reconstitution of the vertebra to its original
height, although some residual compression deformity
normally persists. There are many other causes of vertebra
plana as outlined in the differential diagnosis. One of these
is avascular necrosis, which when idiopathic is termed
vertebral osteochondrosis or CalvKummelVerneuil
disease.

Diagnosis
Eosinophilic granuloma causing vertebra plana.

Differential diagnosis
For vertebra plana:
Idiopathic.
Infection.
Neoplastic (metastasis, leukaemia).
Trauma.
Steroids.
Haemangioma.
For platyspondyly:
Thanatophoric dwarfism.
Osteogenesis imperfecta.
Morquios disease.
Spondyloepiphyseal dysplasia congenital.
Kniest syndrome.

274

Answer 154

Musculoskeletal Imaging

Practical tips
The vertebral body affected has a uniformly flat coin
on edge appearance.
Intervertebral discs are typically spared with normal
disc spaces.
The posterior elements are spared.
Marked generalized osteopenia suggests osteogenesis
imperfecta.
Malignant causes may affect more than one vertebral
body.

Case 155

from birth. Platyspondyly may be generalized, affecting all


the vertebral bodies, multiple, affecting some but not all
the vertebral bodies, or localized, involving just one
vertebral body.

Further management
The underlying cause should be determined if possible.
Eosinophilic granuloma is the most common cause in
children and usually resolves spontaneously with age.

Whereas vertebra plana is the term used to describe


flattening of a previously normal vertebral body,
platyspondyly refers to flattening of the vertebral bodies

CASE 155
History
A young adult presented with
abdominal pains.

155a
275

Answer 155

Musculoskeletal Imaging

ANSWER 155
Observations (155a)

Practical tips

Three well defined densities in the right upper quadrant are


projected outside the contour of the right kidney and are
likely to represent gallstones. There is mild, diffuse
osteosclerosis and increased trabeculation of the bony
skeleton with vertebral endplate infarctions causing Hshaped lumbar vertebrae. Sclerosis and flattening of the left
femoral head suggest avascular necrosis. The spleen is noted
to be atrophic and calcified. Overall the features are
consistent with a diagnosis of sickle cell disease.

Ancillary signs of sickle cell disease on the radiograph


depend on the area of the body imaged:
Abdominal radiograph:
H-shaped vertebrae due to endplate infarctions.
Avascular necrosis of the femoral heads causing
flattening and fragmentation.
Gallstones (secondary to haemolytic anaemia).
Splenic atrophy and calcification.
Renal papillary necrosis may be caused by sickle
cell disease due to sloughing of papillae from
infarction; a sloughed papilla may be seen within
the renal calyx (on an IVU) (155c).

Diagnosis
Sickle cell disease.

Discussion
Sickle cell disease is an inherited disorder mostly seen in
Afro-Caribbeans. The sickling of the red blood cells leads to
increased blood viscosity, occlusion of small vessels and
bone infarction leading to necrosis. Chronic haemolytic
anaemia also ensues. Marrow hyperplasia leads to
coarsening of the trabeculae and may cause diffuse
osteosclerosis. A CXR in a 17-year-old female with sickle
cell disease is shown (155b). This demonstrates diffuse
osteosclerosis with H-shaped thoracic vertebrae (due to
endplate infarctions) and cardiomegaly (secondary to
chronic anaemia).

276

Chest radiograph:
H-shaped vertebrae due to endplate infarctions.
Avascular necrosis of the humeral heads.
Cardiomegaly (due to chronic anaemia).
Skull radiograph:
Hair on end appearance of skull vault due to
marrow hyperplasia.
Widening of diploic space.

Further management
Treatment is generally supportive with multiple
transfusions being necessary. There is a high incidence of
infection of bone and lung. Skeletal pain can occur not
only from osteomyelitis but also from bone marrow
infarction.

155b

155c

155b CXR in a patient with sickle cell


disease demonstrating diffuse
osteosclerosis with cardiomegaly and Hshaped thoracic vertebrae.

155c Sloughed papilla.

Musculoskeletal Imaging

Case 156

CASE 156
History
An 80-year-old woman presented with
left arm and right leg pain.

156

277

Answer 156

Musculoskeletal Imaging

ANSWER 156
Observations (156)
A full body bone scan is presented. There is widespread,
diffuse increased skeletal uptake with more focal areas of
increased uptake located in the skull, spine, ribs, pelvis and
limbs. No renal uptake of isotope is seen, however there is
some uptake in the bladder. The findings are therefore
consistent with a superscan and the most likely cause is
widespread metastases.

Diagnosis
Superscan due to widespread metastases.

Differential diagnosis
Of causes of a superscan:
Diffuse skeletal metastases.
Renal osteodystrophy.
Osteomalacia.
Hyperparathyroidism.
Hyperthyroidism.
Myelofibrosis.
Leukaemia.
Aplastic anaemia.
Widespread Pagets disease.

Practical tips

Discussion
Diffusely increased activity in the bones on an isotope bone
scan can result in an absent kidney sign where there is little
or no activity in the kidneys but good visualization of the
urinary bladder. This is termed a superscan. The key
finding is the absence of activity in the kidneys, which can
be easily missed when the diffuse increase in skeletal activity
is overlooked. In fact many such scans were reported as
normal in the past until the importance of absent renal
activity secondary to diffuse skeletal uptake was realized.

156

278

The most common cause is skeletal metastases, most of the


other causes being metabolic. In this particular case, there
are multiple foci of increased uptake on a background of
increased activity, however more difficult cases of
superscan might show homogeneous activity that would
be more easily overlooked. The diffuse increase in activity
is usually more prominent in the axial skeleton, calvaria,
mandible, sternum, costochondral junctions and long
bones. The femoral cortices become visible and there is
also increased metaphyseal activity.
Radiological signs in superscan are:
Diffuse increased skeletal activity.
Prominent uptake in axial skeleton, long bones and
sternum.
Absent/little uptake in the kidneys.
Visualization of bladder.
Increased bone to soft tissue ratio.
Increased metaphyseal and periarticular uptake.
Visible femoral cortices.

156 No renal
uptake.

Diffusely increased activity in the bones on a


superscan is particular prominent in the sternum
producing a tie sternum.
Metabolic causes tend to cause more diffuse uptake
whereas metastatic causes may produce more focal
areas of increased uptake.

Further management
When a superscan is seen, the most important factor is to
determine whether the cause is due to metabolic or
malignant disease. This may be evident from the history
and clinical examination.

Musculoskeletal Imaging

CASE 157

Case 157

157a

History
An elderly patient presented with leg
pains.

279

Answer 157

Musculoskeletal Imaging

ANSWER 157
Observations (157a)

Practical tips

There is diffuse, thick periosteal reaction along the


metadiaphyses of the left tibia and fibula. Bandages are also
noted around the leg and an elongated soft tissue opacity in
the upper medial leg may well indicate a varix. There is no
arterial calcification of note. The most likely cause of the
periosteal reaction is chronic venous insufficiency, though
it is still advisable to obtain a chest radiograph to exclude
hypertrophic pulmonary osteoarthropathy from an occult
lung tumour.
Venous insufficiency.

On identifying diffuse periosteal reaction on


radiographs of the limbs always ask for a CXR as this
can be the first manifestation of a primary lung
tumour.
Bandages and phleboliths point to chronic venous
insufficiency and arterial calcification to arterial
disease.
Though all five causes can affect the hands, remember
thyroid acropachy in particular with this distribution.
Fluorosis is something that is unlikely to be seen
outside of exam vivas but remember ligamentous
calcification is characteristic!

Differential diagnosis

Further management

Of diffuse bilateral periosteal reaction in adults:


Hypertrophic pulmonary osteoarthropathy (HPOA).
Vascular insufficiency.
Pachydermoperiostosis.
Thyroid acropachy.
Fluorosis.

This depends on the cause. HPOA is the most common


cause and the most important management aspect is to
exclude the presence of an underlying lung tumour.

Diagnosis

Discussion
Focal periosteal reaction demands careful assessment for an
underlying bone lesion including tumour, infection,
fracture, etc. However, diffuse reaction that is bilateral and
may well affect multiple bones is a different scenario
requiring this brief differential diagnosis:
HPOA is the most likely cause for such an appearance
in adults. The condition presents with painful swelling
and clubbing may occur. The periosteal reaction is of
variable thickness and typically affects the lower half of
the arm and leg. Soft tissue swelling and periarticular
osteoporosis may be appreciable radiologically. A chest
radiograph should be requested to exclude a lung
tumour. Other causes of hypertrophic
osteoarthropathy include fibrotic lung disease,
suppurative lung disease, liver cirrhosis and
inflammatory bowel disease.
Vascular insufficiency (arterial or venous) is almost
always seen in the lower limbs.
Thyroid acropachy periosteal reaction typically
affects the radial side of the thumb and index fingers.
Clubbing of the fingers may be present. An example is
shown (157b), although the thumb and index fingers
are affected, periosteal reaction is also seen along most
of the phalanges and the 5th metacarpal.
Pachydermoperiostosis is an autosomal dominant
condition, typically seen in young black males. It is
relatively pain free and self-limiting but causes skin
thickening and clubbing. Enlarged paranasal sinuses
are also seen. Periosteal reaction most commonly
affects the lower half of the arm and leg, though hands
can also be affected. Periosteal reaction looks very
similar to HPOA but begins around ligament and
tendon insertions, i.e. close to the epiphysis.
Fluorosis ligamentous calcification present.

280

157b

157b Hand radiograph of a patient with


thyroid acropachy demonstrating
periosteal reaction along the diaphyses
of most of the phalanges and the 1st
and 5th metacarpals.

Musculoskeletal Imaging

CASE 158

Case 158

158

History
A 14-year-old male presented with
arm pain.

281

Answer 158

Musculoskeletal Imaging

ANSWER 158
Observations (158)
Within the midshaft of the diaphysis of the humerus there
is a relatively well defined lytic lesion that has a narrow zone
of transition. There is endosteal scalloping and smooth
lamellated periosteal reaction. The lesion has nonaggressive
features and in this age group eosinophilic granuloma or
infection is the most likely diagnosis.

Diagnosis
Eosinophilic granuloma.

Differential diagnosis

Osteomyelitis.
Fibrous dysplasia.
Leukaemia.
Lymphoma.

Radiographic appearances are varied. Lesions typically


appear lytic but may have reactive sclerosis. Margins can
be well demarcated or poorly defined and they may even
have a permeative appearance. A lamellated periosteal
reaction is often seen. Invasion of overlying soft tissue may
result if the lesion penetrates through the cortex. In the
skull, the lesion is often round with a punched out
appearance and uneven destruction of the inner and outer
skull tables results in a double contour or bevelled edge
appearance. If there is more than one lesion, these may
coalesce producing geographical skull. In the spine, the
vertebral body is the most common site of involvement.
Lung involvement occurs in 10%. Treatment of the bone
lesions consists of conservative therapy or surgical
treatment such as curettage or excision.

Practical tips
Discussion
Langerhans cell histiocytosis is a spectrum of disease
characterized by idiopathic proliferation of histiocytes
producing focal or systemic manifestations. Eosinophilic
granuloma is the term used to describe the disease when
limited to bone and is mostly seen in patients aged between
5 and 30 years. The clinical and radiological features may
mimic infection as well as other benign and malignant
diseases. The cause and pathogenesis of the condition are
unknown. Clinical manifestations relate to the affected
bone with local pain, tenderness and masses commonly
observed. Patients may have low-grade fever or elevated
inflammatory markers, which confuses the clinical picture
with infection. The disease may occur in any bone,
although there is a predilection for the flat bones with more
than half occurring in the skull, mandible, ribs and pelvis.
Lesions are solitary in 5075% of cases. Approximately onethird of lesions occur in the long bones, most commonly
the femur followed by the humerus and tibia. Most lesions
occur in the diaphysis, and in general the growth plate acts
as a barrier to extension.

282

Always keep this condition in mind when forming


differential diagnoses for lytic bone lesions in the young
patient the appearances are varied and the clinical picture
may be confusing.

Further management
The prognosis of eosinophilic granuloma is excellent with
spontaneous resolution of bony lesions occurring in 618
months.

Further reading
Levine S, Lambiase R, Petchprapa C (2003). Cortical
lesions of the tibia: characteristic appearances at
conventional radiography. Radio Graphics 23:
157177.
Stull M, Kransdorf M, Devaney K (1992). Langerhans
cell histiocytosis of bone. Radio Graphics 12(4):
801823.

Musculoskeletal Imaging

Case 159

CASE 159
History
A 24-year-old male presents with knee instability
following a football injury 2 months previously.

159a

283

Answer 159

Musculoskeletal Imaging

ANSWER 159
Observations (159a)
Sagittal T2 weighted images of the knee demonstrate a
rupture of the anterior cruciate ligament, no intact fibres
being demonstrated. In keeping with this, there is mild
anterior tibial translocation. The posterior cruciate ligament
(PCL) is intact.
There is loss of the normal bow tie appearance of the
lateral meniscus with non-visualization of the body and
much of the posterior horn. Furthermore, abnormal low
signal tissue is present in the intercondylar region just lateral
to the PCL. These findings are indicative of a bucket
handle tear of the lateral meniscus with a fragment of
meniscus displaced medially. A joint effusion is also present.

Diagnosis
Anterior cruciate ligament (ACL) rupture with bucket
handle tear of the lateral meniscus.

Differential diagnosis
None.

mobile fragment that flips through approximately 180,


much like a handle flipping from one side of a bucket to
the other. In such circumstances, the residual part of
meniscus will be reduced in thickness and will not be seen
on the usual number of sagittal scans. If the bow tie is
seen on less than two contiguous sagittal images, a bucket
handle tear must be excluded. The mobile fragment
should then be sought elsewhere in the joint, e.g. medially
in the intercondylar region; anterior to the posterior
cruciate ligament (PCL) producing the double PCL sign
(159d); in the anterior joint, in front of the anterior horn
of the meniscus.
Other types of meniscal tear include:
Oblique and horizontal linear signal change within
the meniscus that extends to the inferior or superior
surface.
Radial (also known as parrot beak tear) a vertical
tear through the free edge will produce an absent
bow tie sign similar to a bucket handle tear.
However, the defect is only small and so the defect in
the bow tie is much smaller.

Discussion
The ACL is best evaluated on T1 weighted images and
fibres should run parallel to the roof of the intercondylar
notch (159b). ACL tears most commonly leave no normal
residual fibres visible on MRI. Sometimes, residual fibres
of the ACL are seen, but following a more horizontal
course than normal.
Sagittal images of normal menisci show a bow tie
appearance on at least two contiguous slices (159c). This
is because the normal meniscus is approximately 9 mm in
width and the sagittal images are 34 mm in thickness.
Thus, at least two sagittal slices should pass through a
contiguous section of meniscus.
Bucket handle tears constitute about 10% of meniscal
tears. The vertical tear through the inner edge produces a

159b

159b Sagittal T1 weighted image


shows an intact anterior cruciate
ligament.

284

As an aside, if the bow tie appearance of the meniscus


is seen on more than two sagittal images this can be
indicative of a discoid meniscus. This is probably a
congenital abnormality where the meniscus has a more
disc-like shape than the normal C-shape due to a wider
than normal body. They are more prone to tearing and can
be symptomatic even without being torn.

Further management
A complete ACL tear causes instability that is treated
by surgical repair with a prosthetic or tendon graft.
Meniscal tears often require arthroscopic
debridement.

159c

159c Sagittal T2 weighted image


shows a normal lateral meniscus.

159d

159d Sagittal T2 weighted image


shows the double PCL sign of a
bucket handle tear of the medial
meniscus

Musculoskeletal Imaging

CASE 160

Case 160

160a

History
A female patient presented with joint
pains.

285

Answer 160

Musculoskeletal Imaging

ANSWER 160
Observations (160a)
There is subperiosteal resorption of the radial aspect of the
middle phalanges of the index and middle fingers. No
evidence of marginal erosions or brown tumours is seen
though the trabecular pattern is coarsened. The findings
are consistent with hyperparathyroidism.

Diagnosis
Hyperparathyroidism.

Discussion
The uncontrolled production of parathyroid hormone in
hyperparathyroidism is primary, secondary or tertiary.
Primary hyperparathyroidism is caused by a parathyroid
adenoma. The raised parathyroid hormone levels lead to
resorption of bone and hypercalcaemia. Secondary
hyperparathyroidism is usually a consequence of renal
insufficiency, where chronic hypocalcaemia leads to
parathyroid hyperplasia. Some patients with secondary
hyperparathyroidism then go on to develop the tertiary
form, whereby a parathyroid adenoma arises within a
chronically overstimulated hyperplastic parathyroid gland.
The cardinal radiological feature is subperiosteal bone
resorption. The different sites affected are shown below,
the classical location being the radial aspect of the middle
phalanx of the index and middle fingers. Figure 160b
shows another pattern of erosion band-like zones in the
middle of the terminal tufts. Bone softening may result in
wedged vertebrae, kyphoscoliosis and bowing of long
bones. Parathyroid hormone-stimulated focal osteoclastic
activity can cause brown tumours, which are characteristically expansile, lytic, well demarcated lesions. These can
be the solitary sign of hyperparathyroidism in 3% of cases.
Figure 160e shows a pathological fracture through a brown
tumour.

160b

160b Left hand


radiograph
demonstrating
lucent bands of
resorption across
the mid-portions
of the distal
phalanges of the
first three digits.

Radiological features of hyperparathyroidism are as


follows:
Bone resorption:
Radial aspect middle phalanx of 2nd and 3rd
fingers (160a).
Terminal phalangeal tufts (160b).
Distal end of clavicles and superior aspect of ribs
on CXR.
Medial aspect proximal tibia (160c).
Medial femoral and humeral necks.
Lamina dura of skull and teeth producing floating
teeth.
Pepper-pot skull due to trabecular resorption
(160d).
Pseudo-widening of joints, e.g. sacroiliac joints.
Marginal erosions of the hands.
Bone softening:
Wedged vertebrae.
Kyphoscoliosis.
Bowing of long bones.
Brown tumour (160e).
Osteosclerosis (more common in secondary
hyperparathyroidism).
Rugger jersey spine.
Soft tissue calcification:
Periarticular.
Chondrocalcinosis.
Arterial.
Renal calculi.
Medullary nephrocalcinosis.

160c

160c AP radiograph of both knees


demonstrates subperiosteal resorption at
the medial aspect of both tibial metaphyses.

286

Answer 160

Musculoskeletal Imaging

Practical tips

Further management

On CXR look for subperiosteal resorption at the


superior aspects of the ribs Figure 160f demonstrates
very subtle resorption at the superior aspects of the
posterior left 7th and 8th ribs. Also look for erosion of
the lateral ends of the clavicles (160g).
Hyperparathyroidism is associated with renal failure so
there may be a haemodialysis line on the film and
prominent soft tissue calcification. Lucent bone lesions
may be due to brown tumours.
On AXR look for a peritoneal dialysis catheter. There
may be renal calculi or medullary nephrocalcinosis. A
rugger jersey spine may be seen along with widening
of the sacroiliac joints due to resorption.

Primary hyperparathyroidism is treated by surgical


resection of the parathyroid gland. After clinical and
serological diagnosis of hyperparathyroidism, US and/or
sestamibi scintigraphy of the neck is often performed to
locate the adenoma pre-operatively (160h). In suspected
ectopic parathyroid adenoma, MRI or scintigraphy may be
required to locate the tumour.

160f

160d

160f CXR in a patient with


hyperparathyroidism with subtle
erosions of the superior aspects of the
left posterior 7th and 8th ribs.

160g

160d Diffuse bone resorption of the


calvaria producing a pepper-pot skull
appearance.

160e
160g AP radiograph of right
shoulder demonstrating erosion of
the lateral end of the right clavicle.

160h

160e Radiograph of the humerus in a


patient with brown tumours; there are
lytic lesions with an associated
pathological fracture.

160h Sestamibi scan with images at 10


min and 90 min post injection show
retained tracer in a right parathyroid
adenoma.

287

This page intentionally left blank

Chapter 5

PAEDIATRIC IMAGING

The approach to paediatric imaging is essentially a


composite of the suggested approaches in the other
chapters. The approach with paediatric films is as for adult
films but the differential diagnosis list will be completely
different in many cases. Some additional points when
approaching paediatric films are:
It is very useful to know whether the child has had a
premature birth as conditions such as hyaline
membrane disease and necrotizing enterocolitis are
essentially diseases of the premature neonate. In
addition, patterns of disease can vary between infants
born prematurely and those born at term. For
example, hypoxic/ischaemic brain injury in the
premature infant leads to periventricular leukomalacia,
a pattern of injury rather different to that otherwise
seen.
Particularly in the child under 2 years, always consider
the possibility of non accidental injury (NAI) in
suspected trauma. However, while it is important to
be vigilant for NAI, a false assumption can have severe
consequences for the family and must not be declared
likely without due consideration. Although certain
injuries such as metaphyseal fractures and depressed
skull fractures may be highly suspicious for NAI, a
multidisciplinary approach should be used with
involvement of a specialist paediatric radiologist and
appropriate clinical correlation.

The distribution of categories of disease is very


different in children and adults. For example,
degenerative disease is largely a feature of adult
medicine, and while malignancy is certainly seen in
children, it is far less common than in adults.
Conversely, congenital disorders are a much bigger
consideration in children and there are many such rare
conditions that remain outside the realm of the
general radiologists experience. For example, there
are many varieties of skeletal dysplasia causing
widespread abnormalities of the skeleton. In the
examination viva for general radiological training, it is
unlikely that you will be expected to know specific
details of the less common varieties. However, it is
reasonable to expect you to recognize that a skeletal
dysplasia is likely and suggest specialist review by a
paediatric radiologist with an interest in such
disorders.
Imaging that utilizes ionizing radiation has an
associated risk that must always be balanced against
the potential benefit of diagnosis. This is even more
important in the child, where the risk of ionizing
radiation is greater. As such, while CT imaging is now
commonplace in the early work-up of adult illness,
greater reliance may be placed on modalities such as
ultrasound and MRI in the paediatric population.

289

This page intentionally left blank

Paediatric Imaging

CASE 161

Cases 161, 162

161

History
Stillborn fetus.

CASE 162

162

History
An 8-year-old child presented with
headaches.

291

Answers 161, 162

Paediatric Imaging

ANSWER 161
Observations (161)

Discussion

This radiograph is of a stillborn baby as there is no air seen


within the lungs. The cut umbilical cord is seen, confirming
that this is a newborn. The thoracic cage is narrow and
there is squaring of the iliac wings. There is bowing of the
long bones with flaring of the metaphyses producing a
characteristic telephone handle shape. There is no
evidence of fractures to suggest osteogenesis imperfecta.
The most likely diagnosis is thanatophoric dysplasia.

Thanatophoric dysplasia, which is also known as


thanatophoric dwarfism, is one of the more common
causes of lethal neonatal dysplasia. The most common is
osteogenesis imperfecta.

Diagnosis
Thanatophoric dysplasia.

Differential diagnosis
Of lethal neonatal dysplasia:
Osteogenesis imperfecta.
Thanatophoric dysplasia.
Jeunes syndrome (asphyxiating thoracic dysplasia).
The narrow elongated thorax contains a normal size
heart but leaves little room for the lungs. There is an
80% mortality rate from respiratory failure.

Practical tips
It may be possible to identify the specific cause of the lethal
neonatal dysplasia from certain features on the babygram:
Osteogenesis imperfecta will cause generalized
osteopenia with bowing of long bones and multiple
fractures. Multiple wormian bones may also be seen in
the skull.
Thanatophoric dysplasia is associated with a narrow
thoracic cage and telephone handle long bones. The
iliac wings may be small and squared.
Narrowing of the thoracic cage and small squared iliac
wings are also seen in Jeunes syndrome. The ribs may
also be small and horizontal in this condition.

Further management
No further management options.

ANSWER 162
Observations (162)

Discussion

There is significant widening of the coronal suture. The


skull vault also has a copper beaten appearance. The
combination of findings suggests raised intracranial pressure
and urgent CT or MRI should be advised.

Abnormal widening of the cranial sutures is suggested if


there is widening of >10 mm at birth, >3 mm at 2 years
and >2 mm at 3 years. The appearance of wide sutures may
just be a normal variant but there are several pathological
causes, as listed Suture widening due to elevated
intracranial pressure is unlikely after 10 years of age.

Diagnosis
Suture diastasis due to raised intracranial pressure.

Practical tips
Differential diagnosis
Of the causes of suture diastasis follows the mnemonic
TRIM:
Traumatic diastasis.
Raised intracranial pressure:
Intracerebral tumour.
Hydrocephalus.
Subdural collection.
Infiltration (of the sutures):
Leukaemia.
Lymphoma.
Neuroblastoma.
Metabolic:
Hypoparathyroidism.
Rickets.
Hypophosphatasia.

292

Suture diastasis, copper beaten skull appearance and


erosion of the dorsum sella are classical plain film
signs of raised intracranial pressure.
A tense fontanelle is a useful clinical sign to confirm
raised intracranial pressure.

Further management
In most cases, clinical suspicion of elevated intracranial
pressure will lead directly to CT or MRI evaluation, but
this is clearly the next step should such findings be
encountered on plain films with no other explanation.

Paediatric Imaging

CASE 163

Cases 163, 164

163

History
Emergency plain CT of the brain in a
4-year-old child with acute seizures and
hypoxia.

CASE 164

164

History
A newborn presented with breathing
difficulties at birth.

293

Answers 163, 164

Paediatric Imaging

ANSWER 163
Observations (163)

Severe hypoxic-ischaemic injury producing the reversal


sign.

the first 24 hr may show subtle hypoattenuation of the


basal ganglia and insular cortex with effacement of the
cisterns around the midbrain. Subsequent CT scans (at
2472 hr) show diffuse cerebral oedema with effacement
of the sulci and cisterns, and decreased grey matter white
matter differentiation.
CT features of the so-called reversal sign are
hypodensity of the cerebrum with reduced, lost, or even
reversed grey matterwhite matter differentiation. The
thalami, brainstem and cerebellum are relatively spared and
retain a more normal density, thus appearing denser than
the cerebrum. This finding indicates irreversible brain
damage and a universally poor prognosis.

Discussion

Further reading

When hypoxia, ischaemia or circulatory arrest occur in


children, diffuse hypoxic-ischaemic brain injury can ensue.
The cerebral circulation redistributes to the most vital areas,
i.e. hindbrain so that the cerebrum is first affected. CT in

Barkovich A (2005). Paediatric Neuro imaging, 4th edn.


Lippincott, Williams and Wilkins, Baltimore, pp.
240241.

The CT scan image shows diffuse cerebral oedema with


effacement of the sulci and gyri. The grey and white matter
of the cerebral hemispheres are low in attenuation resulting
in loss of the normal corticomedullary differentiation. There
is sparing of the cerebellum and brainstem which have an
increased density by comparison. The features are typical
of the acute reversal sign indicating that there has been
severe hypoxic-ischaemic brain injury.

Diagnosis

ANSWER 164
Observations (164)
An endotracheal tube is in place in a satisfactory position.
The right hemithorax is filled by a large septated lucent
mass, which is causing mediastinal shift to the contralateral
side. The hemidiaphragm is not visualized. The most likely
cause is a congenital diaphragmatic hernia, however cystic
adenomatoid malformation should be considered.

Practical tips

Congenital diaphragmatic hernia (CDH).

Bochdalek occurs at the Back of the thorax and


accounts for 90% of CDH.
Morgagni occurs More on the right (heart prevents
development on left) and accounts for the other 10%.

Discussion

Further management

Congenital diaphragmatic hernia results from failure of


closure of the pleuroperitoneal fold during gestation and
presents with respiratory distress soon after birth. It is the
most common thoracic fetal anomaly, affecting 1 in 2,500
livebirths. The left side is much more commonly affected
than the right. The solid abdominal organs can be
herniated as well as bowel. The two main types of hernia
are the posterolateral Bochdalek and the anteromedial
Morgagni hernias. Survival depends on the size of the
hernia, as this determines the degree of pulmonary
hypoplasia that will have resulted. There is an association
with CNS neural tube defects.
The main differential diagnosis on imaging is cystic
adenomatoid malformation (CAM). This congenital cystic
abnormality of the lung results from arrest of the normal

Many cases will be expected from antenatal scans and


delivery in an appropriate setting can be arranged.
Unexpected cases present a medical emergency and may
first be suspected when bowel sounds are heard in the
chest of an infant with respiratory distress. If suspected,
formal airway intubation should be undertaken as soon as
possible as bag and mask ventilation may further distend
the upper GI tract with air. Adequate oxygenation often
requires ventilation or perhaps extracorporeal membrane
oxygenation (ECMO). Ultimately, surgical correction is
required.
Cross-sectional imaging can be helpful in better
characterizing the anatomy and differentiating a hernia
from CAM.

Diagnosis

294

bronchoalveolar differentiation in utero . The imaging


features can be almost identical to those of a diaphragmatic
hernia, although the cysts may be fluid filled and thus
appear solid.

Paediatric Imaging

CASE 165

Case 165

165a

History
A 29-week premature neonate
presented with respiratory distress a few
hours after birth.

295

Answer 165

Paediatric Imaging

ANSWER 165
Observations (165a)
An NG tube lies in the stomach and there is an umbilical
artery catheter in a satisfactory position with the tip at the
level of L4 vertebra. The lungs are small in volume with a
fine reticulogranular pattern affecting all lung zones with
air bronchograms. In view of the history and radiographic
findings, the likely diagnosis is hyaline membrane disease.

Diagnosis
Hyaline membrane disease (HMD).

Discussion
Hyaline membrane disease is one of the most common
causes of respiratory distress in newborns. It is most
common in premature infants but occasionally also occurs
in term infants of diabetic mothers. It is due to lack of
surfactant, an agent responsible for decreasing the surface
tension in alveoli and produced by the type 2 alveolar cells.
Without it the alveoli are poorly distensible and remain
collapsed causing respiratory distress shortly after birth.
Classically the lungs are small in volume with either
ground glass opacity or a fine reticulogranular pattern and
air bronchograms extending out to the lung periphery.
Treatment consists of surfactant therapy and positive
pressure assisted ventilation. However, the elevated airway
pressures may lead to air dissecting through into the

165b

165b Chest radiograph of a child with HMD and left


pneumothorax secondary to ventilation therapy.
Note the left sided air bronchograms.

296

interstitium (pulmonary interstitial emphysema PIE).


This can lead to a sudden deterioration in the infants
condition due to the interstitial air causing obstruction to
the pulmonary veins. This characteristically appears as
elongated bubbles extending to the lung periphery in a
bilateral, symmetrical pattern. Pneumomediastinum and
pneumothorax are other complications of positive pressure
ventilation. Figure 165b demonstrates a left tension
pneumothorax in an infant being treated for hyaline
membrane disease. Note the shift of the mediastinum to
the right and the air bronchograms in the left lung
radiating out to the periphery. Figure 165c is an example
of an infant with pulmonary interstitial emphysema and a
left sided tension pneumothorax displacing the
mediastinum to the right. Elongated lucencies due to air
tracking along the interstitium and lymphatics are seen,
most clearly in the left lung.

Practical tips
Signs of prematurity on the film are reduced
subcutaneous fat and absence of humeral ossification
centres.
Similar lung opacities are seen with neonatal
pneumonia or neonatal retained fluid syndrome,
however unlike HMD, the lung volumes in these
patients will be normal or increased.

165c

165c Chest radiograph of a child with HMD who


developed pulmonary interstitial emphysema and a
left pneumothorax secondary to ventilation therapy.
Note the bubbly interstitial emphysema radiating to
the lung edge.

Answer 165

Paediatric Imaging

Air bronchograms are a characteristic feature of HMD


and are not seen in conditions such as meconium
aspiration syndrome.
Check the position of all lines and tubes on the
neonatal film:
Umbilical artery catheter (165a) has characteristic
U bend as it passes inferiorly from the umbilicus
in the umbilical artery then ascends in the internal
and common iliac arteries and thus into aorta. The
tip should lie either above the renal arteries at
T812 or below them at L34.
Umbilical vein catheter straight course cranially
from umbilicus passing in umbilical veins, into
ductus venosus and IVC to terminate in the right
atrium (165d).

165d
Radiograph
demonstrating
position of an
umbilical
catheter.

Case 166

165d

Further management
Treatment involves oxygenation, ventilation and
administration of surfactant.
Complications of HMD treatment should always be
sought, namely pulmonary interstitial emphysema,
pneumothorax and pneumomediastinum.

CASE 166

166a

History
Unconscious 1-month-old born
at term.

297

Answer 166

Paediatric Imaging

ANSWER 166
Observations (166a)
This axial contrast enhanced CT image shows bilateral and
symmetrical low density in the basal ganglia, involving the
putamina and thalami. There are several possible causes for
this appearance. Hypoxia and hypotension should be selfevident. If not present, levels of glucose and carbon
monoxide should be checked urgently.

Diagnosis
Low-density basal
hypoxia/ischaemia.

ganglia

secondary

to

severe

matter (thalami, putamina and brainstem nuclei), the perirolandic gyri and corticospinal tracts. This case illustrates
such a pattern.
In older children, severe hypoxic-ischaemic injury may
produce a different pattern, that of global cerebral injury.
The resulting cerebral oedema and loss of greywhite
matter differentiation with sparing of the brainstem and
cerebellum can produce the acute reversal sign on CT
described elsewhere (see Case 163).
As a comparative aside, high-density basal ganglia due
to calcification are illustrated (166b) and the differential
diagnosis listed.

Differential diagnosis
Of low-density basal ganglia:
Hypoxia
Hypotension
Hypoglycaemia
Carbon monoxide poisoning
Wilsons disease
Of basal ganglia calcification (mnemonic PIE MAPS):
Physiological the most common cause and
increasingly so with age.
Infection cytomegalovirus (CMV), toxoplasma,
congenital rubella, HIV.
Endocrine hypoparathryroidism (and
pseudo/pseudopseudohypoparathyroidism),
hyperparathyroidism, hypothyroidism.
Metabolic Leigh disease, Fahr disease, Wilsons
disease.
Anoxia at birth, cerebrovascular accident (CVA).
Poisoning carbon monoxide, lead.
Syndromes Downs, Cockaynes syndrome,
neurofibromatosis.

Practical tips
Comparative densities of grey and white matter in the
infant brain are variable on CT depending on stage of
myelination. Normality of the basal ganglia density
can be confirmed by comparison with other grey
matter structures.
MRI is the most sensitive modality for detecting
hypoxic-ischaemic injury but may be logistically
difficult in the acutely unwell infant. Ultrasound may
be practically the easiest imaging option but has
reduced sensitivity.

Further management
Urgent correction of hypoxia, hypotension and
hypoglycaemia is required followed by exclusion of other
causes.

166b

Discussion
Apart from Wilsons disease, the listed differential diagnoses
for this appearance are of acute disorders that ultimately
result in reduced cerebral oxygenation or glucose provision.
The effect of hypoxia/hypotension on the infant brain
depends on whether the infant is term or premature, and
whether the insult is mild or severe. In the premature infant
up to about 34 weeks, it is the deep white matter that is
most vulnerable and hypoxic-ischaemic injury results in
periventricular leukomalacia (PVL) with sparing of subcortical white matter and cortex. Since the corticospinal
tract fibres pass through this area, there is usually resulting
motor impairment.
In the term infant, the pattern of susceptibility is
different. A mild insult results in ischaemia of the
watershed areas of the cerebrum where blood supply is
most tenuous. These are the boundaries between the areas
supplied by the anterior, middle and posterior cerebral
arteries. Vital areas of the brain are protected by redistribution of blood flow. After a severe insult, however, vital
areas of the brain can no longer be protected, and it the
most metabolically active areas at this time of life that are
affected. Thus, ischaemic damage occurs in the deep grey

298

166b Axial CT brain scan in a child showing bilateral


basal ganglia calcification.

Paediatric Imaging

CASE 167

Cases 167, 168

167a

History
A premature neonate presented with
abdominal distension and sepsis.

CASE 168

168

History
An adolescent male presented with a
history of trauma.

299

Answer 167

Paediatric Imaging

ANSWER 167
Observations (167a)

Necrotizing enterocolitis (NEC).

Pneumoperitoneum should be carefully looked for, as


this necessitates immediate surgery. In the supine position,
free gas often collects anteriorly as a large rounded lucency
in the central abdomen producing the football sign.
When perforation occurs in utero , meconium within the
peritoneal cavity calcifies and can be seen on plain
radiography. An example is shown in a neonate (167b)
where there is peritoneal calcification (best seen over the
inferior tip of liver) with a pneumoperitoneum causing a
positive Rigler sign, i.e. visualization of both walls of the
bowel.

Discussion

Practical tips

Up to 80% of cases are related to prematurity and NEC is


the most common gastrointestinal emergency seen in
premature babies, usually occurring in the first 2 weeks of
life. Ischaemia of the bowel is thought to occur secondary
to perinatal stress, hypoxia or infection. Presentation is with
diarrhoea or bloody stools. Radiographic features consist
of distended thick walled bowel which has a bubbly
appearance due to submucosal gas, i.e. pneumatosis
intestinalis. Gas may track from the bowel into the portal
venous system and can be seen on plain radiography
(branching gas extends towards the periphery of the liver
unlike air in the biliary tree, which is central). Such a
finding in adults is an ominous and usually premorbid sign,
but in NEC, this is not the case at all. Bowel strictures are
a potential long term complication.

The earliest radiological sign on plain film is bowel


dilatation (due to ileus).
If a contrast enema is required to exclude obstruction
then water-soluble contrast should be used barium
is contraindicated.

A cord clamp is noted indicating that the neonate is no


more than a few days old. The bowel is abnormal with
gaseous distension and a bubbly appearance to the bowel
wall indicating mural gas. A large pneumoperitoneum is
present with most of the gas adjacent to the liver. The
features are consistent with necrotizing enterocolitis and
perforation. No gas is seen within the portal veins.

Diagnosis

167b

300

Further management
Mortality rates are dependent on the degree of
prematurity, with rates quoted at 5% in term infants and
12% in premature newborns. Initial treatment is supportive
with bowel rest but if serial radiographs or clinical features
show progression or perforation then surgical resection of
necrotic bowel is required.

167b Abdominal radiograph in a newborn


demonstrates visibility of both sides of the bowel
wall, i.e. a positive Rigler sign indicating
pneumoperitoneum. Flecks of calcification in the
right abdomen confirm the diagnosis of antenatal
meconium peritonitis.

Answer 168

Paediatric Imaging

Case 169

ANSWER 168
Observations (168)
No bony injury is seen on this skull radiograph. There are
multiple wormian bones, which at this age is abnormal.
There is a large differential diagnosis but the most likely
cause is idiopathic.

found in the lambdoid, posterior sagittal and temporosquamosal sutures. They are considered abnormal when
seen after 1 year or large and numerous (>10 in number
and larger than 6 x 4 mm).

Practical tips
Diagnosis
Multiple wormian bones.

Differential diagnosis
Of wormian bones, with common causes underlined
(mnemonic PORKCHOPSI):
Pyknodysostosis.
Osteogenesis imperfecta.
Rickets in healing.
Kinky hair syndrome (Menkes).
Cleidocranial dysostosis.
Hypothyroidism/hypophosphatasia.
Otopalatodigital syndrome,
Pachydermoperiostosis.
Syndrome of Downs.
Idiopathic normal in first year of life.

Because of the wide differential diagnosis, it is difficult


to identify a specific cause without the aid of a good
clinical history.
Diffuse osteopenia will be present on the skull
radiograph in cases of rickets and osteogenesis
imperfecta.
Pyknodysostosis, on the other hand, will be associated
with diffuse osteosclerosis on the film.

Further management
Management is dependent on the underlying cause.

Further reading
Cremin B, Goodman H, Spranger J, Beighton P (1982).
Wormian bones in osteogenesis imperfecta and other
disorders. Skeletal Radio lo gy 8(1): 3538.

Discussion
Wormian bones are essentially small bones occurring in the
sutures of the calvaria. These intrasutural ossicles are usually

CASE 169

169a

History
A child with a history of partial seizures.

301

Answer 169

Paediatric Imaging

ANSWER 169
Observations (169a)
Axial T2 weighted MR image at the level of the lateral
ventricles. This demonstrates a large cleft extending
through the full thickness of the left cerebral hemisphere
from the surface of the brain to the left lateral ventricle.
The cleft is lined by grey matter and is filled with CSF. The
findings are consistent with schizencephaly.

Diagnosis
Schizencephaly.

Discussion
During gestation, neurones migrate outwards from the
periventricular germinal matrix to form the normal cerebral
cortices. This migration can be interfered with by several
causes including chromosomal abnormalities, but mostly
the reason is unknown. The result is brain tissue lying in
the wrong place, typically grey matter.
Schizencephaly is a cleft extending through the full
thickness of cerebral hemisphere from the ependyma-lined
wall of ventricle to the brain surface. It is lined by pia and
grey matter that usually shows polymicrogyria, and is often
located around the Sylvian fissure. The lateral end of the
cleft may be open and readily apparent (open lip type) but
can sometimes be closely opposed and easy to miss (closed
lip type). However, even the closed lip type will show a
small irregularity in the wall of the ventricle at the site of

302

the cleft. It is not certain whether this condition is due to


an ischaemic insult leading to germinal matrix infarction
or whether it represents a focal cortical dysplasia.
There are different manifestations of the congenital
neuronal migration anomalies which result in varying
degrees of mental retardation and/or seizures. The
following further patterns are recognized:
Heterotopic grey matter when small collections of
the neurones arrest on their way to the cortex they
can be seen as discrete nodules (most commonly in a
subependymal location) or as a subcortical band.
Thus there are nodular and band heterotopias. These
are isointense to grey matter and show no
enhancement. While sometimes asymptomatic,
seizures and developmental delay can ensue, especially
with band heterotopia. An example of nodular
heterotopic grey matter is shown in an axial T2 MR
image of the brain (169b), where a small area of
heterotopic grey matter is seen in a subependymal
location adjacent to the occipital horn of the left
lateral ventricle.
Polymicrogyria sometimes neurones may migrate to
the cortex but are abnormally distributed, producing
a bumpy appearance to the cortical gyri termed
polymicrogyria. An example is shown in a coronal T1
weighted MRI (169c), where polymicrogyria affecting
the right temporal lobe produces a bumpy appearance

169b

169c

169b Axial T2 MRI of the brain shows a small area of


subependymal heterotopic grey matter adjacent to
the left occipital horn. Note how it is isointense to
the cortical grey matter.

169c Coronal T1 MRI of the brain shows a bumpy


appearance to the right temporal lobe gyri due to
polymicrogyria.

Answer 169

Paediatric Imaging

to the gyri. Note how the normal left temporal lobe


gyri are distinct and crisp.
Pachygyria in some cases the gyri may be poorly
formed (pachygyria). A spectrum exists whereby in the
most severe form the surface of the brain appears
smooth. This is termed lissencephaly (169d). Often
these patients have severe mental retardation and
limited survival.

Case 170

169d

Practical tips
Subependymal nodules in tuberous sclerosis can also cause
nodularity along the walls of the ventricles. However, the
nodules in this condition show a similar signal to white
matter rather than the grey matter seen in heterotopia.

Further management
Medical management of epilepsy.

169d Coronal T1 MRI of the brain of a child with


lissencephaly; the cortices are smooth with almost
no normally formed gyri.

CASE 170

170a

History
A newborn who was born at 41 weeks
presented with hypoxia.

303

Answer 170

Paediatric Imaging

ANSWER 170
Observations (170a)
An endotracheal tube has been placed just above the carina.
There is also an NG tube passing into the stomach. There
are bilateral diffuse patchy opacities in both lungs indicative
of widespread atelectasis and patchy consolidation.
However, the lungs appear hyperinflated and there are
small pleural effusions. In view of the history, the
appearances are likely to be due to meconium aspiration
syndrome.

The radiological features on CXR are:


Bilateral patchy atelectasis and consolidation.
No air bronchograms.
Hyperinflation with areas of air trapping.
Small pleural effusions.
Spontaneous pneumothorax and
pneumomediastinum may result from the air trapping
(170b).

Practical tips
Diagnosis
Meconium aspiration syndrome.

Discussion
Meconium aspiration syndrome is the most common cause
of respiratory distress in newborns born at full or post term.
The large size of the fetus makes delivery difficult. Perinatal
hypoxia and fetal distress lead to meconium defecation in
utero . Aspiration of the meconium into the
tracheobronchial tree then causes obstruction of small
peripheral bronchioles (though only a minority of fetuses
exposed to meconium stained amniotic fluid develop
respiratory symptoms). This results in unevenly distributed
areas of subsegmental atelectasis with alternating areas of
air trapping. The chest radiograph usually begins clearing
within a few days with no long term radiographic sequelae
in the lungs.

170b

304

Most common cause of respiratory distress in term


babies: meconium aspiration.
Most common cause of respiratory distress in preterm
babies: hyaline membrane disease.

Further management
Almost all neonates with meconium aspiration syndrome
make a full recovery of their pulmonary function. Upper
airway suction may be employed and ventilatory support
may be required in more severe cases of respiratory distress.

170b CXR in a neonate with a large


pneumomediastinum. Air outlines the
thymus producing an angels wings
appearance.

Paediatric Imaging

CASE 171

Cases 171, 172

171a

History
A newborn presented
regurgitation of feeds.

with

CASE 172

172

History
Micturating cystourethrogram
(MCUG) was taken in a male
infant with a previously confirmed
urinary tract infection.

305

Answer 171

Paediatric Imaging

ANSWER 171
Observations (171a)

Further management

A feeding tube is seen within a gas-distended blind-ending


pouch representing the oesophagus. Air is noted below the
diaphragm. There is no convincing evidence of aspiration
pneumonia. The features are consistent with oesophageal
atresia with a distal tracheoesophageal fistula.

Surgical repair is required. This can be later complicated


by anastomotic leak, oesophageal stricture or abnormal
motility resulting in dysphagia and/or aspiration
pneumonia.

Diagnosis
Congenital oesophageal atresia with tracheoesophageal
fistula (TOF).

Discussion
Embryologically the primitive foregut tube separates to
form the trachea and oesophagus. Disorders of this
separation presenting in infancy result in various
combinations of oesophageal atresia and TOF. Presentation
is with excessive drooling, regurgitation of feeds or
symptoms of aspiration depending on the type of
abnormality present.
In 90% of cases there is a component of oesophageal
atresia, and the majority of these have an associated
tracheoesophageal fistula. Such a fistula can be proximal,
distal or both (i.e. between the trachea and the proximal
oesophageal segment, the distal segment or both). This
case demonstrates the most common subtype (seen in
around 80%) where there is oesophageal atresia and a distal
TOF. The atresia results in drooling and regurgitation
while the distal TOF results in passage of air from trachea
into stomach and thus the rest of the bowel.
A minority of cases have TOF without oesophageal
atresia and are more likely to present with coughing or
choking during feeds and ultimately aspiration pneumonia.
An example is shown (171b) where water-soluble contrast
has been injected via an NG tube in the oesophagus and is
seen to pass into the trachea via the fistula.

171b

Practical tips
Oesophageal atresia CXR shows a retrotracheal
distended pouch of proximal oesophagus and a
feeding tube may be coiled within it after attempted
passage.
A gasless abdomen indicates no fistula or a proximal
fistula.
Gas in the abdomen indicates presence of a distal
fistula.
Look for consolidation suggesting associated
aspiration pneumonia.
Oesophageal atresia and TOF can be part of a
VACTERL syndrome so check the CXR for
abnormalities:
Vertebral anomalies.
Anorectal anomalies.
Cardiovascular anomalies.
Tracheo-Esophageal fistula.
Renal anomalies.
Limb anomalies.

306

171b Contrast examination via an NG tube


demonstrates a tracheoesophageal fistula.

Answer 172

Paediatric Imaging

Case 173

ANSWER 172
Observations (172)
This MCUG study shows a transverse filling defect at the
posterior urethra with distension of the proximal posterior
urethra. The findings are consistent with posterior urethral
valves.

Diagnosis
Posterior urethral valves.

posterior urethra may be seen during voiding and


vesicoureteral reflux is present in 50%. Bladder
trabeculation and a significant post void residual volume
may be noted. Prognosis depends on the duration of
obstruction prior to corrective surgery and is worse if
associated with vesicoureteral reflux. Approximately threequarters of cases will have been discovered in the first year
of life, though occasionally it can be first noted in
adulthood.

Discussion
Congenital presence of thick folds of mucous membrane
in the posterior urethra is the most common cause of
urinary tract obstruction in boys. The condition is often
suspected on prenatal US where it can lead to
oligohydramnios, hydronephrosis, prune belly and urine
ascites or urinoma due to leak. If obstruction occurs early in
utero then multicystic dysplastic kidney may result. After
birth, MCUG is the investigation of choice to outline the
transverse filling defect caused by the thick mucosal folds.
Distension and elongation of the proximal part of the

CASE 173

Practical tips
Note how diagnosis is still possible when the catheter is in
situ during the voiding phase of the MCUG (172).

Further management
Urological surgical intervention is required with initial
treatment aimed at relieving bladder outlet obstruction and
ablating the valves. Secondary treatment may be required
for vesicoureteral reflux, urinary tract infections, urinary
incontinence and renal dysfunction.

173a

History
A newborn presented with abdominal
distension and failure to pass
meconium.

307

Answer 173

Paediatric Imaging

ANSWER 173
Observations (173a)
An NG tube is in the stomach. There are multiple dilated
loops of bowel in the abdomen. No fluid levels are seen
within the bowel suggesting that the appearances may be
due to meconium ileus. However, other pathologies such as
Hirschsprungs disease and imperforate anus should be
considered.

should be performed using Gastrograffin as this has a


therapeutic effect, helping to clear the meconium by
drawing water into the gut.
With Hirschsprungs disease, the contrast enema will
demonstrate dilated bowel with a transition zone to a distal
aganglionic segment.

Practical tips
Diagnosis
Meconium ileus.

Differential diagnosis

Hirschsprungs disease.
Imperforate anus.
Ileal atresia.
Inguinal hernia.

Discussion
Meconium ileus is the term used to describe small bowel
obstruction in neonates secondary to inspissated meconium
pellets impacted in the distal ileum. The vast majority prove
to have cystic fibrosis and this is the earliest manifestation of
the disease. The diagnosis is confirmed by performing a
contrast enema, which demonstrates multiple round filling
defects (the inspissated meconium) in the distal ileum and
proximal colon (173b). The colon may be very narrow on
the contrast study if it has been unused due to antenatal
obstruction, whereby it is termed a microcolon. The enema

On an AXR of a baby it is almost impossible to tell if


dilated loops of bowel are large or small bowel. The
presence or absence of vomiting/passage of
meconium are more helpful to know with regard to
assessing if there is high or low bowel obstruction.
The hernial orifices should be checked for air
suggesting an inguinal hernia.
A soap bubble appearance may be seen on AXR in
meconium ileus due to the mixture of gas with
meconium.
Fluid levels are not usually present in meconium ileus
because the bowel contents are very viscous.
Look at the sacrum on the AXR, as imperforate anus
is associated with sacral agenesis.

Further management
Water-soluble contrast enema can be useful for both
diagnosis and treatment.
All patients with meconium ileus should have a sweat
test to exclude underlying cystic fibrosis.

173b

173b Gastrograffin enema in a newborn shows


multiple filling defects in the ascending colon and
terminal ileum, which represent inspissated
meconium. Note the dilated small bowel loops.

308

Paediatric Imaging

CASE 174

Cases 174, 175

174a

History
A 4-year-old child presented with
fever and abdominal pain.

CASE 175

175

History
A 4-month-old child presented
with persistent irritability.

309

Answer 174

Paediatric Imaging

ANSWER 174
Observations (174a)
Axial CT of the abdomen with oral contrast and IV contrast
in portal phase. There is a large slightly heterogeneous mass
arising from the right kidney. The mass does not enhance as
much as the renal parenchyma. There is local mass effect
with displacement, but no invasion of the right lobe of liver
or encasement of vessels. A small mass of similar density is
seen near the hilum of the left kidney. The appearances
suggest bilateral Wilms tumours.

Diagnosis

Almost all neuroblastomas contain calcification


whereas only up to 10% of Wilms tumours calcify.
Both tumours can cross the midline and look
similar on imaging, appearing inseparable from the
kidney; however, neuroblastoma tends to encase
surrounding vessels such as the aorta while Wilms
tumours tend to displace surrounding tissues and
structures. An example of a neuroblastoma is
shown (174b) encasing the aorta and coeliac axis
vessels. Neuroblastoma may also extend into the
spinal canal through the neural foramina.

Bilateral Wilms tumours.

Differential diagnosis
Neuroblastoma.

Discussion
Wilms tumour (nephroblastoma) is the most common
abdominal malignancy in young children, most commonly
presenting at age 34 years. The most frequent presentation
is with abdominal mass, though hypertension, pain, fever
and haematuria also occur. The tumour usually grows to a
large size, often measuring over 10 cm. Radiological
features include:
Exophytic mass displacing rather than encasing
adjacent structures.
Less contrast enhancement than normal renal
parenchyma.
Cystic/necrotic areas give it a heterogeneous
appearance.
Invasion of the renal vein and inferior vena cava may
occur in up to 10%.
Tumours are bilateral in 10% and this indicates background
nephroblastomatosis, a state of persistent nephrogenic
blastema that is a precursor to Wilms. Wilms tumour is
associated with the BeckwithWiedemann syndrome
(macroglossia, visceromegaly and omphalocele). Other
associations include aniridia and hemihypertrophy.
The main differential diagnosis is neuroblastoma, a
common malignant tumour of the neural crest that presents
in a similar way to Wilms tumour with a painful abdominal
mass and fever. Typical age of presentation is slightly earlier
however (under 2 years). Hormone secretion from the
tumour (such as catecholamines) may cause other signs,
including hypertension and opsoclonus (chaotic jerky eye
movements). It can arise anywhere in the sympathetic
neural chain including the adrenal gland and the abdominal
sympathetic chain.

Look for evidence of metastatic spread; 70% of


neuroblastomas have malignant spread at presentation
compared to just 10% of Wilms tumours. Also,
Wilms tumours spread to lung, whereas
neuroblastoma spreads to bone.

Further management
Treatment is with radical nephrectomy and chemotherapy.
Preoperative chemotherapy is advocated in cases of bilateral
Wilms tumours and when there is IVC extension of
tumour (occurs in ~5%).

174b

Practical tips
Always check for bilateral tumours when Wilms is
suspected.
Differentiating between neuroblastoma and Wilms
tumour can be difficult radiologically but there are
some specific features that can help:

310

174b CT of a neuroblastoma. The huge mass


displaces the spleen and left kidney and shows
typical encasement of vessels, in this case the aorta
and coeliac axis.

Answer 175

Paediatric Imaging

Case 176

ANSWER 175
Observations (175)
There is bilateral symmetrical thick, smooth periosteal
reaction affecting the diaphyses of the long bones. There
is no fraying or splaying of the metaphyses to suggest
rickets. The most likely diagnosis at this age is Caffeys
disease, however other possibilities such as leukaemia need
to be considered.

6 months. Irritability and fever are the presenting


symptoms and are associated with soft tissue swelling over
the bones. Bilateral symmetrical thick periosteal reaction
is the cardinal radiological feature and most commonly
affects the mandible, clavicle and the long bones. It usually
involves the diaphysis of the bone. In the majority of cases,
there is spontaneous complete recovery by the age of 3
years.

Diagnosis
Caffeys disease.

Differential diagnosis
Of bilateral diffuse periosteal reaction in childhood:
Normal variant before the age of 4 months.
Caffeys disease.
Leukaemia.
Scurvy.
Rickets.
Hypervitaminosis A.
Non accidental injury (NAI).

Practical tips
If there is diffuse periosteal reaction with fractures of
differing ages, NAI must be considered.
With rickets, splaying and fraying of the metaphyses
will be seen.

Further management
When NAI is considered then a careful analysis of previous
radiographs, the clinical presentation and consultation with
a specialist paediatric radiologist must be carried out
because of the repercussions of a misdiagnosis.

Discussion
Infantile cortical hyperostosis (Caffeys disease) is a
proliferative bone disease seen in patients under the age of

CASE 176

176a

History
None available.

311

Answer 176

Paediatric Imaging

ANSWER 176
Observations (176a)
There is widespread bilateral decreased bone density with
healing insufficiency fractures of the radius and ulna
bilaterally. These are associated with thick smooth periosteal
reaction and there is fraying and splaying of the metaphyses.
The features are characteristic of rickets.

Diagnosis
Rickets.

Differential diagnosis
Hypophosphatasia.

Discussion
Rickets is most commonly due to insufficient biologically
active vitamin D, though impaired calcium absorption or
excessive phosphate excretion can occasionally be to blame.
In the western world, pure dietary deficiency of vitamin D
is rarely the sole cause; more often it is due to
malabsorption or impaired vitamin D metabolism in the
liver or kidney.
Rickets is essentially osteomalacia during enchondral
bone growth. Portions of the skeleton that have already
matured show features of osteomalacia, but loss of normal
maturation and mineralization of cartilage cells at the

176b

growth plate lead to the additional distinctive radiological


features of rickets. Osteomalacia is discussed elsewhere in
the book but the radiological features are due to excessive
unmineralized osteoid producing Loosers zones,
osteopenia, cortical tunnelling, indistinct trabeculae and
finally bowing and fractures due to softened bones.
In addition, the following features are seen in rickets:
Widened growth plate loss of normal chondrocyte
maturation and mineralization result in cell build up
here.
Metaphyses are irregular/frayed, splayed and cupped
impaired mineralization causes the frayed irregular
appearance while build up of chondrocytes at the
physis indents the metaphysis producing cupping and
splaying.
Epiphysis osteopenic and irregular.
Periarticular soft tissue swelling.
Apparent periosteal reaction due to subperiosteal
unmineralized osteoid.
Delayed maturation and growth.
Figure 176b demonstrates rickets of the lower limbs note
the typical changes around the metaphyses in the tibia and
also bowing of the fibula. Figure 176c is a CXR of a child
with rickets showing splaying of the anterior ends of the

176c

176c Chest radiograph of a child with rickets and


splaying of the ribs.
176b AP radiograph of both legs shows typical
features of rickets with fraying of the metaphyses.

312

Answer 176

Paediatric Imaging

ribs and the metaphysis of the right humerus. The


appearance of the anterior rib ends is due to changes at the
costochondral junction growth plates and is termed the
rachitic rosary.

Case 177

Further management
Rickets is now usually identified early and treated with
vitamin D supplements. Significant pelvis deformity and
gait disturbances are now rarely seen in the developed
world.

Practical tips
The earliest sign of rickets on the plain film is a
widening of the growth plate.
Loosers zones are rare in rickets compared to
osteomalacia in the fused skeleton.

CASE 177

177a

History
None available.

313

Answer 177

Paediatric Imaging

ANSWER 177
Observations (177a)

Discussion

There is partial fusion of an extra digit with the metacarpal


of the little finger. This essentially represents
polysyndactyly, the possible causes of which include
idiopathic, Ellisvan Creveld syndrome and Carpenter
syndrome.

There are several causes of syndactyly (fusion of digits) and


polydactyly (supernumerary digits), which are both
congenital abnormalities. Ellisvan Creveld syndrome is
also associated with carpal fusion, as is Aperts syndrome.
This is characterized by features in the skull: notably
craniosynostosis of the coronal sutures, hypoplastic mid
face and enlargement of the sella. All of these features are
demonstrated in the lateral skull radiograph in a child with
Aperts syndrome (177b).

Diagnosis
Ellisvan Creveld syndrome.

Differential diagnosis
Of causes of polydactyly:
Idiopathic.
Ellisvan Creveld syndrome.
Carpenter syndrome.
Polysyndactyly syndrome.

Practical tips

Of causes of syndactyly:
Idiopathic.
Aperts syndrome.
Carpenter syndrome.
Downs syndrome.
Polands syndrome.
Neurofibromatosis.

Further management

Some exam cases will have an obvious abnormality as part


of a syndrome that you dont know stating that you
would seek help from a textbook or specialist colleague is
a reasonable answer. You cant know everything!
Poly/syndactyly will be part of a syndrome with multiple
abnormalities.

177b

177b Lateral skull radiograph of a child with Aperts


syndrome demonstrating craniosynostosis of the
coronal sutures, hypoplasia of the midface and
enlargement of the sella.

314

Paediatric Imaging

CASE 178

Case 178

178a

History
None available.

178b

315

Answer 178

Paediatric Imaging

ANSWER 178
Observations (178a, 178b)

Pyles disease.

deformity is often a feature. The expanded metaphyses


tend to be lucent and have the appearance of an
Erlenmeyer flask (named after the wide necked laboratory
flask bearing the name of this German chemist).
Craniometaphyseal dysplasia essentially has the same
features but in addition there are cranial nerve palsies due
to sclerosis of the skull base.
Gauchers disease is a hereditary disorder of lipid storage
common among Ashkenazi Jews. It is characterized by
hepatosplenomegaly with flask-shaped long bones and
generalized osteopenia with strikingly thin cortices.
Avascular necrosis is also a feature.

Differential diagnosis

Practical tips

In this case:
Craniometaphyseal dysplasia.
NiemannPick disease.
Gauchers disease.

Erlenmeyer flask deformity, the metaphyseal expansion of


long bones, is also discussed in Chapter 5. Additional
differentiating features can be found on the radiograph as
to the specific underlying cause of Erlenmeyer flask
deformity:
Diffuse sclerosis and sclerotic vertebral endplates
producing sandwich vertebrae indicate osteopetrosis.
With Pyles disease, there will be relative sclerosis at
the diaphysis and lucency of the metaphysis.
Gauchers disease will also be associated with lucency
and osteopenia but there may be signs of avascular
necrosis of the femoral or humeral heads (loss of
height and fragmentation) and on an AXR massive
hepatosplenomegaly may be seen.
Thalassaemia is associated with coarsened
trabeculation producing a cobweb appearance.
Lead poisoning causes dense metaphyseal bands as
well as Erlenmeyer flask deformity.

There is bowing deformity of the tibia, fibula and humerus.


The metaphyses of the bones are widened producing an
Erlenmeyer flask deformity. The metaphyses are also
relatively lucent when compared with the diaphysis, which
is sclerotic. There are no specific features to indicate lead
poisoning, osteopetrosis or thalassaemia, so the differential
diagnosis lies between Pyles disease and lipidoses such as
Gauchers or NiemannPick disease.

Diagnosis

Of Erlenmeyer flask deformity (mnemonic Lead


GNOME):
Lead.
Gauchers.
NiemannPick disease looks like Gauchers but
without avascular necrosis.
Osteopetrosis.
Metaphyseal dysplasia (Pyles) and craniometaphyseal
dysplasia (same as Pyles disease but there is a history
of cranial nerve palsies).
Ematological!! thalassaemia.

Discussion
Pyles disease is also known as metaphyseal dysplasia. It is
a rare autosomal recessive disorder characterized by flaring
of the ends of long bones with relative constriction and
sclerosis of the central portion of the shafts. Affected
patients are usually asymptomatic and genu valgus

Further management
This condition is usually asymptomatic and requires no
direct management.

178a

178a Metaphyseal widening with


increased lucency.

316

Paediatric Imaging

CASE 179

Case 179

179a

History
None available.

179b

317

Answer 179

Paediatric Imaging

ANSWER 179
Observations (179a, 179b)

Discussion

The lateral skull radiograph (179b) demonstrates thinning


of the calvaria with multiple wormian bones. Bowing
deformities are seen to affect the limbs (179a) and there
are several fractures of differing ages, mostly seen at the
metaphyses of the long bones. There is generalized
osteopenia of the skeleton with marked thinning of the
cortices. The features are consistent with osteogenesis
imperfecta.

Osteogenesis imperfecta is a connective tissue disorder


characterized by fragile bones and blue sclerae. Type 1 is
compatible with life. Type 2 is the lethal form associated
with perinatal death.
The principal radiological features include:
Diffuse osteopenia with thinning of cortices.
Multiple fractures of differing ages with
pseudarthroses and bowing deformity.
Fractures are associated with exuberant callus
formation.
Biconcave vertebral bodies.
Multiple wormian bones in the skull.
Poor dentition.

Diagnosis
Osteogenesis imperfecta.

Differential diagnosis
Of wormian bones with common causes underlined
(mnemonic PORKCHOPSI):
Pyknodysostosis.
Osteogenesis imperfecta.
Rickets in healing.
Kinky hair syndrome (Menkes).
Cleidocranial dysostosis.
Hypothyroidism/hypophosphatasia.
Otopalatodigital syndrome.
Pachydermoperiostosis.
Syndrome of Down.
Idiopathic normal in first year of life.

318

Practical tips
Multiple fractures in children should raise suspicion of non
accidental injury (NAI) and sometimes differentiating this
from osteogenesis imperfecta can be difficult. Predominantly osteogenesis fractures are diaphyseal compared with
metaphyseal NAI fractures but this is not always the case.

Further management
Early medical intervention to increase bone mineral density
and surgical intervention to treat/correct scoliosis and treat
fractures mean that a multidisciplinary approach to the
ongoing treatment is required.

Paediatric Imaging

CASE 180

Cases 180, 181

180a

History
A child presented
vomiting and ataxia.

with

CASE 181

181a

History
A 2-month-old baby presented with
cyanosis.
(see page 322 fo r case answer)

319

Answer 180

Paediatric Imaging

ANSWER 180
Observations (180a)
A series of axial MRI images of skull show a well defined
hyperdense mass in the cerebellar vermis causing
compression and anterior displacement of the 4th ventricle
with obstructive hydrocephalus. A small amount of oedema
surrounds the mass. The features are typical of a
medulloblastoma.

Diagnosis
Medulloblastoma.

Differential diagnosis
Of posterior fossa tumour in children:
Medulloblastoma:
Predominantly midline posterior to 4th ventricle.
Hyperdense on CT with oedema.
Avid enhancement.
20% calcify, 50% necrose.
Juvenile pilocytic astrocytoma:
Typically paracentral, posterior to 4th ventricle.
Majority are cystic with an enhancing mural
nodule, the remainder are solid.
20% calcify; oedema is rare.
Ependymoma:
Arises within 4th ventricle.
Heterogeneous appearance and enhancement.
50% calcify.

180b

320

Hydrocephalus is often communicating type due


to protein exudate obstructing CSF resorption.
Spreads through the exit foramen of fourth
ventricle and wraps around brainstem (plastic
growth). Sagittal and axial T2 weighted MR
images (180b) demonstrate a high signal lesion
arising within the 4th ventricle, wrapping around
the brainstem and spreading via the foramina of
Luschka and Magendie. Hydrocephalus is present
and a syrinx of the upper cervical cord has
developed.
Brainstem glioma:
Within pons, possibly causing pontine expansion
or 4th ventricle displacement posteriorly.
Iso- or hypodense to brain so may be easily missed.
Sagittal T1 weighted MRI with contrast (180c)
demonstrates a large pontine glioma. Note how
the lesion is nonenhancing and almost the same
signal as surrounding brain. Smaller such lesions
can easily be missed due to such imaging
characteristics.
Enhancement often absent or minimal.
Hydrocephalus uncommon (because present with
focal neurology before this occurs).

Discussion
Medulloblastoma is the second most common paediatric
brain tumour and the most common in the posterior fossa.
It is a type of primitive neuroectodermal tumour (PNET)

180b Sagittal and axial T2 MRI of


the brain demonstrating an
ependymoma in the 4th ventricle,
which extends into the foramina of
Luschka and Magendie and causes
obstructive hydrocephalus. A syrinx
of the upper cervical cord has
developed.

Answer 180

Paediatric Imaging

arising from the roof of the 4th ventricle. The majority of


patients affected are under the age of 15 years, with 80% of
lesions arising from the cerebellar vermis, and the rest lying
more laterally in the cerebellum. This lateral location is
more common in older children. They are typically
hyperdense on CT due to dense cellularity, and show avid,
homogeneous enhancement. There is usually surrounding
oedema, 20% show calcification and up to 50% show
necrosis/cystic change. They are highly malignant and
spread occurs via the CSF in up to a third. Medulloblastoma is rarely associated with Gorlins syndrome an
autosomal dominant disorder characterized by multiple
cutaneous basal cell carcinomas during childhood with
mandibular keratocysts and extensive intracranial
calcification of the falx and tentorium.
At least 50% of primary brain tumours in children occur
in the posterior fossa. Brainstem glioma tends to present
with focal neurology due to involvement of the long tracts
and cranial nerve nuclei, while the other three differential
diagnoses listed present by way of mass effect and
obstructive hydrocephalus with headache, vomiting and
ataxia. The salient imaging features of each are listed. It is
important when staging paediatric posterior fossa tumours
to remember the potential for CSF spread with medulloblastoma in particular, but sometimes with ependymoma
too. Post gadolinium scans should therefore include the
whole spine as well as brain to pick up such deposits (drop

180c

180c Sagittal T1 weighted MRI post IV contrast that


shows a large pontine glioma.

metastases). A sagittal T1 MRI post-contrast (180d)


demonstrates an enhancing medulloblastoma in the
posterior fossa causing obstructive hydrocephalus. CSF
spread of tumour has occurred with a metastasis in the
prepontine cistern.

Practical tips
Make sure the post contrast scan looking for drop
metastases is done preoperatively as postoperative
haemorrhage and granulation tissue can cause
confusion.
Haemangioblastoma is primarily a tumour of adults
but can be seen in adolescents in the posterior fossa
when part of von HippelLindau syndrome. It is
typically a cystic mass with enhancing mural nodule,
so has similarities with pilocytic astrocytoma.

Further management
MRI of the spine with intravenous contrast enhancement
should be undertaken to look for drop metastases.
Neurosurgical assessment is then clearly appropriate.

Further reading
Koeller K, Rushing E (2003). From the archives of the
AFIP: medulloblastoma: a comprehensive review with
radiologic-pathologic correlation. Radio Graphics 23:
16131637.

180d

180d Sagittal T1 MRI of the brain with gadolinium


showing an enhancing posterior fossa mass, which
causes obstructive hydrocephalus. A metastasis is
seen in the prepontine cistern. This proved to be a
medulloblastoma.

321

Answer 181

Paediatric Imaging

ANSWER 181
Observations (181a)
There is a right sided aortic arch. The heart is boot shaped
indicating right ventricular hypertrophy. The lungs are not
plethoric, in fact there is a reduction in the calibre of the
pulmonary vessels. Overall, the features are suggestive of
Fallots tetralogy.

Diagnosis
Tetralogy of Fallot.

Differential diagnosis
Of conditions associated with a right sided aortic arch:
Truncus arteriosus.
Tetralogy of Fallot.
Transposition of great vessels.
Pulmonary atresia.
Ventricular septal defect (VSD).

Discussion
Tetralogy of Fallot is one of the most common causes of
cyanotic congenital heart disease and is composed of the
following: obstruction of right ventricular outflow, large
VSD, right ventricular hypertrophy and an overriding aorta.
Radiological features on CXR are:
Concavity in the region of the pulmonary artery,
which is small.
Enlarged aorta.

Normal sized heart.


Boot shaped heart due to right ventricular
hypertrophy.
Right sided aortic arch in 25% of cases.
Decreased calibre of pulmonary vessels.
Right sided aortic arch may also be seen in patients with
another cause of neonatal cyanotic congenital heart disease
transposition of the great vessels. However unlike Fallots
there is increased pulmonary vascularity and the heart has
an egg on its side appearance on the CXR due to the fact
that the mediastinum is narrow because of the abnormal
relationship of the great vessels. An example is shown
(181b), though in this particular case the aortic arch is left
sided.

Practical tips
Fallots tetralogy, pulmonary stenosis and tricuspid
atresia cause cyanosis with oligaemic lungs.
Transposition of the great vessels, truncus arteriosus
and total anomalous pulmonary venous drainage
cause cyanosis with plethoric lungs.

Further management
Cardiology referral for echocardiography and consideration
for a palliative shunt or complete surgical repair.

181b

181b CXR in a child with transposition of the great


vessels. There is pulmonary vascular congestion and
the heart has an egg on its side appearance.

322

Paediatric Imaging

Case 182

CASE 182
History
An 11-month-old child with cleft palate
and neurological abnormalities.

182

323

Answer 182

Paediatric Imaging

ANSWER 182
Observations (182)
The posterior fossa is enlarged with absence of the
cerebellar vermis, hypoplasia of the cerebellar hemispheres
and the presence of a large posterior fossa cyst, which is in
continuity with the 4th ventricle. A ventriculoperitoneal
shunt is seen in the right lateral ventricle. There is
ventriculomegaly but the sulci are not effaced. The features
are consistent with DandyWalker malformation.

Diagnosis
DandyWalker malformation.

Discussion
The DandyWalker malformation is a congenital
malformation whereby the posterior fossa is enlarged and
the tentorium cerebelli is elevated, however the cerebellar
hemispheres are hypoplastic. Absence or hypoplasia of the
cerebellar vermis is present with a posterior fossa cyst
directly connected to the 4th ventricle. Ventriculomegaly
and dysgenesis of the corpus callosum are associated
findings. Most affected patients die in infancy. The less
severe form, DandyWalker variant, is more common and
is not associated with enlargement of the posterior fossa.
The associated posterior fossa cyst is smaller and the
cerebellar vermis is hypoplastic rather than absent.

There are numerous associated CNS anomalies, for


example corpus callosum dysgenesis, holoprosencephaly,
gyral dysplasia, grey matter migration anomalies and
encephalocele. Associated anomalies outside the CNS
include cleft palate, polydactyly and cardiac defects.

Practical tips
A mega cisterna magna may mimic DandyWalker
malformation, however there is no cerebellar vermis
abnormality, continuity with or abnormality of the
4th ventricle.
A posterior fossa arachnoid cyst may also mimic these
appearances.
If the posterior fossa is not enlarged and the
cerebellar vermis is hypoplastic rather than absent,
consider DandyWalker variant rather than
malformation.

Further management
The associated CNS abnormalities can be better identified
on MRI. Treatment often involves insertion of a ventricular
shunt, as in this case, to relieve hydrocephalus. Genetic
counselling may be appropriate for the family.

182

182 Absence of cerebellar vermis (left); sulci are not effaced (bottom
left); ventriculomegaly (right); hypoplasia of cerebellar hemispheres
(top right).

324

Paediatric Imaging

CASE 183

Case 183

183

History
A newborn presented with bilious
vomiting.

325

Answer 183

Paediatric Imaging

ANSWER 183
Observations (183)
Dextrocardia is present with situs solitus. A right femoral
line is noted. An NG tube is present in the stomach, which
is dilated with air. There is a double bubble appearance
of the dilated stomach and duodenal cap with no gas seen
distally. The findings are consistent with duodenal atresia.

Diagnosis
Duodenal atresia, possibly part of VACTERL syndrome.

Differential diagnosis
Of double bubble on abdominal radiograph:
Annular pancreas.
Duodenal diaphragm.
Peritoneal band.
Choledochal cyst.

The characteristic cardiac abnormality is a septal defect but


dextrocardia, as in this case, has been described.

Discussion
Duodenal atresia is due to failure of recanalization of the
duodenum at around 10 weeks and is the most common
cause of congenital duodenal obstruction. The other major
cause is annular pancreas, and both are associated with
Downs syndrome. The obstruction is just beyond the
ampulla in the majority of cases and the double bubble
results from gas-fluid levels in the first part of duodenum
and stomach. Gas may be seen more distally in the bowel
if there is duodenal stenosis rather than complete atresia
(though atresia is twice as common).

183

183 Double bubble.

326

Duodenal atresia is associated with the VACTERL


syndrome, a non-random association of congenital
abnormalities affecting multiple systems, summarized by
the mnemonic VACTERL. Three or more of the
associated defects are required to make the diagnosis. The
mnemonic is as follows:
Vertebral anomalies.
Anorectal anomalies imperforate anus.
Cardiovascular anomalies most commonly
endocardial cushion defects.
Tracheo-Esophageal fistula.
Renal anomalies may be associated with a single
umbilical artery.
Limb anomalies e.g. radial dysplasia, polydactyly,
syndactyly.

Practical tips
Always check the double bubble radiograph for
VACTERL associations, e.g. vertebral anomalies on the
film.

Further management
Fluid and electrolyte imbalance must be corrected along
with decompression of the stomach via NG tube insertion.
Surgical correction is then required, usually with good
outcome.

Paediatric Imaging

CASE 184

Case 184

184a

History
A child with dwarfism.

327

Answer 184

Paediatric Imaging

ANSWER 184
Observations (184a)
Lateral spinal radiograph shows vertebral bodies are
flattened with central beaking anteriorly. There is also
widening of the intervertebral disc spaces and posterior
vertebral scalloping. The findings are suggestive of
Morquios syndrome.

Diagnosis
Morquios syndrome.

Differential diagnosis
Hurlers syndrome.
Achondroplasia.

Discussion
Morquios syndrome is a rare metabolic disorder classified
as one of the mucopolysaccharidoses. It is autosomal
recessive and presents in childhood with characteristic
skeletal deformity and dwarfism. Patients also have deafness
and cardiac dysfunction, however they may well live to
adulthood. Atlantoaxial subluxation is a feature and there
may be absence of the odontoid peg. Radiograph of the

184b

cervical spine in the same patient (184b) shows multiple


flattened vertebral bodies and absence of the peg.
Hurlers syndrome (another of the mucopolysaccharidoses) and achondroplasia can have similar radiological features; in particular, both may cause posterior
vertebral body scalloping and anterior vertebral body
beaking.
The radiological features of Morquios syndrome are
described below:
Spine:
Posterior vertebral scalloping.
Widening of intervertebral disc spaces.
Congenital flattening of the vertebral bodies
(platyspondyly).
Anterior beaking of vertebral bodies.
Atlantoaxial subluxation.
Kyphoscoliosis.
Pelvis:
Fragmentation and flattening of femoral heads
(184c).
Flared iliac wings (184c).

184c

184c AP radiograph of pelvis showing fragmentation


and flattening of the femoral heads with flaring of
the iliac wings.

184b Lateral cervical spine demonstrating


absence of the peg with flattening and
posterior scalloping of the vertebral
bodies.

328

Answer 184

Paediatric Imaging

Lower limbs:
Sloping of superior margin of tibial plateau laterally
(184d).
Genu valgus deformity (184d).
Hands and feet:
Proximal tapering of the metacarpal bones
producing bullet-shaped metacarpals (184e).
Short widened tubular bones with metaphyseal
irregularity (184e).

Further management

Practical tips
It may be very difficult on imaging to differentiate
Morquios from achondroplasia or the other mucopolysaccharidoses. Some features may help radiological
differentiation:
Caudal narrowing of the spinal canal is not a feature of
the mucopolysaccharidoses however it is present in
achondroplasia; therefore assess the interpedicular
distance on the AP of the spine.
The anterior vertebral body beaks in Morquios tend
to be in the Middle of the vertebral body whereas in
Achondroplasia and Hurlers syndrome they are
Anteroinferior.

184d

If the spine radiograph includes the craniocervical


junction always assess the peg as this may be absent in
Morquios and there may be atlantoaxial subluxation.
On a pelvic radiograph flaring of the iliac wings will
be seen in achondroplasia and the
mucopolysaccharidoses, however in achondroplasia
the sacrum may be horizontal in orientation therefore
appearing absent (see Case 151).
Mortality/morbidity are related to atlantoaxial instability
due to odontoid peg hyperplasia. In addition, respiratory
complications are common due to chest wall deformity.

Further reading
Wakely S (2006). The posterior vertebral scalloping sign.
Radio lo gy 239: 607609.

184e

184e Radiograph of both hands shows bullet shaped


metacarpals.

184d AP radiograph of both knees shows


genu valgum with sloping of the superior
margins of the tibial plateau.

329

This page intentionally left blank

Chapter 6

BREAST IMAGING

The breast can be evaluated with a number of different


imaging modalities: the main ones utilized in diagnosis are
mammography, ultrasound and MR imaging. Imageguided biopsies from specific areas in the breast can be
obtained using any of these techniques to aid diagnosis and
treatment of breast diseases/conditions.
With the development of more complex surgical
procedures and oncological treatments, many breast cancer
cases are often assessed by multiple imaging modalities
including mammography, ultrasound, CT, MRI and
nuclear medicine, incorporating PET/CT. This chapter
focuses on a few salient breast imaging cases complex
multimodality staging is not discussed.

MAMMOGRAPHY
Until recently, most mammographic images were obtained
as hard film copy. Today, many breast imaging departments
have full-field digital mammography (FFDM) equipment
enabling electronic storage and image manipulation to aid
interpretation. In standard mammography two views of
each breast are taken: the mediolateral oblique (MLO)
projection and the craniocaudal (CC) or superioinferior
(SI) projection. Particular care is taken with image
acquisition in mammography inadequate positioning may
result in suboptimal images and missed diagnoses. During
image acquisition the breast is compressed to even out the
tissue thickness and hold the breast still in order to
minimize blurring of the image caused by motion.
When viewing bilateral mammograms, both sides are
assessed at the same time in a back-to-back or mirror
image format, as shown opposite. By convention, the CC
views are arranged with the lateral (outer) aspect at the top
of the image and the medial (inner) at the bottom of the
image. On the MLO view the inferior extent of the pectoral
muscle should be at least at nipple level. Small densities and
areas of asymmetry may be more apparent when viewing
both sides simultaneously. In addition to a global overview,
specific inspection of all areas is required. The examination
may be evaluated by dividing each image into thirds and
going back and forth between the right and left sides
looking specifically for global and focal asymmetry,
distortion, possible masses and calcification. Other signs to
assess for are skin and nipple retraction, skin thickening,
trabecular thickening and axillary lymphadenopathy. Small
masses and areas of microcalcification should be looked at
under magnification, using workstation tools with digital
images or a magnifying glass with conventional analogue
films. Previous mammograms may aid interpretation and
assessing the significance of focal findings.

As in other aspects of imaging with ionizing radiation,


there is always a slight chance of cancer from excessive
exposure to radiation. However, the benefit of an accurate
diagnosis far outweighs the risk. The effective radiation
dose from a mammogram is about 0.7 mSv; about the
same as the average person receives from background
radiation in 3 months.
The proportion of glandular tissue to fatty tissue within
the breast changes with age there is more glandular tissue
in younger women and as a result the background density
on mammograms is generally dense. The glandular tissue
tends to involute with age; the background density of
mammograms in older women is generally lucent.
Mammography plays a central part in the early detection
of breast cancers. Screening mammograms lead to early
detection of cancers, when they are most curable and
breast-conservation therapies suitable management options.
In the UK, the NHS Breast Screening Programme saves
over 1,400 lives per year. Currently, women in the UK are
invited every 3 years for bilateral two-view mammography
between the ages of 50 and 70 years. The screening

Left MLO

Right MLO

Left CC

Right CC

Normal format for mammograms.

331

Breast Imaging

programme in the UK is due to be extended to include


women between ages 47 and 73.
Diagnostic mammography is used to evaluate a patient
with abnormal clinical findings and may also be done after
an abnormal screening mammography in order to evaluate
the area of concern on the screening examination. Initial
mammographic images themselves are not usually enough
to determine the existence of a benign or malignant disease
with certainty. And while mammography is the best
screening tool for breast cancer available today,
mammograms do not detect all breast cancers. Also, a
small proportion of mammograms indicate that a cancer
could possibly be present when it is not (a false-positive
result). Between 5 and 15% of screening mammograms are
equivocal and further evaluation with additional
mammograms or ultrasound may be required. In addition
to standard views, spot compression views which may
merge out background tissue from a lesion and
magnification views to further assess calcification may be
undertaken. Most of these tests turn out to be normal. If
there is an abnormal finding, a follow-up or biopsy may
have to be performed. Most of these biopsies result in a
benign diagnosis.

BREAST ULTRASOUND
Breast ultrasound is used to characterize abnormalities
detected by physical examination or potential abnormalities
seen on mammography. Ultrasound imaging can help to
determine if an abnormality is solid (which may be a
malignancy, a benign tumour such as a fibroadenoma or
other nonmalignant tissue) or fluid-filled (such as a benign
cyst). As ultrasound provides real-time images, it is often
used to guide biopsy procedures.

BREAST MR IMAGING
MR imaging of the breast is not usually a replacement for
mammography or ultrasound imaging but rather a
supplemental tool for detecting and staging breast cancer
and other breast abnormalities. MR imaging of the breast
may be performed to:
Identify early breast cancer not detected through
other means, especially in women with dense breast
tissue and those at high risk for the disease.
Evaluate abnormalities detected by mammography or
ultrasound in equivocal cases.
Assess multiple tumour locations, especially prior to
breast conservation surgery.
Assess the effect of chemotherapy.
Determine the integrity of breast implants.
Dynamic contrast enhanced MRI to evaluate the breast
parenchyma for cancer has a high sensitivity (~90%), with
a lower variable specificity (4080%) a relatively high
number of false-positive results can be generated. Close
attention to scanning technique, full standard breast
imaging workup and integration of all breast imaging
findings during scan interpretation increases diagnostic
yield.

COMPUTER-AIDED DETECTION
Computer-aided detection (CAD) uses pattern recognition
software to help read medical images. Such techniques
bring features on medical images to the attention of the
film reader and may decrease false-negative readings when
films are single read. The use of CAD has been evaluated in
both mammography and breast MRI, but has not been
used widely in the UK, where screening mammograms are
interpreted by two human readers.

IMAGE GUIDED INTERVENTION


Lumps or abnormalities in the breast are often detected by
physical examination, or by mammography or other
imaging studies. However, it is not always possible to tell
from these imaging tests whether a growth is benign or
cancerous. Usually the preferred modality for intervention
is ultrasound, from both the operators perspective and
that of the patients (breast compression is not required as
is the case with a mammographic stereotactic biopsy or
MRI biopsy).
Image guidance may be used in four biopsy procedures:
Fine needle aspiration (FNA): rarely used in isolation
when evaluating breast lesions; it may be the method
of choice when sampling axillary nodes.
Core biopsy: uses a hollow needle, usually 14G, to
remove one sample of breast tissue per insertion. This
process is usually repeated three to six times.
Vacuum assisted device (VAD): where vacuum
pressure is used to pull tissue from the breast through
the needle, often 11G, into the sampling chamber.
The device rotates positions and collects a greater
volume of tissue than standard core biopsy. Small
benign lesions such as fibroadenoma may be excised
by this method. The diagnostic yield from sampling
calcification is usually greater using this technique
than standard core biopsy. A small marker coil may be
placed at the site so that it can be located in the
future if necessary.
Wire localization: in which a guide wire is placed into
a nonpalpable lesion/suspicious area or at the site of a
marker coil to enable surgical excision biopsy. This
may be undertaken when the diagnosis remains
uncertain after a breast biopsy procedure.

EVALUATION OF IPSILATERAL AXILLA


Once a breast cancer is suspected or confirmed, the
ipsilateral axilla may be staged clinically or with ultrasound:
the area between the axillary vein, latissimus dorsi muscle
and medial border of the pectoralis minor muscle is
carefully inspected. Any nodes of suspicious configuration
(signs include loss of uniform reniform shape, loss of
echogenic fatty hilum and eccentric cortical thickening)
should undergo FNA or core biopsy to assess for metastatic
spread. If the axillary FNA or core biopsy histology is
negative in proven cases of breast malignancy then
combined aniline blue dye and scintigraphic sentinel node
surgical biopsy is usually performed.
Pro fesso r Iain D Lyburn, BSc, MRCP, FRCR
Glo ucestershire Breast Screening Service

332

Breast Imaging

CASE 185

Case 185

185

History
Screening mammograms in a 61-yearold woman.

333

Answer 185

Breast Imaging

ANSWER 185
Observations (185)

Practical tips

There is a semi-spiculate microlobulated dense mass in the


upper inner quadrant of the right breast. The remainder of
the right breast is normal. The left breast is normal. The
suspicious mass in the upper inner right breast likely
represents a unifocal carcinoma.

Look closely for associated microcalcification in the


lesion.
Avoid satisfaction of survey: once one
lesion/abnormality is identified, continue searching
for multifocal (other lesions in the same quadrant,
same duct system or within 4 cm of the affected
breast), multicentric (other lesions in a different
quadrant, different duct system or separated by >4 cm
in the affected breast) or contralateral disease.
Look in the axilla for nodes malignant involvement
cannot be diagnosed, but the presence of large nodes
should be commented upon and the suggestion of
ultrasound guided sampling made.

Diagnosis
Breast carcinoma (invasive ductal carcinoma; IDC).

Differential diagnosis
For stellate lesion:
Malignancy.
Postsurgical scar (ask about a history of previous
surgery).
Fat necrosis (often post-trauma or surgery).
Radial scar (a lesion with central scar formation and
radiating hyperplastic ducts).

Discussion
The most common mammographic sign of an invasive
breast cancer is a mass: a space-occupying lesion that is seen
in at least two mammographic projections. The typical
features on mammography of a mass due to an invasive
cancer are irregular shape, ill-defined or spiculate margins
and high radiographic density. In about 40% of cases the
mass is associated with calcification of malignant
configuration pleomorphic and irregular.
Mammographic ill-defined masses require further
evaluation. Ultrasound will often demonstrate a hypoechoic
irregular mass with ill-defined borders. Image guided core
biopsy should be performed if not possible with ultrasound then under mammography guided stereotaxis.
In such cases, ultrasound of the ipsilateral axilla should
be performed and any nodes of suspicious configuration
should undergo fine needle aspiration (FNA) or core biopsy
to assess for metastatic spread (see introduction to this
chapter).
Invasive ductal carcinoma is the most common
indistinctly marginated carcinoma. IDC may be divided
into various specific subtypes: the majority of ductal
malignancies fall into the generalized category of lesions
that are undifferentiated and have no particular
distinguishing histological features; these are termed not
otherwise specified (NOS) and account for 65% of invasive
breast cancers.

334

Further management
Recall for clinical examination, ultrasound +/ further
mammographic views with the intention to proceed to
image guided core biopsy.

Breast Imaging

CASE 186

Case 186

186a

History
A 56-year-old woman with a firm mass
in the central left breast.

186b

335

Answer 186

Breast Imaging

ANSWER 186
Observations (186a, 186b)
Bilateral mammograms show that in the upper central and
lateral aspects of the left breast there is extensive
pleomorphic microcalcification. Within the left axilla there
is a 21 mm diameter rounded node. The right breast
appears normal. Ultrasound of the superior central left
breast demonstrates an irregular solid mass.
The extensive microcalcification is of a configuration
suspicious for malignancy in the central left breast. There
is possible metastatic spread to the large axillary node. The
ultrasound demonstrates a probable malignancy. Further
investigation with urgent biopsy of the lesion is required.

Diagnosis
Ductal carcinoma in situ (DCIS) with involved axillary
nodes.

Differential diagnosis
For pleomorphic calcification on mammograms:
DCIS.
Atypical ductal hyperplasia.
Fat necrosis.
Fibrocystic change.
For enlarged axillary nodes:
Ipsilateral breast malignancy.
Infection/inflammation of ipsilateral breast or arm.
Collagen vascular disease/rheumatoid arthritis.
Lymphoproliferative diseases: lymphoma and
leukaemia.
Metastases (melanoma, lung, contralateral breast).
HIV adenopathy.

Look carefully for signs of an invasive focus within the


calcification search for a spiculate mass or stromal
deformity. If not identified, assess further with
ultrasound and possibly MR imaging.

Further management
Further imaging assessment is suggested: mammographic
magnification orthogonal views (typically craniocaudal and
mediolateral) may evaluate morphology and distribution
of the microcalcification. Ultrasound guided biopsy of the
solid lesion and mammographic stereotactic biopsies to
obtain a sample containing calcification should be
performed. The suspicious node should be sampled under
ultrasound guidance. In this case, the biopsies showed:
ultrasound solid lesion invasive carcinoma; stereotactic
cores DCIS; axillary node malignant cells.
Once a diagnosis of malignancy has been established,
surgical referral is required. In this case the patient
underwent a mastectomy and axillary lymph node
clearance. Histological findings were of a 12 mm IDC in
the superior central breast with extensive (7 cm)
intermediate grade DCIS; 4 out of 15 axillary nodes were
involved with tumour.

186a

Discussion
Pleomorphic microcalcification may be defined as irregular
calcifications of varying sizes and shapes, usually <0.5 mm
in size. Orthogonal mammographic views may clarify the
characteristics for instance, linear and segmental
distributions suggest that the calcification is ductal in
origin, whereas regional or diffuse multiple bilateral groups
are less likely to represent a ductal process.
A small percentage of malignant lesions arise from the
stromal elements of the breast. Ninety per cent of breast
cancers have cellular features that are similar to ductal
epithelium and are consequently classified as ductal cancers.
When confined to the duct they are termed ductal
carcinoma in situ (DCIS). When the cells have breached
the basement membrane around the duct and invaded the
surrounding tissues, they are termed invasive ductal
carcinoma (IDC).
The diagnosis of DCIS is associated with the possibility
of associated invasive disease. If no invasive focus is
identified on mammography further assessment with
ultrasound and possibly MR imaging with a view to
potentially finding an invasive component may be
performed.

Practical tips
Not infrequently, microcalcification is of equivocal
configuration if there is doubt there is a low
threshold to proceed to biopsy.

336

186a Left MLO: axillary lymph


node (upper arrow) and
extensive microcalcification
(lower arrow).

Breast Imaging

Cases 187, 188

CASE 187

CASE 188

History

History

A 33-year-old woman with a soft


mobile smooth left breast lump.

A 64-year-old woman with a swollen


erythematous right breast.

187a

188

187b

337

Answers 187, 188

Breast Imaging

ANSWER 187
Observations (187a, 187b)
The breasts are heterogeneously dense. There are semiovoid low-density opacities in both breasts. There is a large
dominant lesion in the upper outer left breast.
Ultrasound of the left breast lump demonstrates a well
defined smooth margined anechoic mass with through
transmission. The appearance of the lesion assessed with
ultrasound is in keeping with a benign simple cyst.

Diagnosis
Benign simple cysts.

Differential diagnosis
Of a smooth low-density lesion on mammography:
Simple cyst.
Oil cyst.
Fibroadenoma.
~1.5% of circumscribed round lesions may be
malignancies.

perimenopausally in many women, but can be found in


women of all ages. Cysts may develop after commencing
oestrogen (hormone) replacement therapy.
On mammography, cysts appear as semi-ovoid masses
with variable margins and density. There may be a
peripheral halo and/or rim egg shell calcification. On
ultrasound, cysts usually appear as well defined, anechoic
masses with posterior acoustic enhancement. In some, high
specular echoes shift in position as gain is increased
(gurgling cysts). Posterior enhancement is not always
demonstrable, particularly if the cyst is small or close to the
chest wall. If there is any question as to the cystic nature
of a lesion, aspiration is recommended. On occasion lesions
appear cystic on ultrasound, but aspiration is unsuccessful
thick proteinaceous fluid may be too gelatinous to be
aspirated.

Practical tips

Of an echoic lesion on ultrasound:


Simple cyst.
Complicated (proteinaceous) cyst.
Duct ectasia.
Intraductal/intracystic papilloma look carefully on
ultrasound for a mural lesion.

Avoid satisfaction of survey: look for other, more


suspicious lesions.
Cysts often recur after aspiration.
If the lesion is clearly a simple cyst, aspiration is not
required unless the symptoms of the mass are
distressing.
If there is any doubt as to the nature of the cystic
lesion, core biopsy is suggested.

Discussion

Further management

Cysts are asymptomatic in many women. Presentation is


variable. A palpable mass or masses may develop rapidly and
is/are associated with tenderness. They develop

If there is any doubt about the mammographic


appearances, further evaluation with ultrasound should be
undertaken.

ANSWER 188
Observations (188)
There is diffuse trabecular prominence throughout the right
breast which is of generalized increased density. The skin
is thickened. There are enlarged nodes in the right axilla.
The left breast is normal.

Diagnosis
Probable inflammatory right breast cancer with axillary
nodal involvement.

Differential diagnosis
For diffuse trabecular/skin thickening:
Post radiotherapy change.
Progressive systemic sclerosis.
Obstruction of the superior vena cava.
Lymphoma.
Infection/inflammatory mastitis most common in
lactating women.
Trauma.
Generalized oedema due to causes such as congestive
heart failure or nephritic syndrome.

Discussion
Ultrasound or MR imaging may be used to find a discrete

338

invasive focus which could be biopsied. The diagnosis


could also be obtained from skin punch biopsy or from
image guided core biopsy of an axillary node.
Inflammatory breast cancer may be defined by clinical
diagnosis dependent on findings of oedema, erythema and
peau dorange or on histological findings of metastatic
breast cancer in dermal lymphatics. The definition is
debatable: not all women with clinical findings suggestive
of inflammatory breast cancer have involved dermal
lymphatics and not all patients with tumour cells in the
dermal lymphatics present with signs of inflammation.
Inflammatory malignancies account for 1% of all breast
cancers and up to 40% of locally advanced breast cancers.
The differentiation between mastitis and inflammatory
carcinoma may be difficult.

Practical tips
Patients often undergo neoadjuvant chemotherapy
prior to mastectomy.
Consider inflammatory breast cancer when an
inflamed breast fails to respond to a brief course of
antibiotics.

Breast Imaging

Cases 189, 190

CASE 189

CASE 190

History

History

A 42-year-old man with a soft mobile


tender left breast lump.

Screening mammograms in a 57-yearold woman.

189

190

339

Answers 189, 190

Breast Imaging

ANSWER 189
Observations (189)
In the left breast there is a fan shaped density emanating
from the nipple which gradually blends into surrounding
fatty tissue. The right breast appears normal.

Diagnosis
Gynaecomastia.

Differential diagnosis
Of a breast lump in a male:
Gynaecomastia.
Male breast cancer (circumscribed or spiculate mass
usually evident; often eccentric to the nipple).
Pseudogynaecomastia (fatty enlargement with no
ductal or stromal proliferation; secondary to obesity).
Diabetic mastopathy (firm mass in patient with
longstanding type 1 diabetes mellitus).
Abscess (erythema; acute history).

Discussion
Gynaecomastia usually appears as a fan shaped density
emanating from the nipple, gradually blending into
surrounding fat. Three mammographic patterns of
gynaecomastia have been described: nodular, dendritic and
diffuse. There may be prominent extensions into the
surrounding fat and, in some cases, an appearance similar
to that of a heterogeneously dense female breast. Although

there are characteristic mammographic features that allow


breast cancer in men to be recognized (round/spiculate
subareolar mass typically eccentric to the nipple), there is
substantial overlap between these features and the
mammographic appearance of benign lesions. Male breast
cancer is rare, accounting for <1% of all male cancers.
Gynaecomastia is characterized by hyperplasia of ductal
and stromal elements of the male breast. It manifests
clinically as a soft, mobile, tender mass in the retroareolar
region. Gynaecomastia has been associated with an
increased serum level of oestradiol and a decreased level of
testosterone this may occur with physiological changes
at puberty and senescence and be caused by endocrine
disorders, systemic diseases, neoplasms and certain drugs
including anabolic steroids, cimetidine, spironolactone and
marijuana.

Further management
On the diagnosis of gynaecomastia it is important to
correlate the imaging findings with the clinical history.
Many cases of gynaecomastia are idiopathic but
underlying causes should be investigated serum hormone
levels should be taken. Ask for a drug history and the
presence of signs of chronic renal insufficiency, cirrhosis
and a testicular mass; other imaging investigations
pertaining to the patient may raise one of these
possibilities.

ANSWER 190
Observations (190)

Bilateral calcification of benign configuration.

system. These calcifications tend to be coarser and


larger (usually >1 mm in diameter) than malignant
calcifications.
Skin or dermal usually spherical and lucent-centre
calcifications at the periphery of the breast.
Suture usually seen at a known surgical site. The
calcifications may be linear or tubular.

Differential diagnosis

Discussion

For benign calcification:


Vascular usually secondary to medial atherosclerosis.
May be associated with diabetes and
hyperparathyroidism. Often demonstrates a
characteristic train track configuration.
Fat necrosis peripheral calcification in a lucent mass;
history of trauma or surgery.
Fibroadenoma involution popcorn-like
calcifications usually beginning at the periphery and
then involving the central portion.
Milk of calcium a benign process that can be
diagnosed with magnification views of orthogonal
projections: on the CC view, calcifications appear
poorly defined and smudgy; when imaged on the
MLO view, the calcifications are seen as sharply
defined and crescent shaped or linear.
Plasma cell mastitis and duct ectasia large rod-like
calcifications oriented along the axes of the ductal

Calcification is a frequent finding on mammograms. The


arrangement of calcification aids categorization as to
whether it is benign or malignant. Clustered (occupying a
volume <1 l of tissue), linear and segmental calcification
may be secondary to benign or malignant processes.
Regionally and diffusely distributed calcifications are most
likely due to benign processes. These calcifications are
scattered in a large volume of the breast and do not
necessarily conform to a ductal distribution.

In the anterior aspects of both breasts there are smooth well


defined spherical calcifications with lucent centres. No
abnormal masses or distortion are seen.

Diagnosis

340

Practical tips
If calcification is of equivocal configuration, there is a low
threshold to proceed to biopsy.

Further management
No intervention required. Routine recall for screening.

Breast Imaging

CASE 191

Case 191

191a

History
A 44-year-old woman with fullness in
the medial left breast on clinical
examination. There is no discrete mass.

191b

191c

341

Answer 191

Breast Imaging

ANSWER 191
Observations (191a, 191b, 191c)

Discussion

Bilateral mammograms show fibroglandular densities


scattered in both breasts. There is general increased density
in the medial left breast. The right breast appears normal.
Ultrasound of the upper inner left breast (191b) shows
ill-defined architectural distortion with indistinct margins
and posterior shadowing. Appearances are suspicious of an
invasive malignancy and further imaging with breast MR
imaging is suggested to assess extent of disease.
The single axial view of a T1 fat saturation gadolinium
enhanced scan (191c) demonstrates extensive ill-defined
microlobulated inhomogeneous enhancement within the
upper inner/central left breast, which would be consistent
with an invasive carcinoma.

Invasive lobular carcinoma accounts for 812% of invasive


breast malignancies.
ILC most commonly presents as a spiculate mass on
mammography, but not infrequently may manifest as
isolated architectural distortion or focal asymmetry.
Asymmetrical density refers to a relative increase in the
volume of density as compared with the corresponding
area in the other breast. Such asymmetry usually represents
a normal variation in distribution of fibroglandular tissue.
Occasionally, asymmetrical density is a sign of breast
cancer. ILC may be difficult to detect mammographically
due to the insidious growth pattern. Ultrasound may help
depict mammographically subtle or occult ILC, but often
underestimates the size of the lesion and multifocality/
multicentricity many centres now stage the breast with
MRI prior to making a decision on the type of surgical
management (breast conserving technique or mastectomy).

Diagnosis
Invasive lobular carcinoma (ILC).

Differential diagnosis
For mammographic asymmetry:
Normal variant dominant glandular tissue (stable
compared to previous mammograms, usually
nonpalpable).
Summation artefact due to superimposed normal
structures (thins on spot compression mammograms;
ultrasound normal).
Hormone influences between 20% and 40% of
women commenced on hormone replacement therapy
(HRT) develop increased density which may be focal
or generalized.
Malignancy (invasive ductal carcinoma [IDC], invasive
lobular carcinoma [ILC], ductal carcinoma in situ
[DCIS]) this must be considered when there is
asymmetrical density that is newly developed when
compared to previous mammograms, and that persists
on spot compression mammograms; and/or when
encountering a hypoechoic mass on ultrasound.

Practical tips
Check thoroughly for multicentric and contralateral
malignancies, which are of a high proportion in ILC.

Further management
Please refer to the introductory section in this chapter for
indications for breast MRI.

191c

191c T1 weighted fat saturated image


post gadolinium enhancement shows an
ill-defined area of hyperenhancement.

342

Further Reading
Aids to Radio lo gical Differential Diagno sis
Stephen Chapman, Richard Nakielny
WB Saunders, 4th edn, 2003

Neuro radio lo gy The Requisites


Robert I Grossman, David M Yousem
Mosby, 2nd edn, 2003

Clinical Imaging: An Atlas o f Differential Diagno sis


Ronald L Eisenberg
Lippincott Williams and Wilkins, 4th edn, 2002

Paediatric Neuro imaging


A Barkovich
Lippincott Williams and Wilkins, 4th edn, 2005

Diagno stic Imaging Head and Neck


Ric Harnsberger
Amirsys, 2004

Radio lo gy Review Manual


Wolfgang Dahnert
Lippincott Williams and Wilkins, 6th edn, 2007

Fundamentals o f Diagno stic Radio lo gy


William E Brant, Clyde A Helms
Lippincott Williams and Wilkins, 3rd edn, 2006

Self-Assessment Co lo ur Review o f Neuro imaging


Kirsten Forbes, Sanjay Shetty, Michael Lev, Joseph
Heiserman
Manson Publishing Ltd, 2008.

Grainger and Alliso ns Diagno stic Radio lo gy: A


Textbo o k o f Medical Imaging
Ronald G Grainger, David J Allison, Adrian K Dixon
Churchill Livingstone, 4th edn, 2001
Imaging o f Diseases o f the Chest
David M Hansell, Peter Armstrong, David A Lynch, H
Page McAdams
Mosby, 4th edn, 2004

Self-Assessment Co lo ur Review o f Tho racic Imaging


Sue Copley, David M. Hansell, Nestor L. Mller
Manson Publishing Ltd, 2005
Textbo o k o f Radio lo gy and Imaging
David Sutton, Rodney Reznek, Janet Murfitt
Churchill Livingstone, 7th edn, 2002

Magnetic Reso nance Imaging o f the Brain and Spine


Scott W Atlas
Lippincott-Raven, 2nd edn, 1996

343

Index of Differential Diagnoses


Figures refer to case numbers
achalasia 57
secondary 57
achondroplasia 184
acro-osteolysis (SHARTEN) 152
ankylosing spondylitis 143
anterior mediastinal mass 10
anus, imperforate 173
aortic arch, right sided 181
aphthous ulceration 54
arthropathy
erosive 143
nonerosive deforming 132
atlantoaxial subluxation 120
atypical medulloblastoma 104
avascular necrosis (AVN) (DRIED
HIP) 139
axillary nodes, enlarged 186
basal ganglia
calcification 166
low-density 166
Behets disease 38
bladder
calcification 62
pear shaped (HELP) 70
radiolucent filling defects on IVU
56
small 60
bone lesions, permeative 118
bone within bone appearance
(SHARPS POOL) 145
bony sclerosis 140
bowel (small), smooth thickened
folds 59
bowel, obstruction 46, 47
brain, white matter lesions 97
breasts
anechoic lesion 187
benign calcification 190
enlarged axillary nodes 187
lump in male 189
mammographic asymmetry 191
pleomorphic calcification 186
skin thickening 188
smooth low-density lesion 187
stellate lesion 185
trabecular thickening 188
Brodies abscess 134
calcaneus, lucent lesion in 146
calcification
basal ganglia 166
bladder wall 62
breast: benign 190
fingertip 152
gall bladder 82
hilar 1

344

calcification (co ntinued)


liver metastases 52
ocular 91
periarticular soft tissue 121
pleomorphic: breast 186
popcorn, of lymph nodes 25
spleen 25
calcified liver lesion 40
Candida oesophagitis 38
Carolis disease 80
cavitating lesion, solitary 13
central nervous system (CNS)
lesions, ring enhancing 90
central pontine myelinolysis 105
centrilobular nodules 2
cerebellopontine (CP) angle, lesions
in 94
cerebral atrophy, diffuse 105
cerebral territory infarct 113
Chagas disease 57
children, osteosclerosis in 119
chondrosarcoma 141
clavicle, lateral end erosion 6
congenital diaphragmatic hernia
(CDH) 164
congenital pseudarthrosis 127
congenital thoracic outlet syndrome
28
consolidation and lymphadenopathy
2
corpus callosum, lesions 110
Crohns disease 38
cystic adenomatoid malformation
(CAM) 164
cystic lung disease 27
cystic mandibular lesions 95
cysts: breast 187
double bubble on abdominal
radiograph 183
double density sign 148
dysplasia, lethal neonatal 161

gastric folds, thickened 42, 54


gastric varices 42
gastrointestinal haemorrhage 39
giant cell tumour (GCT) (in child)
134
ground glass opacity 29
haemangioblastoma 104
hair on end sign (STAN) 129
HELP (pear shaped bladder) 70
hemithorax
complete opacification of 4, 14
veil-like opacification of 14
hepatic lesions
calcified 40
with central scar 85
hyperechoic 71
herpes simplex (HSV) encephalitis
113
hilar calcification 1
hilar enlargement, bilateral 1
Hirschsprungs disease 173
Hurlers syndrome 184
hyperdensity, unilateral 31
hyperechoic hepatic lesions 71
hypertransradiancy, unilateral 31
hypophosphatasia 176
ileal atresia 173
imperforate anus 173
inferior rib notching 32
inguinal hernia 173
interlobular septal thickening 29
intrasellar/suprasellar mass lesion
106
irregular epiphyses 135
ivory vertebra sign (Mets...LP HIM)
137
juvenile pilocytic astrocytoma 104
juxtacortical haematoma 136
Kerley B lines 12

eosinophilic granuloma 148, 158


epiphyseal lesions 133
epiphyses, irregular 135
Erlenmeyer flask deformity (Lead
GNOME) 178
erosive arthropathy 143
Ewings sarcoma 125
extraosseous osteosarcoma 136
fallen fragment sign 115
fibrous dysplasia 127, 158
fingertip calcification 152
flitting pneumonia 21
fractures, nonunion of 127
gallbladder, calcification 82

large bowel obstruction 46


lethal neonatal dysplasia 161
leukaemia 158
linitis plastica appearance (CALM
RAGE) 76
lipidoses 178
liver
calcified lesion 40
calcified metastases 52
hyperechoic lesions 71
lesions, with central scar 85
liver disease, polycystic 80
lung disease, cystic 27
lung nodules
centrilobular 2

Index of Differential Diagnoses


lung nodules (co ntinued)
miliary 4
multiple 8
multiple cavitating 5
perilymphatic 1
pin-point high-density 20
pulmonary 8, 33
solitary pulmonary 11
subpleural 12
lungs
cavitating lung nodules 5
high-density opacification 20
multiple nodules 8
pin-point high-density nodules
20
pulmonary fibrosis 17, 18
solitary nodule/mass 11
lymph nodes, popcorn calcifications
25
lymphadenopathy and consolidation
2
lymphangitis, tumours causing 12
lymphoma 141, 158
lytic lesion in digits (SEGA GAME
F) 144
lytic/sclerotic destructive lesion
141
Madelung deformity (TILDti) 153
male breast lump 189
mandible: cystic lesions 95
meconium ileus 173
mediastinal mass
anterior 10
posterior 10
medulloblastoma, atypical 104
melorheostosis 122
metacarpal, short 4th 147
metastases
brain 104
hemipelvis 141
lung 24
sclerotic 122, 137
metatarsal, short 4th 147
middle mediastinal mass 10
miliary nodules 4
miliary opacities, increased density
25
Morquios syndrome 184
myositis ossificans 136
neonatal dysplasia, lethal 161
nephrogram, persistent dense 58
neuroblastoma 174
neurofibromatosis 127
neuropathic joint 138
nonerosive deforming arthropathy
132
ocular calcification 91
oesophageal mass lesion 34
oesophageal strictures 38
oesophageal varices 42

opacification
ground glass 29
high-density 20
over hemithorax 14
ophthalmic vein (superior)
distension 103
optic nerve thickening 99
orbital pseudotumour 88
osteochondritis dissecans 131
osteochondroma 136
osteogenesis imperfecta 127
osteoid osteoma 148
osteomyelitis 158
osteonecrosis, spontaneous 131
osteopoikilosis 122
osteosarcoma 125, 134, 136
osteosclerosis
in adults 140
in children 119, 145
pancreas, annular 44
parosteal osteosarcoma 136
pear shaped bladder (HELP) 70
pencil in cup deformity 143
periarticular soft tissue calcification
121
perilymphatic nodules 1
periosteal reaction, diffuse bilateral
157, 175
periventricular white matter 97
permeative bone lesions 118
persistent dense nephrogram 58
pilocytic astrocytoma, juvenile 104
pineal region mass 92
platyspondyly 154
pleomorphic calcification: breast
186
pleural lesions 9
pleural thickening, diffuse 2
pneumoconiosis, mass lesion with
7
pneumomediastinum, causes of 34
pneumonia, flitting 21
pneumoperitoneum, causes of 75
polycystic liver disease 80
polydactyly, causes of 177
pontine glioma 105
posterior fossa tumour 180
posterior mediastinal mass 10
posterior vertebral scalloping 151
primary sclerosing cholangitis (PSC)
45
pseudarthrosis, congenital 127
pseudotumour, orbital 88
psoriatic arthropathy 143
pulmonary fibrosis
lower zone 17
upper zone 18
pulmonary nodules 8, 33
Pyles disease 178
radiolucent bladder filling defects on
IVU 56

radiolucent filling defects in the


ureters 77
renal papillary necrosis (SAD ROPE)
78
rheumatoid arthritis 143
ribs, notching 32
rickets 176
sacroiliitis 116
sclerosis (dense), and cortical
thickening 148
sclerotic metastases 122, 137
sclerotic/lytic destructive lesion
141
septal (Kerley B) lines 12
septal thickening
nodular interlobular 29
smooth interlobular 29
skull, hair on end 129
splenomegaly 140
small bowel
obstruction 47
smooth thickened folds 59
solitary cavitating lesion 13
spleen, calcification 25
spontaneous osteonecrosis 131
stellate lesion: breast 159
subluxation, atlantoaxial 120
subpleural nodules 12
superior ophthalmic vein distension
103
superior rib notching 32
superscan, causes of a 156
suprasellar/intrasellar mass lesion
106
suture diastasis (TRIM) 162
syndactyly, causes of 177
target lesions 64
terminal ileal disease 53
thoracic outlet syndrome, acquired
28
thumb-printing 53
thyroid ophthalmopathy 88
Trypano so ma cruzi infection 57
tumours, causing lymphangitis 12
unilateral hyperdensity 31
unilateral hypertransradiancy 31
ureters, radiolucent filling defects
77
urinary tract, gas in 63
varices, gastric 42
ventricular lesion 89
vertebra, posterior scalloping 151
vertebra plana 154
white matter lesions on MRI 97
Wilms tumours, bilateral 174
wormian bones (PORKCHOPSI)
168, 179

345

General Index
Figures refer to case numbers
achalasia 57
achondroplasia 151, 184
acoustic neuroma 94
acromegaly 98
acro-osteolysis 152
adamantinoma 95
adenomyosis 81
adrenal adenoma 67
air bronchogram, hyaline membrane
disease 165
air trapping, meconium aspiration
syndrome 170
albendazole 40
alcohol consumption/alcoholism
105
erosive gastritis 54
pancreatitis 72
allergic bronchopulmonary
aspergillosis (ABPA) 21, 36
alveolar microlithiasis 20
ameloblastoma 95
amputation, lymphoma recurrence
118
aneurysmal bone cyst 95, 115
angiodysplasia 39
angiography
arteriovenous malformations 33
coarctation of the aorta 32
focal nodular hyperplasia 85
mesenteric 39
angiomyolipoma 48
angiotensin converting enzyme
(ACE) serum levels 1
ankylosing spondylitis 18, 116,
120, 127
anterior cruciate ligament 159
antinuclear antibodies (ANA) 17
anus, imperforate 44, 173
aorta, coarctation 32
aortic aneurysms, thoracic 10
aortic arch, right sided 26, 181
aortic stenosis, angiodysplasia
association 39
aortic valve, bicuspid 32
aortitis, ankylosing spondylitis 18
Aperts syndrome 177
aphthous ulcers, Crohns disease 53
arachnoid cyst 89, 94
arteriovenous malformations 102
arthropathy of haemochromatosis
50
asbestosis 3, 15
Aspergillus
allergic bronchopulmonary
aspergillosis 21, 36
cystic fibrosis 36

346

secondary infection 7
upper lobe segment 18
aspiration pneumonia 171
asthma
allergic bronchopulmonary
aspergillosis 21
global cerebral anoxia 114
atelectasis
meconium aspiration syndrome
170
rounded 15
atherosclerotic disease
coronary artery 30
renal artery stenosis 58
subclavian steal syndrome 19
atlantoaxial subluxation 120, 184
atrial fibrillation 23
Auerbachs plexus degeneration 57
autonephrectomy, renal tuberculosis
74
avascular necrosis of femoral head
117, 135, 139
balloon occlusion, pulmonary
arteriovenous malformation 33
balloon angioplasty, subclavian steal
syndrome 19
bamboo spine 18, 120
barium aspiration 20
basal ganglia, low-density 166
biliary stasis, Carolis disease 80
biliary tree
gas 41, 47
hepatic cysts 80
biopsy
core 185
ultrasound guided 186
birds beak sign 46, 57
bladder
calcification 62
cancer 62
neurogenic 60
pear shaped 70
squamous cell carcinoma 62
stones 56
trabeculation 172
tuberculosis 62, 74
bladder tumour, pseudoureterocele
56
blood transfusion, iron overload 50
bone
achondroplasia 151, 184
acromegaly 98
aneurysmal cyst 95, 115
Caffeys disease 175
chondrosarcoma 141
diaphyseal aclasis 130
Ellisvan Creveld syndrome 177

bone (co ntinued)


enchondroma 144
Ewings sarcoma 125
fibrous dysplasia 149
growth plate widening 176
intraosseous lipoma 146
ivory osteoma 96
Madelung deformity 147, 153
melorheostosis 122
osteochondritis dissecans 131
osteogenesis imperfecta 179
osteoid osteoma 134, 148
osteomalacia 126, 176
osteopetrosis 119, 140, 145
osteopoikilosis 122
osteosclerosis 119, 140
parosteal osteosarcoma 136
polysyndactyly 177
Pyles disease 178
resorption 160
simple cyst 115
slipped upper femoral epiphysis
117
trabeculation 124
bone within bone appearance 140,
145
bowel
ischaemia 167
see also large bowel; small bowel
bowel loop displacement 84
bowel obstruction, endometrioma
83
bowel strictures 167
bow tie sign, meniscal 159
brain abscess 33
brain damage, irreversible 163
brainstem glioma 180
brainstem infarction 100
brainstem tumours 180
breast
benign calcification 187, 190
gynaecomastia 140, 189
breast cancer
ductal carcinoma in situ 186
inflammatory 188
invasive ductal carcinoma 185,
186
invasive lobular carcinoma 191
metastatic spread 115, 185
microcalcification 185, 186
miliary metastases 4
recurrence 31
sclerotic bone metastases 31, 137,
140
breast cysts, benign 187
breast fibroadenoma, calcified 11
Brodies abscess 134
bronchi, mucous-filled 21

General Index
bronchiectasis
airway walls 27
cystic in allergic
bronchopulmonary aspergillosis
21
traction 1
bronchioles, mucoid impaction 21
bronchogenic carcinoma
metastatic 24
miliary metastases 4
bronchogenic cyst, intrapulmonary
13
brown tumours 160
bucket handle tear, meniscal 159
bulla, right sided 35
butterfly glioblastoma multiforme
110
caecal volvulus 46
Caffeys disease 175
calcification
ankylosing spondylitis differential
diagnosis 18
atrial 23
basal ganglia 166
benign breast 190
benign breast cysts 187
bladder in schistosomiasis 62
breast cancer 185, 186
breast fibroadenoma 11
chondrosarcoma 141
enchondroma 144
gall bladder 82
intraosseous lipoma 146
ivory osteoma 96
ligamentous insertion 119
liver lesions 40
liver metastases 52
lung 7, 8, 11, 20
mammography 185, 186
microcalcification in breast cancer
185, 186
myositis ossificans progressiva
123
obstructing gallstone 41
optic drusen 91
ovarian dermoid cysts 84
peritoneal 167
pleural 15
pleural pseudotumour 11
popcorn of lymph nodes 25
scleroderma 152
spleen 25
testicular microlithiasis 73
tumoral calcinosis 121
ureters in schistosomiasis 62
vascular 138
calvaria, wormian bones 168
CalvKummelVerneuil disease
154
Candida 63, 79
Caplans syndrome 6
capsule endoscopy, Crohns disease
53

cardiac granulomas 22
cardiac sarcoid 22
cardiomegaly, pulmonary arterial
hypertension 16
cardiothoracic ratio
pleural pseudotumour 11
pulmonary oedema due to heart
failure 29
Carolis disease 80
caroticocavernous fistula 103
carpal tunnel syndrome 150
cavernoma 102
cavernous haemangioma 71
cavernous sinus, caroticocavernous
fistula 103
central nervous system (CNS)
tumours 89
central pontine myelinolysis 105
cerebral abscess 90
cerebral anoxia, global 114
cerebral hemispheres, schizencephaly
169
cerebral injury, global 114, 166
cerebrovascular accident 33
cervical ribs, bilateral 28
cervical spine 120
Charcot joint 138
chemotherapy, nephroblastoma
174
chest wall
abnormalities 31
lipoma 9
chocolate cyst 83
cholangitis 80
cholecystectomy, porcelain
gallbladder 82
chondroblastoma 133
chondrocalcinosis 50
chondrosarcoma 141
cigarette smoking, asbestos exposure
15
circumferential resection margin
(CRM) 52
cirrhosis risk with haemochromatosis
50
Claudes syndrome 100
cleidocranial dysostosis 127
multiple wormian bones 168
coal workers
progressive massive fibrosis 7
rheumatoid arthritis 6
coarctation of the aorta 32
Codmans triangle 125
coeliac disease 44, 69
colectomy 49
colloid cyst 89
colon
adenocarcinoma 65
metastases 61
pneumatosis 66
polyps 49
strictures 65
vascular ectasia 39
colorectal carcinoma 52

computed tomography (CT)


angiography
angiodysplasia association 39
coarctation of the aorta 32
congenital anomalies
coarctation of the aorta 32
neuronal migration 169
congenital diaphragmatic hernia
(CDH) 164
congenital oesophageal atresia with
tracheo-oesophageal fistula 171
continuous diaphragm sign 34
cor pulmonale, cystic fibrosis 36
coronary artery atherosclerotic
disease 30
corpus callosum
agenesis 107
butterfly lesions 110
Cowdens syndrome 49
cranial suture diastasis 162
craniometaphyseal dysplasia 178
craniopharyngioma 106
CREST syndrome 17
crocidolite 15
Crohns disease 53
erosive gastritis 54
primary sclerosing cholangitis
association 45
ulcerative colitis differentiation 65
CronkhiteCanada syndrome 49
cystic adenomatoid malformation
(CAM) 164
cystic fibrosis 36
allergic bronchopulmonary
aspergillosis 21
meconium ileus 36, 173
cystitis, schistosomiasis 62
cytomegalovirus (CMV)
oesophagitis 79
DandyWalker malformation 182
dentigerous cyst 95
dermatomyositis 121
dermoid cyst 89
ruptured 112
detrusor muscle denervation 60
diabetes mellitus
cerebral abscess 90
emphysematous pyelitis 63
neuropathic foot 138
renal papillary necrosis 78
diaphragmatic hernia, congenital
164
diaphyseal aclasis with sarcomatous
transformation 130
double bubble sign 44, 183
double density sign 148
double PCL sign 159
Downs syndrome
annular pancreas association 44,
183
duodenal atresia 183
drug-induced conditions, pulmonary
fibrosis 18

347

General Index
ductal carcinoma in situ (DCIS)
186
duodenal atresia 44, 183
dwarfism, thanatophoric 161
dyspnoea
Langerhans cell histocytosis 27
left upper lobe tumour with
lymphangitis carcinomatosa 12
right sided bulla 35
Echino co ccus granulo sus 40
EhlersDanlos syndrome 132
Eisenmengers syndrome 16
Ellisvan Creveld syndrome 177
embolization, arteriovenous
malformations 33
emphysematous pyelitis 63
emphysematous pyelonephritis 63
encephalitis, herpes simplex virus
113
enchondroma 144
endometrioma 83
endometriosis 81, 83
endoscopic retrograde
cholangiopancreatography (ERCP)
annular pancreas 44
primary sclerosing cholangitis
45
endotracheal tube position 14
enlarged vestibular aqueduct
syndrome 86
eosinophilic granuloma 154, 158
eosinophils, pulmonary infiltration
21
ependymoma 89, 180
epidermoid cyst 94
epilepsy
haemoangioblastoma 104
tuberous sclerosis 101
Erlenmeyer flask deformity 119,
140, 178,
Escherichia co li, emphysematous
pyelitis 63
Ewings sarcoma 125
extra-corporeal membrane
oxygenation (ECMO) 164
extrinsic allergic alveolitis (EAA) 17
eyes
optic drusen 91
thyroid ophthalmopathy 88
18 fluoro-2-deoxyglucose (FDG)
24
fallen fragment sign 95
Fallots tetralogy 181
familial adenomatous polyposis
(FAP) 49
feline oesophagus 38
femoral epiphysis, slipped upper
117
femoral head
avascular necrosis 117, 135, 139
congenital multicentric ossification
135

348

femur
congenital pseudarthrosis 127
parosteal osteosarcoma 136
fibrodysplasia ossificans progressiva
123
fibromuscular dysplasia 58
fibrous dysplasia 149
figure of 3 sign 32
fine needle aspiration (FNA) 185
fistulae, Crohns disease 53
fluorosis 119, 140, 157
focal nodular hyperplasia (FNH),
hepatic 85
Fongs disease 128
fontanelle, tense 162
football sign 75, 167
fracture, slipped upper femoral
epiphysis 117
frontal sinus osteoma 96
fungal infections
Candida 63, 79
Pneumo cystis carinii pneumonia
37
see also allergic bronchopulmonary
aspergillosis (ABPA); Aspergillus
gadolinium BOPTA 85
gallbladder, porcelain 82
gallbladder carcinoma 82
gallstone ileus 41, 47
gallstones
annular pancreas association 44
Crohns disease 53
ectopic 41
pancreatitis 72
Gardners syndrome 44, 49
gastric adenoma 49
gastric carcinoma, linitis plastica
76
gastric leiomyoma 64
gastric lipoma 64
gastric varices 42
gastritis, erosive 54
Gastrograffin 173
gastrointestinal mucosa, cobblestone
53
gastro-oesophageal reflux 38
gastro-oesophageal sphincter,
pneumatic dilatation 57
Gauchers disease 119, 178
germ cell tumours 92
giant cell tumour 115, 133, 134
glioblastoma multiforme, butterfly
110
glomus jugulotympanicum tumour
108
gluten-free diet 69
gluten-sensitive enteropathy 69
gouty arthropathy 143
granulomatous disease
Crohns disease 53
Langerhans cell histiocytosis
27
Wegeners granulomatosis 5

grey matter
basal ganglia density 166
heterotopic 169
groin, hernial orifices 47
ground glass density lesion 149
gynaecomastia 140, 189
haemangioblastoma 104, 180
haemangioma 129, 137
haemochromatosis 50
haemoptysis, cystic fibrosis 36
haemosiderosis, pulmonary 23
hair on end sign 129, 155
hearing loss
acoustic neuroma 94
enlarged vestibular aqueduct
syndrome 86
glomus jugulotympanicum tumour
108
heart failure 29
hepatic abscess 80
hepatic adenoma 85
hepatocellular carcinoma
focal nodular hyperplasia
differential diagnosis 85
risk with haemochromatosis 50
hepatocytes, focal nodular
hyperplasia 85
herpes oesophagitis 79
herpes simplex virus (HSV)
encephalitis 113
high resolution computed
tomography (HRCT)
interstitial lung disease 17
primary tuberculosis 2
pulmonary sarcoidosis 1
hip joint, primary tuberculosis 2
Hirschsprungs disease 173
Histo plasma capsulatum
(histoplasmosis) 25
HIV infection
oesophagitis 79
Pneumo cystis carinii pneumonia
37
horizontal meniscal tear 159
humeral head, avascular necrosis 6,
18, 53, 65
Hurlers syndrome 184
hyaline membrane disease (HMD)
165, 170
hydatid disease 40
hydrocephalus 106, 180
hydronephrosis
horseshoe kidney 51
posterior urethral valves 172
renal obstruction 68
hypercalcaemia, pancreatitis 72
hyperparathyroidism 152, 160
renal medullary nephrocalcinosis
43
hypertension
nephroblastoma 174
neurofibromatosis 58
systemic 58

General Index
hypertrophic pulmonary
osteoarthropathy (HPOA) 36,
157
hypothyroidism, multiple wormian
bones 168
hypoxic-ischaemic injury 163
low-density basal ganglia 166
hysterectomy, adenomyosis 81
ileum, jejunalization 69
iliac horns, bilateral posterior 128
immunocompromised patients,
oesophagitis 79
imperforate anus 44, 173
infantile cortical hyperostosis 175
infections
pancreatitis 72
renal scarring 68
see also fungal infections
infertility
adenomyosis 81
endometrioma 83
inflammatory bowel disease
primary sclerosing cholangitis 45
see also Crohns disease; ulcerative
colitis
inflammatory malignancy 188
inguinal hernia 47
intracranial pressure, raised 162
intraosseous lipoma 146
intraperitoneal gas, free 75
intussusception, coeliac disease 69
invasive ductal carcinoma (IDC) of
breast 185, 186
iron overload 50
ivory osteoma 96
ivory vertebra sign 137
Jaccouds arthritis 132
jejunum folds 69
joints
primary tuberculosis 2
see also knee joint
juvenile pilocytic astrocytoma 104,
180
juvenile recurrent parotitis 87
Kerley B lines 12
kidneys
horseshoe 51
medullary sponge 43
multicystic dysplastic 172
partial duplex 78
Wilms tumour 51, 174
Klebsiella, emphysematous pyelitis
63
Kleins lines 117
knee joint
osteochondritis dissecans 131
tumoral calcinosis 121
knee instability 159
Kupffer cells, focal nodular
hyperplasia 85

kyphosis, ankylosing spondylitis


18
lamda sign 1
Langerhans cell histiocytosis 27,
158
large bowel
obstruction in sigmoid volvulus
46
occult bleeding 39
lead poisoning 178
left atrial apical enlargement 23
left ventricular infarct 30
leontiasis ossea 149
LeriWeil disease 153
ligamentous insertion calcification
119
light bulb sign 71
linitis plastica 76
lipoma
chest wall 9
gastric 64
intraosseous 146
lissencephaly 169
liver
calcified metastases 52
Carolis disease 80
cavernous haemangioma 71
focal nodular hyperplasia 85
hydatid disease 40
transplantation 45
liver overlap sign 46
loose body formation 138
Loosers zones 126, 176
lung(s)
ground glass opacity 165
left upper lobe tumour with
lymphangitis carcinomatosa 12
lobar collapse 14
lung calcification 7, 8
alveolar microlithiasis 20
pleural pseudotumour 11
lung cancer
miliary metastases 4
pulmonary asbestosis 15
lung cavitation 7, 8
Aspergillus fungus ball 18
pleural pseudotumour 11
pulmonary metastases 14
lung disease, interstitial 17
lung nodules
alveolar microlithiasis 20
cavitating 5
histological characterization 8
metastases with underlying
osteosarcoma 8
pleural pseudotumour 11
Pneumo cystis carinii pneumonia
37
lymph nodes
breast cancer spread 185, 186
inflammatory breast cancer 188
popcorn calcifications 25

lymphadenopathy
primary tuberculosis 2
pulmonary sarcoidosis 1
lymphangioleiomyomatosis 27
lymphangitis carcinomatosa, with left
upper lobe tumour 12
lymphoma 110, 140
osteoblastic response 137
recurrence following amputation
118
staging 24
Madelung deformity 147, 153
see also pseudo-Madelung
deformity
Maffucis syndrome 144
magnetic resonance angiography
(MRA), coarctation of the aorta
32
magnetic resonance
cholangiopancreatography
(MRCP)
annular pancreas 44
primary sclerosing cholangitis
45
magnetic resonance imaging (MRI),
cardiac 30
malabsorption, coeliac disease 69
malignant mesothelioma 3, 15
mammography 185, 186
benign breast cysts 187
calcifications 185, 186, 190
gynaecomastia 189
invasive lobular carcinoma 191
marble bone disease 119
mastectomy, previous breast cancer
31
mastocytosis 137, 140
McCuneAlbright syndrome 149
mebendazole, hydatid disease 40
meconium aspiration syndrome
170
meconium ileus 36, 173
meconium peritonitis 167
mediastinal mass
bronchogenic cyst 13
thoracic aortic aneurysm 10
medullary sponge kidney 43
medulloblastoma 104, 180
mega-oesophagus 57
melorheostosis 122
meningioma 89, 94
meningitis, chemical 112
meniscus sign 40
meniscus, tears 159
mesenteric angiography,
angiodysplasia association 39
mesenteric desmoid tumour, with
familial adenomatous polyposis
(FAP) 49
mesenteric ischaemia 59
mesenteric metastases 61
mesentery, tumour spread 61

349

General Index
metacarpals
head flattening 50
idiopathic shortening of 4th 147
metastases
adrenal adenoma differentiation
67
breast cancer 115, 185
bronchogenic carcinoma 24
colonic 61
colorectal carcinoma 52
drop 180
lung 8
nephroblastoma 174
osteoblastic 137
pelvic 115
prostate carcinoma 140
pulmonary 130
sclerotic 122
sclerotic bone from breast cancer
31, 137, 140
superscan 156
metastatic calcinosis 20
Meyers dysplasia 135
microcolon 173
microlithiasis, testicular 73
midbrain infarction 100
middle cerebral artery (MCA)
aneurysm 114
infarct 113
miliary nodules 25
MISME syndrome 94
mitral valve disease 23
mitral valvotomy 23
Morquios syndrome 120, 135,
184,
Moulage sign 69
mucocele, sphenoid sinus 109
mucopolysaccharidoses 184
multiple endocrine neoplasia (MEN)
108
multiple epiphyseal dysplasia 135
multiple sclerosis 97
myelofibrosis 140
myocardial fibrosis, cardiac sarcoid
22
myometrial hyperplasia 81
myositis ossificans 121
progressiva 123
nailpatella syndrome 128
nasal region, Wegeners
granulomatosis 5
necrotizing enterocolitis 167
neonates
duodenal atresia 183
meconium aspiration syndrome
170
meconium ileus 173
see also prematurity
neoterminal ileum, Crohns disease
53
nephroblastoma 174
nephrocalcinosis, renal medullary
43

350

neuroblastoma 174
neurofibroma 111
mediastinal mass 10
neurofibromatosis 127
angiomyolipoma 48
CXR changes 27
hypertension 58
neurofibromatosis type 1 111, 127
optic glioma 99
skin nodules 8
neurofibromatosis type 2 94
neurogenic bladder 60
neuronal migration, congenital
anomalies 169
neuropathic foot, diabetic 138
non accidental injury (NAI) 175,
179
oblique meniscal tear 159
odontogenic keratocyst 95
odontoid peg 120
oesophageal atresia, congenital with
tracheo-oesophageal fistula 171
oesophageal cancer 34
oesophageal rupture 34
oesophageal stents 34
oesophageal strictures 38
oesophageal tumour, with left lower
lobe collapse and cavitating
pulmonary metastases 14
oesophageal varices 42
erosive gastritis 54
oesophagitis 79
oesophagus
achalasia 57
feline 38
leiomyoma 55
oligohydramnios, posterior urethral
valves 172
Olliers disease 144
onion peel sign 40
opsoclonus, neuroblastoma 174
optic chiasm 99
compression 106
optic drusen 91
optic nerve glioma 99
oropharyngeal candidiasis 79
OslerWeberRendu syndrome 33
multiple haemangiomas 71
osteitis condensans ilii 116
osteoarthropathy
Crohns disease 53
hypertrophic 36, 157
osteochondritis dissecans 131
osteochondroma 130
osteochondromatosis, synovial 121,
135
osteochondrosis, vertebral 154
osteofibrous dysplasia 149
osteogenesis imperfecta 127, 161,
179
multiple wormian bones 168
osteoid osteoma 134, 148
osteomalacia 126, 176

osteo-onychodysplasia 128
osteopenia 168, 176
osteopetrosis 119, 140, 145
osteopoikilosis 122
osteosarcoma 125, 134
conventional 136
lung metastases 8
parosteal 136
osteosclerosis 119, 140
diffuse 155
ovarian cancer 61
ovarian dermoid cysts 84
pachydermoperiostosis 157
pachygyria 169
Pagets disease 137, 140
pancreas
annular 44, 183
pseudocyst formation 72
pancreatic insufficiency, cystic
fibrosis 36
pancreatitis 72
panda sign 1
paranasal sinus osteoma 96
parotitis, juvenile recurrent 87
parrot beak tear
meniscal 159
patent ductus arteriosus 32
pectoral muscle atrophy/absence
31
pelvic haematoma 70
pelvic overlap sign 46
peptic stricture feline oesophagus
38
percutaneous valve balloon dilatation
23
pericarditis, histoplasmosis 25
peritoneal calcification 167
periventricular leukomalacia 166
PeutzJeghers disease 49
phakomatoses 101
pilocytic astrocytoma 104
pineal germinoma 92
pituitary macroadenoma 106
pleural calcification, asbestos
exposure 15
pleural effusions
asbestos exposure 15
meconium aspiration syndrome
170
primary tuberculosis 2
pulmonary oedema due to heart
failure 29
pleural fluid, oblique fissure 11
pleural lesions, chest wall lipoma
9
pleural plaques
asbestos exposure 15
malignant mesothelioma 3
pleural pseudotumour 11
pneumatocele, Pneumo cystis carinii
pneumonia 37
pneumatosis
colon 66

General Index
pneumatosis (co ntinued)
cystoides intestinalis 66
intestinalis 65, 167
pneumobilia 41
pneumoconiosis 6, 7
Pneumo cystis carinii pneumonia
(PCP) 37
pneumomediastinum 34, 165
meconium aspiration syndrome
170
pneumonia, aspiration 171
pneumoperitoneum 34, 75
necrotizing enterocolitis 167
pneumothorax 31
bulla differential diagnosis 35
cystic fibrosis 36
endometrioma 83
Langerhans cell histiocytosis 27
meconium aspiration syndrome
170
Pneumo cystis carinii pneumonia
37
positive pressure ventilation
complication 165
polycystic kidney disease 48
polydactyly 177
polyhydramnios, annular pancreas
44
polymicrogyria 169
portal hypertension, oesophageal
varices 42
positive pressure ventilation 165
positron emission tomography
(PET), metastatic bronchogenic
carcinoma 24
posterior cruciate ligament 159
double PCL sign 159
posterior inferior cerebellar artery
infarction 100
posterior urethral valves 172
Potts disease 142
prematurity
hyaline membrane disease 165,
170
necrotizing enterocolitis 167
primary sclerosing cholangitis 45
primitive neuroectodermal tumour
(PNET) 180
progressive massive fibrosis 7
prostate carcinoma, metastases 140
Pro teus, emphysematous pyelitis 63
prune belly, posterior urethral valves
172
pseudodiverticulosis, intramural 38
pseudohyperparathyroidism 147
pseudo-Madelung deformity 130,
153
Pseudo mo nas, emphysematous
pyelitis 63
pseudoureteroceles 56
psoas abscess 142
psoriasis 116
psoriatic arthropathy 143, 152
pulmonary arterial hypertension 16

pulmonary arteriovenous
malformation 33
pulmonary asbestosis 3, 15
pulmonary embolus 31
pulmonary fibrosis
drug-induced 18
lower zone 15, 17
upper zone 15, 17, 18
pulmonary haemosiderosis 23
pulmonary interstitial emphysema
(PIE) 165
pulmonary malignancy
asbestosis 15
systemic sclerosis 17
pulmonary metastases, cavitating
14
pulmonary nodules
follow-up 6
mitral valve disease 23
oesophageal tumour 14
rheumatoid lung 6
sarcoidosis 1
pulmonary oedema 31
heart failure 29
pulmonary opacity
barium aspiration 20
ground glass 29
left upper lobe tumour with
lymphangitis carcinomatosa 12
lung lobe collapse 14
metastatic calcinosis 20
pleural pseudotumour 11
pulmonary sarcoidosis 1
pulmonary venous hypertension
mitral valve disease 23
pulmonary oedema due to heart
failure 29
pyelitis, emphysematous 63
pyeloureteritis cystica 77
pyknodysostosis 119, 168
Pyles disease 119, 178
Rathke cleft cyst 106
Raynauds phenomenon 17
thoracic outlet syndrome 28
rectum
stented tumour with calcified liver
metastases 52
ulcerative colitis 65
Reiters syndrome 116, 143
renal adenocarcinoma 51
renal artery stenosis 58
renal ectopia, crossed fused 68
renal lesions, fat 48
renal medullary nephrocalcinosis 43
renal obstruction, hydronephrosis
68
renal osteodystrophy 119, 140
renal papillary necrosis 78
renal scarring, infections 68
renal stones, horseshoe kidney 51
renal tuberculosis with
autonephrectomy 74
renal tubular acidosis 43

reninangiotensin system,
overactivity 58
respiratory distress, meconium
aspiration syndrome 170
reversal sign 114, 163
rheumatic heart disease 23
rheumatoid arthritis 120, 143
rheumatoid factor 17
rheumatoid lung 6
rheumatologic syndromes,
histoplasmosis 25
ribs
bilateral cervical 28
exostoses 130
lesions 9
notching 32
rickets 168, 175, 176
right ventricular hypertrophy, mitral
valve disease 23
Rigler sign 75, 167
Riglers triad 41
Rokitansky nodule 84
S sign of Golden 14
sacral agenesis 173
sacroiliac joint disease
Crohns disease 53
ulcerative colitis 65
sacroiliac joint fusion 139
sacroiliitis 116, 143
sandwich vertebrae 140
sarcoidosis
cardiac sarcoid 22
pulmonary 1
schistosomiasis 62
schizencephaly 169
scleroderma 132, 152
sclerosis
Charcot joint 138
diffuse 119
septal thickening, interlobular
29
sialectasis, juvenile punctate 87
sickle cell disease 129, 140, 155
siderosis, transfusion 50
sigmoid colon, displacement 84
sigmoid volvulus 46
silicosis 18
simple bone cyst 115
Sjgrens syndrome 87
skin folds, lung edge artefact 35
skin nodules, neurofibromatosis type
1 8
slipped upper femoral epiphysis
(SUFE) 117
small bowel
adenoma 49
ischaemia 59
loop dilatation 69
strictures 59
thickening in Crohns disease 53
small bowel obstruction 41
adhesions 47
mechanical 41, 47

351

General Index
sphenoid sinus mucocele 109
spleen, calcification 25
spondyloarthropathy 143
spondylitis, tuberculous 142
Staphylo co ccus aureus, Brodies
abscess 134
steroid use
avascular necrosis of femoral head
139
avascular necrosis of humeral head
6, 18, 53, 65
cerebral abscess 90
Meyers dysplasia 135
rheumatoid disease 6
stillbirth 161
string of beads sign 47
subarachnoid haemorrhage 114
subclavian artery stenosis 19
subclavian steal syndrome 19
subependymal hamartomas 101
subependymal nodules 169
sulphur colloid scans, focal nodular
hyperplasia 85
superior mesenteric vein thrombosis
59
superscan, metastases 156
surfactant therapy 165
SwyerJames syndrome 31
syndactyly 177
syndesmophytes 18, 116
synovial osteochondromatosis 121,
135
systemic lupus erythematosus (SLE)
132
systemic sclerosis 17
teeth, ovarian dermoid 84
tension pneumothorax
intrapulmonary bronchogenic cyst
13
positive pressure ventilation
complication 165
terminal ileum
Crohns disease 53
ulcerative colitis 65
testicular microlithiasis 73
tetralogy of Fallot 181
thalassemia 119, 178
major 124, 129
thanatophoric dysplasia 161
thoracic aortic aneurysm 10
thoracic outlet syndrome 28
thyroid acropachy 157
thyroid cancer, miliary metastases 4
thyroid ophthalmopathy 88
TNM staging, colorectal carcinoma
52
total mesorectal excision (TME)
procedure 52
toxic megacolon, perforation 65
tracheo-oesophageal atresia 44

352

tracheo-oesophageal fistula with


congenital oesophageal atresia
171
transfusion siderosis 50
transitional cell carcinoma 51
transjugular intrahepatic
portosystemic shunt (TIPS) 42
transposition of the great vessels
181
triangle sign 75
tuberculoma 2
tuberculosis
bladder 62, 74
miliary 2
Pneumo cystis carinii pneumonia
37
primary 2
reactive 2
renal with autonephrectomy 74
ureteral 74
tuberculous spondylitis 142
tuberous sclerosis 27, 101
angiomyolipoma 48
subependymal nodules 169
tumoral calcinosis, knee joint 121
Turcot syndrome 49
Turners syndrome 147, 153
ulcerative colitis 53, 65
linear gas opacity 66
primary sclerosing cholangitis
association 45
umbilical vessel catheters, hyaline
membrane disease 165
upper limb ischaemia, thoracic outlet
syndrome 28
urachus sign 75
ureteroceles, bilateral 56
ureteropelvic junction obstruction
51
ureters
calcification 62
filling defect 78
pyeloureteritis cystica 77
strictures 62
tuberculosis 74
urinary calculi 56
urinary tract infection
pyeloureteritis cystica 77
ureteroceles 56
urine ascites, posterior urethral valves
172
urinoma, posterior urethral valves
172
VACTERL syndrome 171, 183
valvulae conniventes 47
atrophic 59
vascular ectasia, colonic circulation
39
venous angiomas 102

venous insufficiency 157


venous sinus thrombosis 93
ventilation, mechanical 164, 165
ventricular septal defect
coarctation of the aorta association
32
pulmonary arterial hypertension
16
vertebra
collapse in avascular necrosis 6
ivory 137
plana 154
posterior scalloping 151
sandwich 140
vertebral osteochondrosis 154
vertebrobasilar insufficiency,
subclavian steal syndrome 19
vertebroplasty 137
vesicoureteric reflux
horseshoe kidney 51
schistosomiasis 62
vestibular aqueduct, enlarged 86
villous atrophy, coeliac disease 69
viscus, perforated 75
visual loss
sphenoid sinus mucocele 109
thyroid ophthalmopathy 88
vitamin D deficiency 126, 176
vomiting, annular pancreas 44
von HippelLindau syndrome 104,
180
Wallenbergs syndrome 100
water lily sign 40
Wegeners granulomatosis 5
Wilms tumour
bilateral 174
horseshoe kidney association 51
wormian bones, multiple 119, 168,
179

Das könnte Ihnen auch gefallen